Academia.eduAcademia.edu

Pembukaan OSN 2007

Pembukaan OSN 2007 Simposium Guru 2008 di Makassar, Sulawesi Selatan KATA PENGANTAR Alhamdulillah Penulis ucapkan kepada Allah, SWT karena dengan karunia-Nya Penulis dapat menyelesaikan penulisan buku ini. Buku ini Penulis t ulis sebagai salah sat u j awaban akan masih kurangnya buku-buku Olimpiade Mat emat ika yang ada di Indonesia. Buku ini berisi soal dan solusi Olimpiade Mat emat ika Tingkat Kabupat en/ Kot a, Tingkat Provinsi dan Tingkat Nasional yang berlangsung di Indonesia dari t ahun 2002-2009 dan dapat dipergunakan dalam menyiapkan siswa-siswa menuj u Olimpiade Sains Nasional pada t ahun-t ahun berikut nya. Ucapan t erima kasih kepada semua pihak yang t elah membant u dalam penyelesaian buku ini, khususnya buat rekan-rekan dalam f orum www. olimpiade. org yang t elah memberikan dorongan moril kepada Penulis, baik yang pernah bert emu secara langsung dengan Penulis maupun yang sampai saat ini belum pernah bert emu langsung dengan Penulis. Tak lupa t erima kasih j uga Penulis ucapkan kepada ist eri t ercint a Penulis, Rosya Hast aryt a, S. Si, yang t elah memberi dukungan yang besar kepada Penulis sert a j uga t elah melahirkan put eri pert ama kami, Kayyisah Haj idah, pada t anggal 2 Desember 2009. Penulis merasa bahwa buku ini masih j auh dari sempurna. Unt uk it u Penulis mengharapkan saran dan kriit ik dari Pembaca yang budiman sebagai bahan perbaikan buku ini. Unt uk korespondensi, pembaca dapat mengirimkan email ke eddyhbkl@yahoo. com. Akhir kat a semoga buku ini dapat bermanf aat yang sebesar-besarnya bagi Pembaca sekalian. Bengkulu, Desember 2009 EDDY HERMANTO, ST eddyhbkl@yahoo. com ii DAFTAR ISI HALAMAN JUDUL KATA PENGANTAR DAFTAR ISI ……………………………………………………………………… ……………………………………………………………………… ……………………………………………………………………… i ii iii OLIMPIADE MATEMATIKA INDONESIA TAHUN 2 002 Soal Olimpiade Mat emat ika Tk. Kabupat en/ Kot a Tahun 2002 Solusi Olimpiade Mat emat ika Tk. Kabupat en/ Kot a Tahun 2002 Soal Olimpiade Mat emat ika Tk. Provinsi Tahun 2002 Solusi Olimpiade Mat emat ika Tk. Provinsi Tahun 2002 Soal Olimpiade Sains Nasional Bidang Mat emat ika Tahun 2002 Solusi Olimpiade Sains Nasional Bidang Mat emat ika Tahun 2002 …………………… …………………… …………………… …………………… …………………… …………………… 1 5 12 17 30 32 OLIMPIADE MATEMATIKA INDONESIA TAHUN 2 003 Soal Olimpiade Mat emat ika Tk. Kabupat en/ Kot a Tahun 2003 Solusi Olimpiade Mat emat ika Tk. Kabupat en/ Kot a Tahun 2003 Soal Olimpiade Mat emat ika Tk. Provinsi Tahun 2003 Solusi Olimpiade Mat emat ika Tk. Provinsi Tahun 2003 Soal Olimpiade Sains Nasional Bidang Mat emat ika Tahun 2003 Solusi Olimpiade Sains Nasional Bidang Mat emat ika Tahun 2003 …………………… …………………… …………………… …………………… …………………… …………………… 39 43 51 56 69 73 OLIMPIADE MATEMATIKA INDONESIA TAHUN 2 004 Soal Olimpiade Mat emat ika Tk. Kabupat en/ Kot a Tahun 2004 Solusi Olimpiade Mat emat ika Tk. Kabupat en/ Kot a Tahun 2004 Soal Olimpiade Mat emat ika Tk. Provinsi Tahun 2004 Solusi Olimpiade Mat emat ika Tk. Provinsi Tahun 2004 Soal Olimpiade Sains Nasional Bidang Mat emat ika Tahun 2004 Solusi Olimpiade Sains Nasional Bidang Mat emat ika Tahun 2004 …………………… …………………… …………………… …………………… …………………… …………………… 79 82 87 92 107 111 OLIMPIADE MATEMATIKA INDONESIA TAHUN 2 005 Soal Olimpiade Mat emat ika Tk. Kabupat en/ Kot a Tahun 2005 Solusi Olimpiade Mat emat ika Tk. Kabupat en/ Kot a Tahun 2005 Soal Olimpiade Mat emat ika Tk. Provinsi Tahun 2005 Solusi Olimpiade Mat emat ika Tk. Provinsi Tahun 2005 Soal Olimpiade Sains Nasional Bidang Mat emat ika Tahun 2005 Solusi Olimpiade Sains Nasional Bidang Mat emat ika Tahun 2005 …………………… …………………… …………………… …………………… …………………… …………………… 117 121 127 132 144 148 OLIMPIADE MATEMATIKA INDONESIA TAHUN 2 006 Soal Olimpiade Mat emat ika Tk. Kabupat en/ Kot a Tahun 2006 Solusi Olimpiade Mat emat ika Tk. Kabupat en/ Kot a Tahun 2006 Soal Olimpiade Mat emat ika Tk. Provinsi Tahun 2006 Solusi Olimpiade Mat emat ika Tk. Provinsi Tahun 2006 Soal Olimpiade Sains Nasional Bidang Mat emat ika Tahun 2006 Solusi Olimpiade Sains Nasional Bidang Mat emat ika Tahun 2006 …………………… …………………… …………………… …………………… …………………… …………………… 157 160 166 171 183 187 OLIMPIADE MATEMATIKA INDONESIA TAHUN 2 007 Soal Olimpiade Mat emat ika Tk. Kabupat en/ Kot a Tahun 2007 Solusi Olimpiade Mat emat ika Tk. Kabupat en/ Kot a Tahun 2007 …………………… …………………… 195 198 iii Soal Olimpiade Mat emat ika Tk. Provinsi Tahun 2007 Solusi Olimpiade Mat emat ika Tk. Provinsi Tahun 2007 Soal Olimpiade Sains Nasional Bidang Mat emat ika Tahun 2007 Solusi Olimpiade Sains Nasional Bidang Mat emat ika Tahun 2007 …………………… …………………… …………………… …………………… 204 209 223 227 OLIMPIADE MATEMATIKA INDONESIA TAHUN 2 008 Soal Olimpiade Mat emat ika Tk. Kabupat en/ Kot a Tahun 2008 Solusi Olimpiade Mat emat ika Tk. Kabupat en/ Kot a Tahun 2008 Soal Olimpiade Mat emat ika Tk. Provinsi Tahun 2008 Solusi Olimpiade Mat emat ika Tk. Provinsi Tahun 2008 Soal Olimpiade Sains Nasional Bidang Mat emat ika Tahun 2008 Solusi Olimpiade Sains Nasional Bidang Mat emat ika Tahun 2008 …………………… …………………… …………………… …………………… …………………… …………………… 236 240 248 253 268 272 OLIMPIADE MATEMATIKA INDONESIA TAHUN 2 009 Soal Olimpiade Mat emat ika Tk. Kabupat en/ Kot a Tahun 2009 Solusi Olimpiade Mat emat ika Tk. Kabupat en/ Kot a Tahun 2009 Soal Olimpiade Mat emat ika Tk. Provinsi Tahun 2009 Solusi Olimpiade Mat emat ika Tk. Provinsi Tahun 2009 Soal Olimpiade Sains Nasional Bidang Mat emat ika Tahun 2009 Solusi Olimpiade Sains Nasional Bidang Mat emat ika Tahun 2009 …………………… …………………… …………………… …………………… …………………… …………………… 279 283 292 297 310 314 iv SELEKSI OLIMPIADE TINGKAT KABUPATEN/ KOTA TAHUN 2002 TIM OLIMPIADE MATEMATIKA INDONESIA TAHUN 2003 Bidang Mat emat ika Waktu : 90 Menit DEPARTEMEN PENDIDIKAN NASIONAL DIREKTORAT JENDERAL PENDIDIKAN DASAR DAN MENENGAH DIREKTORAT PENDIDIKAN MENENGAH UMUM TAHUN 2002 1 OLIMPIADE MATEMATIKA TINGKAT KABUPATEN/ KOTA TAHUN 2002 1 Bagian Pertama (2 ) (4 ) 4 8 1. Bilangan A. ¼ 8 2 sama dengan B. ½ C. 1 D. 2 E. 8 2. Bando selalu berkat a bohong. Suat u hari dia berkat a kepada t et angganya, Andi : “ Paling t idak salah sat u diant ara kit a t idak pernah berbohong. ” Dari inf ormasi ini kit a merasa past i bahwa A. Andi selalu berbohong D. Andi sesekali berkat a benar B. Andi sesekali berbohong E. Andi t idak pernah berkat a apa pun C. Andi selalu berkat a benar 3. Bilangan n t erbesar sehingga 8n membagi 4444 adalah A. 8 B. 22 C. 29 D. 44 4. Pernyat aan manakah yang benar ? A. Jika x < 0 maka x 2 > x C. Jika x2 > x maka x > 0 2 B. Jika x > 0 maka x > 0 D. Jika x 2 > x maka x < 0 E. 88 E. Jika x < 1 maka x 2 < x n 5. Misalkan x −n sama dengan ⎛⎜ 1 ⎞⎟ unt uk set iap bilangan real x. Maka a3 − a −3 sama dengan ⎝ x⎠ E. bukan diant ara A, B, C dan D C. ⎛⎜ a − 1 ⎞⎟⎛⎜ a 2 − 2 + 1 ⎞⎟ A. ⎛⎜ a − 1 ⎞⎟⎛⎜ a 2 + 1 + 1 ⎞⎟ 2 a ⎠⎝ a ⎠⎝ a ⎠ a2 ⎠ ⎝ ⎝ B. ⎛⎜ 1 − a ⎞⎟⎛⎜ a 2 − 1 + 1 ⎞⎟ D. ⎛⎜ 1 − a ⎞⎟⎛⎜ 1 + 1 + a 2 ⎞⎟ 2 2 a a ⎝ ⎝a ⎠⎝ ⎠⎝ a ⎠ ⎠ 6. Lima ekor kambing memakan rumput seluas 5 kali ukuran lapangan bola dalam 5 hari. Berapa hari yang diperlukan oleh 3 ekor kambing unt uk menghabiskan rumput seluas 3 kali lapangan bola ? A. 2 B. 3 C. 4 D. 5 E. 6 7. Jika unt uk set iap x, y bilangan real berlaku x$y = xy − x + y maka (x + y)$(x − y) sama dengan ⋅⋅ A. x2 − y2 + 2x C. x2 − y2 + 2y E. x 2 − y2 B. x2 − y2 − 2x D. x2 − y2 − 2y 8. Berapa banyak pasang bilangan bulat posit if (a, b) yang memenuhi A. 1 B. 2 C. 3 D. 4 2 1 1 1 + = ? a b 6 E. 5 9. Unt uk nilai a yang manakah garis lurus y = 6x memot ong parabola y = x 2 + a t epat di sat u t it ik?⋅ A. 7 B. 8 C. 9 D. 10 E. 11 10. Digit 1, 9, 9, 8 dalam 1998 mempunyai j umlah t ot al 1 + 9 + 9 + 8 = 27. Bilangan berikut nya yang mempunyai j umlah digit 27 t erj adi di ant ara t ahun A. 2500 dan 2700 C. 2901 dan 3100 E. 9901 dan 9999 B. 2701 dan 2900 D. 3101 dan 9900 2 Bagian Kedua 11. Pada suat u segit iga ABC, sudut C t iga kali besar sudut A dan sudut B dua kali besar sudut A. Berapakah perbandingan (rasio) ant ara panj ang AB dengan BC ? 12. Bando dan Bandi ingin mengecat pagar, Bando dapat menyelesaikan pengecat an pagar oleh dirinya sendiri dalam wakt u 3 j am, sedangkan Bandi dapat menyelesaikannya dalam 4 j am. Pada pukul 12: 00 siang mereka mulai mengecat pagar bersama-sama. Akan t et api pada suat u ket ika mereka bert engkar. Mereka bert engkar selama 10 menit dan dalam masa it u t idak sat upun yang melakukan pengecat an. Set elah pert engkaran t ersebut Bandi pergi dan Bando meyelesaikan pengecat an pagar sendirian. Jika Bando menyelesaikan pengecat an pada pukul 14: 25, pada pukul berapakah pert engkaran dimulai ? 13. Berapakah j umlah digit -digit bilangan 22002 ⋅ 52003 ? 14. Berapa banyak bilangan posit if yang kurang dari 10. 000 yang berbent uk x 8 + y8 unt uk suat u bilangan bulat x > 0 dan y > 0 ? 15. Tent ukan bilangan n t erkecil sehingga set iap subhimpunan dari {1, 2, 3, ⋅⋅⋅, 20} yang beranggot akan n unsur past i mengandung dua anggot a yang selisihnya 8. 16. Garis AB dan CD sej aj ar dan berj arak 4 sat uan. Misalkan AD memot ong BC di t it ik P diant ara kedua garis. Jika AB = 4 dan CD = 12, berapa j auh P dari garis CD ? 17. Misalkan a dan b bilangan real yang berbeda sehingga a a + 10b =2 + b b + 10a Tent ukan nilai a . b 18. Bilangan bulat posit if p ≥ 2 disebut bilangan prima j ika ia hanya mempunyai f akt or 1 dan p. Tent ukan nilai penj umlahan semua bilangan prima diant ara 1 dan 100 yang sekaligus bersif at : sat u lebihnya dari suat u bilangan kelipat an 5 dan sat u kurangnya dari suat u bilangan kelipat an 6. 3 19. Misalkan a = 12 2 2 3 2 10012 + + +L+ 1 3 5 2001 b = 12 2 2 3 2 10012 + + +L+ 3 5 7 2003 dan Tent ukan bilangan bulat yang nilainya paling dekat ke a − b. 20. Suat u persegi panj ang berukuran 8 kali 2√2 mempunyai t it ik pusat yang sama dengan suat u lingkaran berj ari-j ari 2. Berapakah luas daerah irisan ant ara persegi panj ang dan lingkaran t ersebut ? 4 SELEKSI OLIMPIADE TINGKAT KABUPATEN/ KOTA 2002 TIM OLIMPIADE MATEMATIKA INDONESIA 2003 Prestasi itu diraih bukan didapat !!! SOLUSI SOAL Bidang Mat emat ika Disusun oleh : Eddy Hermant o, ST 5 Olimpiade Matematika Tk Kabupaten/ Kota 2002 BAGIAN PERTAMA 1. (Jawaban : C) ∴ (2 ) (4 ) 4 8 8 2 = 2 32 416 = 2 32 2 32 =1 2. (Jawaban : B) Ingkaran dari : paling t idak salah sat u di ant ara kit a t idak pernah berbohong adalah : ∴ Kedua-duanya pernah berbohong 3. (Jawaban : C) 4444 = 444 ⋅ 1144 = 1622 ⋅ 1144 = 822 ⋅ 222 ⋅ 1144 = 822 ⋅ (23) 7 ⋅ 2 ⋅ 1144 = 829 ⋅ 2 ⋅ 1144 Karena 8 t idak membagi (2 ⋅ 1144) , maka : ∴ nmaks = 29 4. (Jawaban : A) Dasar t eori : Jika x<0 maka x2 > x Jika 0 < x < 1 maka x2 < x Jika x>1 maka x2 > x A. Benar B. Salah karena j ika x 2 > 0 dimungkinkan x < 0 at au x > 0 C. Salah. Karena x 2 > x maka x (x −1) > 0 sehingga x < 0 at au x > 1 D. Salah karena j ika x 2 > x dimungkinkan x < 0 at au x > 1 E. Salah karena unt uk x < 0 maka x 2 > x ∴ Pernyat aan yang benar adalah : j ika x < 0 maka x 2 > x 5. (Jawaban : A) (a3 − b3) = (a − b)(a2 +ab + b2) 3 ⎛1⎞ a 3 − a − 3 = a 3 − ⎜⎜ ⎟⎟ ⎝a ⎠ 2 ⎛ 1 ⎞⎛⎜ 2 1 ⎛ 1 ⎞ ⎞⎟ 3 −3 a − a = ⎜⎜ a − ⎟⎟ a + a ⋅ + ⎜⎜ ⎟⎟ a ⎠⎜ a ⎝a ⎠ ⎟ ⎝ ⎠ ⎝ ⎛ 1 ⎞⎛ 1 ⎞ ∴ a 3 − a − 3 = ⎜⎜ a − ⎟⎟⎜⎜ a 2 + 1 + 2 ⎟⎟ a ⎠⎝ a ⎠ ⎝ 6. (Jawaban : D) Kecepat an makan unt uk 1 ekor kambing, v k = 1 lap. bola/ 5 hari / 5 kambing. Vk = 1/ 5 lap bola/ hari/ kambing Banyaknya rumput yang dimakan, nr dirumuskan dengan : SMA Negeri 5 Bengkulu Eddy Hermanto, ST 6 Olimpiade Matematika Tk Kabupaten/ Kota 2002 Nr = vk ⋅ nhari ⋅ nkambing 3 = 1/ 5 ⋅ nhari ⋅ 3 ∴ nhari = 5 hari 7. (Jawaban : D) (x + y) $ (x − y) = (x + y) (x − y) − (x + y) + (x − y) ∴ (x + y) $ (x − y) = x 2 − y2 − 2y 8. (Jawaban : ?) Karena b > 0 maka Karena 1 1 1 + = a b 6 1 1 sehingga a > 6 < a 6 a +b 1 maka = ab 6 ⋅⋅⋅⋅⋅⋅⋅⋅ (1) 6(b − 6 ) + 36 b −6 36 ⋅⋅⋅⋅⋅⋅⋅⋅⋅⋅⋅⋅⋅⋅⋅⋅⋅⋅⋅⋅ (2) a =6+ b −6 Karena a > 6 maka (b − 6) > 0 ⋅⋅⋅⋅⋅⋅⋅⋅⋅⋅⋅⋅ (3) Karena a bilangan bulat maka (b −6) adalah f akt or dari 36 dan karena (b − 6) > 0 maka nilai (b − 6) yang memenuhi adalah 1; 2; 3; 4; 6; 9; 12; 18 at au 36. Unt uk b−6=1 b−6=2 b−6=3 b−6=4 b−6=6 b=7 b=8 b=9 b = 10 b = 12 a = 42 a = 24 a = 18 a = 15 a = 12 b−6=9 b − 6 = 12 b − 6 = 18 b − 6 = 36 b = 15 b = 18 b = 24 b = 42 a = 10 a=9 a=8 a=7 Pasangan bilangan bulat (a, b) yang memenuhi adalah : { (7, 42) ; (8, 24) ; (9, 18) ; (10, 15) ; (12, 12) ; (15, 10) ; (18, 9) ; (24, 8) ; (42, 7) } ∴ Maka banyaknya pasangan (a, b) yang memenuhi adalah 9 a = 9. (Jawaban : C) Karena 6x = x 2 + a maka x 2 −6x + a = 0 Disk = 62 − 4(1)(a) = 36 − 4a Syarat agar y = 6x memot ong parabola y = x 2 + a di sat u t it ik adalah Disk = 0 36 − 4a = 0 ∴ a=9 10. (Jawaban : B) Misal bilangan selanj ut nya adalah ABCD, maka A = 2 karena 1 + 9 + 9 + 9 ≠ 27. B + C + D = 25 Karena diinginkan B sekecil-kecilnya, maka (C + D) harus sebesar-besarnya dan karena B ≤ 9; C ≤ 9 dan D ≤ 9 maka (C + D) maks = 18 sehingga Bmin = 25 − 18 = 7. Maka t ahun berikut nya yang digit nya berj umlah 27 adalah 2799 ∴ Maka t ahun berikut nya yang digit nya berj umlah 27 t erj adi di ant ara t ahun 2701 dan 2900 SMA Negeri 5 Bengkulu Eddy Hermanto, ST 7 Olimpiade Matematika Tk Kabupaten/ Kota 2002 BAGIAN KEDUA 11. ∠C = 3∠A dan ∠B = 2∠A Karena ∠A + ∠B + ∠C = 180o maka ∠A + 2∠A + 3∠A = 180o sehingga ∠A = 30o ∠C = 3∠A = 90o AB BC = sin ∠C sin ∠A AB sin 90° ∴ = =2 sin 30° BC 12. Misal kecepat an Bando mengecat vo = 1 pagar / 3 j am = 1/ 3 pagar/ j am Kecepat an Bandi mengecat v i = 1 pagar / 4 j am = 1/ 4 pagar/ j am t 1 adalah lamanya wakt u Bando dan Bandi mengecat bersama (dalam j am) Maka banyaknya pagar yang dicat oleh mereka np1 adalah : np1 = vo⋅t 1 + v1⋅t 1 n p1 = 1 1 7 t1 + t1 = t1 3 4 12 t 2 adalah lamanya wakt u Bando mengecat pagar sendirian set elah pert engkaran (dalam j am) np2 = vo⋅t 2 np2 = 1 t2 3 Karena t t ot al adalah wakt u dari 12. 00 sampai 14. 25 maka t t ot al = Lama pert engkaran 10 menit at au 29 j am 12 1 j am 6 t t ot al = t 1 + lama pert engkaran + t 2 29 1 = t1 + +t 2 12 6 9 9 t 1 + t 2 = . Maka t 2 = − t 1 4 4 7 1 t1 + t 2 12 3 ⎞ ⎛ 7 1 9 1 = t 1 + ⎜⎜ − t 1 ⎟⎟ 12 3⎝4 ⎠ 12 = 7t 1 + 9 − 4t 1 sehingga t 1 = 1 j am n p1 + n p 2 = 1 = Maka pert engkaran dimulai 1 j am set elah pukul 12. 00 ∴ Pert engkaran dimulai pukul 13. 00 13. N = 22002 ⋅ 52003 = 5 ⋅ (2⋅5) 2002 = 5 ⋅ 102002 N = 500000⋅⋅⋅⋅⋅ ( Sebuah bilangan yang t erdiri dari 2003 digit dengan digit pert ama 5 diikut i digit 0 sebanyak 2002 kali) ∴ Jumlah digit N = 5 + 0 + 0 + 0 + ⋅⋅⋅ = 5 SMA Negeri 5 Bengkulu Eddy Hermanto, ST 8 Olimpiade Matematika Tk Kabupaten/ Kota 2002 14. Misal P = x 8 + y8 ; maka P < 104 Karena x 8 > 0 dan y 8 > 0 maka x 8 < 104 dan y8 < 104 2 x < 10 dan y2 < 10 Maka x = 1; 2; at au 3 dan y = 1; 2; at au 3 Unt uk x = 1 dan y = 1 maka P = 18 + 18 = 2 < 10000 (memenuhi) Unt uk x = 1 dan y = 2 at au x = 2 dan y = 1 maka P = 18 + 28 = 257 < 10000 (memenuhi) Unt uk x = 1 dan y = 3 at au x = 3 dan y = 1 maka P = 18 + 38 = 6562 < 10000 (memenuhi) Unt uk x = 2 dan y = 2 maka P = 28 + 28 = 512 < 10000 (memenuhi) Unt uk x = 2 dan y = 3 at au x = 3 dan y = 2 maka P = 28 + 38 = 6817 < 10000 (memenuhi) Unt uk x = 3 dan y = 3 maka P = 38 + 38 = 13122 > 10000 (t idak memenuhi) Maka nilai P yang memenuhi adalah 2; 257; 6562; 512; 6817 ∴ Banyaknya nilai yang berbent uk x 8 + y8 dengan x, y bilangan bulat adalah 5 15. Misal a − b = 8. Kemungkinan 2 nilai yang berselisih 8 adalah : 20 − 12 18 − 10 16 − 8 14 − 6 12 − 4 10 − 2 19 − 11 17 − 9 15 − 7 13 − 5 11 − 3 9−1 Bilangan 9; 10; 11; 12 berperan 2 baik sebagai a maupun b. Jika kedelapan bilangan berikut : a. 9 c. 11 e. 5 at au 13 g. 7 at au 15 b. 10 d. 12 f . 6 at au 14 h. 8 at au 16 t idak t ermasuk dalam nunsur , maka t idak akan ada 2 unsur dari nunsur yang berselisih 8. Maka unt uk n = 20 − 8, masih dimungkinkan t idak ada 2 unsur dari n unsur yang berselisih 8. ∴ nminimal = 13 16. Dibuat garis EF t egak lurus AB maupun CD sert a melalui t it ik P. Karena ∠CPD = ∠APB dan AB sej aj ar dengan CD, maka ∆APB sebangun dengan ∆CPD. EP CD 12 = = =3 PF AB 4 1 PF = ⋅ EP ⋅⋅⋅⋅⋅⋅⋅⋅⋅⋅ (1) 3 EP + PF = 4 EP + 1 ⋅ EP = 4 3 ∴ EP = 3 sat uan SMA Negeri 5 Bengkulu Eddy Hermanto, ST 9 Olimpiade Matematika Tk Kabupaten/ Kota 2002 a + 10 a a + 10b 17. Karena + = 2 maka a + b =2 b b + 10a a b 1 + 10 Misal b a x + 10 = x , maka = 2−x b 1 + 10x x + 10 = 2 − 10x 2 + 19x (5x − 4) (x − 1) = 0 x = 1 at au x = 4 5 ∴ Karena a ≠ b, maka x ≠ 1 maka a 4 = b 5 18. 1 < p < 100 Dari pernyat aan selanj ut nya, maka : p = 1 + 5x dengan x adalah bilangan bulat . Karena 1 < 1 + 5x < 100 maka 0 < 5x < 99 0 < x < 20 ⋅⋅⋅⋅⋅⋅⋅⋅⋅⋅⋅⋅⋅ (1) p = 6y − 1 dengan y adalah bilangan bulat . Karena 1 < 6y − 1 < 100 maka 2 < 6y < 101 0 < y < 17 ⋅⋅⋅⋅⋅⋅⋅⋅⋅⋅⋅⋅⋅⋅ (2) 1 + 5x = 6y − 1 5x = 2(3y − 1) ⋅⋅⋅⋅⋅⋅⋅⋅⋅⋅⋅⋅⋅ (3) 3y − 1 = 5t dan x = 2t dengan t adalah bilangan bulat t = 3y − 1 ⋅⋅⋅⋅⋅⋅⋅⋅⋅⋅⋅⋅⋅⋅⋅ (4) 5 Karena t adalah bilangan bulat , maka 5 membagi (3y − 1) sehingga (3y − 1) adalah bilangan dengan angka sat uan 0 at au 5. Maka y harus suat u bilangan dengan angka sat uan 2 at au 7. Karena 0 < y < 17, maka y = 2 at au 7 at au 12. Jika y = 2 maka p = 6(2) − 1 = 11 (bilangan pima) Jika y = 7 maka p = 6(7) − 1 = 41 (bilangan pima) Jika y = 12 maka p = 6(12) − 1 = 71 (bilangan pima) ∴ Maka j umlah seluruh bilangan prima = 11 + 41 + 71 = 123 19. a − b = 12 ⎛ 2 2 12 +⎜ − 1 ⎜⎝ 3 3 ⎞ ⎛ 32 2 2 ⎟+⎜ − ⎟ ⎜ 5 5 ⎠ ⎝ ⎞ ⎛ 4 2 32 ⎟+⎜ ⎟ ⎜ 7 − 7 ⎠ ⎝ ⎛ 10012 1000 2 ⎞ ⎟ +L+ ⎜ ⎜ 2001 − 2001 ⎟ ⎝ ⎠ Mengingat (x 2 − y2) = (x + y) (x − y), maka persamaan di at as menj adi : a − b = 1 + (1) + (1) + (1) + L + (1) − a − b = 1001 ⋅ 1 − ⎞ 10012 ⎟− ⎟ 2003 ⎠ 10012 2003 10012 2003 SMA Negeri 5 Bengkulu Eddy Hermanto, ST 10 Olimpiade Matematika Tk Kabupaten/ Kota 2002 1001 ⋅ (2003 − 1001) 2003 1001 ⋅ 1002 a −b = 2003 1002 dengan mengingat 2003 ≈ 2 ⋅ 1001 a −b ≈ 2 a −b = ∴ a − b ≈ 501 20. Dari soal diket ahui bahwa DE = 8 dan EF = 2√2 OA = OB = 2 OC = 1 1 ⋅ EF = ⋅ 2 2 = 2 2 cos α = 2 2 OC . Maka α = 45o = 2 OA ∠AOB = 90o 90° ⋅ πr 360 ( ) 1 ⋅ π 22 = π 4 1 1 Luas ∆OAB = ⋅ OA ⋅ OB ⋅ sin ∠AOB = ⋅ 2 ⋅ 2 ⋅ sin 90° = 2 2 2 Luas j uring OAB = 2 = Luas t embereng AB = Luas j uring OAB − Luas ∆OAB = π − 2 Luas arsir = Luas lingkaran − 2 ⋅ Luas t embereng AB Luas arsir = π (r) 2 − 2 ⋅ ( π − 2) Luas arsir = 4π − 2π + 4 ∴ Luas arsir = 2 π + 4 SMA Negeri 5 Bengkulu Eddy Hermanto, ST 11 SELEKSI OLIMPIADE MATEMATIKA INDONESIA 2003 TINGKAT PROVINSI Bidang Mat emat ika Bagian Per t ama Waktu : 90 Menit DEPARTEMEN PENDIDIKAN NASIONAL DIREKTORAT JENDERAL PENDIDIKAN DASAR DAN MENENGAH DIREKTORAT PENDIDIKAN MENENGAH UMUM TAHUN 2002 12 OLIMPIADE MATEMATIKA TINGKAT PROVINSI TAHUN 2002 BAGIAN PERTAMA 1. Misalkan A = ( −1) −1, B = ( −1) 1 dan C = 1−1. Berapakah A + B + C ? 2. Jika y = x −1 , t uliskan x sebagai f ungsi dari y. 2x + 3 3. Misalkan S = (x − 2) 4 + 8(x − 2) 3 + 24(x − 2) 2 + 32(x − 2) + 16. Apakah S j ika dit uliskan dalam sesedikit mungkin suku penj umlahan ? 4. Bilangan real 2, 525252⋅⋅⋅ adalah bilangan rasional, sehingga dapat dit ulis dalam bent uk m , n dimana m, n bilangan-bilangan bulat , n ≠ 0. Jika dipilih m dan n yang relat if prima, berapakah m+n? 5. Misalkan M dan m bert urut -t urut menyat akan bilangan t erbesar dan bilangan t erkecil di ant ara semua bilangan 4-angka yang j umlah keempat angkanya adalah 9. Berapakah f akt or prima t erbesar dari M − m ? 6. Tinj au persamaan yang berbent uk x 2 + bx + c = 0. Berapa banyakkah persamaan demikian yang memiliki akar-akar real j ika koef isien b dan c hanya boleh dipilih dari himpunan {1, 2, 3, 4, 5, 6} ? 7. Diket ahui t iga bilangan k, m dan n. Pernyat aan “ Jika k ≥ m, maka k > n” adalah t idak benar. Apakah pernyat aan yang benar dalam hal ini ? 8. Sebuah saluran air seharusnya dibuat dengan menggunakan pipa berdiamet er 10 cm. Akan t et api yang t ersedia hanyalah pip-pipa kecil yang berdiamet er 3 cm. Supaya kapasit as saluran t idak lebih kecil daripada yang diinginkan, berapakah banyaknya pipa 3 cm yang perlu dipakai sebagai penggant i sat u pipa 10 cm ? 9. Sebuah segit iga samasisi, sebuah lingkaran dan sebuah persegi memiliki keliling yang sama. Di ant ara ket iga bangun t ersebut , manakah yang memiliki luas t erbesar ? 10. Segit iga ABC memiliki panj ang sisi AB = 10, BC = 7, dan CA = 12. Jika set iap sisi diperpanj ang menj adi t iga kali panj ang semula, maka segit iga yang t erbent uk memiliki luas berapa kali luas ∆ABC ? 11. Sebanyak n orang pengurus sebuah organisasi akan dibagi ke dalam empat komisi mengikut i ket ent uan berikut : (i) set iap anggot a t ergabung ke dalam t epat dua komisi, dan (ii) set iap dua komisi memiliki t epat sat u anggot a bersama. Berapakah n ? 13 12. Didef inisikan a∗b = a + b + ab unt uk semua bilangan real a, b. Jika S = {a bilangan real a∗( −a) > a} t uliskan S sebagai sebuah selang (int erval). 13. Garis t engah sebuah set engah lingkaran berimpit dengan alas AB dari ∆ABC. Tit ik sudut C bergerak sedemikian rupa, sehingga t it ik t engah sisi AC selalu t erlet ak pada set engah lingkaran. Berupa apakah lengkungan t empat kedudukan t it ik C ? 14. Berapakah bilangan bulat posit if t erbesar yang membagi semua bilangan 15 − 1, 25 − 2, ⋅⋅⋅, n5 − n ? 15. Jika 2002 = a1 + a2 ⋅ 2! + a3 ⋅ 3! + ⋅⋅⋅ + an ⋅ n! , dimana ak adalah bilangan bulat , 0 ≤ ak ≤ k, k = 1, 2, ⋅⋅⋅, n, dan an ≠ 0, t ent ukan pasangan t erurut (n, an). 16. Berapakah sisa pembagian 43 4343 oleh 100 ? 17. Empat pasang suami-ist eri membeli karcis unt uk 8 kursi sebaris pada suat u pert unj ukan. Dua orang akan duduk bersebelahan hanya kalau keduanya pasangan suami ist eri at au berj enis kelamin sama. Berapa banyakkah cara menempat kan keempat pasang suami-ist eri ke 8 kursi t ersebut ? 18. Ada berapa banyakkah bilangan 4-angka berbent uk abcd dengan a ≤ b ≤ c ≤ d ? 19. Kit a gambarkan segibanyak berat uran (reguler) R dengan 2002 t it ik sudut besert a semua diagonalnya. Berapakah banyaknya segit iga yang t erbent uk yang semua t it ik sudut nya adalah t it ik sudut R, t et api t idak ada sisinya yang merupakan sisi R ? 20. Suat u lomba marat on diikut i oleh empat SMU : Merak, Merpat i, Pipit dan Walet . Set iap SMU mengirimkan lima pelari. Pelari yang masuk f inish ke-1, 2, 3, 4, 5, 6 memperoleh nilai bert urut t urut 7, 5, 4, 3, 2, 1. Nilai set iap SMU adalah j umlah nilai kelima pelarinya. SMU dengan nilai t erbesar adalah j uara lomba. Di akhir lomba t ernyat a SMU Pipit menj adi j uara dan t idak ada dua pelari yang masuk f inish bersamaan. Ada berapa banyakkah kemungkinan nilai SMU pemenang ? 14 SELEKSI OLIMPIADE MATEMATIKA INDONESIA 2003 TINGKAT PROVINSI Bidang Mat emat ika Bagian Kedua Waktu : 120 Menit DEPARTEMEN PENDIDIKAN NASIONAL DIREKTORAT JENDERAL PENDIDIKAN DASAR DAN MENENGAH DIREKTORAT PENDIDIKAN MENENGAH UMUM TAHUN 2002 15 OLIMPIADE MATEMATIKA TINGKAT PROVINSI TAHUN 2002 BAGIAN KEDUA 1. Lima buah bilangan asli berbeda, k, l, m, n dan p, akan dipilih. Kelima inf ormasi berikut t ernyat a cukup unt uk mengurut kan kelima bilangan t ersebut : (a) diant ara set iap dua bilangan, salah sat u bilangan mest i membagi bilangan yang lainnya, (b) m adalah bilangan yang t erbesar at au yang t erkecil, (c) p t idak boleh membagi sekaligus m dan k, (d) n ≤ l − p, dan (e) k membagi n at au p membagi n, t et api t idak sekaligus kedaunya. Tent ukan urut an yang mungkin bagi k, l, m, n dan p 2. Tent ukan semua bilangan bulat posit if p sehingga 3p + 25 j uga bulat posit if . 2p − 5 3. Diberikan sebuah bilangan 6-angka. Bukt ikan bahwa keenam angka bilangan t ersebut dapat disusun ulang sedemikian rupa, sehinggga j umlah t iga angka pert ama dan j umlah t iga angka t erakhir berselisih t idak lebih dari 9. 4. Diberikan segit iga sama sisi ABC dan sebuah t it ik P sehingga j arak P ke A dan ke C t idak lebih j auh dari j arak P ke B. Bukt ikan bahwa PB = PA + PC j ika dan hanya j ika P t erlet ak pada lingkaran luar ∆ABC. 5. Bangun dat ar pada gambar disebut t et romino-T. Misalkan set iap pet ak t et romino menut upi t epat sat u pet ak pada papan cat ur. Kit a ingin menut up papan cat ur dengan t et romino-t et romino sehingga set iap pet ak t et romino menut up sat u pet ak cat ur t anpa t umpang t indih. (a) Tunj ukkan bahwa kit a dapat menut up papan cat ur biasa, yait u tetromino-T papan cat ur dengan 8 X 8 pet ak, dengan menggunakan 16 t et romino-T. (b) Tunj ukkan bahwa kit a t idak dapat menut up papan ‘ cat ur’ 10 X 10 pet ak dengan 25 t et romino-T. 16 SELEKSI OLIMPIADE MATEMATIKA INDONESIA 2003 TINGKAT PROVINSI TAHUN 2002 Prestasi itu diraih bukan didapat !!! SOLUSI SOAL Bidang Mat emat ika Bagian Pertama Disusun oleh : Eddy Hermant o, ST 17 Olimpiade Matematika Tk Provinsi 2002 Solusi Bagian Pert ama BAGIAN PERTAMA 1. A + B +C = 1 (− 1) 1 + (− 1) + ∴ A + B + C = −1 2. y = 1 1 (1) 1 = −1 − 1 + 1 x −1 2x + 3 2yx + 3y = x − 1 x − 2yx = 3y + 1 x (1 − 2y) = 3y + 1 ∴ x = 3y + 1 1 − 2y 3. (a + b) 4 = a0b 4 + 4a1b 3 + 6a2b 2 + 4a3b 1 + a4b 0 S = 20 ⋅ (x − 2) 4 + 4 ⋅ 21 ⋅ (x −2) 3 + 6 ⋅ 22 ⋅ (x − 2) 2 + 4 ⋅ 23 ⋅ (x − 2) 1 + 24 ⋅ (x − 2) 0 Mengingat t eori di at as, maka : S = ( 2 + (x −2) ) 4 ∴ S = x4 4. Misal X = 2, 525252⋅⋅⋅ maka 100X = 252, 525252⋅⋅⋅ 100X − X = 252, 525252⋅⋅⋅ − 2, 525252⋅⋅⋅ 99X = 250 X = 250 99 Karena 250 dan 99 relat if prima, maka m = 250 dan n = 99 ∴ m + n = 250 + 99 = 349 5. Misal bilangan it u adalah : abcd Agar abcd sebesar-besarnya maka a harus sebesar-besarnya. Maka a = 9. Karena a = 9, agar a + b + c + d = 9, maka b = 0 ; c = 0; d = 0. Maka M = 9000 Agar abcd sekecil-kecilnya maka a harus sekecil-kecilnya dan karena a ≠ 0, maka a = 1. b j uga harus sekecil-kecilnya, maka b = 0. c j uga harus sekecil-kecilnya, maka c = 0. Karena a + b + c + d = 9, maka d = 8. Akibat nya m = 1008 M − m = 9000 − 1008 = 7992 = 8 ⋅ 999 = 8 ⋅ 27 ⋅ 37 M − m = 23 ⋅ 33 ⋅ 37 ∴ Maka f akt or prima t ebesar dari M − m adalah 37 SMA Negeri 5 Bengkulu Eddy Hermanto, ST 18 Solusi Olimpiade Matematika Tk Provinsi 2002 Bagian Pert ama 6. Agar akar-akar persamaan t ersebut real maka Diskriminan = b 2 − 4⋅ (1) ⋅c ≥ 0. Maka 4c ≤ b 2 Karena 1 ≤ c ≤ 6, maka 4 ≤ 4c ≤ 24 Unt uk b = 1 maka 4c ≤ 1. Akibat nya t idak ada nilai c yang memenuhi Unt uk b = 2 maka 4c ≤ 4. Akibat nya nilai c yang memenuhi ada sat u, yait u c = 1 Unt uk b = 3 maka 4c ≤ 9. Akibat nya nilai c yang memenuhi ada dua, yait u c = 1 ; 2 Unt uk b = 4 maka 4c ≤ 16. Akibat nya nilai c yang memenuhi ada empat , yait u c = 1 ; 2; 3; 4 Unt uk b = 5 maka 4c ≤ 25. Akibat nya nilai c yang memenuhi ada enam, yait u c = 1 ; 2; 3; 4; 5; 6 Unt uk b = 6 maka 4c ≤ 36. Akibat nya nilai c yang memenuhi ada enam, yait u c = 1 ; 2; 3; 4; 5; 6 ∴ Maka banyaknya pasangan yang memenuhi ada : 0 + 1 + 2 + 4 + 6 + 6 = 19 7. p Æ q ≡ ∼q Æ ∼p ≡ ∼p ∨ q ∼ (p Æ q) ≡ ∼ ( ∼p ∨ q) ≡ p ∧ ∼q p: k≥m q: k>n Karena q : k > n, maka ingkaran dari q adalah ∼q ≡ k ≤ n ∴ Pernyat aan yang benar adalah : k ≥ m dan k ≤ n. Penulisan lain adalah m ≤ k ≤ n. 8. Kapasit as pipa t ergant ung dari luas penampangnya. Lpakai ≥ Lseharusnya n ⋅ ¼ ⋅ π (3) 2 ≥ ¼ ⋅ π ⋅ (10) 2 9n ≥ 100 n ≥ 11, 111⋅⋅⋅ ∴ nmin = 12 9. Misal masing-masing keliling bangun = K Unt uk segit iga j elas 3s = K. Karena s = K/ 3 maka Luas = ½ s2 sin 60o = 1 3K 36 2 K2 K maka Luas = π R2 = 4π 2π 2 K K maka Luas = s2 = Unt uk persegi, 4s = K. Karena s = 16 4 Unt uk lingkaran, 2πR = K. Karena R = Karena π = 3, 142⋅⋅⋅ < 4 dan √3 < 2 , maka 1 1 1 2 3 > > = > 4π 16 18 36 36 ∴ Karena 1 1 3 , maka bangun yang memiliki luas t erbesar adalah : lingkaran > > 4π 16 36 10. Luas segit iga semula = ½ ab sin C Luas segit iga akhir = ½ (3a)(3b)sin C = 9 ⋅ ½ ab sin C Luas segit iga akhir = 9 ⋅ Luas segit iga semula ∴ Perbandingan luas segit iga akhir dengan luas segit iga semula adalah = 9 SMA Negeri 5 Bengkulu Eddy Hermanto, ST 19 Solusi Olimpiade Matematika Tk Provinsi 2002 Bagian Pert ama 11. (a) set iap anggot a t ergabung ke dalam t epat dua komisi (b) set iap dua komisi memiliki t epat sat u anggot a bersama Karena ada 4 komisi maka banyaknya pasangan komisi yang bisa dibuat adalah 4C2 = 6. Karena banyaknya pasangan komisi ada 6 maka banyaknya anggot a minimal adalah 6 sebab j ika kurang dari 6 maka akan ada seorang anggot a yang t ergabung dalam lebih dari 2 komisi. Jika t erdapat lebih dari 6 anggot a maka akan ada seorang anggot a yang masuk dalam sebuah komisi t et api t idak masuk ke dalam t iga komisi lain. Hal ini bert ent angan dengan (a) bahwa seorang anggot a t ergabung ke dalam t epat dua komisi. Akibat nya banyaknya anggot a ada 6 orang. Cont oh pembagian keenam anggot a ke dalam empat komisi yang memenuhi (a) dan (b) adalah : Misalkan komisi t ersebut adalah A, B, C, D dengan ai menyat akan anggot a ke-i dengan 1 ≤ i ≤ 6. Komisi A Komisi B Komisi C Komisi D a1 a1 a2 a3 a2 a4 a4 a5 a3 a5 a6 a6 ∴ Jadi, banyaknya pengurus agar memenuhi syarat t ersebut adalah 6 12. a ∗(-a) = a + ( −a) + a ⋅ ( −a) = − a2 S = { a bilangan real | − a2 > a } = { a bilangan real | a (a + 1) < 0 } ∴ S = { a bilangan real | −1 < a < 0 } 13. AB adalah diamet er dan D t erlet ak pada lingkaran. Maka ∠ADB = 90o Karena AD = CD dan BD ⊥ AC maka ∆ABC adalah segit iga sama kaki dengan AB = BC. Karena BC = AB = diamet er lingkaran yang berart i bernilai t et ap dan B adalah t it ik yang t et ap maka lengkung yang t erj adi adalah berupa set engah lingkaran dengan pusat t it ik B. ∴ Lengkung yang t erj adi adalah berupa setengah lingkaran 14. 15 − 1 = 0 ; 25 − 2 = 30. Unt uk n > 2 maka n 5 − n > 30. Semua bilangan membagi 0. Karena salah sat u bilangan t ersebut adalah 30 maka nilai maksimum bilangan yang membagi 15 − 1, 25 − 2, ⋅⋅⋅ , n5 − n adalah 30. Akan dibukt ikan bahwa 30 membagi n5 − n unt uk set iap n bilangan asli. Alt ernat if 1 : Misal : N = n 5 − n = n (n4 − 1) = n (n 2 − 1) (n2 + 1) = (n − 1) n (n + 1) (n 2 + 1) Karena (n − 1) , n dan (n + 1) adalah t iga bilangan berurut an maka N past i habis dibagi 3! = 6. • Unt uk n = 5k Karena n adalah f akt or dari N dan n habis dibagi 5 maka N past i habis dibagi 5 • Unt uk n = 5k + 1 n − 1 = 5k Karena (n − 1) adalah f akt or dari N dan (n − 1) habis dibagi 5 maka N past i habis dibagi 5 • Unt uk n = 5k + 2 SMA Negeri 5 Bengkulu Eddy Hermanto, ST 20 Solusi Olimpiade Matematika Tk Provinsi 2002 Bagian Pert ama n2 + 1 = (5k + 2) 2 + 1 = 25k 2 + 20k + 5 = 5 (5k 2 + 4k + 1) Karena (n2 + 1) adalah f akt or dari N dan (n 2 + 1) habis dibagi 5 maka N past i habis dibagi 5 • Unt uk n = 5k + 3 n2 + 1 = (5k + 3) 2 + 1 = 25k 2 + 30k + 10 = 5 (5k 2 + 6k + 2) Karena (n2 + 1) adalah f akt or dari N dan (n 2 + 1) habis dibagi 5 maka N past i habis dibagi 5 • Unt uk n = 5k + 4 n + 1 = 5k + 5 = 5 (k + 1) Karena (n + 1) adalah f akt or dari N dan (n + 1) habis dibagi 5 maka N past i habis dibagi 5 Karena unt uk n = 5k ; n = 5k + 1 ; n = 5k + 2 ; n = 5k + 3 dan n = 5k + 4 semuanya menghasilkan N habis dibagi 5 maka N past i habis dibagi 5 unt uk n bilangan bulat posit if . Karena N habis dibagi 6 dan 5 sert a 6 dan 5 relat if prima maka N past i habis dibagi 6⋅5 = 30 Alt ernat if 2 : n5 − n = (n − 1) n (n + 1) (n2 + 1) = (n − 1) n (n + 1) (n 2 − 4 + 5) n5 − n = (n − 1) n (n + 1) (n2 − 4) + 5 (n − 1) n (n + 1) n5 − n = (n − 2) (n − 1) n (n + 1) (n + 2) + 5 (n − 1) n (n + 1) Karena (n − 2), (n − 1) , n, (n + 1) dan (n + 2) adalah lima bilangan bulat berurut an maka perkalian (n − 2) (n − 1) n (n + 1) (n + 2) habis dibagi 5! = 120 at au j uga habis dibagi 30 sebab 30 membagi 120. Karena (n − 1) , n dan (n + 1) adalah 3 bilangan berurut an maka (n − 1) n (n + 1) past i habis dibagi 3! = 6. Maka 5 (n − 1) n (n + 1) habis dibagi 5 ⋅ 6 = 30. ∴ Bilangan nilai maksimum bilangan yang membagi 15 − 1, 25 − 2, ⋅⋅⋅ , n5 − n adalah 30. 15. Misal T = a1 + a2⋅2! + a3⋅3! + ⋅⋅⋅ + an⋅n! Karena 7! = 5040 dan 6! = 720 maka nmaksimum = 6. Jika n = 5 maka Tmaks = 1 + 2⋅2! + 3⋅3! + 4⋅4! + 5⋅5! = 1 + 4 + 18 + 96 + 600 = 719 < 2002 T = 2002 hanya j ika n = 6 Karena unt uk n = 5 makaTmaks = 719 maka 2002 − 719 = 1283 ≤ a6⋅6! ≤ 2002 yang dipenuhi hanya j ika a6 = 2 Maka a1 + a2⋅2! + a3⋅3! + a4⋅4! + a5⋅5! = 2002 − 2⋅6! = 562 Jika n = 4 maka Tmaks = 1 + 2⋅2! + 3⋅3! + 4⋅4! = 119 562 − 119 = 443 ≤ a5⋅5! ≤ 562 yang dipenuhi hanya j ika a5 = 4 Maka a1 + a2⋅2! + a3⋅3! + a4⋅4! = 562 − 4⋅5! = 562 − 480 = 82 Jika n = 3 maka Tmaks = 1 + 2⋅2! + 3⋅3! = 23 82 − 23 = 59 ≤ a4⋅4! ≤ 82 yang dipenuhi hanya j ika a4 = 3 Maka a1 + a2⋅2! + a3⋅3! = 82 − 3⋅4! = 82 − 72 = 10 Jika n = 2 maka Tmaks = 1 + 2⋅2! = 9 10 − 9 = 1 ≤ a3⋅3! ≤ 10 yang dipenuhi hanya j ika a3 = 1 Maka a1 + a2⋅2! = 10 − 1⋅3! = 10 − 6 = 4 Jika n = 1 maka Tmaks = 1 = 1 4 − 1 = 3 ≤ a2⋅2! ≤ 4 yang dipenuhi hanya j ika a2 = 2 Maka a1 = 4 − 2⋅2! = 4 − 4 = 0 ∴ Pasangan t erurut (n, an) adalah { (1, 0) ; (2, 2) ; (3, 1) ; (4, 3) ; (5, 4) ; (6, 2) } SMA Negeri 5 Bengkulu Eddy Hermanto, ST 21 Olimpiade Matematika Tk Provinsi 2002 Solusi Bagian Pert ama 16. Alt ernat if 1 : Dua digit t erakhir dari 431 adalah 43 Dua digit t erakhir dari 432 adalah 49 Dua digit t erakhir dari 433 adalah 07 Dua digit t erakhir dari 434 adalah 01 Dua digit t erakhir dari 435 adalah 43 ⋅⋅⋅⋅⋅⋅ dst . Karena 43 = 4⋅10 + 3 maka 2 digit t erakhir dari 4343 sama dengan dua digit t erakhir dari 433 yait u 07. Sehingga 4343 = ⋅⋅⋅⋅⋅⋅07 = 100t + 7 = 4k + 7 dengan t dan k adalah bilangan bulat . ( ) 43 43 = 43 4 k + 7 = 43 4 k ⋅ 43 7 = 43 4 43 k ⋅ 43 7 Karena dua digit t erakhir dari 434 adalah 01 maka dua digit t erakhir dari (434) k adalah j uga 01. Dua digit t erakhir dari 437 sama dengan dua digit t erakhir dari 433 yait u 07. 4343 sama dengan dua digit t erakhir dari perkalian dua digit Maka dua digit t erakhir dari 43 t erakhir (434) k dengan dua digit t erakhir dari 437. 4343 Karena 01 x 07 = 07. Maka 2 digit t erakhir dari 43 adalah 07. Alt ernat if 2 : Karena 4343 = (4 ⋅ 11 − 1) 43 maka 4343 ≡ ( −1) 43 (mod 4) 4343 ≡ −1 (mod 4) ≡ 3 (mod 4) Berart i 4343 = 4k + 3 dengan k adalah bilangan asli. 43 43 43 43 43 43 43 43 43 43 43 43 43 43 43 = 434k+3 = (1849) 2k ⋅ 433 ≡ (49) 2k ⋅ 4343 (mod 100) ≡ (2401) k ⋅ 7 (mod 100) sebab 4343 ≡ 7 (mod 100) ≡ 1k ⋅ 7 (mod 100) ≡ 7 (mod 100) Karena 43 ∴ 43 4343 4343 ≡ 7 (mod 100) berart i 43 43 43 = 100p + 7 dengan p adalah bilangan asli. j ika dibagi 100 akan bersisa 7 17. Misal S = suami dan I = ist eri Kemungkinan susunannya adalah : a. SIISSIIS at au ISSIISSI Karena yang berdekat an haruslah pasangan suami ist eri maka kasus ini seolah-olah menempat kan 4 pasangan suami ist eri dalam 4 t empat . Banyaknya cara = 2 ⋅ 4P4 = 48. b. SIISSSII at au ISSIIISS Karena ada 3 pasang kursi yang harus diisi 3 pasang suami ist eri maka banyaknya cara menyusun = 2 ⋅ 4C3 ⋅ 3! = 48 c. SSIISSII at au IISSIISS Kasus ini sama dengan (a). Banyaknya cara adalah 48. d. SIIISSSI at au ISSSIIIS Karena ada 3 pasang kursi yang harus diisi 3 pasang suami ist eri maka banyaknya cara adalah 2 ⋅ 4 ⋅ 3! = 48 e. SSIIISSI at au IISSSIIS Kasus ini sama dengan (c). Banyaknya cara ada 48 cara. f . SIIIISSS at au ISSSSIII Ada 2 pasang kursi yang harus diisi oleh 2 pasang suami ist eri. Banyaknya cara = 4C2 ⋅ 2! . Empat kursi lain t erdiri dari 2 kursi diisi oleh 2 perempuan dan 2 kursi lainnya diisi 2 lelaki. SMA Negeri 5 Bengkulu Eddy Hermanto, ST 22 Olimpiade Matematika Tk Provinsi 2002 Solusi Bagian Pert ama Maka banyaknya cara = 2 ⋅ ( 4C2 ⋅ 2! ) ⋅ 2! ⋅ 2! = 96 g. SSIIIISS at au IISSSSII Soal ini mirip dengan bagian (f ). Banyaknya cara ada 96. h. SSSIIIIS at au IIISSSSI Soal ini j uga mirip dengan bagian (f ). Banyaknya cara ada 96. i. SSSSIIII at au IIIISSSS Pasangan yang di t engah dipilih dari 4 pasangan yang lain. Maka banyaknya cara = 2 ⋅ 4 ⋅ 3! ⋅ 3! = 288 Maka banyaknya cara = 48 + 48 + 48 + 48 + 48 + 96 + 96 + 96 + 288 = 816 cara ∴ Jadi, banyaknya cara menempat kan keempat pasang suami ist eri ke-8 kursi adalah 816. 18. a. Unt uk a = 1 • Unt uk a = 1 dan b = 1. Unt uk c = 1 maka nilai d ada 9 kemungkinan. Unt uk c = 2 ada 8 kemungkinan. ⋅⋅⋅⋅ dst . Maka unt uk a = 1 dan b = 1 ada 9 + 8 + 7 + 6 + 5 + 4 + 3 + 2 + 1 = 45 kemungkinan. • Unt uk a = 1 dan b = 2 Sama dengan unt uk a = 1 dan b = 1 dikurangi dengan unt uk c = 1. Maka unt uk a = a dan b = 2 ada 45 − 9 = 36 kemungkinan. • Unt uk a = 1 dan b = 3 Ada 36 − 8 = 28 kemungkinan M dst Unt uk a = 1 ada 45 + 36 + 28 + 21 + 15 + 10 + 6 + 3 + 1 b. Unt uk a = 2 Sama dengan unt uk a = 1 dikurangi unt uk b = 1 Unt uk a = 2 ada 36 + 28 + 21 + 15 + 10 + 6 + 3 + 1 M dst Misalkan banyaknya bilangan = N. N = 1 ⋅ 45 + 2 ⋅ 36 + 3 ⋅ 28 + 4 ⋅ 21 + 5 ⋅ 15 + 6 ⋅ 10 + 7 ⋅ 6 + 8 ⋅ 3 + 9 ⋅ 1 = 495 ∴ Banyaknya bilangan yang memenuhi a ≤ b ≤ c ≤ d adalah 495 19. SMA Negeri 5 Bengkulu Eddy Hermanto, ST 23 Solusi Olimpiade Matematika Tk Provinsi 2002 Bagian Pert ama Misal : A = Banyaknya segit iga seluruhnya yang dapat dibent uk t ermasuk yang sisinya merupakan sisi R. B = Banyaknya segit iga yang dapat dibent uk dengan hanya 1 sisinya yang merupakan sisi R. C = Banyaknya segit iga yang dapat dibent uk dengan 2 sisinya merupakan sisi R. • Banyaknya segit iga seluruhnya yang dapat dibent uk t ermasuk yang sisinya merupakan sisi R Segit iga dibent uk dari 3 t it ik yang t idak segaris, maka banyaknya segit iga yang dapat dibent uk adalah 2002C3 • = 2002 ⋅ 2001 ⋅ 2000 = 2002 ⋅ 667⋅ 1000 6 Banyaknya segit iga yang dapat dibent uk dengan hanya 1 sisinya yang merupakan sisi R. Unt uk membent uk segit iga ini maka 2 dari 3 t it iknya harus berurut an, namun ket iga t it iknya t idak berurut an. Misal kedua t it ik t ersebut adalah n dan n+1, maka t it ik ket iga t idak boleh n−1 at au n+2. Banyaknya 2 t it ik yang berurut an ada 2002 kemungkinan, yait u 1-2, 2-3, 3-4, 4-5, ⋅⋅⋅, 2001-2002, 2002-1. Misalkan t it ik yang kit a pilih adalah 2-3, maka t it ik ket iga t idak boleh t it ik 1 at au 4, maka banyaknya kemungkinan 1 t it ik ket iga adalah 1998 cara. Banyaknya segit iga yang dapat dibent uk adalah 1998 x 2002 • Banyaknya segit iga yang dapat dibent uk dengan 2 sisinya merupakan sisi R Unt uk membent uk segit iga ini maka ke-3 t it iknya harus berurut an. Banyaknya segit iga yang dapat dibent uk adalah 2002, yait u 1-2-3, 2-3-4, 3-4-5, ⋅⋅⋅, 2001-2002-1, 2002-1-2. Banyaknya segit iga dimaksud adalah = A − B − C = 2002 ⋅ 667 ⋅ 1000 − 1998 ⋅ 2002 − 2002 = 2002 (667 ⋅ 1000 − 1999) = 1331332002 ∴ Banyaknya segit iga yang semua t it ik sudut nya adalah t it ik sudut R, t et api t idak ada sisinya yang merupakan sisi R adalah 1. 331. 332. 002 20. Nilai t ot al = 7 + 5 + 4 + 3 + 2 + 1 = 22 Nilai maksimum yang dapat diperoleh SMU Pipit adalah 7 + 5 + 4 + 3 + 2 = 21 Misal nilai minimum SMU Pipit adalah x maka nilai sisa adalah 22 − x. Nilai minimum yang dapat diperoleh adalah j ika nilai sisa yang ada t erdist ribusi merat a kepada ket iga SMU yang lain. Misal nilai masing-masing ket iga SMU yang lain adalah k, maka : x + 3k = 22 dan x > k 3x > 22 − x. Maka x > 22/ 4. Jika x = 6 maka nilai sisa = 22 − 6 = 16. Ada 2 SMU mendapat nilai 5 dan sat u SMU mendapat nilai 6. Hal yang t idak boleh karena berart i t idak ada pemenang. Jika x = 7 maka nilai sisa = 22 − 7 = 15. Yang berart i ket iga SMU yang lain masing–masing mendapat nilai 5. Nilai 5 dapat diperoleh dari 5 ; 3 + 2 dan 4 + 1 yang berart i memenuhi syarat . Maka nilai maksimum SMU Pipit = 21 sedangkan nilai minimunmnya = 7. Semua nilai dari 7 sampai 21 semua dapat diperoleh dari kombinasi : 7, 5, 4, 3, 2, 1. Nilai dari 7 sampai dengan 21 ada 15. ∴ Banyaknya kemungkinan nilai SMU pemenang adalah 15 SMA Negeri 5 Bengkulu Eddy Hermanto, ST 24 SELEKSI OLIMPIADE MATEMATIKA INDONESIA 2003 TINGKAT PROVINSI TAHUN 2002 Prestasi itu diraih bukan didapat !!! SOLUSI SOAL Bidang Mat emat ika Bagian Kedua Disusun oleh : Eddy Her mant o, ST 25 Olimpiade Matematika Tk Provinsi 2002 Solusi Bagian Kedua BAGIAN KEDUA 1. • Jika m adalah bilangan yang t erbesar Berdasarkan (c) dan (a), maka p membagi m sedangkan k membagi p sehingga m > p > k Berdasarkan (d), l ≥ n + p, maka l > n dan l > p sehingga m > l > p > k Berdasarkan (e) : ¾ Jika k membagi n maka n membagi p sehingga p > n > k. Urut an yang mungkin adalah m>l>p>n>k ¾ Jika p membagi n maka n membagi k sehingga k > n > p. Karena p > k maka hal ini merupakan sebuah kont radiksi. • Jika m adalah bilangan t erkecil Berdasarkan (c) dan (a), maka m membagi p dan p membagi k sehingga k > p > m Berdasarkan (e) : ¾ Jika k membagi n maka n membagi p sehingga p > n > k. Karena k > p maka hal ini merupakan sebuah kont radiksi. ¾ Jika p membagi n maka n membagi k sehingga k > n > p. Akibat nya k > n > p > m. Berdasarkan (d) : ¾ Karena n = l − p maka l = n + p dan karena p < n maka n < l < 2n. Karena n harus membagi l maka hal t ersebut t idak mungkin. ¾ Karena n < l − p maka l > p+ n. Sehingga t idak dapat dit ent ukan yang lebih besar ant ara l dan k, maka urut an yang mungkin adalah : k > l > n > p > m at au l > k > n > p > m. ∴ Semua urut an yang mungkin bagi k, l, m, n dan p adalah : 1. m > l > p > n > k at au 2. k > l > n > p > m atau 3. l > k > n > p > m 2. Alt ernat if 1 : Misal m = p + 30 3p + 25 2 p − 5 + p + 30 ⋅⋅⋅⋅⋅⋅⋅⋅⋅⋅ (1) = =1+ 2p − 5 2p − 5 2p − 5 Ambil p + 30 = 2p − 5 maka p = 35. • Unt uk p > 35, maka p + 30 < 2p − 5 sehingga p + 30 < 1 sehingga t idak mungkin m bilangan 2p − 5 bulat . • Unt uk 0 < p < 35 Semakin besar nilai p, maka perbandingan p + 30 dan 2p − 5 akan semakin kecil sehingga nilai m semakin kecil mendekat i sat u. Karena m > 0 maka 2p − 5 > 0. Akibat nya p ≥ 3 Bent uk di at as dapat j uga diubah menj adi : 2pm − 5m = 3p + 25 p(2m −3) = 5m + 25 p = 5m + 25 ⋅⋅⋅⋅⋅⋅⋅⋅⋅⋅ (2) 2m − 3 dengan 2m − 3 > 0 at au m > 1. SMA Negeri 5 Bengkulu Eddy Hermanto, ST 26 Olimpiade Matematika Tk Provinsi 2002 Solusi Bagian Kedua Berdasarkan persamaan (1) Jika p = 3 maka m = 1 + 33/ 1 = 34 ( bilangan bulat ) Jika p = 4 maka m = 37/ 3 ( bukan bilangan bulat ) Jika p = 5 maka m = 40/ 5 = 8 ( bilangan bulat ) Jika p = 6 maka m = 43/ 7 ( bukan bilangan bulat ) Jika p = 7 maka m = 36/ 9 ( bukan bilangan bulat ) Jika p = 8 maka m = 49/ 11 ( bukan bilangan bulat ) Jika p = 9 maka m = 52/ 13 = 4 ( bilangan bulat ) Karena semakin besar nilai p maka nilai m semakin kecil, maka sesuai persamaan (1) dicoba : Jika m = 3 maka p = 40/ 3 ( bukan bilangan bulat ) Jika m = 2 maka p = 35 ( bilangan bulat ) Alt ernat if 2 : Karena m = 3 p + 25 maka 2mp − 5m = 3p + 25. 2p − 5 4mp − 10m = 6p + 50 (2m − 3)(2p − 5) = 50 + 15 (2m − 3)(2p − 5) = 65 2m − 3 dan 2p − 5 masing-masing adalah f akt or dari 65. Fakt or dari 65 adalah ±1, ±5, ±13, ±65. Jika 2p − 5 = −1 dan 2m − 3 = −65 maka p = 2 dan m = −31 (t idak memenuhi p dan m asli) Jika 2p − 5 = 1 dan 2m − 3 = 65 maka p = 3 dan m = 34 (memenuhi p dan m asli) Jika 2p − 5 = −5 dan 2m − 3 = −13 maka p = 0 dan m = −5 (t idak memenuhi p dan m asli) Jika 2p − 5 = 5 dan 2m − 3 = 13 maka p = 5 dan m = 8 (memenuhi p dan m asli) Jika 2p − 5 = −13 dan 2m − 3 = −5 maka p = −4 dan m = −1 (t idak memenuhi p dan m asli) Jika 2p − 5 = 13 dan 2m − 3 = 5 maka p = 9 dan m = 4 (memenuhi p dan m asli) Jika 2p − 5 = −65 dan 2m − 3 = −1 maka p = −30 dan m = 1 (t idak memenuhi p dan m asli) Jika 2p − 5 = 65 dan 2m − 3 = 1 maka p = 35 dan m = 2 (memenuhi p dan m asli) ∴ Bilangan bulat posit if p sehingga 3p + 25 j uga bulat posit if adalah 3 ; 5 ; 9 at au 35 2p − 5 3. Misal ke-6 angka it u A, B, C, D, E, F dengan A ≥ B ≥ C ≥ D ≥ E ≥ F dengan 0 ≤ A, B, C, D, E, F ≤ 9. Penyusunan bilangan yang benar sehingga didapat selisih t iga bilangan pert ama dengan t iga bilangan t erakhir seminimal mungkin adalah ACEBDF. Misal T = A + C + E − B − D − F T = (A − F) + (C − B) + (E − D) Jelas bahwa A − F ≤ 9. Tanda kesamaan akan t erpenuhi hanya apabila A = 9 dan F = 0. Karena C ≤ B dan E ≤ D maka C − B ≤ 0 dan E − D ≤ 0. Tanda kesamaan t erj adi hanya j ika C = B dan E = D. Maka T = (A − F) + (C − B) + (E − D) ≤ 9 + 0 + 0 = 9 ∴ Terbukti bahwa j umlah tiga angka pertama dan j umlah tiga angka terakhir suat u bilangan enam angka dapat disusun sedemikian rupa sehingga berselisih tidak lebih dari 9 SMA Negeri 5 Bengkulu Eddy Hermanto, ST 27 Solusi Olimpiade Matematika Tk Provinsi 2002 Bagian Kedua 4. Pembukt ian Teorema Pt olemy ABCP adalah segiempat t alibusur at au dengan kat a lain t it ik P t erlet ak pada lingkaran luar segit iga ABC dengan t it ik P t erlet ak pada busur AC. Misal ∠APB = α. Dibuat segit iga PCT dengan CT adalah perpanj angan BC dan ∠CPT = α. Karena ABCP adalah segi empat t ali busur maka ∠BAP + ∠BCP = 180o sehingga ∠BAP = ∠PCT. Karena ∠APB = ∠CPT dan ∠BAP = ∠PCT maka ∆BAP sebangun dengan ∆PCT. PA AB PB ⋅⋅⋅⋅⋅⋅⋅⋅ (1) = = PC CT PT AB CT = ⋅ PC ⋅⋅⋅⋅⋅⋅⋅⋅⋅⋅⋅⋅ (2) PA PA PC . Dari persamaan (1) j uga didapat : = PB CT PA PC maka ∆APC sebangun dengan ∆BPT Karena ∠APC = ∠BPT = α + ∠BPC dan = PB PT PA PC AC ⋅⋅⋅⋅⋅⋅⋅ (3) Akibat nya = = PB PT BT PB BT = ⋅ AC ⋅⋅⋅⋅⋅⋅⋅⋅⋅⋅⋅⋅(4) PA Akibat nya BT = BC + CT Subt it usikan pers. (2) dan (4) PB AB ⋅ AC = BC + ⋅ PC PA PA PB ⋅ AC = PA ⋅ BC + PC ⋅ AB ( Teorema Pt olemy ) Jika ∆ABC adalah segit iga sama sisi, maka AC = BC = AB, maka : PB = PA + PC (Terbukt i) at au Jika PB = PA + PC dan karena AB = BC = AC, maka PB ⋅ AC = PA ⋅ BC + PC ⋅ AB ∴ Sesuai dengan Teorema Pt olemy, maka ABCP adalah segi empat t ali busur at au dengan kat a lain titik P t erletak pada lingkaran luar segitiga ABC. SMA Negeri 5 Bengkulu Eddy Hermanto, ST 28 Solusi Olimpiade Matematika Tk Provinsi 2002 Bagian Kedua 5. a. Karena pet ak 4 x 4 dapat dit ut upi oleh 4 buah Tet romino-T, maka t ent unya kita dapat menutup petak catur 8 x 8 dengan 16 buah Tet romino-T . b. Andaikan 25 t et ronimo t ersebut dapat menut up papan ‘ cat ur’ 10 x 10 pet ak. Sebuah t et romino-T akan menut upi 1 buah pet ak hit am dan 3 buah pet ak put ih at au 1 buah pet ak put ih dan 3 buah pet ak hit am pada papan cat ur. Karena 1 dan 3 bilangan ganj il sert a banyaknya Tet romino-T ada 25 yang j uga merupakan bilangan ganj il maka ke-25 Tet romino-T t ersebut akan menut upi sej umlah ganj il pet ak hit am dan sej umlah ganj il pet ak put ih pada papan cat ur. Hal ini kont radiksi dengan kenyat aan bahwa pada papan cat ur 10 x 10 t erdapat 50 pet ak hit am dan 50 pet ak put ih. Terbukti bahwa kita tidak dapat menutup papan ‘ catur’ 10 X 10 pet ak dengan 25 t et romino-T . SMA Negeri 5 Bengkulu Eddy Hermanto, ST 29 SELEKSI TIM OLIMPIADE MATEMATIKA INDONESIA 2003 OLIMPIADE SAINS NASIONAL 2002 YOGYAKARTA, 10 SEPTEMBER 2002 Bidang Mat emat ika Waktu : 4 Jam DEPARTEMEN PENDIDIKAN NASIONAL DIREKTORAT JENDERAL PENDIDIKAN DASAR DAN MENENGAH DIREKTORAT PENDIDIKAN MENENGAH UMUM TAHUN 2002 30 OLIMPIADE SAINS NASIONAL 2002 YOGYAKARTA, 10 SEPTEMBER 2002 BI DANG : MATEMATI KA WAKTU : 4 JAM 1. Bukt ikan bahwa n 4 − n2 habis dibagi oleh 12 unt uk sebarang bilangan bulat n > 1 2. Lima buah dadu (enam-muka) akan dilempar sat u demi sat u, lalu hasil kelima angka yang muncul akan dihit ung. Manakah yang lebih besar peluang t erj adinya hasil kali 180 at au hasil kali 144 ? 3. Tent ukan semua solusi dari sist em persamaan x+y+z=6 x 2 + y2 + z2 = 12 x 3 + y3 + z3 = 24 4. Diberikan segit iga ABC dengan AC > BC. Pada lingkaran luar segit iga ABC t erlet ak t it ik D yang merupakan t it ik t engah busur AB yang memuat t it ik C. Misalkan E adalah t it ik pada AC sehingga DE t egak lurus pada AC. Bukt ikan bahwa AE = EC + CB 5. Sembilan dari sepuluh bilangan berikut : 4, 5, 6, 7, 8, 12, 13, 16, 18, 19 akan diisikan ke dalam pet akkosong pada t abel 3 x 5 di samping. Sesudah semua pet ak t erisi, j umlah bilangan pada set iap baris akan sama. Demikian pula halnya j umlah bilangan pada set iap kolom akan sama. Tent ukan semua pengisian pet ak yang mungkin. 6. Tent ukan semua bilangan prima p yang membuat 4p 2 + 1 dan 6p 2 + 1 keduanya bilangan prima. 7. Misalkan ABCD sebuah belah ket upat dengan ∠A = 60o dan P adalah t it ik pot ong kedua diagonal AC dan BD. Misalkan Q, R dan S t iga t it ik pada (keliling) belah ket upat . Jika PQRS j uga membent uk belah ket upat , t unj ukkan bahwa t epat sat u di ant ara Q, R, S berimpit dengan t it ik sudut belah ket upat ABCD. 31 SELEKSI TIM OLIMPIADE MATEMATIKA INDONESIA 2003 OLIMPIADE SAINS NASIONAL 2002 YOGYAKARTA, 10 SEPTEMBER 2002 Prestasi itu diraih bukan didapat !!! SOLUSI SOAL Bidang Mat emat ika Disusun oleh : Eddy Hermant o, ST 32 Solusi Olimpiade Sains Nasional 2002 Bidang : Mat emat ika 1. Alt ernat if 1 : n4 − n2 = n2(n2 − 1) = n2(n − 1)(n + 1) (n − 1), n dan (n + 1) adalah 3 bilangan bulat berurut an maka 3! = 6 membagi (n − 1)n(n + 1). Maka 3⏐ n 4 − n2. Jika n genap maka 4⏐n2 sedangkan j ika n ganj il maka 4⏐n2 − 1. Maka 4⏐ n2(n2 − 1) = n4 − n2 Karena 3 dan 4 relat if prima maka n 4 − n 2 habis dibagi 3 ⋅ 4 = 12. Alt ernat if 2 : n4 − n2 = n2(n2 − 1) = n2(n − 1)(n + 1) = (n − 1)n(n + 1)(n + 2 − 2) n4 − n2 = (n − 1)n(n + 1)(n + 2) − 2(n − 1)n(n + 1) n − 1, n, n + 1 dan n + 2 adalah 4 bilangan bulat berurut an maka 4! = 24⏐(n − 1)n(n + 1)(n + 2). (n − 1), n dan (n + 1) adalah 3 bilangan bulat berurut an maka 3! = 6 membagi (n − 1)n(n + 1). Maka 12⏐2(n − 1)n(n + 1). Maka 12⏐ n4 − n2 ∴ Terbukti bahwa n4 − n habis dibagi 12 untuk sebarang bilangan bulat n > 1. 2. 180 = 22 ⋅ 32 ⋅ 5 Maka kemungkinan lima mat a dadu yang memenuhi perkaliannya = 180 adalah (1, 3, 3, 4, 5), (1, 2, 3, 5, 6), (1, 1, 5, 6, 6), (2, 2, 3, 3, 5) dan permut asinya yang secara berurut an banyaknya kemungkinan t ersebut adalah 5! 5! 5! , , 5! , 2! 2!⋅2! 2!⋅2! Peluang hasil kali mat a dadu sama dengan 180 adalah = 1 65 ⎛ 5! 5! ⎞ 240 ⎟ = 5 ⎜⎜ + 5!+2 ⋅ 2!⋅2! ⎟⎠ 6 ⎝ 2! 144 = 24 ⋅ 32 Maka kemungkinan lima mat a dadu yang memenuhi perkaliannya = 144 adalah (1, 1, 4, 6, 6), (1, 2, 2, 6, 6), (1, 2, 3, 4, 6), (1, 3, 3, 4, 4), (2, 2, 3, 3, 4), (2, 2, 2, 3, 6) dan permut asinya yang 5! 5! 5! 5! 5! , , 5! , , , . 2!⋅2! 2!⋅2! 2!⋅2! 2!⋅2! 3! 5! ⎞ 260 5! 1 ⎛ Peluang hasil kali mat a dadu sama dengan 144 adalah = 5 ⎜⎜ 4 ⋅ + 5!+ ⎟⎟ = 5 3! ⎠ 6 ⎝ 2!⋅2! 6 secara berurut an banyaknya kemungkinan t ersebut adalah ∴ Maka peluang yang lebih besar adalah terj adinya hasil kali 144. 3. Alt ernat if 1 : x + y + z = 6 ⋅⋅⋅⋅⋅⋅⋅⋅⋅⋅⋅⋅⋅⋅⋅⋅⋅⋅⋅⋅⋅⋅⋅⋅⋅⋅⋅⋅⋅⋅⋅⋅⋅⋅⋅⋅⋅⋅⋅⋅ (1) x 2 + y2 + z2 = 12 ⋅⋅⋅⋅⋅⋅⋅⋅⋅⋅⋅⋅⋅⋅⋅⋅⋅⋅⋅⋅⋅⋅⋅⋅⋅⋅⋅⋅⋅⋅⋅⋅⋅⋅⋅⋅⋅⋅⋅⋅ (2) x 3 + y3 + z3 = 24 ⋅⋅⋅⋅⋅⋅⋅⋅⋅⋅⋅⋅⋅⋅⋅⋅⋅⋅⋅⋅⋅⋅⋅⋅⋅⋅⋅⋅⋅⋅⋅⋅⋅⋅⋅⋅⋅⋅⋅⋅ (3) (x + y + z) 2 = 62 x 2 + y2 + z2 + 2(xy + xz + yz) = 36 xy + xz + yz = 12 ⋅⋅⋅⋅⋅⋅⋅⋅⋅⋅⋅⋅⋅⋅⋅⋅⋅⋅⋅⋅⋅⋅⋅⋅⋅⋅⋅⋅⋅⋅⋅⋅⋅⋅⋅⋅⋅⋅⋅⋅ (4) Alt ernat if 1. a : (x + y + z)( x 2 + y2 + z2) = 6 ⋅ 12 = 72 x 3 + y3 + z3 + xy2 + xz2 + x 2y + x 2z + yz2 + y2z = 72 xy2 + xz2 + x 2y + x 2z + yz2 + y2z = 48 ⋅⋅⋅⋅⋅⋅⋅⋅⋅⋅⋅⋅⋅⋅⋅⋅⋅⋅⋅⋅⋅⋅⋅⋅⋅⋅⋅⋅⋅⋅⋅⋅⋅⋅⋅⋅⋅⋅⋅⋅ (5) SMA Negeri 5 Bengkulu Eddy Hermanto, ST 33 Olimpiade Sains Nasional 2002 Solusi Bidang : Mat emat ika (x + y + z)( xy + xz + yz) = 6 ⋅ 12 = 72 xy2 + xz2 + x 2y + x 2z + yz2 + y2z + 3xyz = 72. Maka 48 + 3xyz = 72 xyz = 8 ⋅⋅⋅⋅⋅⋅⋅⋅⋅⋅⋅⋅⋅⋅⋅⋅⋅⋅⋅⋅⋅⋅⋅⋅⋅⋅⋅⋅⋅⋅⋅⋅⋅⋅⋅⋅⋅⋅⋅⋅ (6) Dari persamaan (1), (4) dan (6) dapat disimpulkan bahwa x, y dan z adalah akar-akar persamaan t 3 − 6t 2 + 12t − 8 = 0 sehingga (t − 2) 3 = 0 Maka x = y = z = 2 ∴ Semua solusi yang memenuhi sistem persamaan tersebut hanya x = y = z = 2. Alt ernat if 1. b : Dengan AM-GM x 2 + y2 ≥ 2xy x 2 + z2 ≥ 2xz y2 + z2 ≥ 2yz Tanda kesamaan t erj adi j ika dan hanya j ika x = y = z Maka 12 = x 2 + y2 + z2 ≥ xy + xz + yz = 12 ⋅⋅⋅⋅⋅⋅⋅⋅⋅⋅⋅⋅⋅⋅⋅⋅⋅⋅⋅⋅⋅⋅⋅⋅ (7) Berdasarkan persamaan (2) dan (4) maka x = y = z Karena x + y + z = 6 maka x = y = z = 2. Set elah dicek t ernyat a x = y = z = 2 memenuhi ket iga persamaan. ∴ Semua solusi yang memenuhi sistem persamaan tersebut hanya x = y = z = 2. Alt ernat if 2 : Dari persamaan (1) dan (2) didapat x2 + y2 + z2 x+ y+z = 3 3 ⋅⋅⋅⋅⋅⋅⋅⋅⋅⋅⋅⋅⋅⋅⋅⋅⋅⋅⋅⋅⋅⋅⋅⋅⋅⋅⋅⋅⋅⋅⋅⋅⋅ (8) Berdasarkan ket aksamaan QM-AM maka x2 + y2 + z2 x + y + z ≥ 3 3 ⋅⋅⋅⋅⋅⋅⋅⋅⋅⋅⋅⋅⋅⋅⋅⋅⋅⋅⋅⋅⋅⋅⋅⋅⋅⋅ (9) Tanda kesamaan t erj adi j ika dan hanya j ika x = y = z. Berdasarkan persamaan (8) dan (9) dapat disimpulkan bahwa x = y = z = 2. Set elah dicek t ernyat a x = y = z = 2 memenuhi ket iga persamaan. ∴ Semua solusi yang memenuhi sistem persamaan tersebut hanya x = y = z = 2. Alt ernat if 3 : (x − 2) 2 + (y − 2) 2 + (z − 2) 2 = x 2 + y2 + z2 + 12 − 4(x + y + z) (x − 2) 2 + (y − 2) 2 + (z − 2) 2 = 12 + 12 − 4(6) = 0 Karena bilangan kuadrat t idak mungkin negat if maka kesamaan t erj adi hanya j ika x = y = z = 2. Set elah dicek t ernyat a x = y = z = 2 memenuhi ket iga persamaan. ∴ Semua solusi yang memenuhi sistem persamaan tersebut hanya x = y = z = 2. SMA Negeri 5 Bengkulu Eddy Hermanto, ST 34 Olimpiade Sains Nasional 2002 Solusi Bidang : Mat emat ika 4. Misalkan ∠ACB = γ Karena ∆ABD dan ∆ABC memiliki alas yang sama dan t it ik A, B, C dan D semuanya t erlet ak pada sat u lingkaran yang sama maka ∠ADB = ∠ACB = γ. Tit ik F adalah pert engahan AB dengan DF t egak lurus AB. Maka AD = BD. Karena DF t egak lurus AB dan F pert engahan maka ∠ADF = ∠FDB = ½ γ. Karena O pusat lingkaran maka ∠AOB = 2∠ACB = 2γ sert a ∠AOB = ∠FOB = γ AD = BD = AB ⋅⋅⋅⋅⋅⋅⋅⋅⋅⋅⋅⋅⋅⋅⋅⋅⋅⋅⋅⋅⋅⋅⋅⋅⋅⋅⋅⋅⋅⋅⋅⋅⋅⋅⋅⋅⋅⋅⋅⋅⋅⋅⋅⋅⋅⋅⋅⋅⋅ (1) 1 2 sin γ 2 ABCD adalah segiempat t alibusur, maka sesuai t eorema Pt olemy berlaku : AC ⋅ BD = DC ⋅ AB + AD ⋅ BC Karena AD = BD maka DC = ( AC − BC ) 1 2 sin γ 2 ⋅⋅⋅⋅⋅⋅⋅⋅⋅⋅⋅⋅⋅⋅⋅⋅⋅⋅⋅⋅⋅⋅⋅⋅⋅⋅⋅⋅⋅⋅⋅⋅⋅⋅⋅⋅⋅⋅⋅⋅⋅⋅⋅⋅⋅⋅⋅⋅⋅ (2) Karena DE t egak lurus AC maka pada ∆ADE berlaku AD2 = DE2 + AE2 ⋅⋅⋅⋅⋅⋅⋅⋅⋅⋅⋅⋅⋅⋅⋅⋅⋅⋅⋅⋅⋅⋅⋅⋅⋅⋅⋅⋅⋅⋅⋅⋅⋅⋅ (3) Pada ∆DEC berlaku DC2 = DE2 + EC2 ⋅⋅⋅⋅⋅⋅⋅⋅⋅⋅⋅⋅⋅⋅⋅⋅⋅⋅⋅⋅⋅⋅⋅⋅⋅⋅⋅⋅⋅⋅⋅⋅⋅⋅⋅⋅⋅⋅ (4) Dari persamaan (3) dan (4) didapat : AD2 − DC2 = AE2 − EC2 Mengingat AE = AC − EC maka : AD2 − DC2 = AC2 − 2 AC ⋅EC ( AB ) 2 − ( AC − BC ) 2 = AC 2 − 2AC ⋅ EC 1 4 sin 2 γ 2 Mengingat bahwa 2sin 2 ½ γ = 1 − cos γ dan AB2 = AC2 + BC2 − 2 AC BC cos γ maka : AC ⋅ BC ⋅ ( 2 − 2 cos γ ) = AC 2 − 2AC ⋅ EC 2 − 2 cos γ BC = AC − 2EC Karena AC = AE + EC maka AE = EC + CB (t erbukt i) ∴ Terbukti bahwa AE = EC + CB. SMA Negeri 5 Bengkulu Eddy Hermanto, ST 35 Solusi Olimpiade Sains Nasional 2002 Bidang : Mat emat ika 5. Karena j umlah pada set iap barisan sama dan j umlah pada set iap kolom sama maka j umlah ke-15 bilangan t ersebut akan habis dibagi 3 dan 5 yang berart i j umlah ke-15 bilangan t ersebut habis dibagi 15. Jumlah ke-16 bilangan = 3+4+5+6+7+8+9+10+11+12+13+16+17+18+19+20 = 178 Karena 178 ≡ 13 (mod 15) maka bilangan yang harus dibuang adalah 13. Karena j umlah bilangan = 165 maka j umlah masing-masing baris = 165 : 5 = 33 dan j umlah pada masing-masing kolom = 165 : 3 = 55. Berdasarkan hal t ersebut maka j elas bahwa G = 5. H + J = 13. Pasangan yang mungkin memenuhi adalah (6, 7) at au (7, 6) A + B = 23. Pasangan yang mungkin memenuhi adalah (4, 19), (19, 4). Pasangan (7, 16) dan (16, 7) t idak mungkin memenuhi sebab 7 past i berada pada baris ke-5. Jika A = 4 dan B = 19 maka A + D + 3 + G + 20 = 55. Akibat nya D = 23 (t idak ada bilangan 23). Maka nilai yang mungkin memenuhi hanya A = 19 dan B = 4 yang dipenuhi oleh D = 8. Pada baris ke-2, C + D + 9 = 33. Maka C = 16. Pada baris ke-3 berlaku E + F = 30. Pasangan (E, F) yang mungkin hanya (12, 18) at au (18, 12). Jika E = 18 maka 10 + C + E + 11 + H = 55. Akibat nya H = 0 (t idak ada bilangan 0) Maka kemungkinan nilai E hanya j ika E = 12. Maka F = 18 dan H = 6 yang berakibat J = 7 Dengan mengecek kembali semua bilangan t ersebut maka semuanya t erpenuhi. ∴ Hanya ada satu kemungkinan pengisian petak yaitu : 6. Karena p prima maka 4p 2 + 1 dan 6p 2 + 1 keduanya bilangan prima > 5. Alt ernat if 1 : Jika p 2 ≡ ±1 (mod 5) maka 4p 2 + 1 ≡ 0 (mod 5) yang t idak mungkin merupakan bilangan prima. Jika p 2 ≡ ±2 (mod 5) maka 6p 2 + 1 ≡ 0 (mod 5) yang t idak mungkin merupakan bilangan prima. Sedangkan j ika p 2 ≡ 0 (mod 5) maka bilangan prima p yang memenuhi hanya p = 5. Unt uk p = 5 maka 4p 2 + 1 = 101 dan 6p 2 + 1 = 151 yang keduanya merupakan bilangan prima. Alt ernat if 2 : Angka sat uan bilangan kuadrat adalah 0, 1, 4, 5, 6, 9. Jika angka sat uan p 2 adalah 0 maka angka sat uan p j uga 0 yang membuat t idak mungkin p prima. Jika angka sat uan p 2 adalah 5 maka angka sat uan p j uga 5. Bilangan prima p yang memenuhi hanya j ika p = 5 maka 4p 2 + 1 = 101 dan 6p 2 + 1 = 151 yang keduanya merupakan bilangan prima. SMA Negeri 5 Bengkulu Eddy Hermanto, ST 36 Solusi Olimpiade Sains Nasional 2002 Bidang : Mat emat ika Jika angka sat uan p 2 adalah 1 at au 6 maka angka sat uan 4p 2 + 1 adalah 5 yang membuat t idak mungkin 4p 2 + 1 bilangan prima. Jika angka sat uan p 2 adalah 4 at au 9 maka angka sat uan 6p 2 + 1 adalah 5 yang membuat t idak mungkin 5p 2 + 1 bilangan prima. ∴ Maka nilai p prima yang memenuhi 4p2 + 1 dan 6p2 + 1 hanya p = 5. 7. ABCD adalah belah ket upat sehingga AB = BC = CD = DA. Tidak mungkin Q, R dan S ket iga berimpit dengan t it ik sudut belah ket upat ABCD sebab akan menyebabkan t erdapat t iga t it ik P dan dua di ant ara Q, R dan S akan sej aj ar. Misalkan t erdapat dua t it ik, misalkan Q dan R, yang berhimpit dengan t it ik sudut ABCD. Salah sat u PQ at au PR adalah merupakan sisi belah ket upat PQRS. Tanpa mengurangi keumuman misalkan PQ adalah sisi belah ket upat PQRS. Maka PQ akan sej aj ar dengan salah sat u diagonal ABCD. Karena R berhimpit dengan t it ik sudut belah ket upat ABCD maka t idak mungkin ada ruas garis sej aj ar PQ dengan salah uj ungnya merupakan t it ik sudut ABCD. Misalkan t it ik Q t erlet ak pada sisi belah ket upat ABCD sehingga PQ t idak sej aj ar dengan salah sat u sisi ABCD sert a PQ merupakan salah sat u sisi belah ket upat PQRS. Misalkan j uga t it ik R adalah t it ik sehingga PR j uga merupakan sisi belah ket upat PQRS. Tidak mungkin PR sej aj ar sisisisi ABCD sebab akan membuat panj ang PR = ½AB ≠ PQ. Maka t it ik S t idak akan mungkin t erlet ak pada sisi yang sama dengan Q dan R sebab akan menyebabkan QS at au QR sej aj ar sisi ABCD, padahal PQ maupun PR t idak sej aj ar sisi ABCD. Misalkan t erdapat dua t it ik di ant ara Q, R at au S yang t erlet ak pada sisi yang sama. Misalkan t it iki t ersebut adalah QR. Tidak mungkin QR diagonal sebab akan menyebbakan t it ik S t erlet ak di luar ABCD. Akibat nya PS harus sej aj ar QR maka t it ik S adalah pert engahan dari sisi AB. BC, CD at au DA. Panj ang PS = ½AB. Dengan pusat P dan j ari-j ari PS dibuat lingkaran yang memot ong belah ket upat ABCD di pert engahan sisi AB, BC, CD, DA, B at au D Akibat nya t it ik Q at au R haruslah t erlet ak pada pert engahan sisi ABCD. Maka t it ik keempat haruslah t erlet ak pada A, B, C at au D. Jika t idak t erdapat dua t it ik yang t erlet ak pada sisi yang sama. Maka akan t erdapat dua t it ik yang t erlet ak pada sisi yang sej aj ar. Tanpa mengurangi keumuman misalakn sisi yang sej aj ar t ersebut adalah AD dan BC. Jika t it ik Q t erlet ak pada bagian A-13. Misalkan j uga t it ik Q adalah t it ik 1. Dari t it ik Q dibuat garis lurus melalui P dan meoot ng sisi BC di t it ik K. Karena ∆APD kongruen dengan ∆BPC (ket iga SMA Negeri 5 Bengkulu Eddy Hermanto, ST 37 Olimpiade Sains Nasional 2002 Solusi Bidang : Mat emat ika sudut nya sama dan AD = BC), maka PQ = PK. Dengan P sebagai pusat dan j ari-j ari PQ dibuat sebuah lingkaran yang akan memot ong sisi BC di t it ik K. Maka agar t erbent uk belah ket upat PQRS, haruslah t it ik K merupakan salah sat u t it ik sudut belah ket upat PQRS. Padahal Q, P dan K berada pada sat u garuis lurus. Maka t idak mungkin dapat dibent uk belah ket upat PQRS dengan salah sat u t it ik t erlet ak pada sisi A-13. Jika t it ik Q t erlet ak pada bagian 13-D. Tanpa mengurangi keumuman misalkan t it ik Q adalah t it ik 5. Dengan pusat P dan j ari-j ari PQ dapat dibuat sebuah lingkaran yang memot ong belah ket upat ABCD di t it ik 6 sampai 12. Dengan cara yang sama dengan sebelumnya maka 5-P-9, 6-P10, 7-P-11, 8-P-12 masing-masing merupakan garis lurus. Karena Q adalah t it ik 5 maka sat u t it ik yang t erlet ak pada sisi BC adalah t it ik 10, misalkan t it ik ini adalah R. Dari t it ik Q dibuat garis sej aj ar PR yang hanya akan memot ong sisi BC sehingga pada sisi BC akan t erdapat dua t it ik di ant ara Q, R dan S dan sesuai penj elasan sebelumnya hal t ersebut t idak akan t erpenuhi . Maka dapat disimpulkan bahwa PQRS akan berbent uk belah ket upat hanya j ika dua t it ik di ant ara Q, R at au S merupakan pert engahan sisi AB, BC, CD at au DA dan sat u di anat ar berhimpit dengan t it ik A, B, C at au D. ∴ Terbukti bahwa j ika PQRS membentuk belah ketupat maka tepat satu di ant ara Q, R at au S berhimpit dengan titik sudut belah ketupat ABCD. SMA Negeri 5 Bengkulu Eddy Hermanto, ST 38 SELEKSI OLIMPIADE TINGKAT KABUPATEN/ KOTA TAHUN 2003 TIM OLIMPIADE MATEMATIKA INDONESIA TAHUN 2004 Bidang Mat emat ika Waktu : 90 Menit DEPARTEMEN PENDIDIKAN NASIONAL DIREKTORAT JENDERAL PENDIDIKAN DASAR DAN MENENGAH DIREKTORAT PENDIDIKAN MENENGAH UMUM TAHUN 2003 39 OLIMPIADE MATEMATIKA TINGKAT KABUPATEN/ KOTA TAHUN 2003 1 Bagian Pertama 1. Ada berapa banyak diant ara bilangan-bilangan 20000002, 20011002, 20022002, 20033002 yang habis dibagi 9 ? A. 0 B. 1 C. 2 D. 3 E. 4 2. Ada berapa banyak bilangan 4-angka (digit ) yang semua angkanya genap dan bukan merupakan kelipat an 2003 ? A. 499 C. 624 E. Tidak sat upun diant aranya B. 500 D. 625 3. Hari ini usiaku 1/ 3 kali usia ayahku. Lima t ahun yang lalu, usiaku 1/ 4 kali usia ayahku pada wakt u it u. Berapakah usiaku sekarang ? A. 12 B. 15 C. 17 D. 20 E. 21 4. Sebuah kelas t erdiri dari 40 siswa. Diant aranya, 20 siswa menyukai pelaj aran Mat emat ika, 15 orang menyukai pelaj aran Biologi, 15 orang menyukai pelaj aran Bahasa Inggris dan lima orang menyukai ket iganya. Banyaknya siswa yang menyukai sedikit nya sat u dari ket iga pelaj aran t ersebut adalah ? A. 10 C. 20 E. Tidak sat upun diant aranya B. 15 D. 25 5. Masing-masing dari kelima pernyat aan berikut benar at au salah. (a) pernyat aan (c) dan (d) keduanya benar (b) pernyat aan (d) dan (e) t idak keduanya salah (c) pernyat aan (a) benar (d) pernyat aan (c) salah (e) pernyat aan (a) dan (c) keduanya salah. Berapa banyak diant ara kelima pernyat aan di at as yang benar ? A. 0 B. 1 C. 2 D. 3 E. 4 6. Misalkan x dan y adalah bilangan t aknol yang memenuhi xy = x =x −y y Berapakah nilai x + y ? A. − 3 2 B. − 1 2 C. 0 D. 40 1 2 E. 3 2 7. Di dalam suat u lingkaran L1 berj ari-j ari 1 dan berpusat di t it ik asal dilukis suat u lingkaran L2 yang bersinggungan dengan lingkaran L1 , dan dengan sumbu-x dan sumbu-y posit if . Jari-j ari lingkaran L2 adalah ? A. 1 3 B. 2 5 C. 2 −1 D. 1 2 E. 2 − 2 8. Misalkan 3a = 4, 4b = 5, 5c = 6, 6d = 7, 7e = 8, dan 8f = 9. Berapakah hasil kali abcdef ? A. 1 B. 2 C. 6 D. 3 E. 10 3 9. Misalkan N adalah bilangan bulat t erkecil yang bersif at : bersisa 2 j ika dibagi 5, bersisa 3 j ika dibagi oleh 7, dan bersisa 4 j ika dibagi 9. Berapakah hasil penj umlahan digit -digit dari N ?⋅ A. 4 B. 8 C. 13 D. 22 E. 40 10. Suat u garis melalui t it ik (m, −9) dan (7, m) dengan kemiringan m. Berapakah nilai m ? A. 1 B. 2 C. 3 D. 4 E. 5 2 Bagian Kedua 11. Misalkan f suat u f ungsi yang memenuhi ⎛1⎞ 1 f ⎜⎜ ⎟⎟ + f (− x ) = 2x ⎝x ⎠ x unt uk set iap bilangan real x ≠ 0. Berapakah nilai f (2) ? 12. Jika a dan b bilangan bulat sedemikian sehingga a2 − b 2 = 2003, maka berapakah nilai a2 + b 2 ? (Diket ahui bahwa 2003 merupakan bilangan prima) 13. Dari sepuluh orang siswa akan dibent uk 5 kelompok, masing-masing beranggot a dua orang. Berapa banyaknya cara membent uk kelima kelompok ini ? 14. Misalkan bahwa f (x) = x 5 + ax 4 + bx 3 + cx 2 + dx + c dan bahwa f (1) = f (2) = f (3) = f (4) = f (5). Berapakah nilai a ? 15. Berapakah hasil perkalian ⎛ 1 ⎞⎛ 1 ⎞⎛ 1 ⎞ ⎛ 1 ⎞ ⎟⎟ ⎜⎜1 − 2 ⎟⎟⎜⎜1 − 2 ⎟⎟⎜⎜1 − 2 ⎟⎟ L ⎜⎜1 − 2 ⎠⎝ 3 ⎠⎝ 4 ⎠ ⎝ 2003 2 ⎠ ⎝ 16. Iwan selalu berbohong pada hari Senin, Selasa, Rabu dan berkat a j uj ur pada hari-hari lainnya. Di lain pihak Budi selalu berbohong pada hari Kamis, Jumat , Sabt u dan berkat a j uj ur pada hari-hari lainnya. Pada suat u hari t erj adi percakapan berikut : Iwan : Kemarin saya berbohong Budi : Saya j uga Pada hari apa percakapan t ersebut t erj adi ? 41 17. Segit iga ABC adalah segit iga samasisi dengan panj ang sisi 1 sat uan. Melalui B dibuat garis yang t egak lurus BC. Garis t ersebut berpot ongan dengan perpanj angan garis AC di t it ik D. Berapakah panj ang BD ? ⎣α ⎦ sebagai bilangan bulat yang kurang dari at au sama dengan α. Sebagai cont oh ⎣4,9 ⎦ = 4 dan ⎣7 ⎦ = 7. Jika x dan y bilangan real sehingga ⎣ x ⎦ = 9 dan ⎣ y ⎦ = 12, maka nilai t erkecil yang mungkin dicapai oleh ⎣y − x ⎦ adalah ? 18. Unt uk set iap bilangan real α, kit a def inisikan 19. Unt uk menent ukan wakilnya dalam cabang lari 110 m gawang put era, sebuah SMU mengadakan seleksi yang diikut i 5 orang siswa. Dalam seleksi t ersebut diadakan t iga kali lomba yang pada set iap lomba, pelari t ercepat diberi nilai 5, sedangkan peringkat di bawahnya bert urut -t urut mendapat nilai 3, 2, 1, 1. Tidak ada dua pelari yang menempat i peringkat yang sama. Jika pemenang seleksi diberikan kepada yang nilai t ot alnya paling t inggi pada ket iga lomba, berapakah nilai t erendah yang mungkin dicapai oleh pemenang seleksi ? 20. Misalkan a, b, c, d, e, f , g, h, i menyat akan bilangan-bilangan bulat posit if berbeda yang kurang dari at au sama dengan sembilan. Jika j umlah set iap t iga bilangan dalam set iap lingkaran bernilai sama, berapakah nilai a + d + g ? 42 SELEKSI OLIMPIADE TINGKAT KABUPATEN/ KOTA 2003 TIM OLIMPIADE MATEMATIKA INDONESIA 2004 Prestasi itu diraih bukan didapat !!! SOLUSI SOAL Bidang Mat emat ika Disusun oleh : Eddy Hermant o, ST 43 Olimpiade Matematika Tk Kabupaten/ Kota 2003 BAGIAN PERTAMA 1. (Jawaban : A) Teori : Sebuah bilangan bulat habis dibagi 9 j ika j umlah digit bilangan t ersebut habis dibagi 9. Jumlah digit 20000002 = 2 + 0 + 0 + 0 + 0 + 0 + 0 + 2 Jumlah digit 20011002 = 2 + 0 + 0 + 1 + 1 + 0 + 0 + 2 Jumlah digit 20022002 = 2 + 0 + 0 + 2 + 2 + 0 + 0 + 2 Jumlah digit 20033002 = 2 + 0 + 0 + 3 + 3 + 0 + 0 + 2 ∴ Banyaknya bilangan yang habis dibagi 3 adalah 0 = 4 (Tidak habis dibagi 9) = 6 (Tidak habis dibagi 9) = 8 (Tidak habis dibagi 9) = 10 (Tidak habis dibagi 9) 2. (Jawaban : A) Angka pert ama ada 4 kemungkinan : 2, 4, 6, 8. Angka ke-2, ke-3 dan ke-4 masing-masing ada 5 kemungkinan. Banyaknya bilangan empat angka yang semua digit nya genap ada : 4 x 5 x 5 x 5 = 500 bilangan. Bilangan kelipat an 2003 yang t erdiri dari 4 angka adalah : 2003, 4006, 6009, 8012. Yang semua digit nya bilangan genap hanya 4006. ∴ Banyaknya bilangan 4 angka yang semua digit nya genap dan bukan merupakan kelipat an 2003 ada : 500 − 1 = 499 bilangan 3. (Jawaban : B) Misal usiaku saat (X − 5) = X −5= ini = X dan usia ayahku saat ini = Y, maka : 1 X = Y 3 dan 1 (Y − 5) 4 1 (3X − 5) 4 4X − 20 = 3X − 5 X = 15 ∴ Usiaku saat ini 15 t ahun 4. (Jawaban : ?) Misalkan M adalah himpunan siswa yang menyukai Mat emat ika ; B adalah himpunan siswa yang menyukai Biologi dan I adalah himpunan siswa yang menyukai Bahasa Inggris. SMA Negeri 5 Bengkulu Eddy Hermanto, ST 44 Olimpiade Matematika Tk Kabupaten/ Kota 2003 Misalkan n(M∪B∪I) = T. Maka banyaknya siswa yang menyukai paling sedikit 1 mat a pelaj aran adalah T. Misalkan banyaknya siswa yang t idak menyukai sat upun dari ket iga pelaj aran t ersebut adalah k n(M∪B∪I) = n(M) + n(B) + n(I) − n(M∩B) − n(M∩I) − n(B∩I) + n(M∩B∩I) T = 40 − k = 20 + 15 + 15 − (e + g) − (d + g) − (f + g) + g T = 40 − k = 40 − d − e − f . Maka k = d + e + f Tampak ada yang kurang pada soal. Kemungkinan maksud soal : a. k = 0, banyaknya siswa yang menyukai hanya 1 pelaj aran ? n(M∪B∪I) = n(M) + n(B) + n(I) − n(M∩B) − n(M∩I) − n(B∩I) + n(M∩B∩I) 40 = 20 + 15 + 15 − (e + g) − (d + g) − (f + g) + g d+e+g=0 Karena d ≥ 0 ; e ≥ 0 dan f ≥ 0 maka d = 0 ; e = 0 dan f = 0 a + d + e + g = 20 sehingga a = 20 − 5 − 0 − 0 = 15 c + d + f + g = 15 sehingga c = 15 − 5 − 0 − 0 = 10 b + e + f + g = 15 sehingga b = 15 − 5 − 0 − 0 = 10 Banyaknya siswa yang menyukai hanya 1 pelaj aran adalah = a + b + c = 35 b. n(M∪B∪I) = 40 dan pert anyaan sesuai dengan soal Maka j elas a + b + c + d + e + f + g = 40 ( Cat at an : Jawaban asli soal ini adalah 25 , t api bagaimana mendapat kannya ? ) 5. (Jawaban : D) Misalkan (a) benar maka (c) dan (d) benar Berdasarkan (d) hal ini merupakan kont radiksi. Maka (a) salah. Karena (a) salah maka (c) j uga salah sehingga (d) dan (e) benar. Akibat nya (b) j uga benar. Pernyat aan yang benar adalah (b) ; (d) dan (e). ∴ Banyaknya pernyat aan yang benar ada : 3 6. (Jawaban : A) xy = x y ; y≠0 xy2 = x ⋅⋅⋅⋅⋅⋅⋅⋅⋅⋅⋅ (1) a. Unt uk x = 0 x = x − y . Maka 0 = 0 − y sehingga y = 0 (Tidak memenuhi syarat awal bahwa y ≠ 0) y b. Unt uk x ≠ 0 Berdasarkan pers (1) maka y 2 = 1 sehingga y = 1 at au y = −1 * Unt uk y = 1 x = x − y . Maka x = x − 1. Karena 0 = − 1 maka t idak ada nilai x yang memenuhi y * Unt uk y = −1 x 1 = x − y . Maka −x = x + 1 sehingga x = − 2 y 3 1 ∴ x + y = − + (− 1) = − 2 2 SMA Negeri 5 Bengkulu Eddy Hermanto, ST 45 Olimpiade Matematika Tk Kabupaten/ Kota 2003 7. (Jawaban : C) OB adalah j ari-j ari lingkaran besar dengan pusat O. Misal j ari-j ari lingkaran dalam = r, maka AB = r Karena OD = OC = r maka OA = r √2 OB = OA + AB 1 = r √2 + r ∴ r = 1 2 +1 = 2 −1 8. (Jawaban : B) Karena 3a = 4 maka a = 3log 4 Karena 5c = 6 maka c = 5log 6 Karena 7e = 8 maka e = 7log 8 abcdef = 3log 4 ⋅ 4log 5 ⋅ 5log 6 ⋅ ∴ abcdef = 2 Karena 4b = 5 maka b = 4log 5 Karena 6d = 7 maka d = 6log 7 Karena 8f = 9 maka f = 8log 9 6 log 7 ⋅ 7log 8 ⋅ 8log 9 = 3log 9 = 2 9. (Jawaban : C) Alt ernat if 1 : Karena N bersisa 2 j ika dibagi 5 maka N = 5m + 2. Bilangan-bilangan N adalah 2, 7, 12, 17, 22, ⋅⋅⋅⋅ Karena N bersisa 3 j ika dibagi 7 maka N = 7n + 3. Bilangan-bilangan N adalah 3, 10, 17, 24, 31, ⋅⋅⋅ Karena persekut uan t erkecilnya 17 maka bilangan yang bersisa 2 j ika dibagi 5 dan bersisa 3 j ika dibagi 7 akan berbent uk N = (5 ⋅ 7) p + 17 = 35p + 17 dengan p adalah bilangan bulat . Bilanganbilangan N adalah 17, 52, 87, 122, 157, 192, ⋅⋅⋅⋅⋅⋅ N bersisa 4 j ika dibagi 9. Maka N = 9t + 4. Bilangan-bilangan N adalah 4, 13, 22, 31, 40, 49, 58, 67, 76, 85, 94, 103, 112, 121, 130, 139, 148, 157, 166, ⋅⋅⋅⋅⋅⋅ Karena persekut uan t erkecilnya adalah 157, maka bilangan yang bersisa 2 j ika dibagi 5, bersisa 3 j ika dibagi 7 dan bersisa 4 j ika dibagi 9 akan berbent uk N = (35 ⋅ 9)k + 157 N = 315k + 157 Nmin = 157 j ika k = 0 ∴ Jumlah digit dari Nmin adalah = 1 + 5 + 7 = 13 Alt ernat if 2 : Karena N bersisa 2 j ika dibagi 5 maka N = 5m + 2 unt uk suat u bilangan bulat t ak negat if m. N ≡ 3 (mod 7) 5m + 2 ≡ 3 (mod 7) 5m ≡ 1 (mod 7) Nilai m yang memenuhi haruslah berbent uk m = 7k + 3 unt uk suat u bilangan bulat t ak negat if k. N = 5m + 2 = 5(7k + 3) + 2 = 35k + 17 N ≡ 4 (mod 9) 35k + 17 ≡ 4 (mod 9) ≡ 22 (mod 9) 35k ≡ 5 (mod 9) Nilai k yang memenuhi haruslah berbent uk k = 9p + 4 unt uk suat u bilangan bulat t ak negat if p. N = 35k + 17 = 35(9p + 4) + 17 = 315p + 157. Nmin = 157 j ika p = 0 ∴ Jumlah digit dari Nmin adalah = 1 + 5 + 7 = 13 SMA Negeri 5 Bengkulu Eddy Hermanto, ST 46 Olimpiade Matematika Tk Kabupaten/ Kota 2003 10. (Jawaban : C) y2 −y1 x2 −x1 m − (− 9 ) m = 7−m Gradien = m + 9 = 7m − m 2 (m − 3) 2 = 0 ∴ m=3 BAGIAN KEDUA ⎛1⎞ 1 ⎟⎟ + f (− x ) = 2x ⎝x ⎠ x 11. f ⎜⎜ ⎛ 1⎞ ⎜⎜ − ⎟⎟ = 1 ⋅⋅⋅⋅⋅⋅⋅⋅⋅⋅⋅⋅⋅ ⎝ 2⎠ ⎛ 1⎞ 1 Unt uk x = −2 maka f ⎜⎜ − ⎟⎟ − f (2 ) = −4 ⎝ 2⎠ 2 ⎛ 1⎞ 2f ⎜⎜ − ⎟⎟ − f (2) = −8 ⋅⋅⋅⋅⋅⋅⋅⋅⋅ ⎝ 2⎠ Unt uk x = 1 maka f (2 ) + 2f 2 (1) (2) Kurangkan persamaan (1) dengan persamaan (2) 2f (2) = 9 ∴ f(2) = 9 2 12. a2 − b 2 = 2003. Maka (a + b) (a − b) = 2003 ⋅ 1 * Unt uk a + b = 2003 dan ( a − b) = 1 didapat 2a = 2004. Maka a = 1002 dan b = 1001 a2 + b 2 = (1002) 2 + (1001) 2 = 2006005 * Unt uk (a + b) = 1 dan (a − b ) = 2003 didapat 2a = 2004. Maka a = 1002 dan b = − 1001 a2 + b 2 = (1002) 2 + ( − 1001) 2 = 2006005 ∴ a2 + b 2 = 2006005 13. Alt ernat if 1: * Jika 2 orang siswa akan dibent uk 1 kelompok Banyaknya cara ada 1 * Jika 4 orang siswa (misal A, B, C dan D) akan dibent uk menj adi 2 kelompok yang masingmasing beranggot a 2 orang SMA Negeri 5 Bengkulu Eddy Hermanto, ST 47 Olimpiade Matematika Tk Kabupaten/ Kota 2003 * * * Pasangkan A dengan salah sat u anggot a lainnya. Maka sisanya adalah membent uk 1 kelompok yang masing-masing beranggot a 2 orang. Banyaknya cara ada 1. Karena kemungkinan pasangan A ada 3, maka banyaknya cara dari 4 orang siswa akan dibent uk 2 kelompok yang masing-masing beranggot a dua orang adalah 3 x 1 = 3 cara. Jika 6 orang siswa (misal A, B, C, D, E dan F) akan dibent uk menj adi 3 kelompok yang masing-masing beranggot a 2 orang Pasangkan A dengan salah sat u anggot a lainnya. Maka sisanya adalah membent uk 2 kelompok yang masing-masing beranggot a 2 orang. Banyaknya cara ada 3 x 1. Karena kemungkinan pasangan A ada 5, maka banyaknya cara dari 6 orang siswa akan dibent uk 3 kelompok yang masing-masing beranggot a dua orang adalah 5 x 3 x 1 = 15 cara. Jika 8 orang siswa (misal A, B, C, D, E, F, G dan H) akan dibent uk menj adi 4 kelompok yang masing-masing beranggot a 2 orang Pasangkan A dengan salah sat u anggot a lainnya. Maka sisanya adalah membent uk 3 kelompok yang masing-masing beranggot a 2 orang. Banyaknya cara ada 5 x 3 x 1. Karena kemungkinan pasangan A ada 7, maka banyaknya cara dari 8 orang siswa akan dibent uk 4 kelompok yang masing-masing beranggot a dua orang adalah 7 x 5 x 3 x 1 = 105 cara. Jika 10 orang siswa (misal A, B, C, D, E, F, G, H, I dan J) akan dibent uk menj adi 5 kelompok yang masing-masing beranggot a 2 orang Pasangkan A dengan salah sat u anggot a lainnya. Maka sisanya adalah membent uk 4 kelompok yang masing-masing beranggot a 2 orang. Banyaknya cara ada 7 x 5 x 3 x 1. Karena kemungkinan pasangan A ada 9, maka banyaknya cara dari 10 orang siswa akan dibent uk 5 kelompok yang masing-masing beranggot a dua orang adalah 9 x 7 x 5 x 3 x 1 = 945 cara. Alt ernat if 2 : Pilih salah sat u siswa. Banyaknya cara memasangkan siswa t ersebut dengan siswa lain adalah 9C1. Pilih salah sat u siswa dari 8 siswa yang sisa. Banyaknya cara memasangkan siswa t ersebut dengan siswa yang lain adalah 7C1. Pilih salah sat u siswa dari 6 siswa yang sisa. Banyaknya cara memasangkan siswa t ersebut dengan siswa yang lain adalah 5C1. Pilih salah sat u siswa dari 4 siswa yang sisa. Banyaknya cara memasangkan siswa t ersebut dengan siswa yang lain adalah 3C1. Sisanya adalah 2 orang siswa yang t idak dapat dipilih lagi. Banyaknya cara membent uk kelima kelompok adalah 9C1 ⋅ 7C1 ⋅ 5C1 ⋅ 3C1 ⋅ 1 = 945. ∴ Banyaknya cara membent uk kelima kelompok t ersebut adalah 945 14. Misal f (1) = f (2) = f (3) = f (4) = f (5) = k Dibent uk persamaan polinomial : g(x) = x 5 + ax 4 + bx 3 + cx 2 + dx + c − k g(x) = f (x) − k Jelas bahwa g(1) = g(2) = g(3) = g(4) = g(5) = 0 Berart i bahwa 1; 2; 3; 4 dan 5 adalah akar-akar persamaan polinomial g(x) = 0. x 5 + ax 4 + bx 3 + cx 2 + dx + c − k = 0 x1 + x2 + x3 + x4 + x5 = − B a = − = −a 1 A Karena akar-akarnya adalah 1; 2; 3; 4 dan 5 maka : 1+2+3+4+5=−a ∴ a = − 15 SMA Negeri 5 Bengkulu Eddy Hermanto, ST 48 Olimpiade Matematika Tk Kabupaten/ Kota 2003 1 ⎞⎛ 1 ⎞⎛ 1 ⎞ ⎛ 1 ⎞⎛ 1 ⎞ ⎟⎟ ⎟⎜1 − ⎟⎜1 − 2 ⎟⎟⎜⎜1 − 2 ⎟⎟ L ⎜⎜1 − 2 ⎟⎜ 2 ⎟⎜ 2 ⎠⎝ 3 ⎠⎝ 4 ⎠ ⎝ 2002 ⎠⎝ 2003 2 ⎠ ⎝ ⎛ 1 ⎞ 1 ⎞⎛ 1 ⎞⎛ 1 ⎞⎛ 1⎞ ⎛ 1 ⎞⎛ 1 ⎞⎛ 1 ⎞⎛ 1 ⎞⎛ 1 ⎞⎛ ⎟⎟⎜⎜1 + ⎟⎟⎜⎜1 − ⎟⎟⎜⎜1 + ⎟ = ⎜⎜1 − ⎟⎟⎜⎜1 + ⎟⎟⎜⎜1 − ⎟⎟⎜⎜1 + ⎟⎟⎜⎜1 − ⎟⎟⎜⎜1 + ⎟⎟ L ⎜⎜1 − 2003 ⎟⎠ 2003 ⎠⎝ 2002 ⎠⎝ 2002 ⎠⎝ 4⎠ ⎝ 4 ⎠⎝ 3 ⎠⎝ 3 ⎠⎝ 2 ⎠⎝ 2 ⎠⎝ ⎝ 1 ⎛ 3 2 ⎞ ⎛ 4 3 ⎞ ⎛ 5 4 ⎞ ⎛ 2002 2001 ⎞ ⎛ 2003 2002 ⎞ 2004 ⎟⋅⎜ ⎟⋅ = ⋅ ⎜⎜ ⋅ ⎟⎟ ⋅ ⎜⎜ ⋅ ⎟⎟ ⋅ ⎜⎜ ⋅ ⎟⎟ L ⎜⎜ ⋅ ⋅ 2 ⎝ 2 3 ⎠ ⎝ 3 4 ⎠ ⎝ 4 5 ⎠ ⎝ 2001 2002 ⎟⎠ ⎜⎝ 2002 2003 ⎟⎠ 2003 1 2004 = ⋅ 2 2003 1002 S = 2003 ⎛ 15. S = ⎜⎜1 − S S S ∴ 16. Misalkan pada hari t ersebut Iwan berbohong dan dengan berdasarkan perkat aannya, pada hari sebelumnya Iwan harus berkat a j uj ur. Akibat nya hari t ersebut adalah Senin karena pada hari Minggu Iwan berkat a j uj ur. Pada hari Senin Budi berkat a j uj ur. Maka berdasarkan perkat aannya berart i pada hari Minggu Budi berbohong. Hal t ersebut kont radiksi karena pada hari Minggu Budi berkat a j uj ur. Misalkan pada hari t ersebut Iwan berkat a j uj ur dan dengan berdasarkan perkat aannya, pada hari sebelumnya Iwan harus berkat a bohong. Akibat nya hari t ersebut adalah Kamis karena Rabu Iwan berbohong. Pada hari Kamis Budi berkat a bohong. Maka berdasarkan perkat aannya berart i pada hari Rabu Budi berkat a j uj ur. Hal t ersebut sesuai karena pada hari Rabu Budi berkat a j uj ur. ∴ Percakapan t ersebut t erj adi pada hari Kamis 17. ∠CBA = 60o maka ∠ABD = 30o Jelas ∠ACB = 60o, maka ∠ADB = 90o − ∠ACB = 30o BD AB BD 1 = = , maka sin 120° sin 30° sin ∠BAD sin ∠ADB sin 120° BD = sin 30° ∴ BD = √3 18. Karena Karena 81 = 9 dan ⎣ x ⎦ = 9 dipenuhi oleh 81 ≤ x < 100 = 13 maka ⎣ y ⎦ = 12 dipenuhi oleh 144 ≤ y < 169 100 = 10 maka 144 = 12 dan 169 ⎣ y − x ⎦ min = ⎣ ymin − x maks ⎦ = ⎣ 144 − 99, 99⋅⋅⋅⎦ = ⎣ 44, 00⋅⋅⋅⋅⋅1 ⎦ ∴ ⎣ y − x ⎦ min = 44 19. Nilai t ot al = 3 ⋅ ( 5 + 3 + 2 + 1 + 1 ) = 36 Misal nilai pemenang = x. Maka nilai sisa = 36 − x SMA Negeri 5 Bengkulu Eddy Hermanto, ST 49 Olimpiade Matematika Tk Kabupaten/ Kota 2003 Agar x minimum maka nilai sisa harus t erdist ribusi merat a kepada 4 pelari lain. Misal nilai masing-masing pelari lain = y x + 4y = 36 dengan x > y. Maka 4x > 4y 4x > 36 − x. 5x > 36 Jika x = 8 maka 4y = 28 sehingga y = 7. Kombinasi nilai 7 adalah (5, 1, 1) ; (1, 5, 1) ; (3, 1, 3) ; (2, 3, 2). Karena masing-masing nilai 2, 3 dan 5 t idak lebih dari t iga kali dan nilai 1 t idak lebi h dari 6 kali, maka kombinasi di at as memenuhi. ∴ Nilai minimum pemenang adalah 8 20. 1 ≤ a, b, c, d, e, f , g, h, i ≤ 9 Karena a, b, c, d, e, f , g, h, i adalah bilangan bulat berbeda maka : a + b + c + d + e + f + g + h + i = 1 + 2 + 3 + 4 + 5 + 6 + 7 + 8 + 9 = 45 Misal masing-masing lingkaran berj umlah k dan karena ada 9 lingkaran, maka : (a+1+i)+(b+2+a)+(c+3+b)+(d+4+c)+(e+5+d)+(f +6+e)+(g+7+f )+(h+8+g)+(i+9+h) = 9k (1 + 2 + 3 + 4 + 5 + 6 + 7 + 8 + 9) + 2 (a + b + c + d + e + f + g + h + i) = 9k 45 + 2 ⋅ 45 = 9k k = 15 Karena a + 1 + i = 15 maka a + i = 14 Kemungkinan nilai a dan i adalah : a = 5 dan i = 9 at au a = 9 dan i = 5 at au a = 6 dan i = 8 at au a = 8 dan i = 6. Karena i + 9 + h = 15 maka i + h = 6 Kemungkinan nilai h dan i adalah : h = 1 dan i = 5 at au h = 5 dan i = 1 at au h = 2 dan i = 4 at au h = 4 dan i = 2. Irisan dari kedua persamaan di at as didapat i = 5. Maka h = 1 dan a = 9 Karena b + 2 + a = 15 maka b = 15 − 2 − 9 = 4 Karena c + 3 + b = 15 maka c = 15 − 3 − 4 = 8 Karena d + 4 + c = 15 maka d = 15 − 4 − 8 = 3 Karena e + 5 + d = 15 maka e = 15 − 5 − 3 = 7 Karena f + 6 + e = 15 maka f = 15 − 6 − 7 = 2 Karena g + 7 + f = 15 maka g = 15 − 7 − 2 = 6 ∴ a + d + g = 9 + 3 + 6 = 18 SMA Negeri 5 Bengkulu Eddy Hermanto, ST 50 SELEKSI OLIMPIADE MATEMATIKA INDONESIA 2004 TINGKAT PROVINSI Bidang Mat emat ika Bagian Per t ama Waktu : 90 Menit DEPARTEMEN PENDIDIKAN NASIONAL DIREKTORAT JENDERAL PENDIDIKAN DASAR DAN MENENGAH DIREKTORAT PENDIDIKAN MENENGAH UMUM TAHUN 2003 51 OLIMPIADE MATEMATIKA TINGKAT PROVINSI TAHUN 2003 BAGIAN PERTAMA 1. Jika a dan b bilangan bulat ganj il dengan a > b, berapa banyakkah bilangan bulat genap di ant ara a dan b ? 2. Agung mendapat kan bahwa nilai rat a-rat a dari t iga ulangan mat emat ika yang diikut inya adalah 81. Nilai ulangan pert ama adalah 85. Nilai ulangan ket iga lebih rendah 4 dari nilai ulangan kedua. Berapakah nilai ulangan kedua Agung ? 3. Apakah himpunan j awab dari persamaan |x + 2| + |3x | = 14 ? 4. Keempat bilangan 3, 5, 7 dan 8 akan diisikan ke dalam kot ak-kot ak di samping. Berapakah hasil t erbesar yang dapat diperoleh ? 5. Misalkan x, y, z t iga bilangan asli berbeda. Fakt or persekut uan t erbesar ket iganya adalah 12, sedangkan kelipat annya persekut uan t erkecil ket iganya adalah 840. Berapakah nilai t erbesar bagi x + y + z ? L4 2003 6. Berapakah bilangan bulat posit if k t erkecil sehingga 20032003 144 424 4 3 habis dibagi 9 ? k 7. Persamaan kuadrat 2x 2 − 2(2a + 1)x + a(a − 1) = 0 mempunyai dua akar real x 1 dan x 2. Berapakah nilai a yang memenuhi persamaan kuadrat t ersebut sehingga x 1 < a < x 2 ? 8. Dalam sebuah segit iga ABC siku-siku sama kaki, dibuat persegi PQRS sebagai berikut : Tit ik P pada sisi AB, t it ik Q pada sisi AC, sedangkan t it ik-t it ik R dan S pada sisi miring BC. Jika luas segit iga ABC adalah x, berapakah luas persegi PQRS ? 9. Upik melemparkan n dadu. Ia menghit ung peluang t erj adinya j umlah mat a dadu sama dengan 6. Unt uk n berapakah peluang t ersebut paling besar ? 10. Suat u garis vert ikal membagi segit iga dengan t it ik sudut (0, 0), (1, 1) dan (9, 1) menj adi dua daerah dengan luas yang sama. Apakah persamaan garis t ersebut ? 11. Misalkan m dan n dua bilangan asli yang memenuhi m 2 − 2003 = n2. Berapakah mn ? 12. Berapakah nilai x yang memenuhi 4log ( 2log x) + 2log ( 4log x) = 2 ? 13. Tit ik P t erlet ak di dalam persegi ABCD demikian rupa, sehingga AP : BP : CP = 1 : 2 : 3. Berapakah besar sudut APB ? 14. Dengan mengkombinasikan ket iga warna dasar merah, kuning, dan biru dapat dibent uk warnawarna yang lain. Misalkan t erdapat 5 kaleng cat warna merah, 5 kaleng warna kuning, dan 5 52 kaleng warna biru. Budi boleh memilih kaleng manapun unt uk mencampurkan warna, dan semua cat dalam sebuah kaleng harus dipakai semua. Ada berapa pilihan warna yang dihasilkan ? 15. Pak Ot o membeli dua mobil unt uk dij ual kembali. Ia memperoleh keunt ungan 30% dari mobil pert ama, t et api menderit a kerugian 20% pada mobil kedua. Harga j ual kedua mobil sama. Berapa persenkah keunt ungan (at au kerugian) pak Ot o secara keseluruhan ? [ Cat at an : Semua persent ase t erhadap harga pembelian. Unt uk j awaban, gunakan t anda ‘ −’ unt uk menyat akan kerugian dan t anda ‘ +’ unt uk menyat akan keunt ungan. ] 16. Empat pasang suami ist eri menont on pagelaran orkest ra. Tempat duduk mereka harus dipisah ant ara kelompok suami dan kelompok ist eri. Unt uk masing-masing kelompok disediakan 4 buah t empat duduk bersebelahan dalam sat u barisan. Ada berapa banyak cara memberikan t empat duduk kepada mereka ? 17. Sebuah bola dengan j ari-j ari r dit endang dari B ke A. Bola t ersebut menggelinding sebanyak t epat 10 put aran sebelum membent ur bidang miring dan berhent i. Berapakah j arak dari B ke A ? 18. Berapakah sisa pembagian 1 ⋅ 1! + 2 ⋅ 2! + 3 ⋅ 3! + ⋅⋅⋅ + 99 ⋅ 99! + 100 ⋅ 100! oleh 101 ? 19. Suat u lingkaran mempunyai diamet er AB yang panj angnya merupakan bilangan bulat 2-angka. Tali busur CD t egak lurus pada AB dan memot ong AB di t it ik H. Panj ang CD sama dengan bilangan yang diperoleh dengan menukar let ak kedua angka dari panj ang AB. Jika j arak dari H ke pusat lingkaran merupakan bilangan rasional, berapakah panj ang AB ? 20. Berapakah banyaknya cara memilih t iga bilangan berbeda sehingga t idak ada dua bilangan yang berurut an, j ika bilangan-bilangan t ersebut dipilih dari himpunan {1, 2, 3, ⋅⋅⋅, 9, 10 } ? 53 SELEKSI OLIMPIADE MATEMATIKA INDONESIA 2004 TINGKAT PROVINSI Bidang Mat emat ika Bagian Kedua Waktu : 120 Menit DEPARTEMEN PENDIDIKAN NASIONAL DIREKTORAT JENDERAL PENDIDIKAN DASAR DAN MENENGAH DIREKTORAT PENDIDIKAN MENENGAH UMUM TAHUN 2003 54 OLIMPIADE MATEMATIKA TINGKAT PROVINSI TAHUN 2003 BAGIAN KEDUA 1. Andi, Beni, Coki, Doni dan Edo bermain kancil-serigala. Set iap anak menj adi kancil at au serigala, t et api t idak kedua-duanya. Kancil selalu j uj ur, sement ara serigala selalu berdust a. Andi berkat a bahwa Beni adalah kancil. Coki berkat a bahwa Doni adalah serigala. Edo berkat a Andi bukan serigala. Beni berkat a Coki bukan kancil. Doni berkat a bahwa Edo dan Andi adalah binat ang yang berbeda. Tent ukan banyaknya serigala dalam permainan ini. 2. Tent ukan semua bilangan bulat a dan b sehingga bilangan 2+ a 3+ b merupakan bilangan rasional 3. Tit ik-t it ik P dan Q bert urut -t urut adalah t it ik t engah rusuk AE dan CG pada kubus ABCD. EFGH. Jika panj ang rusuk kubus adalah 1 sat uan, t ent ukan luas segi-empat DPFQ. 4. Bukt ikan bahwa 999! < 500999. [ Cat at an : n! = 1 x 2 x 3 x ⋅⋅⋅ x n. ] 5. Tiga buah t it ik t erlet ak pada daerah yang dibat asi oleh sumbu y dan graf ik persamaan 7x − 3y2 + 21 = 0. Bukt ikan bahwa sedikit nya dua di ant ara ket iga t it ik t ersebut mempunyai j arak t idak lebih dari 4 sat uan. 55 SELEKSI OLIMPIADE MATEMATIKA INDONESIA 2004 TINGKAT PROVINSI TAHUN 2003 Prestasi itu diraih bukan didapat !!! SOLUSI SOAL Bidang Mat emat ika Bagian Pertama Disusun oleh : Eddy Hermant o, ST 56 Olimpiade Matematika Tk Provinsi 2003 Solusi Bagian Pert ama BAGIAN PERTAMA 1. Banyaknya bilangan bulat ant ara a dan b adalah a − b − 1. Karena a dan b ganj il, maka banyaknya bilangan genap di ant ara a dan b lebih sat u dari banyaknya bilangan ganj il di ant ara a dan b. (a − b − 1) + 1 . 2 a −b ∴ Banyaknya bilangan genap di ant ara a dan b adalah 2 Maka banyaknya bilangan bulat genap dirumuskan dengan 2. Misal nilai ulangan ke-2 Agung = x, maka 81 + 2x = 81 ⋅ 3. Maka x = 81 ∴ Nilai ulangan Agung ke-2 = 81 (85) + (x ) + (x − 4) = 81 3 Unt uk x ≤ −2, maka |x + 2| = −x − 2 dan |3x| = −3x |x + 2| + |3x | = 14. Maka −x − 2 − 3x = 14 sehingga x = −4 (memenuhi bahwa x ≤ −2) * Unt uk −2 ≤ x ≤ 0 maka |x + 2| = x + 2 dan |3x | = −3x |x + 2| + |3x | = 14. Maka x + 2 − 3x = 14 sehingga x = −6 (t idak memenuhi bahwa −2 ≤ x ≤ 0) * Unt uk x ≥ 0 maka |x + 2| = x + 2 dan |3x | = 3x |x + 2| + |3x | = 14. Maka x + 2 + 3x = 14 sehingga x = 3 (memenuhi bahwa x ≥ 0) ∴ Himpunan j awab dari persamaan |x + 2| + |3x| = 14 adalah = { −4, 3} 3. * 4. Teori : Agar T − M maksimal , maka T harus sebesar-besarnya dan M harus sekecil -l ecil nya. Jika diinginkan N sebesar-besarnya, maka A dan D harus maksimal dengan A > D sedangkan B dan C harus minimum dan karena B ⋅ C = C ⋅ B, maka t idak ada pengaruh posisi B dan C. Berart i A = 8, B = 3, C = 5, D = 7 at au A = 8, B = 5, C = 3, D = 7 ∴ N =8− 3 41 ⋅5 = 7 7 5. Karena f akt or persekut uan t erbesar dari x, y, z adalah 12, maka x, y, z akan berbent uk x = 12a, y = 12b dan z = 12c dengan a, b dan c adalah bilangan bulat FPB(a, b, c) = 1 Dan karena 840 : 12 = 70, maka a, b dan c masing-masing harus f akt or dari 70. Nilai a, b dan c harus diambil dari f akt or-f akt or 70 yait u : 1, 2, 5, 7, 10, 14, 35 dan 70. Karena diinginkan nilai x + y + z yang t erbesar maka nilai a + b + c j uga harus yang t erbesar. Karena FPB (14, 35, 70), FPB (10, 35, 70), FPB (7, 35, 70), FPB (5, 35, 70) semuanya lebih dari 1 maka a, b dan c diambil dari 2, 35 dan 70 at au 10, 14, 35 dan karena 2 + 35 + 70 > 10 + 14 + 35 maka a, b dan c diambil dari 2, 35 dan 70. ∴ (x + y + z) t erbesar = 12 ⋅ 2 + 12 ⋅ 35 + 12 ⋅ 70 = 1284 SMA Negeri 5 Bengkulu Eddy Hermanto, ST 57 Olimpiade Matematika Tk Provinsi 2003 Solusi Bagian Pert ama L4 2003 6. Misal N = 20032003 144 424 4 3. k Agar N habis dibagi 9 maka j umlah digit N harus habis dibagi 9. Karena 2 + 0 + 0 + 3 = 5 maka j umlah digit N = 5k. ∴ Bilangan bulat posit if k t erkecil yang memenuhi adalah k = 9 7. x 1, 2 = * 4a + 2 ± ( 4a + 2) 2 − 4( 2)(a 2 − a ) 1 1 =a + ± 2a 2 + 6a + 1 2⋅2 2 2 Akar-akarnya real berart i Disk ≥ 0. Maka Disk = (4a + 2) 2 − 4(2)(a2 − a) ≥ 0 8a2 + 24a + 4 ≥ 0 2a2 + 6a + 1 ≥ 0 3 1 − 6 ± 6 2 − 4( 2)(1) 7 =− ± 2⋅2 2 2 3 1 3 1 Nilai a yang memenuhi adalah a ≤ − − 7 at au a ≥ − + 7 2 2 2 2 a 1, 2 = * a < x 2. Maka a < a + 1 1 + 2a 2 + 6a + 1 sehingga 2 2 ⋅⋅⋅⋅⋅⋅⋅⋅⋅⋅⋅⋅⋅⋅⋅⋅⋅⋅⋅⋅⋅⋅⋅ (1) 2a 2 + 6a + 1 > −1 Akar dari suat u bilangan bernilai posit if sehingga semua nilai a memenuhi. * a > x 1. Maka a > a + 1 1 − 2a 2 + 6a + 1 sehingga 2 2 ⋅⋅⋅⋅⋅⋅⋅⋅⋅⋅⋅⋅⋅⋅⋅⋅⋅⋅⋅⋅⋅ (2) 2a 2 + 6a + 1 > 1 2a2 + 6a + 1 > 1 2a (a + 3) > 0 Nilai a yang memenuhi adalah a < −3 at au a > 0 ⋅⋅⋅⋅⋅⋅⋅⋅⋅⋅⋅⋅⋅⋅⋅⋅⋅⋅⋅⋅⋅⋅⋅ (3) Karena 7 < 3 maka − 3 1 3 1 7 < 0. − 7 > −3 dan − + 2 2 2 2 Irisan dari ket iga penyelesaian unt uk a adalah a < −3 at au a > 0 ∴ Maka nilai a yang memenuhi adalah a < −3 atau a > 0 8. SMA Negeri 5 Bengkulu Eddy Hermanto, ST 58 Olimpiade Matematika Tk Provinsi 2003 Solusi Bagian Pert ama Misal PQ = QR = RS = PS = k ∠ACB = ∠ABC = ∠APQ = ∠AQP = ∠BPS = ∠CQR = 45o Maka BS = CR = k BP = CQ = k √2 Luas ∆ABC = k2 = ⎞⎛ ⎞ 1 1 1 1⎛ ( AB )( AC ) = ⎜⎜ k 2 + k 2 ⎟⎟⎜⎜ k 2 + k 2 ⎟⎟ = x 2 2 2 2⎝ ⎠⎝ ⎠ 4x 9 ∴ Luas persegi PQRS = 4x 9 9. Karena nilai t erkecil dadu = 1, maka n ≤ 6. * Unt uk n = 1 Peluang t erj adinya j umlah mat a dadu sama dengan 6 adalah * Unt uk n = 2 Kej adian j umlah mat a dadu sama dengan 6 adalah (1, 5), (2, 4), (3, 3), (4, 2), (5, 1) = 5 Peluang t erj adinya j umlah mat a dadu sama dengan 6 adalah * 10 10 5 1 10 < < < = 4 1296 216 36 6 6 Unt uk n = 5 Kej adian j umlah mat a dadu sama dengan 6 adalah (1, 1, 1, 1, 2), (1, 1, 1, 2, 1), (1, 1, 2, 1, 1), (1, 2, 1, 1, 1), (2, 1, 1, 1, 1) = 5 Peluang t erj adinya j umlah mat a dadu sama dengan 6 adalah * 10 5 1 10 < < = 3 216 36 6 6 Unt uk n = 4 Kej adian j umlah mat a dadu sama dengan 6 adalah (1, 1, 1, 3), (1, 1, 2, 2), (1, 1, 3, 1), (1, 2, 1, 2), (1, 2, 2, 1), (1, 3, 1, 1), (2, 1, 1, 2), (2, 1, 2, 1), (2, 2, 1, 1), (3, 1, 1, 1) = 10 Peluang t erj adinya j umlah mat a dadu sama dengan 6 adalah * 5 1 5 = < 2 36 6 6 Unt uk n = 3 Kej adian j umlah mat a dadu sama dengan 6 adalah (1, 1, 4), (1, 2, 3), (1, 3, 2), (1, 4, 1), (2, 1, 3), (2, 2, 2), (2, 3, 1), (3, 1, 2), (3, 2, 1), (4, 1, 1) = 10 Peluang t erj adinya j umlah mat a dadu sama dengan 6 adalah * 1 6 10 10 5 1 5 < < < < 5 1296 216 36 6 6 Unt uk n = 6 Kej adian j umlah mat a dadu sama dengan 6 adalah (1, 1, 1, 1, 1, 1) = 1 Peluang j umlah mat a dadu sama dengan 6 adalah ∴ Peluang t erbesar adalah j ika n = 1 SMA Negeri 5 Bengkulu 1 5 10 10 5 1 < < < < 5 < 6 1296 216 36 6 6 6 Eddy Hermanto, ST 59 Olimpiade Matematika Tk Provinsi 2003 Solusi Bagian Pert ama 10. Misal persamaan garis vert ikal t ersebut adalah x = k Luas ∆ABC = ½ (9 − 1)(1 − 0) = 4 Persamaan garis melalui (0, 0) dan (9, 1) adalah y = Unt uk x = k maka y = 1 x 9 1 k 9 Luas ∆ II = ½ Luas ∆ABC 1 k) = ½ ⋅ 4 9 ½ (9 − k)(1 − 9−k=±6 k = 3 (memenuhi) at au k = 15 (t idak memenuhi bahwa 0 ≤ k ≤ 9) ∴ Persamaan garis vert ikal t ersebut adalah x = 3 11. m 2 − 2003 = n2 m 2 − n2 = 2003 (m + n)(m − n) = 2003 2003 adalah bilangan prima sehingga persamaan dipenuhi hanya j ika m + n = 2003 dan m − n = 1 Sehingga m = 1002 dan n = 1001 ∴ mn = 1002 ⋅ 1001 = 1003002 12. 4 2 ( log x ) log ( log x ) log 2 1/ 2 2 2 2 log + 2 ( log x ) = 2 4 log ( log x ) = 2 2 1 log x ⋅ ⋅ 2 log x = 2 2 = 4 2 ( log x ) 2 + 3/ 2 =8 x = 24 ∴ x = 16 SMA Negeri 5 Bengkulu Eddy Hermanto, ST 60 Solusi Olimpiade Matematika Tk Provinsi 2003 Bagian Pert ama 13. Misalkan AP = a maka BP = 2a dan CP = 3a Dengan berpusat di B, t it ik P diput ar sej auh 90o menj adi t it ik P’ . maka ∆PBP’ adalah segit iga siku-siku sama kaki. ∠BPP’ = 45o dan PP’ = 2a√2 ∆BPC ≅ ∆AP’ B sehingga AP’ = 3a (AP’ ) 2 = (AP) 2 + (PP’ ) 2 − 2(AP)(PP’ )cos ∠APP’ (3a) 2 = (a) 2 + (2a√2) 2 − 2(a)(2a√2)cos ∠APP’ cos ∠APP’ = 0 ∠APP’ = 90o ∴ ∠APB = ∠APP’ + ∠BPP’ = 90o + 45o = 135 o 14. * Jika t erdapat sedikit nya sat u warna yang t idak ikut dicampur Salah sat u perbandingan yang menghasilkan warna adalah 0: 0: 1. Karena ada 3 warna, maka akan ada 3 warna yang dihasilkan dari perbandingan ini. Perbandingan 0: 0: 2, 0: 0: 3, 0: 0: 4, 0: 0: 5 akan menghasilkan warna yang sama dengan perbandingan 0: 0: 1. Perbandingan lainnya yang memenuhi adalah 0: 1: 1, 0: 1: 2, 0: 1: 3, 0: 1: 4, 0: 1: 5, 0: 2: 3, 0: 2: 5, 0: 3: 4, 0: 3: 5, 0: 4: 5. Banyaknya warna yang dihasilkan adalah 3 x 11 = 33. * Jika t erdapat sedikit nya sat u warna dengan t epat 1 kaleng warna t ersebut yang dicampur Kemungkinan perbandingannya adalah 1: 1: 1, 1: 1: 2, 1: 1: 3, 1: 1: 4, 1: 1: 5, 1: 2: 2, 1: 2: 3, 1: 2: 4, 1: 2: 5, 1: 3: 3, 1: 3: 4, 1: 3: 5, 1: 4: 4, 1: 4: 5, 1: 5: 5. Perbandingan 1: 1: 1 hanya ada 1 kemungkinan. Banyaknya warna yang dihasilkan adalah 1 + 3 x 14 = 43. * Jika t erdapat sedikit nya sat u warna dengan t epat 2 kaleng warna t ersebut yang dicampur Kemungkinan perbandingannya adalah 2: 2: 3, 2: 2: 5, 2: 3: 3, 2: 3: 4, 2: 3: 5, 2: 4: 5, 2: 5: 5. Perbandingan 2: 2: 2 akan menghasilkan warna yang sama dengan perbandingan 1: 1: 1. Hal yang hampir sama berhubungan dengan perbandingan 2: 2: 4 dan 2: 4: 4. Banyaknya warna yang dihasilkan adalah 3 x 7 = 21. * Jika t erdapat sedikit nya sat u warna dengan t epat 3 kaleng warna t ersebut yang dicampur Kemungkinan perbandingannya adalah 3: 3: 4, 3: 3: 5, 3: 4: 4, 3: 4: 5, 3: 5: 5. Banyaknya warna yang dihasilkan adalah 3 x 5 = 15. * Jika t erdapat sedikit nya sat u warna dengan t epat 4 kaleng warna t ersebut yang dicampur Kemungkinan perbandingannya adalah 4: 4: 5, 4: 5: 5. Banyaknya warna yang dihasilkan adalah 3 x 2 = 6. ∴ Banyaknya warna keseluruhan yang dihasilkan adalah 33 + 43 + 21 + 15 + 6 = 118. SMA Negeri 5 Bengkulu Eddy Hermanto, ST 61 Solusi Olimpiade Matematika Tk Provinsi 2003 Bagian Pert ama 15. Misal harga j ual masing-masing mobil = p Misal harga pembelian mobil pert ama = y 1 y1 + 0, 3y1 = p, maka y1 = 10 p 13 Misal harga pembelian mobil kedua = y 2 y2 − 0, 2y2 = p, maka y2 = 5 4 p Harga pembelian t ot al = y 1 + y2 = 5 10 105 p+ p= p 13 52 4 5 10 1 p− p= − p 13 52 4 1 p − 100 52 % Kerugian Pak Ot o = x 100 % = − 105 105 p 52 20 ∴ Kerugian Pak Ot o = − % 21 Selisih = 2p − 16. Banyaknya cara duduk masing-masing kelompok adalah sama dengan permut asi 4 obyek pada 4 t empat = 4P4 = 24. Posisi duduk kelompok ist eri dapat di sebelah kanan maupun di sebelah kiri kelompok suami. ∴ Banyaknya cara memberikan t empat duduk kepada mereka adalah = 2 ⋅ 24 ⋅ 24 = 1152 cara. 17. BC = 10 ⋅ 2πr = 20πr CA = OC ⋅ cot g 30o = r √3 AB = BC + CA = 20πr + r √3 ∴ Jarak dari B ke A = (20 π + √3)r SMA Negeri 5 Bengkulu Eddy Hermanto, ST 62 Olimpiade Matematika Tk Provinsi 2003 Solusi Bagian Pert ama P = 1 ⋅ 1! + 2 ⋅ 2! + 3 ⋅ 3! + ⋅⋅⋅ + 99 ⋅ 99! + 100 ⋅ 100! T = 2 ⋅ 1! + 3 ⋅ 2! + 4 ⋅ 3! + ⋅⋅⋅ + 100 ⋅ 99! + 101 ⋅ 100! = 2! + 3! + 4! + ⋅⋅⋅ + 100! + 101! T − P = (2 − 1) 1! + (3 − 2) 2! + (4 − 3) 3! + ⋅⋅⋅ + (100 − 99) 99! + (101 − 100) 100! T − P = 1! + 2! + 3! + ⋅⋅⋅ + 99! + 100! 2! + 3! + 4! + ⋅⋅⋅ + 100! + 101! − P = 1! + 2! + 3! + ⋅⋅⋅ + 99! + 100! P = 101! − 1! = 101! − 1 101! adalah bilangan yang habis dibagi 101, maka P = 101! − 1 = 101k + 101 − 1 = 101k + 100 ∴ 1 ⋅ 1! + 2 ⋅ 2! + 3 ⋅ 3! + ⋅⋅⋅ + 99 ⋅ 99! + 100 ⋅ 100! dibagi 101 akan bersisa 100 18. Misal 19. Misal panj ang AB = 10a + b OC = ½ AB = ½ (10a + b) Panj ang CD = 10b + a CH = ½ (10b + a) Dengan a dan b adalah bilangan bulat posit if dan 0 < a ≤ 9 , 0 < b ≤ 9 OH = (OC ) − (CH ) 2 2 1 (10a + b ) 2 − (10b + a ) 2 2 3 OH = 11(a + b )(a − b ) 2 OH = Karena OH adalah bilangan rasional dan a + b > a − b maka : a + b = 11k dan a − b = km 2 dengan k dan m adalah bilangan asli sebab a dan b asli. Karena a + b ≤ 18 maka 11k ≤ 18 sehingga nilai k yang memenuhi hanya j ika k = 1. Maka a − b = m 2. Karena a − b < 9 maka nilai m 2 yang mungkin hanya 1 at au 4. Jika a + b = 11 dan a − b = 4 maka t idak mungkin didapat a dan b asli. Jika a + b = 11 dan a − b = 1 maka nilai a dan b yang memenuhi hanya j ika a = 6 dan b = 5. ∴ Panj ang AB = 65 SMA Negeri 5 Bengkulu Eddy Hermanto, ST 63 Solusi Olimpiade Matematika Tk Provinsi 2003 Bagian Pert ama 20. Misal H = {1, 2, 3, ⋅⋅⋅, 9, 10} Alt ernat if 1 : * Banyaknya 2 bilangan berurut an dari himpunan H ada 9 yait u : (1, 2), (2, 3), (3, 4), ⋅⋅⋅, (9, 10) * Menent ukan 3 bilangan dari H yang 2 berurut an namun ket iganya t idak berurut an : Unt uk (1, 2) hanya ada sat u bilangan ket iga yang akan membuat ket iga bilangan t ersebut berurut an, yait u 3. Maka banyaknya cara 3 bilangan diambil dari himpunan H yang 2 bilangannya adalah (1, 2) namun bilangan ket iga bukan 3 ada 7, yait u : (1, 2, 4), (1, 2, 5), ⋅⋅⋅, (1, 2, 10). Banyaknya cara ini j uga sama dengan 2 bilangan di ant aranya adalah (9, 10) Unt uk (2, 3) ada dua bilangan ket iga yang akan membuat ket iga bilangan t ersebut berurut an, yait u 1 dan 4. Maka banyaknya cara 3 bilangan diambil dari himpunan H yang 2 bilangannya adalah (2, 3) namun bilangan ket iga bukan 1 at au 4 ada 6, yait u : (2, 3, 5), (2, 3, 6), ⋅⋅⋅, (2, 3, 10). Banyaknya cara ini j uga sama dengan 2 bilangan di ant aranya adalah (3, 4), (4, 5), ⋅⋅⋅, (8, 9). Banyaknya cara 3 bilangan diambil dari himpunan H yang 2 di ant aranya berurut an namun ket iga bilangan t ersebut t idak berurut an adalah = 2 ⋅ 7 + 7 ⋅ 6 = 56. * Banyaknya cara 3 bilangan diambil dari himpunan H yang ket iganya berurut an = 8, yait u : (1, 2, 3), (2, 3, 4), (3, 4, 5), ⋅⋅⋅, (7, 8, 9), (8, 9, 10). * Banyaknya cara 3 bilangan diambil dari himpunan H = 10C3 = 120. ∴ Banyaknya cara memilih 3 bilangan berbeda dari himpunan H sehingga t idak ada 2 bilangan berurut an = 120 − 56 − 8 = 56 . Alt ernat if 2 : Jika (a, b, c) adalah 3 bilangan dari H yang memenuhi bahwa t idak ada 2 bilangan di ant aranya yang berurut an maka (a, b − 1, c − 2) haruslah merupakan 3 bilangan yang berbeda dan merupakan elemen dari himpunan {1, 2, 3, ⋅⋅⋅, 7, 8}. Banyaknya cara memilih 3 bilangan dari himpunan {1, 2, 3, ⋅⋅⋅, 7, 8} adalah 8C3 = 56 ∴ Banyaknya cara memilih 3 bilangan berbeda dari himpunan H sehingga t idak ada 2 bilangan berurut an = 56 . SMA Negeri 5 Bengkulu Eddy Hermanto, ST 64 SELEKSI OLIMPIADE MATEMATIKA INDONESIA 2004 TINGKAT PROVINSI TAHUN 2003 Prestasi itu diraih bukan didapat !!! SOLUSI SOAL Bidang Mat emat ika Bagian Kedua Disusun oleh : Eddy Her mant o, ST 65 Olimpiade Matematika Tk Provinsi 2003 Solusi Bagian Kedua BAGIAN KEDUA 1. Pernyat aan-pernyat aan : a. Andi berkat a bahwa Beni adalah kancil b. Coki berkat a bahwa Doni adalah serigala c. Edo berkat a Andi bukan serigala d. Beni berkat a Coki bukan kancil e. Doni berkat a bahwa Edo dan Andi adalah binat ang berbeda ¾ Misalkan Andi adalah kancil. Berdasarkan (a) maka Beni adalah kancil. Berdasarkan (d) maka Coki adalah serigala. Berdasarkan (b) maka Doni adalah kancil. Berdasarkan (e) karena Andi kancil maka Edo adalah serigala. Berdasarkan (c) maka Andi adalah serigala. Pernyat aan ini kont radiksi dengan permisalan bahwa Andi adalah kancil. ¾ Misalkan Andi adalah serigala. Berdasarkan (a) Berdasarkan (d) Berdasarkan (b) Berdasarkan (e) Berdasarkan (c) adalah kancil. maka Beni adalah serigala. maka Coki adalah kancil. maka Doni adalah serigala. maka Edo dan Andi sej enis. Karena Andi serigala maka Edo j uga serigala. maka Andi adalah serigala yang berart i sesuai dengan permisalan bahwa Andi Yang t ermasuk kancil adalah Coki dan yang t ermasuk serigala adalah Andi, Beni, Doni dan Edo. ∴ Banyaknya serigala ada 4 2. Karena 2+ a 3+ b adalah bilangan rasional maka 2+ a 3+ b = p dengan a, b, p dan q adalah q bilangan asli dan q ≠ 0 sert a p dan q relat if prima. ( q 2 + q a = p 3 + p b , maka q 2 − p 3 ) = (p 2 b −q a ) 2 2q 2 + 3p 2 − 2 pq 6 = p 2b + q 2a − 2 pq ab Karena a, b, p dan q adalah bilangan asli maka 6 = ab. Pasangan (a, b) yang memenuhi adalah (1, 6) ; (2, 3) ; (3, 2) ; (6, 1). Subt it usikan keempat pasangan ini ke persamaan semula unt uk dicek apakah memenuhi bilangan rasional at au t idak. Set elah dicek maka pasangan (a, b) yang akan membuat persamaan semula merupakan bilangan rasional adalah (3, 2). 2+ a 3+ b = 2+ 3 3+ 2 =1 ∴ a = 3 dan b = 2 SMA Negeri 5 Bengkulu Eddy Hermanto, ST 66 Solusi Olimpiade Matematika Tk Provinsi 2003 Bagian Kedua 3. Karena bidang ADHE sej aj ar dengan BCGF dan bidang ABFE sej aj ar dengan bidang DCGH maka DP sej aj ar FQ dan FP sej aj ar DQ. (1)2 + ⎛⎜ 1 ⎞⎟ PF = DP = DQ = FQ = 2 = ⎝2⎠ PQ sej aj ar AC maka PQ = AC = 1 5 2 2 Al t ernat if 1 : Mencari sudut PFQ. Misal ∠PFQ = α (PQ) 2 = (PF) 2 + (FQ) 2 − 2(PF)(FQ) cos α 2= 5 5 ⎛1 ⎞⎛ 1 ⎞ 5 ⎟ cos α 5 ⎟⎜ + − 2⎜ 4 4 ⎝2 ⎠⎝ 2 ⎠ 2 1 cos α = sehingga sin α = 6 5 5 Luas segi empat DPFQ = (FP)(PD)sin α ⎞ ⎞⎛ 2 ⎞⎛ 1 ⎛1 5 ⎟⎟⎜⎜ 6 ⎟⎟ 5 ⎟⎟⎜⎜ ⎠ ⎠⎝ 5 ⎠⎝ 2 ⎝2 Luas segi empat DPFQ = ⎜⎜ ∴ Luas segi empat DPFQ = 1 6 2 Al t ernat if 2 : Karena PF = DP = DQ = FQ maka segiempat DPFQ adalah belah ket upat . Diagonal PQ = sedangkan diagonal DF adalah diagonal ruang maka FD = Luas segiempat DPFQ = ½ ⋅ PQ ⋅ FD = ½ ⋅ 2 ⋅ 2 3. 3 1 ∴ Luas segi empat DPFQ = 6 2 SMA Negeri 5 Bengkulu Eddy Hermanto, ST 67 Olimpiade Matematika Tk Provinsi 2003 Solusi Bagian Kedua 4. Rat aan Geomet ri ≤ Rat aan Arit mat ika n a 1 ⋅ a 2 ⋅ a 3 L a n −1 ⋅ a n ≤ a 1 + a 2 + a 3 + L + a n −1 + a n n Tanda kesamaan berlaku j ika a1 = a2 = a3 = ⋅⋅⋅ = an-1 = an. Maka : 999 1 ⋅ 2 ⋅ 3L 998 ⋅ 999 < 1 + 2 + 3 + L + 998 + 999 999 1 999 (1 + 999 ) ⋅ 999 2 999 999! < 500 999 999! < ∴ Terbukti bahwa 999! < 500 999 5. 7x − 3y2 + 21 = 0. Maka x = ( 3 y + 7 )( ) 7 y − 7 yang merupakan suat u persamaan parabola dengan puncak di ( −3, 0) dan t it ik pot ong dengan sumbu Y di (0, √7) dan (0, −√7). Tampak bahwa ada 2 daerah. Sat u daerah di at as sumbu X dan sat u daerah lagi di bawah sumbu X. Jarak AB = Jarak AC = (− 3 − 0)2 + (0 − 7 (− 3 − 0 )2 + (0 − (− ) 2 =4 7) ) 2 =4 Unt uk 0 ≤ y ≤ √7, t ampak bahwa j arak t erj auh 2 t it ik t erj adi j ika kedua t it ik t ersebut di A dan B dengan j arak AB = 4. Unt uk −√7 ≤ y ≤ 0, t ampak bahwa j arak t erj auh 2 t it ik t erj adi j ika kedua t it ik t ersebut di A dan C dengan j arak AC = 4. Karena ada 3 buah t it ik dan ada 2 daerah maka sesuai Pigeon Hole Principle (PHP) maka sedikit nya ada 2 t it ik dalam sat u daerah yait u memiliki ordinat 0 ≤ y ≤ √7 at au −√7 ≤ y ≤ 0. ∴ Dari penj elasan di atas dapat disimpulkan bahwa j ika 3 titik terletak pada daerah yang dibatasi oleh sumbu Y dan grafik persamaan 7x − 3y2 + 21 = 0, maka sedikitnya 2 titik di antara ketiga titik tersebut mempunyai j arak tidak lebih dari 4 satuan. SMA Negeri 5 Bengkulu Eddy Hermanto, ST 68 SELEKSI TIM OLIMPIADE MATEMATIKA INDONESIA 2004 OLIMPIADE SAINS NASIONAL 2003 BALIK PAPAN (KALIMANTAN TIMUR), 15 – 19 SEPTEMBER 2003 Bidang Mat emat ika Hari Pert ama Waktu : 180 Menit DEPARTEMEN PENDIDIKAN NASIONAL DIREKTORAT JENDERAL PENDIDIKAN DASAR DAN MENENGAH DIREKTORAT PENDIDIKAN MENENGAH UMUM TAHUN 2003 69 OLIMPIADE SAINS NASIONAL 2003 15 – 19 SEPTEMBER 2003 BALIK PAPAN, KALIMANTAN TIMUR BI DAN G : MATEMATI KA HARI PERTAMA WAKTU : 180 MENIT 1. Bukt ikan bahwa a9 − a habis dibagi 6, unt uk set iap bilangan bulat a. 2. Diberikan sebuah segiempat ABCD sebarang. Misalkan P, Q, R, S bert urut -t urut adalah t it ik-t it ik t engah AB, BC, CD, DA. Misalkan pula PR dan QS berpot ongan di O. Bukt ikan bahwa PO = OR dan QO = OS. 3. Tent ukan semua solusi bilangan real persamaan ⎣x 2⎦ + ⎡x 2⎤ = 2003. [ Catat an : Unt uk sebarang bilangan real α, not asi ⎣α⎦ menyat akan bilangan bulat t erbesar yang lebih kecil at au sama dengan α, sedangkan ⎡α⎤ menyat akan bilangan bulat t erkecil yang lebih besar at au sama dengan α. ] 4. Diberikan sebuah mat riks berukuran 19 x 19, yang set iap komponennya bernilai +1 at au −1. Misalkan pula b i adalah hasil kali semua komponen mat riks di baris ke-i, dan k j adalah hasil kali semua komponen mat riks di kolom ke-j . Bukt ikan bahwa b 1 + k 1 + b 2 + k 2 + ⋅⋅⋅ + b 19 + k 19 ≠ 0. 70 SELEKSI TIM OLIMPIADE MATEMATIKA INDONESIA 2004 OLIMPIADE SAINS NASIONAL 2003 BALIK PAPAN (KALIMANTAN TIMUR), 15 – 19 SEPTEMBER 2003 Bidang Mat emat ika Hari Kedua Waktu : 180 Menit DEPARTEMEN PENDIDIKAN NASIONAL DIREKTORAT JENDERAL PENDIDIKAN DASAR DAN MENENGAH DIREKTORAT PENDIDIKAN MENENGAH UMUM TAHUN 2003 71 OLIMPIADE SAINS NASIONAL 2003 15 – 19 SEPTEMBER 2003 BALIK PAPAN, KALIMANTAN TIMUR BI DAN G : MATEMATI KA HARI KEDUA WAKTU : 180 MENIT 5. Unt uk sebarang bilangan real a, b, c bukt ikan ket aksamaan 5a2 + 5b 2 + 5c2 ≥ 4ab + 4ac + 4bc dan t ent ukan kapan kesamaan berlaku. 6. Balairung sebuah ist ana berbent uk segi-6 berat uran dengan panj ang sisi 6 met er. Lant ai balairung t ersebut dit ut upi dengan ubin-ubin keramik berbent uk segit iga samasisi dengan panj ang sisi 50 cm. Set iap ubin keramik dibagi ke dalam 3 daerah segit iga yang kongruen, lihat gambar. Set iap daerah segit iga diberi sat u warna t ert ent u sehingga set iap ubin memiliki t iga warna berbeda. Raj a menginginkan agar t idak ada dua ubin yang memiliki pola warna sama. Paling sedikit berapa warna yang diperlukan ? 7. Misalkan k, m, n adalah bilangan-bilangan asli demikian, sehingga k > n > 1 dan f akt or persekut uan t erbesar k dan n sama dengan 1. Bukt ikan bahwa j ika k − n membagi k m − nm-1, maka k ≤ 2n − 1. 8. Diket ahui segit iga ABC siku-siku di C dengan panj ang sisi-sisinya merupakan bilangan bulat . Tent ukan panj ang sisi-sisi segit iga t ersebut j ika hasil kali dari dua sisi yang bukan sisi miring sama dengan t iga kali keliling segit iga. 72 SELEKSI TIM OLIMPIADE MATEMATIKA INDONESIA 2004 OLIMPIADE SAINS NASIONAL 2003 BALIK PAPAN (KALIMANTAN TIMUR), 15 – 19 SEPTEMBER 2003 Prestasi itu diraih bukan didapat !!! SOLUSI SOAL Bidang Mat emat ika Disusun oleh : Eddy Hermant o, ST 73 Solusi Olimpiade Sains Nasional 2003 Bidang : Mat emat ika 1. Alt ernat if 1 : a9 − a = a (a8 − 1) a9 − a = a (a4 − 1) (a4 + 1) a9 − a = a (a2 − 1) (a2 + 1) (a4 + 1) a9 − a = (a − 1) a (a + 1) (a2 + 1) (a4 + 1) Karena (a − 1) a (a + 1) adalah perkalian t iga bilangan bulat berurut an maka a9 − a habis dibagi 3! = 6. Alt ernat if 2 : Sebuah bilangan bulat past i akan memenuhi bahwa ia ganj il at au genap . * Jika a genap maka a9 adalah genap Maka a9 − a adalah selisih ant ara dua bilangan genap sehingga a9 − a genap * Jika a ganj il maka a9 adalah ganj il Maka a9 − a adalah selisih ant ara dua bilangan ganj il sehingga a9 − a genap Karena a9 − a genap maka berart i a9 − a habis dibagi 2. Akan dibukt ikan bahwa a9 − a j uga habis dibagi 3. Alt ernat if 2. a : Sebuah bilangan bulat akan memenuhi salah sat u bent uk dari 3k, 3k + 1 at au 3k + 2 * Jika a = 3k ≡ 0 (mod 3) a9 − a = 39k 9 − 3k = 3 (38k 9 − k) yang berart i a9 − a habis dibagi 3 Penulisan lain. a9 − a ≡ 09 − 0 (mod 3) ≡ 0 (mod 3) yang berart i a9 − a habis dibagi 3. * Jika a = 3k + 1 ≡ 1 (mod 3) a9 − a = (3k + 1) 9 − (3k + 1) = 39k 9 + 9C1 38k 8 + 9C2 37k 7 + ⋅⋅⋅ + 9C7 32k 2 + 9C8 3k + 1 − (3k + 1) a9 − a = 39k 9 + 9C1 38k 8 + 9C2 37k 7 + ⋅⋅⋅ + 9C7 32k 2 + 24k = 3p Penulisan lain. a9 − a ≡ 19 − 1 (mod 3) ≡ 0 (mod 3) yang berart i a9 − a habis dibagi 3. a9 − a habis dibagi 3 * Jika a = 3k + 2 ≡ −1 (mod 3) a9 − a = (3k + 2) 9 − (3k + 2) a9 − a = 39 k 9 + 9C1 38 21 k 8 + 9C2 37 22 k 7 + ⋅⋅⋅ + 9C7 32 27 k 2 + 9C8 3k 28 + 1 − (3k + 2) a9 − a = 39 k 9 + 9C1 38 21 k 8 + 9C2 37 22 k 7 + ⋅⋅⋅ + 9C7 32 27 k 2 + 9C8 3k 28 + 29 − (3k + 2) 8 8 9C8 3k 2 − 3k = 3k ( 9C8 ⋅ 2 − 1) yang berat i habis dibagi 3 29 − 2 = 512 − 2 = 510 habis dibagi 3. Penulisan lain. a9 − a ≡ ( −1) 9 − ( −1) (mod 3) ≡ 0 (mod 3) yang berart i a9 − a habis dibagi 3. a9 − a habis dibagi 3 Dapat disimpulkan bahwa a9 − a habis dibagi 3. Alt ernat if 2. b : Teorema Fermat : Unt uk a bilangan bulat dan p prima maka ap − a habis dibagi p. Penulisan dalam bent uk lain adalah ap − a ≡ 0 (mod p) at au bisa j uga ap ≡ a (mod p) Berdasarkan t eorema Fermat maka a3 − a habis dibagi 3 dan (a3) 3 − a3 j uga habis dibagi 3. Maka (a3) 3 − a3 + a3 − a harus habis dibagi 3. Karena (a3) 3 − a3 + a3 − a = a9 − a maka a9 − a habis dibagi 3. Karena 2 dan 3 relat if prima maka a9 − a habis dibagi 2 ⋅ 3 = 6 ∴ Terbukti bahwa a9 − a habis dibagi 6 untuk setiap bilangan bulat a SMA Negeri 5 Bengkulu Eddy Hermanto, ST 74 Olimpiade Sains Nasional 2003 Solusi Bidang : Mat emat ika 2. Alt ernat if 1 : Dengan cara vekt or : AB + BC + CD + DA = 0 ( PQ = PB + BQ = 0,5(AB ) + BC ) ) ( ( SR = SD + DR = 0,5 AD + DC = −0,5 DA + CD = 0,5 AB + BC ) Karena SR = PQ maka ruas gar i s SR dan PQ sej aj ar dan sama panj ang. ) ( = 0,5(BA + AD ) = −0,5(AB QR = QC + CR = 0,5 BC + CD PS = PA + AS ) ( + DA = 0,5 BC + CD ) Karena QR = PS maka ruas garis QR dan PS sej aj ar dan sama panj ang. Akibat nya segiempat PQRS adalah j aj aran genj ang. Karena ∠SOP = ∠QOR dan PS sej aj ar sert a sama panj ang dengan QR maka ∆SOP kongruen dengan ∆QOR yang berakibat QO = OS dan PO = OR Alt ernat if 2 : Pada ∆ABC dan ∆PBQ berlaku ∠ABC = ∠PBQ sert a ∆PBQ sebangun. Maka AC sej aj ar PQ dan AC = 2. PQ Pada ∆ADC dan ∆SDR berlaku ∠ADC = ∠SDR sert a ∆SDR sebangun. Maka AC sej aj ar SR dan CB AB = 2 yang berart i ∆ABC dan = 2 dan QB PA AD CD = 2 dan = 2 yang berart i ∆ADC dan SD RD AC = 2 sehingga SR sej aj ar PQ dan SR = PQ. SR Karena SR ⁄⁄ PQ maka ∠SRP = ∠QPR dan ∠RSQ = ∠PQS dan karena SR = PQ maka ∆SOP kongruen dengan ∆QOR yang berakibat QO = OS dan PO = OR Alt ernat if 3 : PQRS adalah sebuah j aj aran genj ang (Varignon Parallelogram) Menurut sif at j aj aran genj ang, diagonalnya saling membagi dua sama panj ang. ∴ Terbukti bahwa PO = OR dan QO = OS SMA Negeri 5 Bengkulu Eddy Hermanto, ST 75 Solusi Olimpiade Sains Nasional 2003 Bidang : Mat emat ika 3. * Unt uk x 2 ≤ 1001 maka ⎣x 2⎦ ≤ 1001 dan ⎡x 2⎤ ≤ 1001 sehingga ⎣x 2⎦ + ⎡x 2⎤ ≤ 2002 * Unt uk x 2 ≥ 1002 maka ⎣x 2⎦ ≥ 1002 dan ⎡x 2⎤ ≥ 1002 sehingga ⎣x 2⎦ + ⎡x 2⎤ ≥ 2004 * Unt uk 1001 < x 2 < 1002 maka ⎣x 2⎦ = 1001 dan ⎡x 2 ⎤ = 1002 sehingga ⎣x 2⎦ + ⎡x 2⎤ = 2003 Maka persamaan ⎣x 2⎦ + ⎡x 2⎤ = 2003 hanya dipenuhi oleh 1001 < x 2 < 1002 ∴ ⎣x 2⎦ + ⎡x 2⎤ = 2003 hanya dipenuhi oleh 1001 < x < 1002 at au − 1002 < x < − 1001 4. Karena komponen-komponen mat riks bernilai +1 at au −1 maka b i dan k i masing-masing j uga akan bernilai +1 at au −1. Alt ernat if 1 : Andaikan bahwa b 1 + k 1 + b 2 + k 2 + ⋅⋅⋅ + b 19 + k 19 = 0 Maka harus ada 19 di ant ara b 1, k 1, b 2, k 2, ⋅⋅⋅ , b 19, k 19 yang bernilai −1. Kemungkinannya adalah : • Ada p genap di ant ara b 1, b 2, b 3, ⋅⋅⋅ , b 19 yang bernilai −1 dan ada (19 − p) ganj il di ant ara k 1, k 2, k 3, ⋅⋅⋅, k 19 yang bernilai −1. Berdasarkan f akt a bahwa ada p genap di ant ara b 1, b 2, b 3, ⋅⋅⋅ , b 19 yang bernilai −1 maka di ant ara 19 x 19 komponen harus ada komponen bert anda −1 sebanyak (ganj il ⋅ p + genap ⋅ ganj il) = genap Berdasarkan f akt a bahwa ada (19 − p) ganj il di ant ara k 1, k 2, k 3, ⋅⋅⋅ , k 19 yang bernilai −1 maka di ant ara 19 x 19 komponen harus ada sebanyak (ganj il ⋅ (19 − p) + genap ⋅ genap) = ganj il komponen bert anda −1. Hal ini bert ent angan dengan kenyat aan sebelumnya bahwa komponen bert anda −1 harus ada sebanyak genap. Maka t idak mungkin bahwa ada p genap di ant ara b 1, b 2, b 3, ⋅⋅⋅ , b 19 yang bernilai −1 dan ada (19 − p) ganj il di ant ara k 1, k 2, k 3, ⋅⋅⋅ , k 19 yang bernilai −1. • Ada q ganj il di ant ara b 1, b 2, b 3, ⋅⋅⋅ , b 19 yang bernilai −1 dan ada (19 − q) genap di ant ara k 1, k 2, k 3, ⋅⋅⋅, k 19 yang bernilai −1. Berdasarkan f akt a bahwa ada q ganj il di ant ara b 1, b 2, b 3, ⋅⋅⋅ , b 19 yang bernilai −1 maka di ant ara 19 x 19 komponen harus ada komponen bert anda −1 sebanyak (ganj il ⋅ q + genap ⋅ genap) = ganj il Berdasarkan f akt a bahwa ada (19 − q) genap di ant ara k 1, k 2, k 3, ⋅⋅⋅ , k 19 yang bernilai −1 maka di ant ara 19 x 19 komponen harus ada sebanyak (ganj il ⋅ (19 − q) + genap ⋅ ganj il) = genap komponen bert anda −1. Hal ini bert ent angan dengan kenyat aan sebelumnya bahwa komponen bert anda −1 harus ada sebanyak ganj il. Maka t idak mungkin bahwa ada q ganj il di ant ara b 1, b 2, b 3, ⋅⋅⋅ , b 19 yang bernilai −1 dan ada (19 − q) genap di ant ara k 1, k 2, k 3, ⋅⋅⋅ , k 19 yang bernilai −1. Alt ernat if 2 : Pada mat riks berlaku b 1b 2b 3⋅⋅⋅b 19 = k 1k 2k 3⋅⋅⋅k 19 ⋅⋅⋅⋅⋅⋅⋅⋅⋅⋅⋅⋅⋅⋅⋅⋅⋅⋅⋅⋅⋅⋅⋅ (1) Andaikan bahwa b 1 + k 1 + b 2 + k 2 + ⋅⋅⋅ + b 19 + k 19 = 0 ⋅⋅⋅⋅⋅⋅⋅⋅⋅⋅⋅⋅⋅⋅⋅⋅⋅⋅⋅⋅⋅⋅⋅⋅⋅⋅⋅⋅⋅⋅⋅⋅⋅ (2) Karena b i dan k j bernilai +1 at au −1 Maka harus ada 19 di ant ara b 1, k 1, b 2, k 2, ⋅⋅⋅ , b 19, k 19 yang bernilai −1. Jika di ant ara b i ada t erdapat sebanyak n buah yang bert anda −1 maka harus ada sebanyak (19−n) di ant ara k i yang bert anda −1. Tet api n dan (19−n) berbeda parit asnya (salah sat unya ganj il dan sat unya lagi genap), sehingga persamaan (1) t idak mungkin dapat dipenuhi. Akibat nya t idak mungkin b 1 + k 1 + b 2 + k 2 + ⋅⋅⋅ + b 19 + k 19 = 0 ∴ Terbukti b1 + k1 + b2 + k2 + ⋅⋅⋅ + b19 + k19 ≠ 0 SMA Negeri 5 Bengkulu Eddy Hermanto, ST 76 Solusi Olimpiade Sains Nasional 2003 Bidang : Mat emat ika 5. Alt ernat if 1 : (a − b) 2 ≥ 0. Maka a2 + b 2 ≥ 2ab Pert idaksamaan di at as dapat diperoleh pula dari pert idaksamaan AM-GM a2 +b2 ≥ a 2b 2 sehingga a2 + b 2 ≥ 2ab ⋅⋅⋅⋅⋅⋅⋅⋅⋅⋅⋅⋅⋅⋅⋅⋅⋅⋅⋅⋅⋅⋅⋅⋅⋅⋅⋅⋅⋅⋅⋅⋅⋅⋅⋅⋅⋅⋅⋅⋅⋅⋅⋅ (1) 2 Kesamaan t erj adi bila a = b Berdasarkan persamaan (1) didapat : a2 + c 2 ≥ 2ac ⋅⋅⋅⋅⋅⋅⋅⋅⋅⋅⋅⋅⋅⋅⋅⋅⋅⋅⋅⋅⋅⋅⋅⋅⋅⋅⋅⋅⋅⋅⋅⋅⋅⋅⋅⋅⋅⋅⋅⋅⋅⋅⋅ (2) b 2 + c 2 ≥ 2bc ⋅⋅⋅⋅⋅⋅⋅⋅⋅⋅⋅⋅⋅⋅⋅⋅⋅⋅⋅⋅⋅⋅⋅⋅⋅⋅⋅⋅⋅⋅⋅⋅⋅⋅⋅⋅⋅⋅⋅⋅⋅⋅⋅ (3) Jumlahkan persamaan (1), (2) dan (3) a2 + b 2 + c 2 ≥ ab + ac + bc sehingga 4a2 + 4b 2 + 4c 2 ≥ 4ab + 4ac + 4bc ⋅⋅⋅⋅⋅⋅⋅⋅⋅⋅⋅⋅⋅⋅⋅⋅⋅ (4) Bilangan kuadrat bernilai ≥ 0 maka : a2 + b 2 + c 2 ≥ 0 ⋅⋅⋅⋅⋅⋅⋅⋅⋅⋅⋅⋅⋅⋅⋅⋅⋅⋅⋅⋅⋅⋅⋅⋅⋅⋅⋅⋅⋅⋅⋅⋅⋅⋅⋅⋅⋅⋅⋅⋅⋅⋅⋅⋅⋅⋅ (5) Kesamaan t erj adi hanya j ika a = 0, b = 0 dan c = 0 Jumlahkan persamaan (4) + (5) sehingga 5a2 + 5b 2 + 5c 2 ≥ 4ab + 4ac + 4bc Kesamaan t erj adi hanya j ika a = b = c = 0 ∴ Terbukti bahwa 5a2 + 5b2 + 5c2 ≥ 4ab + 4ac + 4bc Alt ernat if 2 : (2a − b) 2 ≥ 0 Tanda kesamaan t erj adi j ika 2a = b. 4a2 + b 2 ≥ 4ab ⋅⋅⋅⋅⋅⋅⋅⋅⋅⋅⋅⋅⋅⋅⋅⋅⋅⋅⋅⋅⋅ (6) Dengan cara yang sama didapat 4b 2 + c 2 ≥ 4bc ⋅⋅⋅⋅⋅⋅⋅⋅⋅⋅⋅⋅⋅⋅⋅⋅⋅⋅⋅⋅⋅ (7) 4c 2 + a2 ≥ 4ac ⋅⋅⋅⋅⋅⋅⋅⋅⋅⋅⋅⋅⋅⋅⋅⋅⋅⋅⋅⋅⋅ (8) Tambahkan persamaan (6), (7) dan (8) didapat 5a2 + 5b 2 + 5c 2 ≥ 4ab + 4ac + 4bc Tanda kesamaan t erj adi j ika 2a = b, 2b = c dan 2c = a yang t erpenuhi hanya j ika a = b = c = 0 ∴ Terbukti bahwa 5a2 + 5b2 + 5c2 ≥ 4ab + 4ac + 4bc 6. Segienam berat uran dibuat dari 6 buah segit iga sama sisi yang kongruen. Luas lant ai balairung = 6 ⋅ ½ ⋅ 6 ⋅ 6 ⋅ sin 60o = 108 sin 60o Luas 1 buah ubin = ½ ⋅ 0, 5 ⋅ 0, 5 ⋅ sin 60o Luas lant ai balairung : Luas 1 buah ubin = 8 ⋅ 108 = 864 Maka unt uk menut upi lant ai balairung dibut uhkan 864 buah ubin Jika ada n buah warna maka banyaknya pola yang dapat dibuat = ( nC3) ⋅ (3 − 1)! = n ( n − 1)( n − 2) 3 n ( n − 1)( n − 2) ≥ 864 3 n (n − 1) (n − 2) ≥ 2592 Unt uk n = 14 maka n (n − 1) (n − 2) = 2184 < 2592 Unt uk n = 15 maka n (n − 1) (n − 2) = 2730 > 2592 ∴ Banyaknya warna minimum yang diperlukan adalah 15 buah SMA Negeri 5 Bengkulu Eddy Hermanto, ST 77 Olimpiade Sains Nasional 2003 Solusi Bidang : Mat emat ika 7. k − n ⏐ k m − nm-1 k − n ⏐ k m − nm + nm − n m-1 k − n ⏐ k m − nm + nm-1 (n − 1) Unt uk m ∈ bilangan asli maka k − n membagi k m − nm . Karena FPB (k, n) = 1 maka FPB(k − n, nm-1) = 1. Akibat nya k − n harus membagi n − 1. Karena k − n membagi n − 1 maka k − n ≤ n − 1 k ≤ 2n − 1 ∴ Terbukti bahwa k ≤ 2n − 1 8. Misalkan sisi-sisi segit iga t ersebut adalah a, b dan c dengan c adalah sisi miring, maka : b = c 2 −a2 ⋅⋅⋅⋅⋅⋅⋅⋅⋅⋅⋅⋅⋅⋅⋅⋅⋅⋅⋅⋅⋅⋅⋅⋅⋅⋅⋅⋅⋅⋅⋅ (1) ab = 3(a + b + c) ⋅⋅⋅⋅⋅⋅⋅⋅⋅⋅⋅⋅⋅⋅⋅⋅⋅⋅⋅⋅⋅⋅⋅⋅⋅⋅⋅⋅⋅ (2) Karena a, b dan c adalah bilangan bulat maka sekurang-kurangnya salah sat u di ant ara a at au b adalah kelipat an 3. Tanpa mengurangi keumuman misalkan a = 3k dengan k ∈ bilangan asli (sama saj a j ika dimisalkan b = 3k) maka : 3k c k c 2 2 − 9k − 9k 2 2 = 3(3k + = (3k + c 2 − 9k c − 9k 2 2 2 + c) + c) (k − 1) c − 9k = c + 3k (k − 1) (c + 3k )(c − 3k ) = (c + 3k ) (k − 1) (c − 3k ) = (c + 3k ) (k − 1) (c − 3k ) = c − 3k + 6k (c − 3k )(k − 2k ) = 6k Karena k ≠ 0 maka (c − 3k )(k − 2 ) = 6 2 2 2 2 2 2 2 ⋅⋅⋅⋅⋅⋅⋅⋅⋅⋅⋅⋅⋅⋅⋅⋅⋅⋅⋅⋅⋅⋅⋅⋅⋅⋅⋅⋅⋅⋅⋅⋅⋅⋅⋅⋅⋅⋅⋅⋅ (3) Karena c, k ∈ bilangan asli maka (k − 2) past i membagi 6 dan karena c > 3k maka (k − 2) > 0 Nilai k yang memenuhi adalah 3; 4; 5; 8 c = 3k + 6 k −2 ⋅⋅⋅⋅⋅⋅⋅⋅⋅⋅⋅⋅⋅⋅⋅⋅⋅⋅⋅⋅⋅⋅⋅⋅⋅⋅⋅⋅⋅⋅⋅⋅⋅⋅⋅⋅⋅⋅⋅⋅⋅⋅⋅⋅⋅⋅⋅⋅⋅ (4) Unt uk k = 3 maka a = 9 sehingga c = 15 dan b = 12 ⋅⋅⋅⋅⋅⋅⋅⋅⋅⋅⋅⋅⋅⋅⋅⋅⋅⋅⋅⋅⋅⋅⋅⋅⋅ Unt uk k = 4 maka a = 12 sehingga c = 15 dan b = 9 ⋅⋅⋅⋅⋅⋅⋅⋅⋅⋅⋅⋅⋅⋅⋅⋅⋅⋅⋅⋅⋅⋅⋅⋅⋅ Unt uk k = 5 maka a = 15 sehingga c = 17 dan b = 8 ⋅⋅⋅⋅⋅⋅⋅⋅⋅⋅⋅⋅⋅⋅⋅⋅⋅⋅⋅⋅⋅⋅⋅⋅⋅ Unt uk k = 8 maka a = 24 sehingga c = 25 dan b = 7 ⋅⋅⋅⋅⋅⋅⋅⋅⋅⋅⋅⋅⋅⋅⋅⋅⋅⋅⋅⋅⋅⋅⋅⋅⋅ Subt it usikan persamaan (5), (6), (7), (8) ke persamaan (2) yang t ernyat a semuanya ∴ Panj ang sisi-sisi segitiga yang memenuhi adalah : * a=9 b = 12 c = 15 * a = 12 b=9 c = 15 * a=8 b = 15 c = 17 * a = 15 b=8 c = 17 * a=7 b = 24 c = 25 * a = 24 b=7 c = 25 SMA Negeri 5 Bengkulu (5) (6) (7) (8) memenuhi. Eddy Hermanto, ST 78 SELEKSI OLIMPIADE TINGKAT KABUPATEN/ KOTA TAHUN 2004 TIM OLIMPIADE MATEMATIKA INDONESIA TAHUN 2005 Bidang Mat emat ika Waktu : 90 Menit DEPARTEMEN PENDIDIKAN NASIONAL DIREKTORAT JENDERAL PENDIDIKAN DASAR DAN MENENGAH DIREKTORAT PENDIDIKAN MENENGAH UMUM TAHUN 2004 79 OLIMPIADE MATEMATIKA TINGKAT KABUPATEN/ KOTA TAHUN 2004 Bagian Pertama Pilih sat u j awaban yang benar. Dalam hal t erdapat lebih dari sat u j awaban yang benar, pilih j awaban yang paling baik. 1. Jika a dan b adalah bilangan real yang memenuhi a + b = 3 dan a2 + ab = 7, maka a adalah A. 3/ 7 B. 5/ 7 C. 3/ 4 D. 7/ 5 E. 7/ 3 2. Bilangan 2004 memiliki f akt or selain 1 dan 2004 sendiri sebanyak A. 3 B. 4 C. 6 D. 10 E. 12 3. Misalkan k bilangan bulat . Nilai 4k+1 x 5k−1 sama dengan A. 4 x 20k 5 B. 4 x 202k 5 C. 16 x 20k-1 D. 202k E. 20 k 2 −1 4. Unt uk a dan b bilangan bulat dengan a ≠ 0, not asi a⏐b menyat akan “ a membagi b” . Pernyat aan berikut yang sal ah adalah A. Jika a⏐b dan a⏐c, maka maka a⏐(bc) B. Jika a⏐c dan b ⏐c, maka (ab) ⏐c C. Jika a⏐b dan a⏐c, maka a⏐(b + c) D. Unt uk set iap bilangan bulat a ≠ 0 berlaku a⏐0 E. Jika a⏐b, maka a⏐(bc), unt uk set iap bilangan bulat c. 5. Di suat u hot el, rat a-rat a 96% kamar t erpakai sepanj ang sebulan liburan kenaikan kelas dan rat a-rat a 72% kamar t erpakai sepanj ang sebelas bulan lainnya. Maka rat a-rat a pemakaian kamar sepanj ang t ahun di hot el t ersebut adalah A. 70% B. 74% C. 75% D. 80% E. 84% 6. Dalam ket idaksamaan berikut , besar sudut dinyat akan dalam radian. Ket idaksamaan yang benar adalah A. sin 1 < sin 2 < sin 3 C. sin 1 < sin 3 < sin 2 E. sin 3 < sin 1 < sin 2 B. sin 3 < sin 2 < sin 1 D. sin 2 < sin 1 < sin 3 7. Sebuah kot ak berisi 6 bola merah dan 6 bola put ih. Secara acak diambil dua bola sekaligus. Peluang unt uk mendapat kan dua bola berwarna sama adalah A. 5 12 B. 5 11 C. 1 2 D. 5 9 E. 5 7 8. Segit iga dengan panj ang sisi 6 dan 8 memiliki luas t erbesar j ika sisi ket iganya memiliki panj ang A. 6 B. 8 C. 10 D. 12 E. 15 80 9. Pada sebuah segi6 berat uran, rasio panj ang ant ara diagonal t erpendek t erhadap diagonal t erpanj ang adalah A. 1 : 3 B. 1 : 2 C. 1 : 3 D. 2 : 3 E. 3 :2 10. Nomor polisi mobil-mobil di suat u negara selalu t erdiri dari 4 angka. Jika j umlah keempat angka pada set iap nomor j uga harus genap, mobil yang bisa t erdaf t ar di negara it u paling banyak ada A. 600 B. 1800 C. 2000 D. 4500 E. 5000 Bagian Kedua Isikan hanya j awaban saj a pada t empat yang disediakan 11. Jika x 2 4 x z = ⋅⋅⋅⋅ dan maka = = 3 5 y y z 12. Jika 2004 dibagi ke dalam t iga bagian dengan perbandingan 2 : 3 : 5, maka bagian t erkecil adalah ⋅⋅⋅⋅⋅ 13. Unt uk dua bilangan bulat a dan b, penulisan a * b menyat akan sisa t ak negat if ab j ika dibagi 5. Nilai ( −3) * 4 = ⋅⋅⋅⋅ 14. Jika luas segit iga ABC sama dengan kelilingnya, maka j ari-j ari lingkaran dalam segit iga ABC adalah ⋅⋅⋅⋅ 15. Agar bilangan 20 + 21 + 22 + ⋅⋅⋅ + 2n sedekat mungkin kepada 2004, haruslah n = ⋅⋅⋅⋅ 16. Jika log p + log q = log (p + q), maka p dinyat akan dalam q adalah p = ⋅⋅⋅⋅ 17. Luas sebuah segit iga siku-siku adalah 5. Panj ang sisi miring segit iga ini adalah 5. Maka keliling segit iga t ersebut adalah ⋅⋅⋅⋅ 18. Jika x dan y dua bilangan asli dan x + y + xy = 34, maka nilai x + y = ⋅⋅⋅⋅⋅ 19. Sepuluh t im mengikut i t urnamen sepakbola. Set iap t im bert emu sat u kali dengan set iap t im lainnya. Pemenang set iap pert andingan memperoleh nilai 3, sedangkan yang kalah memperoleh nilai 0. Unt uk pert andingan yang berakhir seri, kedua t im memperoleh nilai masing-masing 1. Di akhir t urnamen, j umlah nilai seluruh t im adalah 124. Banyaknya pert andingan yang berakhir seri adalah ⋅⋅⋅⋅⋅ 20. Delegasi Indonesia ke suat u pert emuan pemuda int ernasional t erdiri dari 5 orang. Ada 7 orang pria dan 5 orang wanit a yang mencalonkan diri unt uk menj adi anggot a delegasi. Jika dipersyarat kan bahwa paling sedikit seorang anggot a it u harus wanit a, banyaknya cara memilih anggot a delegasi adalah ⋅⋅⋅⋅ 81 SELEKSI OLIMPIADE TINGKAT KABUPATEN/ KOTA 2004 TIM OLIMPIADE MATEMATIKA INDONESIA 2005 Prestasi itu diraih bukan didapat !!! SOLUSI SOAL Bidang Mat emat ika Disusun oleh : Eddy Hermant o, ST 82 Olimpiade Matematika Tk Kabupaten/ Kota 2004 BAGIAN PERTAMA 1. (Jawaban : E) a + b = 3 dan a2 + ab = 7, maka a(a + b) = 7 sehingga a(3) = 7 ∴ a= 7 3 2. (Jawaban : D) 2004 = 22 ⋅ 501 = 22 ⋅ 3 ⋅ 167 dan 167 adalah bilangan prima. Maka banyaknya f akt or posit if dari 2004 t ermasuk 1 dan 2004 = (2 +1)(1 + 1)(1 + 1) = 12 Banyaknya f akt or 2004 selain 1 dan 2004 adalah = 12 − 2 = 10 Fakt or dari 2004 selain 1 dan 2004 adalah : 2, 3, 4, 6, 12, 167, 334, 501, 668, 1002. Banyaknya f akt or ada 10 ∴ Banyaknya f akt or ada 10 3. (Jawaban : A at au C) 4k+1 x 5k-1 = 4 x 4k x 5k 4 x 20k at au = 5 5 4k+1 x 5k-1 = 16 x 4k-1 x 5k-1 = 16 x 20k-1 ∴ 4k+1 x 5k-1 sama dengan 4 x 20 k atau 16 x 20 k-1 5 Cat at an : Jawaban yang dikirimkan dari panit ia pusat menyat akan hanya A saj a yang benar. Namun dalam hit ungan t ernyat a C j uga bernilai sama. 4. (Jawaban : B) - A benar karena j ika a⏐b maka a⏐(bc) - B salah karena yang benar adalah j ika a⏐c dan b ⏐c, maka (ab) ⏐c2 - C benar - D benar - E benar sesuai dengan A ∴ Pernyat aan yang salah adalah B 5. (Jawaban : B) Rat a-rat a % pemakaian kamar set ahun = 1 ⋅ 96% + 11 ⋅ 72% = 74 % 1 + 11 ∴ Rat a-rat a pemakaian kamar sepanj ang t ahun di hot el t ersebut adalah 74 % 6. (Jawaban : E) 1 rad ≈ 57, 3o sehingga 2 rad ≈ 114, 6o dan 3 rad ≈ 171, 9o sin 114, 6o = sin (180 − 114, 6) o = sin 65, 4o sin 171, 9o = sin (180 − 171, 9) o = sin 8, 1o SMA Negeri 5 Bengkulu Eddy Hermanto, ST 83 Olimpiade Matematika Tk Kabupaten/ Kota 2004 Unt uk 0 ≤ x ≤ 90o berlaku bahwa sin x 1 < sin x 2 j ika x 1 < x 2 ∴ Ket idaksamaan yang benar adalah sin 3 < sin 1 < sin 2 Cat at an : Jawaban yang dikirimkan dari panit ia pusat menyat akan bahwa j awaban yang benar adalah B, namun bisa dibukt ikan bahwa seharusnya j awaban yang benar adalah E. Jawaban soal ini j uga bisa dibukt ikan dengan hit ungan dengan alat hit ung berupa kalkulat or at au komput er. 7. (Jawaban : B) 2 bola berwarna sama bisa didapat dari keduanya berwarna merah at au keduanya berwarna put ih. P(A) = 6 C 2 ⋅6 C 0 C ⋅ C 30 5 + 6 0 6 2 = = 66 11 12 C 2 12 C 2 ∴ Peluang unt uk mendapat kan dua bola berwarna sama adalah 5 11 8. (Jawaban : C) Misal segit iga t ersebut adalah segit iga ABC. Luas segit iga = ½ ab sin C Karena a dan b bernilai konst an, maka luas segit iga akan maksimum j ika sin C bernilai maksimum. Maksimum sin C = 1 unt uk C = 90o yang berart i segit iga ABC siku-siku di C. c= 6 2 + 8 2 = 10 ∴ Panj ang sisi ket iga agar segit iga t ersebut memiliki luas t erbesar adalah 10 . 9. (Jawaban : E) Misal sisi segi-6 berat uran t ersebut adalah a dan O adalah pusat segi-6 berat uran. Karena bangun adalah segi-6 berat uran maka berlaku : OA = OB = OC = OD = OE = OF = AB = BC = CD = DE = EF = AF = a ∠AFO = ∠OFE = 60o (AE) 2 = (AF) 2 + (FE) 2 − 2(AF)(FE) cos 120o (AE) 2 = a2 + a2 − 2 ⋅ a ⋅ a ⋅ ( −½) (AE) = a 3 (AD) = (AO) + (OD) = a + a = 2a (AE) : (AD) = 3 :2 ∴ Rasio panj ang diagonal t erpendek t erhadap diagonal t erpanj ang adalah SMA Negeri 5 Bengkulu 3 :2 Eddy Hermanto, ST 84 Olimpiade Matematika Tk Kabupaten/ Kota 2004 10. (Jawaban : D) Unt uk plat angka pert ama t idak boleh 0. Agar j umlah keempat angka t ersebut genap, maka keempat angka t ersebut harus genap at au keempat nya harus ganj il at au 2 genap dan 2 ganj il. • Jika keempat angka t ersebut genap maka banyaknya plat = 4 x 5 x 5 x 5 = 500 • Jika keempat angka t ersebut ganj il maka banyaknya plat = 5 x 5 x 5 x 5 = 625 • Jika keempat angak t ersebut t erdiri dari 2 genap dan 2 ganj il Misal angka genap = p dan angka ganj il = j Banyaknya susunan angka genap dan ganj il ada 4! = 6, yait u : ppj j , pj pj , pj j p, j j pp, j pj p, 2!⋅2! j ppj . Unt uk susunan ppj j , pj pj , pj j p, banyaknya plat unt uk masing-masing susunan = 4x5x5x5 = 500. Unt uk susunan j j pp, j pj p, j ppj , banyaknya plat unt uk masing-masing susunan = 5x5x5x5 = 625. ∴ Mobil yang bisa t erdaf t ar di negara it u paling banyak = 500 + 625 + 3(500) + 3(625) = 4500. BAGIAN KEDUA 11. x x z 2 4 5 = : = : = 3 5 6 z y y x 5 ∴ = z 6 12. Bagian yang t erkecil = 4008 2 ⋅ 2004 = 10 2+3+5 ∴ Bagian yang t erkecil adalah 400, 8 13. ( −3) ⋅ 4 = −12 = ( −3) ⋅ 5 + 3 Maka : −12 dibagi 5 akan bersisa 3 ∴ ( −3) * 4 = 3 14. Misal j ari-j ari lingkaran dalam sama dengan r dan ket iga sisinya adalah a, b dan c, maka : Luas segit iga = ½ r (a + b + c) Luas segit iga = ½ r ⋅ Keliling segit iga Karena Luas segit iga sama dengan Keliling segit iga maka r = 2 ∴ Jari-j ari lingkaran dalam segit iga ABC adalah 2 15. 2o + 21 + 22 + ⋅⋅⋅ + 2n = 2 o (2 n +1 − 1) = 2n+1 − 1 2 −1 Diinginkan 2n+1 − 1 sedekat mungkin ke 2004 sedangkan 210 = 1024 dan 211 = 2048, maka n = 10 ∴ n = 10 SMA Negeri 5 Bengkulu Eddy Hermanto, ST 85 Olimpiade Matematika Tk Kabupaten/ Kota 2004 16. log p + log q = log (p + q) log (pq) = log (p + q) pq = p + q p(q − 1) = q ∴ p= q q −1 17. Misal sisi siku-siku segit iga t ersebut adalah a dan b. Luas segit iga = ½ ab = 5 ab = 10 ⋅⋅⋅⋅⋅⋅⋅⋅⋅⋅⋅⋅⋅ (1) dan a2 + b 2 = 52 = 25 ⋅⋅⋅⋅⋅⋅⋅⋅⋅⋅⋅⋅⋅ (2) (a + b) 2 − 2ab = 25 (a + b) 2 − 2⋅ 10 = 25 sehingga a + b = Keliling segit iga = 5 + a + b 45 = 3 5 ∴ Keliling set iga t ersebut = 5 + 3 5 18. x + y + xy = 34 (x + 1) (y + 1) = 34 + 1 = 35 = 5 ⋅ 7 Karena x dan y bilangan asli maka persamaan hanya dipenuhi j ika x + 1 = 5 dan y + 1 = 7 at au x + 1 = 7 dan y + 1 = 5. Akibat nya x = 4 dan y = 6 at au x = 6 dan y = 4 x + y = 4 + 6 = 6 + 4 = 10 ∴ x + y = 10 19. Jika dalam pert andingan ada salah sat u yang menang maka nilai t ot al kedua t im = 3. Jika dalam pert andingan berakhir seri maka nilai t ot al kedua t im = 1 + 1 = 2 at au ada 1 nilai yang hilang per pert andingan yang berakhir seri. Banyaknya pert andingan keseluruhan = 10C2 = 45 pert andingan. Jumlah nilai unt uk seluruh t im maksimum t erj adi j ika t idak ada pert andingan yang berakhir seri, yait u 3 x 45 = 135. Karena di akhir t urnamen, j umlah nilai seluruh t im adalah 124, maka banyaknya pert andingan yang berakhir seri = 135 − 124 = 11 ∴ Banyaknya pert andingan yang berakhir seri = 11 20. Susunan delegasi yang mungkin adalah 4 pria dan 1 wanit a at au 3 pria dan 2 wanit a at au 2 pria dan 3 wanit a at au 1 pria dan 4 wanit a at au 5 wanit a . Banyaknya cara memilih anggot a delegasi = 7C4 ⋅ 5C1 + 7C3 ⋅ 5C2 + 7C2 ⋅ 5C3 + 7C1 ⋅ 5C4 + 7C0 ⋅ 5C5 = 35 ⋅ 5 + 35 ⋅ 10 + 21 ⋅ 10 + 7 ⋅ 5 + 1 ⋅ 1 = 175 + 350 + 210 + 35 + 1 = 771 cara. ∴ Banyaknya cara memilih anggot a delegasi ada 771. SMA Negeri 5 Bengkulu Eddy Hermanto, ST 86 SELEKSI OLIMPIADE MATEMATIKA INDONESIA 2005 TINGKAT PROVINSI Bidang Mat emat ika Bagian Per t ama Waktu : 90 Menit DEPARTEMEN PENDIDIKAN NASIONAL DIREKTORAT JENDERAL PENDIDIKAN DASAR DAN MENENGAH DIREKTORAT PENDIDIKAN MENENGAH UMUM TAHUN 2004 87 OLIMPIADE MATEMATIKA TINGKAT PROVINSI TAHUN 2004 BAGIAN PERTAMA 1. Misalkan x dan y adalah bilangan real t ak nol. Jika 1 1 + = 10 dan x + y = 40, berapakah xy ? x y 2. Sebot ol sirup bisa digunakan unt uk membuat 60 gelas minuman j ika dilarut kan dalam air dengan perbandingan 1 bagian sirup unt uk 4 bagian air. Berapa gelas minuman yang diperoleh dari sebot ol sirup j ika perbandingan larut an adalah 1 bagian sirup unt uk 5 bagian air ? 3. Penduduk Jawa Tengah adalah 25 % dari penduduk pulau Jawa dan 15 % dari penduduk Indonesia. Berapa persen penduduk Indonesia yang t inggal di luar pulau Jawa ? 4. Ket ika menghit ung volume sebuah t abung, Dina melakukan kesalahan. Ia memasukkan diamet er alas ke dalam rumus volume t abung, padahal seharusnya j ari-j ari alas yang dimasukkan. Berapakah rasio hasil perhit ungan Dinas t erhadap hasil yang seharusnya ? 5. Tiga lingkaran melalui t it ik pusat koordinat (0, 0). Pusat lingkaran pert ama t erlet ak di kuadran I, pusat lingkaran kedua berada di kuadran II dan pusat lingkaran ket iga berada pada kuadran III. Jika P adalah sebuah t it ik yang berada di dalam ket iga lingkaran t ersebut , di kuadran manakah t it ik ini berada ? 6. Diberikan bert urut -t urut (dari kiri ke kanan) gambar-gambar pert ama, kedua dan ket iga dari suat u barisan gambar. Berapakah banyaknya bulat an hit am pada gambar ke-n ? 7. Diberikan segit iga ABC dengan perbandingan panj ang sisi AC : CB = 3 : 4. Garis bagi sudut luar C memot ong perpanj angan BA di P (t it ik A t erlet ak di ant ara t it ik-t it ik P dan B). Tent ukan perbandingan panj ang PA : AB. 8. Berapakah banyaknya barisan bilangan bulat t ak negat if (x, y, z) yang memenuhi persamaan x + y + z = 99 ? 9. Tent ukan himpunan semua bilangan asli n sehingga n(n − 1)(2n − 1) habis dibagi 6. 10. Tent ukan semua bilangan real x yang memenuhi x 2 < ⏐2x − 8⏐. 11. Dari ant ara 6 buah kart u bernomor 1 sampai 6 diambil dua kart u secara acak. Berapakah peluang t erambilnya dua kart u yang j umlah nomornya adalah 6 ? 12. Pada sebuah t rapesium dengan t inggi 4, kedua diagonalnya saling t egak lurus. Jika salah sat u dari diagonal t ersebut panj angnya 5, berapakah luas t rapesium t ersebut ? 88 ⎛ 2 ⎞⎛ 2 ⎞⎛ 2⎞ 2 ⎞ ⎛ ⎟. 13. Tent ukan nilai dari ⎜⎜1 − ⎟⎟⎜⎜1 − ⎟⎟⎜⎜1 − ⎟⎟ L ⎜⎜1 − 2005 ⎟⎠ 7⎠ ⎝ 5 ⎠⎝ 3 ⎠⎝ ⎝ 14. Sant i dan Tini berlari sepanj ang sebuah lint asan yang berbent uk lingkaran. Keduanya mulai berlari pada saat yang sama dari t it ik P, t et api mengambil arah berlawanan. Sant i berlari 1½ kali lebih cepat daripada Tini. Jika PQ adalah garis t engah lingkaran lint asan dan keduanya berpapasan unt uk pert ama kalinya di t it ik R, berapa deraj at kah besar ∠RPQ ? 15. Pada sisi-sisi SU, TS dan UT dari ∆STU dipilih t it ik-t it ik P, Q dan R bert urut -t urut sehingga SP = 14 SU, TQ = 12 TS dan UR = 13 UT. Jika luas segit iga STU adalah 1, berapakah luas ∆PQR ? ( 16. Dua bilangan real x, y memenuhi x + x 2 )( +1 y + y 2 ) + 1 = 1 . Berapakah nilai x + y ? 17. Berapakah banyak minimal t it ik yang harus diambil dari sebuah persegi dengan panj ang sisi 2, agar dapat dij amin senant iasa t erambil dua t it ik yang j arak ant ara keduanya t idak lebih dari 1 2 2 ? 18. Misalkan f sebuah f ungsi yang memenuhi f (x) f (y) − f (xy) = x + y, unt uk set iap bilangan bulat x dan y. Berapakah nilai f (2004) ? 19. Not asi f pb(a, b) menyat akan f akt or persekut uan t erbesar dari bilangan bulat a dan b. Tiga bilangan asli a1 < a2 < a3 memenuhi f pb(a1, a2, a3) = 1, t et api f pb(ai , aj ) > 1 j ika i ≠ j , i, j = 1, 2, 3. Tent ukan (a1, a2, a3) agar a1 + a2 + a3 minimal. 20. Didef inisikan a o b = a + b + ab, unt uk semua bilangan bulat a, b. Kit a kat akan bahwa bilangan bulat a adalah f akt or dari bilangan bulat c bilamana t erdapat bilangan bulat b yang memenuhi a o b = c. Tent ukan semua f akt or posit if dari 67. 89 SELEKSI OLIMPIADE MATEMATIKA INDONESIA 2005 TINGKAT PROVINSI Bidang Mat emat ika Bagian Kedua Waktu : 120 Menit DEPARTEMEN PENDIDIKAN NASIONAL DIREKTORAT JENDERAL PENDIDIKAN DASAR DAN MENENGAH DIREKTORAT PENDIDIKAN MENENGAH UMUM TAHUN 2004 90 OLIMPIADE MATEMATIKA TINGKAT PROVINSI TAHUN 2004 BAGIAN KEDUA 1. Tent ukan semua (x, y, z), dengan x, y, z bilangan-bilangan real, yang memenuhi sekaligus ket iga persamaan berikut : x 2 + 4 = y3 + 4x − z3 y2 + 4 = z3 + 4y − x 3 z2 + 4 = x 3 + 4z − y3 2. Pada segit iga ABC diberikan t it ik-t it ik D, E, dan F yang t erlet ak bert urut -t urut pada sisi BC, CA dan AB sehingga garis-garis AD, BE dan CF berpot ongan di t it ik O. Bukt ikan bahwa AO BO CO + + =2 AD BE CF 3. Beni, Coki dan Doni t ingggal serumah dan belaj ar di sekolah yang sama. Set iap pagi ket iganya berangkat pada saat yang sama. Unt uk sampai ke sekolah Beni memerlukan wakt u 2 menit , Coki memerlukan wakt u 4 menit , sedangkan Doni memerlukan wakt u 8 menit . Selain it u t ersedia sebuah sepeda yang hanya dapat dinaiki sat u orang. Dengan sepeda, set iap orang memerlukan wakt u hanya 1 menit . Tunj ukkan bahwa adalah mungkin bagi ket iganya unt uk sampai ke sekolah dalam wakt u t idak lebih dari 2 34 menit . 4. Bukt ikan bahwa t idak ada bilangan asli m sehingga t erdapat bilangan-bilangan bulat k, e, dengan e ≥ 2, yang memenuhi m(m 2 + 1) = k e. 5. Tit ik let is pada bidang adalah t it ik yang mempunyai koordinat berupa pasangan bilangan bulat . Misalkan P1, P2, P3, P4, P5 adalah lima t it ik let is berbeda pada bidang. Bukt ikan bahwa t erdapat sepasang t it ik (Pi , Pj ), i ≠ j , demikian, sehingga ruas garis Pi Pj memuat sebuah t it ik let is selain Pi dan Pj . 91 SELEKSI OLIMPIADE MATEMATIKA INDONESIA 2005 TINGKAT PROVINSI TAHUN 2004 Prestasi itu diraih bukan didapat !!! SOLUSI SOAL Bidang Mat emat ika Bagian Pertama Disusun oleh : Eddy Hermant o, ST 92 Solusi Olimpiade Matematika Tk Provinsi 2004 Bagian Pert ama BAGIAN PERTAMA 1. 1 1 x +y 40 + = 10 . Maka = 10 . Karena x + y = 40 maka = 10 x y xy xy ∴ xy = 4 2. Keadaan I : Misalkan dalam 1 gel as t erdapat a bagian sirup maka banyaknya bagian air adal ah 4a bagian. Karena dalam sat u gelas t erdapat a bagian sirup maka dalam sat u bot ol sirup t erdapat 60a bagian sirup. Sedangkan dalam 1 gel as t erdapat 5a bagian. Keadaan II : Jika dalam gelas t erdapat b bagian sirup, maka banyaknya bagian air adal ah 5b bagian. Karena dalam sat u gelas t erdapat b bagian sirup maka dalam x gelas t erdapat bx bagian sirup. Sedangkan dalam 1 gel as t erdapat 6b bagian. Dari keadaan I dan keadaan II didapat 5a = 6b. Misalkan dari campuran t ersebut dapat dibuat x gelas, maka : bx = 60a = 12 ⋅ (6b) sehingga x = 72 ∴ Banyaknya gelas yang diperoleh adalah 72 gelas 3. Misalkan penduduk Jawa t engah = JT Penduduk Jawa = J Penduduk Indonesia = I JT = 25% J JT = 15% I 25% J = 15% I J = 60% I Karena penduduk Jawa = 60% penduduk Indonesia maka ∴ Penduduk Indonesia yang t inggal di luar pul au Jawa = 40% 4. Volume seharusnya = πr 2t Volume perhit ungan Dina = πD2t = 4πr 2t Rasio perhit ungan Dinas t erhadap hasil seharusnya = 4πr 2t =4 πr 2t ∴ Rasio perhit ungan Dina t erhadap hasil seharusnya = 4 5. * Karena lingkaran pert ama berpusat di kuadran I dan melalui t it ik (0, 0) maka semua t it ik yang t erlet ak di dalam l ingkaran pert ama t idak akan mungkin t erlet ak di kuadran III. * Karena lingkaran pert ama berpusat di kuadran II dan melalui t it ik (0, 0) maka semua t it ik yang t erlet ak di dalam lingkaran pert ama t idak akan mungkin t erlet ak di kuadran IV. * Karena lingkaran pert ama berpusat di kuadran III dan melalui t it ik (0, 0) maka semua t it ik yang t erlet ak di dalam lingkaran pert ama t idak akan mungkin t erlet ak di kuadran I. ∴ Tit ik P hanya mungkin t erlet ak di kuadran II. SMA Negeri 5 Bengkulu Eddy Hermanto, ST 93 Olimpiade Matematika Tk Provinsi 2004 Solusi Bagian Pert ama 6. Jika panj ang sisi segit iga adal ah k t it ik maka banyaknya bul at an hit am = 2k − 1. Pada gambar ke-n panj ang sisi segit iga = n + 2 t it ik. Banyaknya bulat an hit am = 2(n + 2) − 1 = 2n + 3 ∴ Banyaknya bulat an hit am pada gambar ke-n adalah 2n + 3 7. Karena CP adalah garis bagi maka berlaku AC : CB = PA : PB. Maka PA = 3 PB 4 PB = PA + AB 4 PA = PA + PB. 3 PA = 3 AB ∴ PA : AB = 3 : 1 8. Alt ernat if 1 : * Unt uk x = 0, maka y + z = 99. Banyaknya pasangan (y, z) yang memenuhi * Unt uk x = 1, maka y + z = 98. Banyaknya pasangan (y, z) yang memenuhi * Unt uk x = 2, maka y + z = 97. Banyaknya pasangan (y, z) yang memenuhi * Unt uk x = 3, maka y + z = 96. Banyaknya pasangan (y, z) yang memenuhi ada 100 yait u (0, 99), (1, 98), (2, 97), ⋅⋅⋅, (99, 0) ada 99 yait u (0, 98), (1, 97), (2, 96), ⋅⋅⋅, (98, 0) ada 98 yait u (0, 97), (1, 96), (2, 95), ⋅⋅⋅, (97, 0) ada 97 yait u (0, 96), (1, 95), (2, 94), ⋅⋅⋅, (96, 0) M * Unt uk x = 99, maka y + z = 0 Banyaknya pasangan (y, z) yang memenuhi ada 1 yait u (0, 0) Banyaknya barisan bil angan bul at (x, y, z) yang memenuhi = 100 + 99 + 98+ ⋅⋅⋅ + 1 = 100 (100 + 1) 2 ∴ Banyaknya barisan bil angan bul at (x, y, z) yang memenuhi persamaan x + y + z = 99 ada 5050. Alt ernat if 2 : Misalkan x 1 + x 2 + x 3 + ⋅⋅⋅ + x n = r dengan x i bulat ≥ 0 unt uk i = 1, 2, ⋅⋅⋅, n. Maka banyaknya pasangan (x 1, x 2, ⋅⋅⋅, x n) yang memenuhi adalah (r + (n − 1))! r!(n − 1)! = r+n-1Cn-1 Diket ahui x + y + z = 99 dengan x, y , z ≥ 0 dan x, y, z bulat . Banyaknya t ripel bil angan bulat t ak negat if (x, y, z) yang memenuhi = 101! 99!⋅2! = 5050. ∴ Banyaknya barisan bil angan bul at (x, y, z) yang memenuhi persamaan x + y + z = 99 ada 5050. SMA Negeri 5 Bengkulu Eddy Hermanto, ST 94 Olimpiade Matematika Tk Provinsi 2004 Solusi Bagian Pert ama 9. n(n − 1)(2n − 1) = n(n − 1)(2n + 2 − 3) = 2n(n − 1)(n + 1) − 3n(n − 1) (n − 1), n, (n + 1) adal ah 3 bil angan bulat berurut an, maka (n − 1)n(n + 1) habis dibagi 3! = 6. n(n − 1) j uga habis dibagi 2! = 2. Maka 3n(n − 1) past i habis dibagi 6. Akibat nya berapa pun nilai n bilangan asl i akan memenuhi n(n − 1)(2n − 1) habis dibagi 6. ∴ Himpunan semua n asli sehingga n(n − 1)(2n − 1) habis dibagi 6 adalah {n⏐n ∈ bilangan asli} Jika x ≤ 4 maka ⏐2x − 8⏐ = 8 − 2x Pert idaksamaan menj adi x 2 < 8 − 2x (x + 4) (x − 2) < 0 −4 < x < 2 Ket aksamaan di at as memenuhi syarat awal x ≤ 4. * Jika x ≥ 4 maka ⏐2x − 8⏐ = 2x − 8 Pert idaksamaan menj adi x 2 < 2x − 8 x 2 − 2x + 8 < 0 (x − 1) 2 + 7 < 0 Ruas kiri adalah def init posit if sehingga t idak ada penyelesaian x yang memenuhi. ∴ Penyelesaian x yang memenuhi pert idaksamaan x 2 < ⏐2x − 8⏐ adalah −4 < x < 2 10. * 11. Banyaknya pasangan kart u yang j umlahnya 6 ada 2 yait u (1, 5) dan (2, 4) Peluang t erambilnya 2 kart u yang j umlahnya nomornya 6 adalah 2 6C 2 ∴ Peluang t erambilnya 2 kart u yang j umlah nomornya 6 adalah 2 15 12. Alt ernat if 1 : Misal ∠ACD = α maka ∠GOD = ∠CAB = ∠BOF = α sin α = CE FG 4 = = CA CA 5 ⋅⋅⋅⋅⋅⋅⋅ (1) sehingga cos α = 3 5 ⋅⋅⋅⋅⋅⋅⋅ (2) dan t an α = 4 3 ⋅⋅⋅⋅⋅⋅⋅ (3) Misal CO = a dan GO = b maka OA = 5 − a dan OF = 4 − b sebab FG adal ah t inggi t rapesium. 3 a 5 4 DG = GO t an α = b 3 GC = CO cos α = SMA Negeri 5 Bengkulu Eddy Hermanto, ST 95 Olimpiade Matematika Tk Provinsi 2004 Solusi Bagian Pert ama 3 4 a + b ⋅⋅⋅⋅⋅⋅⋅⋅⋅⋅⋅⋅⋅⋅⋅⋅ (4) 5 3 3 3 AF = OA cos α = (5 − a) =3− a 5 5 4 16 4 = − b FB = OF t an α = (4 − b) 3 3 3 3 16 4 25 3 4 AB = AF + FB = 3 − a + − b= − a− b 5 3 3 3 5 3 1 Luas t rapesium = (DC + AB )FG 2 DC = DG + GC = Dari persamaan (4) dan (5) didapat l uas t rapesium = ∴ Luas t rapesium = 1 2 ⎛ 25 ⎞ 50 ⋅ ⎜⎜ ⎟⎟ ⋅ 4 = 3 ⎝ 3 ⎠ 50 3 Alt ernat if 2 : Misalkan OC = x maka OA = 5 − x Misalkan j uga OD = y dan OB = z. Jelas bahwa ∆OAB sebangun dengan ∆OCD sehingga 5− x z y y+z = maka = x y x 5 ⋅⋅⋅⋅⋅⋅⋅⋅⋅⋅⋅⋅⋅⋅⋅⋅⋅⋅⋅⋅⋅⋅⋅⋅⋅⋅⋅⋅⋅⋅⋅⋅⋅ (5) Misalkan j uga ∠ACD = α maka t g α = 4/ 3 Karena AC t egak lurus BD maka t g α = y/ x = 4/ 3 ⋅⋅⋅⋅⋅⋅⋅⋅⋅⋅⋅⋅⋅⋅⋅⋅⋅⋅⋅⋅⋅⋅⋅⋅⋅⋅ (6) Subt it usikan persamaan (6) ke persamaan (5) Maka y + z = 20 3 Karena AC t egak lurus BD maka luas t rapesium = ½ ⋅ AC ⋅ BD Luas t rapesium = ½ ⋅ 5 ⋅ (y + z) 1 20 ⋅5⋅ 2 3 50 ∴ Luas t rapesium = 3 Luas t rapesium = SMA Negeri 5 Bengkulu Eddy Hermanto, ST 96 Solusi Olimpiade Matematika Tk Provinsi 2004 Bagian Pert ama ⎛ 2 ⎞ 1 3 5 2003 2⎞ ⎛ 2 ⎞⎛ 2 ⎞⎛ ⎟⎟⎜⎜1 − ⎟⎟⎜⎜1 − ⎟⎟ L ⎜⎜1 − ⎟⎟ = ⋅ ⋅ ⋅ L ⋅ 2005 ⎠ 3 5 7 2005 7⎠ ⎝ 5 ⎠⎝ 3 ⎠⎝ ⎝ ⎛ 2 ⎞ 1 2 ⎞⎛ 2⎞ ⎛ 2 ⎞⎛ ⎟⎟ = ∴ ⎜⎜1 − ⎟⎟⎜⎜1 − ⎟⎟⎜⎜1 − ⎟⎟ L ⎜⎜1 − 7⎠ ⎝ 2005 ⎠ 2005 3 ⎠⎝ 5 ⎠⎝ ⎝ 13. ⎜⎜1 − 14. Karena Tini lebih l ambat dari Sant i maka panj ang busur yang dit empuhnya akan lebih pendek dari yang dit empuh Sant i. Misal panj ang busur yang dit empuh Tini = a maka panj ang busur yang dit empuh Sant i = 3 a. 2 3 a = K dengan K adal ah kel iling l ingkaran. 2 2 a= K 5 a 2 α = = 360° K 5 a+ α = 144o Karena O adalah pusat lingkaran maka ∆OPR adalah segit iga sama kaki. ∠RPO = ∠RPQ = ∴ ∠RPQ = 18 o 1 (180o − 144o) 2 15. Misal panj ang sisi TU = a, SU = b dan ST = c sert a ∠UST = α, ∠STU = β dan ∠TUS = γ, maka : SMA Negeri 5 Bengkulu Eddy Hermanto, ST 97 Olimpiade Matematika Tk Provinsi 2004 Solusi Bagian Pert ama 1 1 1 ab sin γ = ac sin β = bc sin α = 1 2 2 2 ⎞ ⎛ ⎞ ⎛ 1 1 1 1 1 Luas ∆SPQ = ⎜⎜ b ⎟⎟⎜⎜ c ⎟⎟ sin α = Luas ∆STU = 8 8 2 ⎝ 4 ⎠⎝ 2 ⎠ Luas ∆STU = Luas ∆TQR = 1 ⎛ 2 ⎞⎛ 1 ⎞ 1 1 ⎜⎜ a ⎟⎟⎜⎜ c ⎟⎟ sin β = Luas ∆STU = 3 3 2 ⎝ 3 ⎠⎝ 2 ⎠ Luas ∆UPR = 1 ⎛ 1 ⎞⎛ 3 ⎞ 1 1 ⎜⎜ a ⎟⎟⎜⎜ b ⎟⎟ sin γ = Luas ∆STU = 4 4 2 ⎝ 3 ⎠⎝ 4 ⎠ Luas ∆PQR = Luas ∆STU − Luas ∆SPQ − Luas ∆TQR − Luas ∆UPR = 1 − ∴ Luas ∆PQR = ( 16. x + x x + x (x + x 2 2 2 7 24 )( +1 y + y +1 = )( y + y ( −1)( −1) = x − x x − x 2 +1 = 2 ) +1 =1 1 +1 x − x ( 1 1 1 − − 8 3 4 2 2 2 +1 ⋅⋅⋅⋅⋅⋅⋅ (1) )( +1 y + y )( +1 y − y 1 y − y 2 +1 2 2 +1 )( +1 y − y ) 2 ) ( +1 = x − x 2 )( +1 y − y 2 +1 ) ⋅⋅⋅⋅⋅⋅⋅ (2) Jumlahkan persamaan (1) dan (2) sehingga 2x = 1 y + y 2 +1 + 1 y − y 2 +1 = 2y ( −1) −x = y ∴ x+y=0 17. Pada sebuah persegi dengan panj ang sisi = a, j arak t erj auh dua t it ik yang t erlet ak pada persegi adalah a 2 j ika kedua t it ik merupakan uj ung-uj ung diagonal bidang persegi t ersebut . SMA Negeri 5 Bengkulu Eddy Hermanto, ST 98 Olimpiade Matematika Tk Provinsi 2004 Solusi Bagian Pert ama Bagi persegi dengan panj ang sisi 2 t ersebut menj adi 16 persegi dengan panj ang sisi masingmasing = 1 sehingga j arak t erj auh 2 t it ik yang t erlet ak pada masing-masing persegi adalah 2 1 2. 2 Jika t erdapat 16 t it ik, maka t it ik-t it ik t ersebut masih dapat didist ribusikan masing-masing 1 t it ik yang t erlet ak di dal am persegi kecil sehingga masih belum dapat dij amin senant iasa t erambil dua t it ik yang j arak ant ara keduanya 1 2 . Jika t erdapat 17 t it ik maka sesuai Pigeon Hole 2 Principle maka sekurang-kurangnya ada sat u persegi kecil berisi sekurang-kurangnya 2 t it ik sehingga dapat dij amin senant iasa t erambil dua t it ik yang j arak ant ara keduanya 1 2. 2 ∴ Jumlah minimal t it ik yang harus diambil dari dalam sebuah persegi dengan panj ang sisi 2 agar dapat dij amin senant iasa t erambil 2 t it ik yang j arak ant ara keduanya 1 2 adal ah 17 . 2 18. f (x)f (y) − f (xy) = x + y * Jika x = 0 dan y = 0, maka f (0)f (0) − f (0) = 0 f (0) ( f (0) − 1 ) = 0. Maka f (0) = 0 at au f (0) = 1 * Jika x = 1 dan y = 0, maka f (1)f (0) − f (0) = 1 • Jika f (0) = 0, maka 0 = 1 yang berart i t idak mungkin f (0) = 0 maka f (0) = 1 • Unt uk f (0) = 1 maka f (1) − 1 = 1 sehingga f (1) = 2 * Jika x = 2004 dan y = 1 maka f (2004)f (1) − f (2004) = 2005 2f (2004) − f (2004) = 2005 sehingga f (2004) = 2005 * Jika x = 2004 dan y = 0 maka f (2004)f (0) − f (0) = 2004 f (2004) − 1 = 2004 sehingga f (2004) = 2005 ∴ f (2004) = 2005 19. f pb(a1, a2, a3) = 1. Karena f pb(ai , aj ) > 1 unt uk i ≠ j , i, j = 1, 2, 3 maka ai dan aj unt uk i ≠ j , i, j = 1, 2, 3 t idak sal ing prima relat if . Misal kan f pb(a1, a2) = q, f pb(a1, a3) = p dan f pb(a1, a2) = r dengan p, q, r > 1. Maka ai dengan i = 1, 2, 3 akan berbent uk : a1 = pq a2 = qr a3 = pr p dan q, q dan r, p dan r masing-masing sal ing prima relat if . 3 bilangan t erkecil (p, q, r) yang memenuhi adal ah (2, 3, 5) sehingga a1 = 2 ⋅ 3 = 6, a2 = 2 ⋅ 5 = 10 dan a3 = 3 ⋅ 5 = 15. ∴ Agar a1 + a2 + a3 minimal maka (a1, a2, a3) = (6, 10, 15) 20. a o b = a + b + ab c = a + b + ab 67 = a + b + ab 67 = (a + 1) (b + 1) − 1 SMA Negeri 5 Bengkulu Eddy Hermanto, ST 99 Solusi Olimpiade Matematika Tk Provinsi 2004 (a + 1) (b + 1) = 68 Fakt or yang sebenarnya dari 68 adalah 1, 2, 4, 17, 34 dan 68 Jika a + 1 = 1 maka a = 0 Jika a + 1 = 2 maka a = 1 Jika a + 1 = 17 maka a = 16 Jika a + 1 = 34 maka a = 33 ∴ f akt or posit if dari 67 adalah 1, 3, 16, 33 dan 67 SMA Negeri 5 Bengkulu Bagian Pert ama Jika a + 1 = 4 maka a = 3 Jika a + 1 = 68 maka a = 67 Eddy Hermanto, ST 100 SELEKSI OLIMPIADE MATEMATIKA INDONESIA 2005 TINGKAT PROVINSI TAHUN 2004 Prestasi itu diraih bukan didapat !!! SOLUSI SOAL Bidang Mat emat ika Bagian Kedua Disusun oleh : Eddy Her mant o, ST 101 Solusi Olimpiade Matematika Tk Provinsi 2004 Bagian Kedua BAGIAN KEDUA 1. x 2 + 4 = y3 + 4x − z3 y2 + 4 = z3 + 4y − x 3 z2 + 4 = x 3 + 4z − y3 Jumlahkan (1) + (2) + (3) ⋅⋅⋅⋅⋅⋅⋅⋅⋅⋅⋅ (1) ⋅⋅⋅⋅⋅⋅⋅⋅⋅⋅⋅ (2) ⋅⋅⋅⋅⋅⋅⋅⋅⋅⋅⋅ (3) x 2 + 4 + y2 + 4 + z2 + 4 = 4x + 4y + 4z (x 2 − 4x + 4) + (y 2 − 4y + 4) + (z2 − 4z + 4) = 0 (x − 2) 2 + (y − 2) 2 + (z − 2) 2 = 0 Karena persamaan kuadrat t idak mungkin negat if , maka persamaan di at as hanya dipenuhi j ika : x − 2 = 0 ; y − 2 = 0 dan z − 2 = 0 Didapat x = 2 ; y = 2 dan z = 2. Subt it usikan hasil ini ke persamaan (1), (2) dan (3) Persamaan (1), (2) 2 + 4 = (2) 3 + 4(2) − (2) 3. Memenuhi 8 = 8 Persamaan (2), (2) 2 + 4 = (2) 3 + 4(2) − (2) 3. Memenuhi 8 = 8 Persamaan (3), (2) 2 + 4 = (2) 3 + 4(2) − (2) 3. Memenuhi 8 = 8 ∴ (x, y, z) yang memenuhi adalah (2, 2, 2) sehingga 2. Dibuat garis t inggi pada segit iga ABC dan segit iga BOC yang masing-masing dit arik dari t it ik A dan O. Garis t inggi ini masing-masing memot ong sisi BC di t it ik P dan K. 1 1 (BC)(AP) dan Luas ∆BOC = (BC)(OK) 2 2 OK Luas ∆BOC ⋅⋅⋅⋅⋅⋅⋅⋅⋅ (1) = Luas ∆ABC AP OK OD ⋅⋅⋅⋅⋅⋅⋅⋅⋅⋅⋅⋅⋅⋅ (2) ∆DAP sebangun dengan ∆DOK sehingga = AD AP Luas ∆BOC OD ⋅⋅⋅⋅⋅⋅⋅⋅⋅⋅⋅⋅⋅⋅⋅⋅⋅⋅⋅⋅⋅⋅⋅⋅⋅ (3) Dari (1) dan (2) didapat = Luas ∆ABC AD OE Luas ∆AOC ⋅⋅⋅⋅ (4) dan Dengan cara yang sama didapat = Luas ∆ABC BE Luas ∆ABC = Luas ∆AOB OF = Luas ∆ABC CF ⋅⋅⋅ (5) Luas ∆BOC + Luas ∆AOC + Luas ∆AOB = Luas ∆ABC Luas ∆AOB Luas ∆BOC Luas ∆AOC + + =1 Luas ∆ABC Luas ∆ABC Luas ∆ABC SMA Negeri 5 Bengkulu Eddy Hermanto, ST 102 Olimpiade Matematika Tk Provinsi 2004 Solusi Bagian Kedua OF OD OE + + =1 AD BE CF OA OB OC OB OC OA 1− +1− +1− = 1 sehingga + + =2 AD BE CF CF AD BE OA OB OC ∴ Terbukt i bahwa + + =2 AD BE CF 3. Misal j arak dari rumah mereka ke sekolah = S Unt uk Doni : Misalkan agar wakt u yang diperl ukan Doni adalah 2 3 menit maka ia harus naik sepeda 4 sej auh X dan sisanya dengan j alan kaki dengan cat at an bahwa Doni t idak pernah ist irahat at au bergerak mundur. X (S − X )8 11 3 + = . Maka 4X + 32S − 32X = 11S sehingga X = S 4 S S 4 Unt uk Coki : Misalkan agar wakt u yang diperlukan Coki adalah 2 3 menit maka ia harus naik sepeda 4 sej auh Y dan sisanya dengan j al an kaki dengan cat at an bahwa Coki t idak pernah ist irahat at au bergerak mundur. Y (S − Y )4 11 5 + = . Maka 4Y + 16S − 16Y = 11S sehingga Y = S 12 S S 4 1 5 3 Karena S = 1 S maka berart i sepeda harus dimundurkan dalam perj alanannya. S+ 4 12 6 Alt ernat if 1: Doni naik sepeda sej auh sampai dal am wakt u 3 S lalu melanj ut kan perj alan dengan j alan kaki. Maka ia akan 4 3 1 3 ⋅ 1 + ⋅ 8 = 2 menit . 4 4 4 Beni akan sampai di t empat di mana sepeda dit inggalkan dalam wakt u 1 1 menit . Agar Coki 2 j uga dapat sampai di sekol ah dalam wakt u 2 3 menit maka Beni harus memundurkan 4 sepedanya menuj u ke arah rumahnya. Anggap Beni memundurkan sepedanya sej auh Z dari t empat di mana sepeda t ersebut dit emukan olehnya. Alt ernat if 1a : 3 menit maka : 4 1 11 3 Z (Z + 0,25S ) ⋅2+ + ⋅ 2 = . Maka 4Z + 8Z + 2S = 5S sehingga Z = S . Art inya 4 4 4 S S Jika yang diinginkan adalah Beni yang mencapai sekolah dalam wakt u 2 posisi sepeda kini berada di t engah-t engah ant ara rumah dan sekolah. Wakt u yang SMA Negeri 5 Bengkulu Eddy Hermanto, ST 103 Olimpiade Matematika Tk Provinsi 2004 Solusi Bagian Kedua ⎛ 1 1⎞ + ⎟⎟ menit = ⎝ 2 4⎠ diperlukan sampai dengan sepeda sampai di t empat t ersebut adalah ⎜1 ⎜ 1 3 menit . Wakt u yang diperlukan Coki unt uk mencapai pert engahan rumah dan sekolah 4 adalah 2 menit > 1 3 menit . Art inya ket ika ia mencapai t empat t ersebut , sepeda t elah 4 berada di sana. 1 3 1 ⋅ 1 = 2 < 2 menit . 4 2 2 3 1 3 ∴ Waktu yang diperlukan oleh Beni = 2 menit ; Coki = 2 menit ; Doni = 2 menit. 4 2 4 Wakt u yang diperlukan Coki unt uk mencapai sekolah adal ah 2 + Alt ernat if 1b : Jika yang diinginkan adalah Coki yang mencapai sekolah dalam wakt u 2 3 menit maka 4 5 S dihit ung dari 12 sesuai dengan hit ungan sebelumnya, sepeda harus dit aruh pada ⎛ 5 1⎞ 1 − ⎟ = S dihit ung dari t empat dimana sepeda dit emukan oleh Beni. ⎝ 12 4 ⎠ 6 sekolah at au ⎜ Wakt u yang diperlukan Beni unt uk mencapai sekolah adalah 3 1 5 1 3 ⋅ 2 + ⋅ 1 + ⋅ 2 = 2 < 2 menit . 4 6 12 2 4 ∴ Waktu yang diperlukan oleh Beni = 2 1 3 3 menit ; Coki = 2 menit ; Doni = 2 menit. 2 4 4 Alt ernat if 2 : 1 S dan melanj ut kan perj alannya dengan j alan kaki. Wakt u yang 2 1 1 1 3 diperlukan unt uk mencapai sekol ah adalah ⋅ 1 + ⋅ 4 = 2 < 2 menit 2 2 2 4 Coki naik sepeda sej auh Beni akan mencapai pert engahan j arak t erlebih dulu. Agar Doni dapat mencapai sekol ah 3 1 menit maka Beni harus memundurkan sepedanya sej auh S. Wakt u yang 4 4 1 1 1 3 diperlukan agar sepeda sampai pada j arak menit . S dari rumah adalah ⋅1 + ⋅1 = 2 4 4 4 3 1 menit . Wakt u yang diperlukan Doni unt uk mencapai j arak ini adalah ⋅ 8 = 2 menit > 4 4 dalam wakt u 2 Art inya sepeda t elah berada di sana saat Doni mencapai t empat t ersebut . 1 1 3 3 ⋅ 2 + ⋅ 1 + ⋅ 2 = 2 menit . 2 4 4 4 3 1 3 ∴ Waktu yang diperlukan oleh Beni = 2 menit ; Coki = 2 menit ; Doni = 2 menit 4 2 4 Wakt u yang diperlukan Beni unt uk mencapai sekolah adal ah SMA Negeri 5 Bengkulu Eddy Hermanto, ST 104 Olimpiade Matematika Tk Provinsi 2004 Solusi Bagian Kedua 4. Anggap t erdapat persamaan yang memenuhi m(m 2 + 1) = k e dengan k dan e bulat dan e > 2 Jika ada m bilangan asl i yang memenuhi, maka ruas kiri ≥ 2 yang berart i k ≥ 2 ⋅⋅⋅⋅⋅⋅⋅⋅⋅⋅⋅⋅⋅⋅ (1) Karena persamaan berbent uk ab = c d dengan a, b, c, d ∈ Asli, maka a membagi c at au c membagi a. Alt ernat if 1 : * Jika k membagi m maka m = p ⋅ k q dengan p bukan kel ipat an k dan q ∈ bilangan bulat dan p ∈ bilangan asli. Persamaan menj adi p 3k 3q + pk q = k e sehingga p 3k 2q + p = k e-q ⋅⋅⋅⋅⋅⋅⋅⋅⋅⋅⋅⋅⋅⋅⋅⋅⋅⋅ (2) ƒ Jika e > q Ruas kanan persamaan (2) adalah sebuah bilangan yang habis dibagi k sedangkan ruas kiri adalah sebuah bilangan yang bersisa p j ika dibagi k dengan p bukan bilangan kelipat an k. Maka t anda kesamaan t idak akan mungkin t erj adi. • Jika e ≤ q Ruas kanan persamaan (2) bernilai ≤ 1 Karena p ≥ 1 dan k ≥ 2 maka p 3k 2q + p ≥ 3 yang berart i t idak ada nilai p dan k yang memenuhi. Maka t idak ada nilai m ∈ bil angan asli yang memenuhi m(m 2 + 1) = k e dengan k membagi m. * Jika m membagi k maka k = rm dengan r ∈ bilangan asl i sebab k ≥ 2 Persamaan akan menj adi m(m 2 + 1) = r em e sehingga m + • 1 = r e m e −2 m ⋅⋅⋅⋅⋅⋅⋅⋅⋅⋅⋅⋅⋅⋅⋅⋅⋅ (3) Jika m = 1 Persamaan (3) menj adi 2 = r e. Karena 2 = 21 maka persamaan hanya akan dipenuhi j ika r = 2 dan e = 1 yang t idak memenuhi syarat bahwa e ≥ 2. • Jika m > 1 Ruas kiri persamaan (3) bukan merupakan bilangan bulat sedangkan ruas kanan merupakan bilangan bul at sebab e ≥ 2. Maka t idak ada nilai m ∈ bil angan asli yang memenuhi m(m 2 + 1) = k e dengan m membagi k. ∴ Terbukti bahwa tidak ada bilangan asli m sehingga terdapat bilangan-bilangan bulat k, e, dengan e ≥ 2, yang memenuhi m(m2 + 1) = k e Alt ernat if 2 : FPB (m, m 2 + 1) = FPB(m, 1) = 1 yang art inya m dan m 2 + 1 relat if prima. Jadi, persamaan m(m 2 + 1) = k e hanya akan t erpenuhi j ika m dan m 2 + 1 memiliki pangkat yang sama. Misalkan m = ae dan m 2 + 1 = b e = a2e + 1. Karena (a2 + 1) e = eCoa2e + eC1a2(e-1) + ⋅⋅⋅ + eCe 1e = a2e + e ⋅ a2(e-1) + ⋅⋅⋅ + 1 > a2e + 1 = m 2 + 1 maka (a2) e < m 2 + 1 = (a2) e + 1 < (a2 + 1) e Dari ket aksamaan di at as didapat m 2 + 1 t erlet ak di ant ara dua bilangan asl i berurut an berpangkat e. Maka t idak mungkin m 2 + 1 berbent uk b e. ∴ Terbukti bahwa tidak ada bilangan asli m sehingga terdapat bilangan-bilangan bulat k, e, dengan e ≥ 2, yang memenuhi m(m2 + 1) = k e SMA Negeri 5 Bengkulu Eddy Hermanto, ST 105 Solusi Olimpiade Matematika Tk Provinsi 2004 Bagian Kedua 5. Misal x ij adalah j arak t it ik Pi dan Pj dalam arah sumbu X dan Misal y ij adalah j arak t it ik Pi dan Pj dalam arah sumbu Y. Jika x ij dan yij keduanya genap, maka dapat dipast ikan bahwa sekurang-kurangnya sat u t it ik let is selain t it ik Pi dan Pj akan t erlet ak pada ruas garis Pi Pj , yait u pada pert engahan ruas garis Pi Pj 1 1 x ij pada arah sumbu X dan yij pada arah sumbu Y t erhadap t it ik Pi 2 2 1 1 maupun Pj dengan x ij dan yij adalah j uga bil angan bulat . 2 2 yang akan berj arak Sif at penj umlahan berikut j uga akan membant u menj elaskan : Bilangan Genap − Bilangan Genap = Bilangan Genap Bilangan Ganj il − Bilanagn Ganj il = Bilangan Genap. Kemungkinan j enis koordinat (dalam bahasa lain disebut parit as) suat u t it ik let is pada bidang hanya ada 4 kemungkinan yait u (genap, genap), (genap, ganj il), (ganj il, ganj il) dan (ganj il, genap). Jika 2 t it ik let is mempunyai parit as yang sama maka sesuai sif at penj umlahan maka dapat dipast ikan kedua t it ik let is memiliki j arak mendat ar dan j arak vert ikal merupakan bilangan genap yang berart i koordinat t it ik t engah dari garis yang menghubungkan kedua t it ik let is t ersebut j uga merupakan bilangan genap. Karena ada 5 t it ik let is sedangkan hanya ada 4 parit as t it ik let is maka sesuai Pigeon Hole Principle (PHP) maka dapat dipast ikan sekurang-kur angnya ada dua t it ik let is yang memiliki parit as yang sama. ∴ Dari penj elasan di atas dapat dibuktikan bahwa j ika P1 , P2 , P3 , P4 , P5 adalah lima titik letis berbeda pada bidang maka terdapat sepasang titik (Pi, Pj ), i ≠ j , demikian, sehingga ruas garis Pi Pj memuat sebuah titik letis selain Pi dan Pj . SMA Negeri 5 Bengkulu Eddy Hermanto, ST 106 SELEKSI TIM OLIMPIADE MATEMATIKA INDONESIA 2005 OLIMPIADE SAINS NASIONAL 2004 PEKAN BARU (RIAU), 24 – 29 AGUSTUS 2004 Bidang Mat emat ika Hari Pert ama Waktu : 180 Menit DEPARTEMEN PENDIDIKAN NASIONAL DIREKTORAT JENDERAL PENDIDIKAN DASAR DAN MENENGAH DIREKTORAT PENDIDIKAN MENENGAH UMUM TAHUN 2004 107 OLIMPIADE SAINS NASIONAL 2004 24 – 29 AGUSTUS 2004 PEKAN BARU, RIAU BI DAN G : MATEMATI KA HARI PERTAMA WAKTU : 180 MENIT 1. Berapa banyaknya pembagi genap dan pembagi ganj il dari 56 − 1 ? 2. Sebuah bak bila diisi dengan keran air dingin akan penuh dalam 14 menit . Unt uk mengosongkan bak yang penuh dengan membuka lubang pada dasar bak, air akan keluar semua dalam wakt u 21 menit . Jika keran air dingin dan air panas dibuka bersamaan dan lubang pada dasar bak dibuka, bak akan penuh dalam 12, 6 menit . Maka berapa lamakah wakt u yang diperlukan unt uk memenuhkan bak hanya dengan keran air panas dan lubang pada dasar bak dit ut up ? 3. 1 * 2 * 3 * 4 * 5 * 6 * 7 * 8 * 9 * 10 Berapa carakah unt uk menyusun deret an t ersebut dengan menggant i menggant i t anda ekspresi “ *” dengan t anda “ +” at au “ −“ sehingga j umlahnya menj adi 29 ? 4. Lingkaran yang berbeda bent uk disusun sebagai berikut : Bukt ikan bahwa ada lingkaran yang melewat i keempat t it ik singgung keempat lingkaran. 108 SELEKSI TIM OLIMPIADE MATEMATIKA INDONESIA 2005 OLIMPIADE SAINS NASIONAL 2004 PEKAN BARU (RIAU), 24 – 29 AGUSTUS 2004 Bidang Mat emat ika Hari Kedua Waktu : 180 Menit DEPARTEMEN PENDIDIKAN NASIONAL DIREKTORAT JENDERAL PENDIDIKAN DASAR DAN MENENGAH DIREKTORAT PENDIDIKAN MENENGAH UMUM TAHUN 2004 109 OLIMPIADE SAINS NASIONAL 2004 24 – 29 AGUSTUS 2004 PEKAN BARU, RIAU BI DAN G : MATEMATI KA HARI KEDUA WAKTU : 180 MENIT 5. x 1 + 4x 2 + 9x 3 + 16x 4 + 25x 5 + 36x 6 + 49x 7 = 1 4x 1 + 9x 2 + 16x 3 + 25x 4 + 36x 5 + 49x 6 + 64x 7 = 12 9x 1 + 16x 2 + 25x 3 + 36x 4 + 49x 5 + 64x 6 + 81x 7 = 123 Berapakah nilai S j ika S = 16x 1 + 25x 2 + 36x 3 + 49x 4 + 64x 5 + 81x 6 + 100x 7 6. Persamaan kuadrat x 2 + ax + b + 1 = 0 dengan a, b adalah bilangan bulat , memiliki akar-akar bilangan asli. Bukt ikan bahwa a2 + b 2 bukan bilangan prima. 7. Bukt ikan bahwa suat u segit iga ABC siku-siku di C dengan a menyat akan sisi dihadapan sudut A, b menyat akan sisi di hadapan sudut B, c menyat akan sisi di hadapan sudut C memiliki diamet er lingkaran dalam = a + b − c. 8. Sebuah lant ai berluas 3 m 2 akan dit ut upi oleh karpet dengan bermacam bent uk sebanyak 5 buah dengan ukuran @ 1m 2. Tunj ukkan bahwa ada 2 karpet yang t umpang t indih dengan luasan t umpang t indih lebih dari 1/ 5 m 2. 110 SELEKSI TIM OLIMPIADE MATEMATIKA INDONESIA 2005 OLIMPIADE SAINS NASIONAL 2004 PEKAN BARU (RIAU), 24 – 29 AGUSTUS 2004 Prestasi itu diraih bukan didapat !!! SOLUSI SOAL Bidang Mat emat ika Disusun oleh : Eddy Hermant o, ST 111 Olimpiade Sains Nasional 2004 Solusi Bidang : Mat emat ika 1. 56 − 1 = (53 + 1)(53 − 1) = 126 ⋅ 124 56 − 1 = 2 ⋅ 32 ⋅ 7 ⋅ 22 ⋅ 31 56 − 1 = 23 ⋅ 32 ⋅ 7 ⋅ 31 Misalkan M = p 1d1 ⋅ p 2d2 ⋅ p 3d3 ⋅ ⋅⋅⋅ ⋅ p ndn dengan p 1, p 2, p 3, ⋅⋅⋅, p n adalah bilangan prima maka banyaknya pembagi posit if dari M adalah (d 1 + 1)(d 2 + 1)(d 3 + 1) ⋅⋅⋅ (d n + 1) Banyaknya pembagi (disebut j uga f akt or) dari 56 − 1 adalah (3 + 1)(2 + 1)(1 + 1)(1 + 1) = 48 Misal K = 32 ⋅ 7 ⋅ 31. Mengingat bahwa bilangan ganj il hanya didapat dari perkalian bilangan ganj il maka semua pembagi dari K past i ganj il. Banyaknya pembagi dari K adalah (2 + 1)(1 + 1)(1 + 1) = 12 Banyaknya pembagi dari K sama dengan banyaknya pembagi ganj il dari 56 − 1 ∴ Banyaknya pembagi ganj il dari 5 6 − 1 adalah 12. Banyaknya pembagi genap dari 5 6 − 1adalah 48 − 12 = 36 (Cat at an : Ke-48 pembagi 56 − 1 adalah : 1, 2, 3, 4, 6, 7, 8, 9, 12, 14, 18, 21, 24, 28, 31, 36, 42, 56, 62, 63, 72, 84, 93, 124, 126, 168, 186, 217, 248, 252, 279, 372, 434, 504, 558, 651, 744, 868, 1116, 1302, 1736, 1953, 2232, 2604, 3906, 5208, 7812, 15624) 2. Misalkan vd = kelaj uan air keluar dari keran air dingin vp = kelaj uan air keluar dari keran air panas vb = kelaj uan air keluar dari lubang di dasar bak X = volume bak X 14 X = 21 vd = vb v d +v p −v b = X 12,6 Dari ket iga persamaan di at as didapat : X +v p 14 X vp = 7 v p = X 5X = 21 63 ⎛5 1 1⎞ ⎜⎜ + − ⎟⎟ ⎝9 3 2⎠ − X 18 ∴ Wakt u yang diperlukan unt uk memenuhkan bak hanya dengan keran air panas dan lubang pada dasar bak dit ut up adalah 18 menit SMA Negeri 5 Bengkulu Eddy Hermanto, ST 112 Solusi Olimpiade Sains Nasional 2004 Bidang : Mat emat ika 3. Misalkan S = 1 + 2 + 3 + 4 + 5 + 6 + 7 + 8 + 9 + 10 = 55 Jika + k kit a gant i dengan − k maka S akan berkurang sebanyak 2k. Karena 55 − 29 = 26 maka bilangan yang yang bert anda “ −” harus berj umlah 13. Jika ada 4 bilangan yang bert anda “ −” maka j umlah minimum bilangan t ersebut = 2 + 3 + 4 + 5 = 14 > 13. Maka banyaknya bilangan yang bert anda “ −” harus kurang dari 4. • Unt uk 2 bilangan yang bert anda “ −” maka pasangan yang mungkin adalah (3, 10), (4, 9), (5, 8), (6, 7). • Unt uk 3 bilangan yang bert anda “ −” maka t ripel yang mungkin adalah (2, 3, 8), (2, 4, 7), (2, 5, 6), (3, 4, 6) ∴ Banyaknya kemungkinan seluruhnya ada 8. (Cat at an : Ke-8 kemungkinan t ersebut adalah : 1 + 2 − 3 + 4 + 5 + 6 + 7 + 8 + 9 − 10 1 − 2 − 3 + 4 + 5 + 6 + 7 − 8 + 9 + 10 1 + 2 + 3 − 4 + 5 + 6 + 7 + 8 − 9 + 10 1 − 2 + 3 − 4 + 5 + 6 − 7 + 8 + 9 + 10 1 + 2 + 3 + 4 − 5 + 6 + 7 − 8 + 9 + 10 1 − 2 + 3 + 4 − 5 − 6 + 7 + 8 + 9 + 10 1 + 2 + 3 + 4 + 5 − 6 − 7 + 8 + 9 + 10 1 + 2 − 3 − 4 + 5 − 6 + 7 + 8 + 9 + 10 4. Misalkan A, B, C dan D adalah keempat pusat lingkaran dan E, F, G dan H adalah t it ik singgung keempat lingkaran. Maka persoalan t ersebut dapat digambarkan sebagai berikut : ∆AEH, ∆BEF, ∆CFG dan ∆DGh semuanya adalah segit iga sama kaki. Misalkan ∠A menyat akan ∠DAC ∠B menyat akan ∠ABC ∠C menyat akan ∠BCD ∠D menyat akan ∠CDA ∠EHA = ∠AEH = 90o − ½ ∠A ∠BEF = ∠BFE = 90o − ½ ∠B ∠CFG = ∠CGF = 90o − ½ ∠C ∠D = 360o − ∠A − ∠B − ∠C ∠DGH = ∠DHG = ½ (180o − (360o − ∠A − ∠B − ∠C)) = ½ ( ∠A + ∠B + ∠C) − 90o ∠HEF = 180o − ∠AEH − ∠BEF = ½ ( ∠A + ∠B) ∠HGF = 180o − ∠DGH − ∠CGF = 180o − ½ ( ∠A + ∠B) ∠HEF + ∠HGF = 180o Karena ∠HEF + ∠HGF = 180o maka segiempat EFGH adalah segiempat t ali busur yang berart i t it ik E, F, G, dan H t erlet ak pada sat u lingkaran. ∴ Terbukti bahwa ada lingkaran yang melewati keempat titik singgung keempat lingkaran. SMA Negeri 5 Bengkulu Eddy Hermanto, ST 113 Solusi Olimpiade Sains Nasional 2004 Bidang : Mat emat ika 5. x 1 + 4x 2 + 9x 3 + 16x 4 + 25x 5 + 36x 6 + 49x 7 = 1 ⋅⋅⋅⋅⋅⋅⋅⋅⋅⋅⋅⋅⋅⋅⋅ (1) 4x 1 + 9x 2 + 16x 3 + 25x 4 + 36x 5 + 49x 6 + 64x 7 = 12 ⋅⋅⋅⋅⋅⋅⋅⋅⋅⋅⋅⋅⋅⋅⋅⋅⋅⋅⋅⋅ (2) 9x 1 + 16x 2 + 25x 3 + 36x 4 + 49x 5 + 64x 6 + 81x 7 = 123 ⋅⋅⋅⋅⋅⋅⋅⋅⋅⋅⋅⋅⋅⋅⋅⋅⋅⋅⋅⋅⋅⋅⋅⋅ (3) Kurangkan (2) dengan (1), 3x 1 + 5x 2 + 7x 3 + 9x 4 + 11x 5 + 13x 6 + 15x 7 = 11 ⋅⋅⋅⋅⋅⋅⋅⋅⋅⋅⋅⋅⋅⋅⋅⋅⋅⋅⋅⋅⋅⋅⋅⋅⋅⋅ Kurangkan (3) dengan (2), 5x 1 + 7x 2 + 9x 3 + 11x 4 + 13x 5 + 15x 6 + 17x 7 = 111 ⋅⋅⋅⋅⋅⋅⋅⋅⋅⋅⋅⋅⋅⋅⋅⋅⋅⋅⋅⋅⋅⋅ Kurangkan (5) dengan (4), 2x 1 + 2x 2 + 2x 3 + 2x 4 + 2x 5 + 2x 6 + 2x 7 = 100 ⋅⋅⋅⋅⋅⋅⋅⋅⋅⋅⋅⋅⋅⋅⋅⋅⋅⋅⋅⋅⋅⋅⋅⋅⋅⋅⋅⋅ Jumlahkan (5) dengan (6), 7x 1 + 9x 2 + 11x 3 + 13x 4 + 15x 5 + 17x 6 + 19x 7 = 211 ⋅⋅⋅⋅⋅⋅⋅⋅⋅⋅⋅⋅⋅⋅⋅⋅⋅⋅⋅⋅⋅ Jumlahkan (3) dengan (7), 16x 1 + 25x 2 + 36x 3 + 49x 4 + 64x 5 + 81x 6 + 100x 7 = 334 ∴ 16x 1 + 25x 2 + 36x 3 + 49x 4 + 64x 5 + 81x 6 + 100x 7 = 334 (4) (5) (6) (7) 6. Misalkan x 1 dan x 2 adalah akar-akar persamaan x 2 + ax + b + 1 = 0 maka : x 1 + x 2 = −a x1 x2 = b + 1 b = x1 x2 − 1 a2 + b 2 = (x 1 + x 2) 2 + (x 1 x 2 − 1) 2 a2 + b 2 = x 12 + x 22 + 2x 1x 2 + (x 1 x 2) 2 − 2 x 1x 2 + 1 a2 + b 2 = (x 1x 2) 2 + x 12 + x 22 + 1 a2 + b 2 = (x 12 + 1) (x 22 + 1) Karena x 1 dan x 2 keduanya adalah bilangan asli maka (x 12 + 1) dan (x 22 + 1) keduanya adalah bilangan asli lebih dari 1. Maka a2 + b 2 adalah perkalian dua bilangan asli masing-masing > 1 yang mengakibat kan a2 + b 2 adalah bukan bilangan prima. ∴ Terbukti a2 + b2 bukan bilangan prima. 7. Misalkan d adalah diamet er lingkaran dalam segit iga dan r adalah j ej ari lingkaran dalam maka : Alt ernat if 1 : ½ r (a + b + c) = Luas segit iga d (a + b + c) = 4 ⋅ Luas segit iga d (a + b + c) = 2ab d (a + b + c) = (a + b) 2 − (a2 + b 2) Karena ABC adalah segit iga siku-siku di C maka : d (a + b + c) = (a + b) 2 − c2 d (a + b + c) = (a + b + c) (a + b − c) d=a+b−c ∴ Terbukti bahwa diamet er lingkaran dalam segitiga tersebut adalah a + b − c. SMA Negeri 5 Bengkulu Eddy Hermanto, ST 114 Olimpiade Sains Nasional 2004 Solusi Bidang : Mat emat ika Alt ernat if 2 : Misalkan O adalah pusat lingkaran dalam ∆ABC. Misalkan j uga garis AB, AC dan BC bert urut -t urut menyinggung lingkaran dalam di t it ik D, E dan F. Jelas bahwa CE = CF = r. Jelas j uga bahwa AD = AE dan BD = BF Maka AE = b − r dan BF = a − r AB = AD + BD c = (b − r) + (a − r) d = 2c = a + b − c ∴ Terbukti bahwa diamet er lingkaran dalam segitiga tersebut adalah a + b − c. 8. Alt ernat if 1 : Misalkan Ai menyat akan karpet ke-i. A1 = A2 = A3 = A4 = A5 = 1 Berdasarkan Prinsip Inklusi Eksklusi maka : (A1 (A2 (A1 (A2 (A1 ∪ A2 ∪ A3 ∪ A4 ∪ A5) = A1 + A2 + A3 + A4 + A5 − (A1 ∩ A2) − (A1 ∩ A3) − (A1 ∩ A4) − (A1 ∩ A5) ∩ A3) − (A2 ∩ A4) − (A2 ∩ A5) − (A3 ∩ A4) − (A3 ∩ A5) − (A4 ∩ A5) + (A1 ∩ A2 ∩ A3) ∩ A2 ∩ A4) + (A1 ∩ A2 ∩ A5) + (A1 ∩ A3 ∩ A4) + (A1 ∩ A3 ∩ A5) + (A1 ∩ A4 ∩ A5) + A2 ∩ A3 ∩ A4) ∩ A3 ∩ A5) + (A2 ∩ A4 ∩ A5) + (A3 ∩ A4 ∩ A5) − (A1 ∩ A2 ∩ A3 ∩ A4) − (A1 ∩ A2 ∩ A3 ∩ A5) ∩ A2 ∩ A4 ∩ A5) − (A1 ∩ A3 ∩ A4 ∩ A5) − (A2 ∩ A3 ∩ A4 ∩ A5) + (A1 ∩ A2 ∩ A3 ∩ A4 ∩ A5) − + + − 3 = 1 + 1 + 1 + 1 + 1 − (A1 ∩ A2) − (A1 ∩ A3) − (A1 ∩ A4) − (A1 ∩ A5) − (A2 ∩ A3) − (A2 ∩ A4) − (A2 ∩ A5) − (A3 ∩ A4) − (A3 ∩ A5) − (A4 ∩ A5) + (A1 ∩ A2 ∩ A3) + (A1 ∩ A2 ∩ A4) + (A1 ∩ A2 ∩ A5) + (A1 ∩ A3 ∩ A4) + (A1 ∩ A3 ∩ A5) + (A1 ∩ A4 ∩ A5) + (A2 ∩ A3 ∩ A4) + (A2 ∩ A3 ∩ A5) + (A2 ∩ A4 ∩ A5) + (A3 ∩ A4 ∩ A5) − (A1 ∩ A2 ∩ A3 ∩ A4) − (A1 ∩ A2 ∩ A3 ∩ A5) − (A1 ∩ A2 ∩ A4 ∩ A5) − (A1 ∩ A3 ∩ A4 ∩ A5) − (A2 ∩ A3 ∩ A4 ∩ A5) + (A1 ∩ A2 ∩ A3 ∩ A4 ∩ A5) 2 + (A1 ∩ A2 ∩ A3) + (A1 ∩ A2 ∩ A4) + (A1 ∩ A2 ∩ A5) + (A1 ∩ A3 ∩ A4) + (A1 ∩ A3 ∩ A5) (A1 ∩ A4 ∩ A5) + (A2 ∩ A3 ∩ A4) + (A2 ∩ A3 ∩ A5) + (A2 ∩ A4 ∩ A5) + (A3 ∩ A4 ∩ A5) (A1 ∩ A2 ∩ A3 ∩ A4) − (A1 ∩ A2 ∩ A3 ∩ A5) − (A1 ∩ A2 ∩ A4 ∩ A5) − (A1 ∩ A3 ∩ A4 ∩ A5) (A2 ∩ A3 ∩ A4 ∩ A5) + (A1 ∩ A2 ∩ A3 ∩ A4 ∩ A5) = (A1 ∩ A2) + (A1 ∩ A3) + (A1 ∩ A4) + (A1 ∩ A5) (A2 ∩ A3) + (A2 ∩ A4) + (A2 ∩ A5) + (A3 ∩ A4) + (A3 ∩ A5) + (A4 ∩ A5) ⋅⋅⋅⋅⋅⋅⋅⋅⋅⋅⋅⋅⋅⋅⋅⋅⋅⋅⋅⋅⋅⋅⋅⋅⋅⋅⋅⋅⋅⋅⋅⋅⋅⋅⋅⋅⋅⋅⋅⋅⋅ (1) + − − + (Aa ∩ Ab ∩ Ac ∩ Ad) merupakan himpunan bagian dari (Aa ∩ Ab ∩ Ac) sehingga (Aa ∩ Ab ∩ Ac ∩ Ad) ≤ (Aa ∩ Ab ∩ Ac) SMA Negeri 5 Bengkulu Eddy Hermanto, ST 115 Olimpiade Sains Nasional 2004 Solusi Bidang : Mat emat ika (Aa ∩ Ab ∩ Ac) merupakan himpunan bagian dari (Aa ∩ Ab) sehingga (Aa ∩ Ab ∩ Ac) ≤ (Aa ∩ Ab) dan set erusnya ⋅⋅⋅⋅⋅⋅⋅⋅⋅ Akibat nya : (A1 ∩ A2 ∩ A3 (A1 ∩ A2 ∩ A3 (A1 ∩ A2 ∩ A4 (A1 ∩ A3 ∩ A4 (A2 ∩ A3 ∩ A4 ∩ ∩ ∩ ∩ ∩ A4) A5) A5) A5) A5) ≤ (A1 ≤ (A1 ≤ (A1 ≤ (A1 ≤ (A2 ∩ ∩ ∩ ∩ ∩ A2 A2 A2 A3 A3 ∩ ∩ ∩ ∩ ∩ A3) A5) A4) A4) A5) at au at au at au at au at au (A1 (A1 (A1 (A1 (A2 ∩ ∩ ∩ ∩ ∩ A2 A2 A2 A3 A3 ∩ ∩ ∩ ∩ ∩ A3 A3 A4 A4 A4 ∩ ∩ ∩ ∩ ∩ A4) A5) A5) A5) A5) ≤ (A1 ∩ ≤ (A1 ∩ ≤ (A1 ∩ ≤ (A3 ∩ ≤ (A2 ∩ A2 A2 A2 A4 A3 ∩ ∩ ∩ ∩ ∩ A4) A3) A5) A5) A4) dan dan dan dan dan set erusnya set erusnya set erusnya set erusnya set erusnya Maka ruas kiri persamaan (1) bernilai minimal 2. Karena ada 10 irisan di ruas kanan persamaan (1) maka dapat dipast ikan sekurang-kurangnya ada 1 di ant ara 10 irisan 2 karpet t ersebut yang memiliki irisan minimal 2/ 10 = 0, 2 m 2. Alt ernat if 2 : Andaikan t idak ada 2 karpet yang t umpang t indih dengan luasan t umpang t indih lebih dari 1/ 5 m 2. Karpet pert ama akan menempat i ruang dengan luas 1 m 2. Maka karpet kedua akan menempat i ruang dengan luas minimum 4/ 5 m 2. Karpet ket iga akan menempat i ruang dengan luas minimum 3/ 5 m 2. Karpet keempat akan menempat i ruang dengan luas minimum 2/ 5 m 2. Karpet kelima akan menempat i ruang dengan luas minimum 1/ 5 m 2. Luas minimum karpet yang diperlukan adalah 1 + 4 3 2 1 + + + = 3 m 2. 5 5 5 5 ∴ Hanya dapat dibuktikan bahwa ada 2 karpet yang tumpang tindih dengan luasan tumpang tindih minimal 1/ 5 m 2 . SMA Negeri 5 Bengkulu Eddy Hermanto, ST 116 SELEKSI OLIMPIADE TINGKAT KABUPATEN/ KOTA TAHUN 2005 TIM OLIMPIADE MATEMATIKA INDONESIA TAHUN 2006 Bidang Mat emat ika Waktu : 3, 5 Jam DEPARTEMEN PENDIDIKAN NASIONAL DIREKTORAT JENDERAL PENDIDIKAN DASAR DAN MENENGAH DIREKTORAT PENDIDIKAN MENENGAH UMUM TAHUN 2005 117 OLIMPIADE MATEMATIKA NASIONAL SELEKSI TINGKAT KOTA/ KABUPATEN TAHUN 2005 Bagian Pertama Pilih sat u j awaban yang benar. Dalam hal t erdapat lebih dari sat u j awaban yang benar, pilih j awaban yang paling baik. 1. Bilangan 1 (1 + 2 )(2 + 3 )(1 − 2 )(2 − 3 ) adalah bilangan A. t akrasional posit if B. t akrasional negat if C. rasional t idak bulat D. bulat posit if E. bulat negat if 2. Pada gambar di samping, a, b, c, d dan e bert urut -t urut menyat akan besar sudut pada t it ikt it ik uj ung bint ang lima yang t erlet ak pada suat u lingkaran. Jumlah a + b + c + d + e = A. 135o B. 180o C. 270o o D. 360 E. t idak dapat dit ent ukan dengan past i 3. Semula harga semangkuk bakso dan harga segelas j us masing-masing adalah Rp. 5000. Set elah kenaikan BBM, semangkuk bakso harganya naik 16% sedangkan harga segelas j us naik 4%. Kenaikan harga dari semangkuk bakso dan segelas j us adalah A. 8% B. 10% C. 12% D. 15% E. 20% 4. Jika a bilangan real yang memenuhi a2 < a, maka A. a negat if C. 1 < a B. a < 1 D. ½ < a < 2 E. t idak ada a yang memenuhi 5. Aries menggambar bagian dari parabola y = x 2 − 6x + 7. Tit ik-t it ik parabola yang muncul dalam gambar memiliki absis mulai dari 0 sampai +4. Maka ordinat t erkecil dan ordinat t erbesar t it ikt it ik pada parabola yang muncul dalam gambar bert urut -t urut adalah A. −2 dan −1 B. −2 dan 7 C. −1 dan 7 D. 0 dan −1 E. 0 dan 7 6. Dua buah dadu dilemparkan bersamaan. Berapakah peluang j umlah angka yang muncul adalah 6 at au 8 ? A. 5 36 B. 7 36 C. 10 36 D. 14 36 E. 35 36 7. Tit ik A(a, b) disebut t it ik let is j ika a dan b keduanya adalah bilangan bulat . Banyaknya t it ik let is pada lingkaran yang berpusat di O dan berj ari-j ari 5 adalah A. 4 B. 6 C. 8 D. 12 E. t idak bisa dipast ikan 118 8. Mana di ant ara 5 ekspresi berikut yang angka t erakhirnya bert urut -t urut bukan 5, 6, 8, 9 at au 0? A. 5 55 5 B. 6 66 6 C. 8 88 8 D. 9 99 9. Diberikan t iga bilangan posit if x, y dan z yang semuanya berbeda. Jika maka nilai A. 1 2 x sama dengan y 3 B. 5 C. 1 10 9 D. 2 1010 E. 10 y x +y x = = , x −z z y E. 10 3 10. Jika diberikan persamaan (x 2 − x − 1) x + 2 = 1, maka banyaknya bilangan bulat x yang merupakan solusi dari persamaan t ersebut adalah A. 2 B. 3 C. 4 D. 5 E. 6 Bagian Kedua Isikan hanya j awaban saj a pada t empat yang disediakan 11. Fakt or prima t erbesar dari 2005 adalah ⋅⋅⋅⋅⋅ 12. Tent ukan semua solusi persamaan ⏐x − 1⏐ + ⏐x − 4⏐ = 2. 13. Misalkan a dan b adalah bilangan real t ak nol yang memenuhi 9a2 − 12ab + 4b 2 = 0. Tent ukan a . b 14. Diberikan dua buah persegi, A dan B, dimana luas A adalah separuh dari luas B. Jika keliling B adalah 20 cm, maka keliling A, dalam cent imet er, adalah ⋅⋅⋅⋅ 15. Seorang siswa mempunyai dua celana berwarna biru dan abu-abu, t iga kemej a berwarna put ih, merah muda dan kuning, sert a dua pasang sepat u berwarna hit am dan coklat . Banyaknya cara siswa t ersebut memakai pakaian dan sepat u adalah ⋅⋅⋅⋅⋅⋅ 16. Tent ukan semua bilangan real x yang memenuhi x 4 + 1 ≤ 2. x4 17. Tent ukan semua bilangan t iga-angka sehingga nilai bilangan it u adalah 30 kali j umlah ket iga angka it u. 18. Nilai sin875o − cos875o = ⋅⋅⋅⋅⋅⋅ 19. Diket ahui bahwa segiempat ABCD memiliki pasangan sisi yang sej aj ar. Segiempat t ersebut memiliki t epat sat u sumbu simet ri lipat j ika ia berbent uk ⋅⋅⋅⋅⋅⋅⋅⋅ 119 20. Tent ukan banyaknya pasangan bilangan bulat posit if (m, n) yang merupakan solusi dari persamaan 4 2 + = 1. m n 120 SELEKSI OLIMPIADE TINGKAT KABUPATEN/ KOTA 2005 TIM OLIMPIADE MATEMATIKA INDONESIA 2006 Prestasi itu diraih bukan didapat !!! SOLUSI SOAL Bidang Mat emat ika Disusun oleh : Eddy Hermant o, ST 121 Olimpiade Matematika Tk Kabupaten/ Kota 2005 BAGIAN PERTAMA 1. (Jawaban : E) 1 (1 + 2 )(2 + 3 )(1 − 2 )(2 − 3 ) ∴ = 1 = −1 (1 − 2) 2 2 − 3 ( ) 1 (1 + 2 )(2 + 3 )(1 − 2 )(2 − 3 ) adalah bilangan bulat negatif. 2. (Jawaban : B) Misalkan penamaan t it ik sepert i pada gambar. Pada ∆EFC berlaku ∠EFC = 180o − (c + e). Maka ∠BFG = c + e Pada ∆AGD berlaku ∠AGD = 180o − (a + d). Maka ∠FGB = a + d Pada ∆FGB berlaku ∠BFG + ∠FGB + ∠FBG = 180o. Maka (c + e) + (a + d) + (b) = 180o. ∴ a + b + c + d + e = 180 o. 3. (Jawaban : B) Kenaikan harga dari semangkuk bakso dan segelas j us = 16% ⋅ 5000 + 4% ⋅ 5000 = 10% 5000 + 5000 ∴ Kenaikan harga dari semangkuk bakso dan segelas j us adalah 10 %. 4. (Jawaban : ?) a2 < a. Maka a(a − 1) < 0 sehingga 0 < a < 1. ∴ Jika a2 < a maka 0 < a < 1. 5. (Jawaban : B) y = x 2 − 6x + 7 Nilai pada uj ung-uj ung int erval, unt uk x = 0 maka y = 7 sedangkan unt uk x = 4 maka y = −1 y maks =− b D ( −6) 2 − 4(1)( 7) = 3. =− = −2 yang didapat unt uk x = − 2a 4a 4(1) ∴ Maka ordinat t erkecil dan ordinat t erbesar adalah −2 dan 7. SMA Negeri 5 Bengkulu Eddy Hermanto, ST 122 Olimpiade Matematika Tk Kabupaten/ Kota 2005 6. (Jawaban : C) Kemungkinan penj umlahan mat a dadu sama dengan 6 ada 5, yait u (1, 5), (2, 4), (3, 3), (4, 2), (5, 1). Kemungkinan penj umlahan mat a dadu sama dengan 8 ada 5, yait u (2, 6), (3, 5), (4, 4), (5, 3), (6, 2). 5+5 36 10 ∴ Peluang j umlah angka yang muncul adalah 6 at au 8 = 36 Peluang j umlah angka yang muncul adalah 6 at au 8 = 7. (Jawaban : D) Persamaan lingkaran yang berpusat di O dan berj ari-j ari 5 adalah x 2 + y2 = 25 Karena 02 + 52 = 32 + 42 = 25 maka pasangan (x, y) bulat yang memenuhi ada 12, yait u (0, 5), (0, −5), (5, 0), ( −5, 0), (3, 4), (3, −4), ( −3, 4), ( −3, −4), (4, 3), (4, −3), ( −4, 3) dan ( −4, −3). ∴ Banyaknya t it ik let is pada lingkaran yang berpusat di O dan berj ari-j ari 5 ada 12. 8. (Jawaban : C) Karena 5k memiliki angka sat uan 5 unt uk set iap k asli maka 5 Karena 6k memiliki angka sat uan 6 unt uk set iap k asli maka 6 55 6 5 memiliki angka t erakhir 5. 66 memiliki angka t erakhir 6. 10 1010 Karena 10k memiliki angka sat uan 0 unt uk set iap k asli maka 10 81 memiliki angka sat uan 8 82 memiliki angka sat uan 4 83 memiliki angka sat uan 2 84 memiliki angka sat uan 6 85 memiliki angka sat uan 8 dst Maka 84k+i ≡ 8i (mod 10) unt uk set iap k dan i bilangan asli. 88 88 memiliki angka t erakhir 0. 8 Karena 8 habis dibagi 4 maka 8 memiliki angka sat uan yang sama dengan 84 yait u 6. 91 memiliki angka sat uan 9 92 memiliki angka sat uan 1 93 memiliki angka sat uan 9 dst Maka 92k+i ≡ 9i (mod 10) unt uk set iap k dan i bilangan asli. Karena 9k ganj il unt uk k asli maka 9 55 66 99 9 memiliki angka sat uan yang sama dengan 91 yait u 9. 88 5 6 8 9 ∴ Maka di ant ara 5 , 6 , 8 , 9 bukan 5, 6, 8, 9 at au 0 adalah 8 88 8 99 10 1010 dan 10 yang angka t erakhirnya bert urut -t urut . 9. (Jawaban : D) Misalkan Maka : y x +y x = = =k x −z z y y = k(x − z) x + y = kz x = ky ⋅⋅⋅⋅⋅⋅⋅⋅⋅⋅⋅⋅⋅⋅⋅⋅⋅⋅⋅⋅⋅⋅⋅⋅⋅⋅⋅⋅⋅ (1) ⋅⋅⋅⋅⋅⋅⋅⋅⋅⋅⋅⋅⋅⋅⋅⋅⋅⋅⋅⋅⋅⋅⋅⋅⋅⋅⋅⋅⋅ (2) ⋅⋅⋅⋅⋅⋅⋅⋅⋅⋅⋅⋅⋅⋅⋅⋅⋅⋅⋅⋅⋅⋅⋅⋅⋅⋅⋅⋅⋅ (3) SMA Negeri 5 Bengkulu Eddy Hermanto, ST 123 Olimpiade Matematika Tk Kabupaten/ Kota 2005 Jumlahkan (1) + (2) + (3) sehingga 2(x + y) = k(x + y). Karena x dan y keduanya posit if maka x + y ≠ 0 sehingga k = 2. x x =2 = k maka y y x sama dengan 2 ∴ nilai y Karena 10. (Jawaban : C) (x 2 − x − 1) x + 2 = 1 Kemungkinan-kemungkinan yang memenuhi adalah : • x + 2 = 0. Maka x = −2 (( −2) 2 − ( −2) − 1) ≠ 0 maka x = −2 memenuhi • x 2 − x − 1 = 1. Maka (x − 2)(x + 1) = 0 x = 2 dan x = −1 keduanya memenuhi • x 2 − x − 1 = −1. Maka x(x − 1) = 0 sehingga x = 0 at au x = 1 Jika x = 0 maka x + 2 = 2 (bilangan genap). Maka x = 0 memenuhi Jika x = 1 maka x + 2 = 3 (bilangan ganj il). Maka x = 1 t idak memenuhi. Nilai-nilai x yang memenuhi adalah −2, −1, 0 dan 2. ∴ Banyaknya bilangan bulat x yang merupakan solusi dari persamaan (x 2 − x − 1) x + 2 = 1 ada 4 . BAGIAN KEDUA 11. 2005 = 5 ⋅ 401 dengan 401 adalah bilangan prima. ∴ Fakt or prima t erbesar dari 2005 adalah 401. 12. ⏐x − 1⏐ + ⏐x − 4⏐ = 2 • Jika x ≤ 1 Maka ⏐x − 1⏐ = 1 − x dan ⏐x − 4⏐ = 4 − x 1 − x + 4 − x = 2 sehingga x = • • 3 (memenuhi karena x ≤ 1) 2 Jika 1 < x ≤ 4 Maka ⏐x − 1⏐ = x − 1 dan ⏐x − 4⏐ = 4 − x x − 1 + 4 − x = 2 sehingga 3 = 2 (t idak memenuhi kesamaan) Jika x > 4 Maka ⏐x − 1⏐ = x − 1 dan ⏐x − 4⏐ = x − 4 x − 1 + x − 4 = 2 sehingga x = 7 (t idak memenuhi x > 4) 2 ∴ Nilai x yang memenuhi persamaan ⏐x − 1⏐ + ⏐x − 4⏐ = 2 adalah x = SMA Negeri 5 Bengkulu 3 . 2 Eddy Hermanto, ST 124 Olimpiade Matematika Tk Kabupaten/ Kota 2005 13. 9a2 − 12ab + 4b 2 = 0 2 ⎞ ⎛ a Unt uk b ≠ 0 maka ⎜⎜ 3 − 2 ⎟⎟ = 0 ⎠ ⎝ b a 2 = ∴ Maka 3 b 14. Luas B = 2 Luas A, maka B = 2A Misalkan panj ang sisi A = x dan panj ang sisi B = y maka Luas B = y 2 = 2x 2 sehingga y = x √2 5 2 2 Keliling B = 4y. Maka 4x √2 = 20 sehingga x = Keliling A = 4x = 10√2 ∴ Keliling A = 10 √2 cm 15. Banyaknya cara siswa t ersebut memakai pakaian dan sepat u = 2 x 3 x 2 = 12 cara ∴ Banyaknya cara siswa t ersebut memakai pakaian dan sepat u adalah 12. 16. x 4 + 1 ≤2 x4 Sesuai dengan ket aksamaan AM-GM maka x Karena x ⎛ ⎜⎜ x ⎝ 2 4 1 − 2 x + 1 ≤ 2 dan x x4 4 + 4 + 1 ≥2 x x4 4 ⋅ 1 =2 x4 1 ≥ 2 maka ket aksamaan hanya dipenuhi j ika x x4 4 + 1 = 2. x4 2 ⎞ ⎟⎟ = 0 ⎠ ∴ Bilangan real x yang memenuhi persamaan adalah x = 1 atau x = −1 17. Misalkan bilangan t ersebut adalah n = 100a + 10b + c 100a + 10b + c = 30(a + b + c) 10(7a − 2b) = 29c c 10 = 7a − 2b 29 Karena 10 dan 29 relat if prima maka 7a − 2b = 29k dan c = 10k. Karena 0 ≤ c ≤ 9 maka nilai k yang memenuhi hanya k = 0 sehingga c = 0. 7a = 2b Karena 2 dan 7 relat if prima sedangkan 0 ≤ a, b ≤ 9 maka nilai a dan b yang memenuhi adalah a = 2 dan b = 7. ∴ Bilangan t iga angka yang memenuhi adalah 270. SMA Negeri 5 Bengkulu Eddy Hermanto, ST 125 Olimpiade Matematika Tk Kabupaten/ Kota 2005 18. sin875o − cos875o = (sin475o + cos475o) (sin475o − cos475o) sin875o − cos875o = ((sin275o+cos275o) 2 − 2(sin 275o)(cos275o)) (sin275o+cos275o)(sin275o − cos275o) Mengingat bahwa sin2 α + cos2 α = 1, sin 2α = 2 sin α cos α dan cos2α − sin2α = cos 2α maka : sin875o − cos875o = (1 − ½ sin2150o)( −cos 120o) ∴ sin875o − cos875o = 7 16 19. Jika segiempat adalah t rapesium sebarang maka belum dapat dipast ikan bangun t ersebut memiliki t epat sat u sumbu simet ri lipat sebab ada kemungkinan t rapesium t ersebut t idak memiliki sumbu simet ri lipat . ∴ Maka bangun t ersebut adalah trapesium sama kaki. 20. 4 2 + =1 m n mn − 4n − 2m = 0 (m − 4)(n − 2) = 8 = 23 Karena 4 dan 2 memiliki parit as yang sama maka m − 4 dan n − 2 memiliki parit as yang sama. Maka kemungkinan-kemungkinan penyelesaiannya adalah : • m − 4 = −2 dan n − 2 = −4 m = 2 dan n = −2 (t idak memenuhi m dan n keduanya bulat posit if ) • m − 4 = 2 dan n − 2 = 4 m = 6 dan n = 6 (memenuhi m dan n keduanya bulat posit if ) • m − 4 = −4 dan n − 2 = −2 m = 0 dan n = 0 (t idak memenuhi m dan n keduanya bulat posit if ) • m − 4 = 4 dan n − 2 = 2 m = 8 dan n = 4 (memenuhi m dan n keduanya bulat posit if ) ∴ Banyaknya pasangan bilangan bulat posit if (m, n) yang memenuhi ada 2. SMA Negeri 5 Bengkulu Eddy Hermanto, ST 126 SELEKSI OLIMPIADE MATEMATIKA INDONESIA 2006 TINGKAT PROVINSI Bidang Mat emat ika Bagian Per t ama Waktu : 90 Menit DEPARTEMEN PENDIDIKAN NASIONAL DIREKTORAT JENDERAL PENDIDIKAN DASAR DAN MENENGAH DIREKTORAT PENDIDIKAN MENENGAH UMUM TAHUN 2005 127 OLIMPIADE MATEMATIKA TINGKAT PROVINSI TAHUN 2005 BAGIAN PERTAMA 1. Jika a sebuah bilangan rasional dan b adalah sebuah bilangan t ak rasional, maka a + b adalah bilangan ⋅⋅⋅⋅⋅⋅ 2. Jumlah sepuluh bilangan prima yang pert ama adalah ⋅⋅⋅⋅⋅ 3. Banyaknya himpunan X yang memenuhi {1, 2} ⊆ X ⊆ {1, 2, 3, 4, 5} adalah ⋅⋅⋅⋅⋅ 4. Jika N = 123456789101112 ⋅⋅⋅ 9899100, maka t iga angka pert ama N adalah ⋅⋅⋅⋅⋅ 5. Misalkan ABCD adalah sebuah t rapesium dengan BC║AD. Tit ik-t it ik P dan R bert urut -t urut adalah t it ik t engah sisi AB dan CD. Tit ik Q t erlet ak pada sisi BC sehingga BQ : QC = 3 : 1, sedangkan t it ik S t erlet ak pada sisi AD sehingga AS : SD = 1 : 3. Maka rasio luas segiempat PQRS t erhadap luas t rapesium ABCD adalah ⋅⋅⋅⋅⋅⋅ 6. Bilangan t iga-angka t erkecil yang merupakan bilangan kuadrat sempurna dan bilangan kubik (pangkat t iga) sempurna sekaligus adalah ⋅⋅⋅⋅⋅ 7. Jika a, b dua bilangan asli a ≤ b sehingga 3+ a 4+ b adalah bilangan rasional, maka pasangan t erurut (a, b) = ⋅⋅⋅⋅⋅⋅ Jika AB = AC, AD = BD, dan besar sudut DAC = 39o, maka besar sudut BAD adalah ⋅⋅⋅⋅⋅ 8. 9. Ket ika mendaki sebuah bukit , seorang berj alan dengan kecepat an 1½ km/ j am. Ket ika menuruni bukit t ersebut , ia berj alan t iga kali lebih cepat . Jika wakt u yang dibut uhkan unt uk melakukan perj alanan bolak-balik dari kaki bukit ke puncak bukit dan kembali ke kaki bukit adalah 6 j am, maka j arak ant ara kaki bukit dan puncak bukit (dalam km) adalah ⋅⋅⋅⋅ 10. Sebuah segienam berat uran dan sebuah segit iga sama sisi mempunyai keliling yang sama. Jika luas segit iga adalah 3 , maka luas segienam adalah ⋅⋅⋅⋅ 11. Dua buah dadu dilemparkan secara bersamaan. Peluang j umlah kedua angka yang muncul adalah bilangan prima adalah ⋅⋅⋅⋅ 128 12. Keliling sebuah segit iga samasisi adalah p. Misalkan Q adalah sebuah t it ik di dalam segit iga t ersebut . Jika j umlah j arak dari Q ke ket iga sisi segit iga adalah s, maka, dinyat akan dalam s, p = ⋅⋅⋅⋅⋅⋅⋅⋅ 13. Barisan bilangan asli (a, b, c) dengan a ≥ b ≥ c, yang memenuhi sekaligus kedua persamaan ab + bc = 44 dan ac + bc = 23 adalah ⋅⋅⋅⋅⋅ 14. Empat buah t it ik berbeda t erlet ak pada sebuah garis. Jarak ant ara sebarang dua t it ik dapat diurut kan menj adi barisan 1, 4, 5, k, 9, 10. Maka k = ⋅⋅⋅⋅ 15. Sebuah kelompok t erdiri dari 2005 anggot a. Set iap anggot a memegang t epat sat u rahasia. Set iap anggot a dapat mengirim surat kepada anggot a lain manapun unt uk menyampaikan seluruh rahasia yang dipegangnya. Banyaknya surat yang perlu dikirim agar semua anggot a kelompok menget ahui seluruh rahasia adalah ⋅⋅⋅⋅⋅ 16. Banyaknya pasangan bilangan bulat (x, y) yang memenuhi persamaan 2xy − 5x + y = 55 adalah ⋅⋅⋅⋅ 17. Himpunan A dan B saling lepas dan A ∪ B = {1, 2, 3, ⋅⋅⋅, 9}. Hasil perkalian semua unsur A sama dengan j umlah semua unsur B. Unsur t erkecil B adalah ⋅⋅⋅⋅ 18. Bent uk sederhana dari adalah ⋅⋅⋅⋅ (2 (2 3 3 )( )( )( )( ) ( ) ( ) ) − 1 33 − 1 4 3 − 1 100 3 − 1 L + 1 33 + 1 4 3 + 1 100 3 + 1 19. Misalkan ABCD adalah limas segit iga berat uran, yait u bangun ruang bersisi empat yang berbent uk segit iga samasisi. Misalkan S adalah t it ik t engah rusuk AB dan T t it ik t engah rusuk CD. Jika panj ang rusuk ABCD adalah 1 sat uan panj ang, maka panj ang ST adalah ⋅⋅⋅⋅ 20. Unt uk sembarang bilangan real a, not asi ⎣a⎦ menyat akan bilangan bulat t erbesar yang lebih kecil dari at au sama dengan a. Jika x bilangan real yang memenuhi x − ⎣x ⎦ t idak akan lebih besar dari ⋅⋅⋅⋅⋅ 129 ⎣x ⎦ + 3 = ⎣x ⎦ + ⎣ 3⎦, maka SELEKSI OLIMPIADE MATEMATIKA INDONESIA 2006 TINGKAT PROVINSI Bidang Mat emat ika Bagian Kedua Waktu : 120 Menit DEPARTEMEN PENDIDIKAN NASIONAL DIREKTORAT JENDERAL PENDIDIKAN DASAR DAN MENENGAH DIREKTORAT PENDIDIKAN MENENGAH UMUM TAHUN 2005 130 OLIMPIADE MATEMATIKA TINGKAT PROVINSI TAHUN 2005 BAGIAN KEDUA 1. Panj ang sisi t erbesar pada segiempat t alibusur ABCD adalah a, sedangkan j ari-j ari lingkaran luar ∆ACD adalah 1. Tent ukan nilai t erkecil yang mungkin bagi a. Segiempat ABCD yang bagaimana yang memberikan nilai a sama dengan nilai t erkecil t ersebut ? 2. Di dalam sebuah kot ak t erdapat 4 bola yang masing-masing bernomor 1, 2, 3 dan 4. Anggi mengambil bola secara acak, mencat at nomornya, dan mengembalikannya ke dalam kot ak. Hal yang sama ia lakukan sebanyak 4 kali. Misalkan j umlah dari keempat nomor bola yang t erambil adalah 12. Berapakah peluang bola yang t erambil selalu bernomor 3 ? 3. Jika α, β dan γ adalah akar-akar persamaan x 3 − x − 1 = 0, t ent ukan 1+α 1+ β 1+ γ + + 1−α 1− β 1− γ 4. Panj ang ket iga sisi a, b, c dengan a ≤ b ≤ c, sebuah segit iga siku-siku adalah bilangan bulat . Tent ukan semua barisan (a, b, c) agar nilai keliling dan nilai luas segit iga t ersebut sama. 5. Misalkan A dan B dua himpunan, masing-masing beranggot akan bilangan-bilangan asli yang berurut an. Jumlah rat a-rat a arit mat ika unsur-unsur A dan rat a-rat a arit mat ika unsur-unsur B adalah 5002. Jika A ∩ B = {2005}, t ent ukan unsur t erbesar yang mungkin dari himpunan A ∪ B. 131 SELEKSI OLIMPIADE MATEMATIKA INDONESIA 2006 TINGKAT PROVINSI TAHUN 2005 Prestasi itu diraih bukan didapat !!! SOLUSI SOAL Bidang Mat emat ika Bagian Pertama Disusun oleh : Eddy Hermant o, ST 132 Solusi Olimpiade Matematika Tk Provinsi 2005 Bagian Pert ama BAGIAN PERTAMA 1. Bilangan rasional + bilangan t ak rasional = bilangan t ak rasional ∴ a + b adalah bilangan tak rasional. 2. Sepuluh bilangan prima pert ama adalah 2, 3, 5, 7, 11, 13, 17, 19, 23, 29. 2 + 3 + 5 + 7 + 11 + 13 + 17 + 19 + 23 + 29 = 129 ∴ Jumlah sepuluh bilangan prima pert ama = 129 3. {1, 2} ⊆ X ⊆ {1, 2, 3, 4, 5} X t erdiri dari sedikit nya 2 unsur dan maksimal 5 unsur dengan 2 unsur di ant aranya haruslah 1 dan 2. Sedangkan sisanya dipilih dari unsur-unsur 3, 4 at au 5. Jika X t erdiri dari 2 unsur maka banyaknya himpunan X = 3C0 = 1 Jika X t erdiri dari 3 unsur maka banyaknya himpunan X = 3C1 = 3 Jika X t erdiri dari 4 unsur maka banyaknya himpunan X = 3C2 = 3 Jika X t erdiri dari 5 unsur maka banyaknya himpunan X = 3C3 = 1 ∴ Banyaknya himpunan X = 1 + 3 + 3 + 1 = 8. 4. N = 123456789101112 ⋅⋅⋅ 9899100 Banyaknya angka 123456789 adalah 9. Karena 10, 11, ⋅⋅⋅, 99 adalah bilangan 2 angka maka banyaknya digit 101112⋅⋅⋅99 adalah genap. Banyaknya angka 100 = 3 Maka banyaknya angka N adalah merupakan bilangan genap. Mengingat 3502 = 122500, 3512 = 123201, 3522 = 123904, 1102 = 12100, 1112 = 12321, 1122 =12544 N adalah 351 at au 111. Akan dibukt ikan bahwa j ika t iga angka pert ama N adalah 111 maka banyaknya digit ⎣N⎦ akan ganj il sedangkan j ika t iga angka pert ama N adalah 351 maka banyaknya digit ⎣N⎦ akan genap. maka kemungkinan t iga angka pert ama dari N = (111 ⋅ 10k + p) 2 = 12321 ⋅102k + 222p ⋅ 10k + p 2 dengan banyaknya angka ⎣p ⎦ t idak lebih dari k. Karena banyaknya angka ⎣p ⎦ t idak lebih dari k maka p < 10k. N < 12321 ⋅ 102k + 222 ⋅ 102k + 102k = 12544 ⋅ 102k 12321 ⋅ 102k < N < 12544 ⋅ 102k Maka banyaknya angka N sama dengan banyaknya angka 12321 ⋅ 102k yang merupakan bilangan ganj il. N = (351 ⋅ 10k + p) 2 = 123201⋅102k + 702p ⋅10k + p 2 dengan banyaknya angka ⎣p ⎦ t idak lebih dari k. Karena banyaknya angka ⎣p ⎦ t idak lebih dari k maka p < 10k. N < 123201 ⋅ 102k + 702 ⋅ 102k + 102k = 123904 ⋅ 102k 123201 ⋅ 102k < N < 123904 ⋅ 102k Maka banyaknya angka N sama dengan banyaknya angka 123201 ⋅ 102k yang merupakan bilangan genap. ∴ Tiga angka pert ama N adalah 351. SMA Negeri 5 Bengkulu Eddy Hermanto, ST 133 Olimpiade Matematika Tk Provinsi 2005 Solusi Bagian Pert ama 5. Misalkan [ PQRS] menyat akan luas segiempat PQRS Misalkan j uga j arak ant ara garis AD dan BC adalah t [ ABCD] = ½ (AD + BC) ⋅ t Karena P dan R berurut an adalah pert engahan AB dan CD maka PR sej aj ar CD dan berlaku : PR = ½ (AD + BC) Jarak t it ik Q ke PR = j arak t it ik S ke PR = ½ t [ PQRS] = [ PQR] + [ PRS] = ½ (½t )(PR) + ½ (½t )(PR) [ PQRS] = (½t )(PR) = ½ (½ (AD + BC) ⋅ t ) = ½ [ ABCD] ∴ Rasio luas segiempat PQRS t erhadap luas t rapesium ABCD adalah 1 : 2 6. Bilangan kuadrat yang j uga merupakan bilangan pangkat t iga adalah bilangan pangkat enam. 26 = 64 dan 36 = 729 ∴ Bilangan t iga-angka t erkecil yang merupakan bilangan kuadrat sempurna dan bilangan kubik (pangkat t iga) sempurna sekaligus adalah 729. 3+ a 7. (q 4+ b 3 − 2p = p dengan a, b, p dan q asli dan a ≤ b sert a p dan q keduanya relat if prima. q ) = (p 2 b −q a ) 2 3q 2 + 4p 2 − 4pq 3 = p 2b + q 2a − 2pq ab Karena a, b, p, q semuanya asli maka 2 3 = ab sehingga ab = 12. Kemungkinan pasangan (a, b) yang memenuhi adalah (1, 12), (2, 6) dan (3, 4) Jika a = 1 dan b = 12 maka Jika a = 2 dan b = 6 maka Jika a = 3 dan b = 4 maka 3+ a 4+ b 3+ a 4+ b 3+ a 4+ b = = 1 yang merupakan bilangan rasional. 2 1 yang bukan merupakan bilangan rasional. 2 = 3 yang bukan merupakan bilangan rasional. 2 ∴ Pasangan t erurut (a, b) adalah (1, 12) SMA Negeri 5 Bengkulu Eddy Hermanto, ST 134 Solusi Olimpiade Matematika Tk Provinsi 2005 Bagian Pert ama 8. Misalkan ∠BAD = α Karena AD = BD maka ∠ABD = α Karena AB = AC maka ∠ACB = α Pada ∆ABC berlaku ( α) + ( α + 39o) + ( α) = 180o. Maka α = 47o ∴ Besarnya sudut BAD = 47 o. 9. vn = 1½ km/ j am dan v t = 4½ km/ j am Misalkan j arak ant ara kaki bukit dan puncak bukit dalam km adalah s. 27 s s + = 6 maka s = km 4 1,5 4,5 ∴ Jarak ant ara kaki bukit dan puncak bukit = 27 km 4 10. Karena keliling segienam berat uran sama dengan keliling segit iga sama sisi maka panj ang sisi segit iga berat uran dua kali panj ang sisi segienam berat uran. Misalkan panj ang sisi segienam berat uran = a maka panj ang sisi segit iga sama sisi = 2a. Luas segit iga sama sisi = ½ (2a) 2 sin 60o = 3 a=1 Pada segienam berat uran, j ari-j ari lingkaran luar segienam berat uran sama dengan panj ang sisinya. Luas segienam berat uran = 6 ⋅ ½ (a2) sin 60o ∴ Luas segienam berat uran = 3 3 2 11. Kemungkinan penj umlahan dua angka dadu bilangan prima adalah 2, 3, 5, 7, at au 11. * Jika j umlah angka dadu = 2 maka banyaknya kemungkinan ada 1, yait u (1, 1) * Jika j umlah angka dadu = 3 maka banyaknya kemungkinan ada 2, yait u (1, 2), (2, 1) * Jika j umlah angka dadu = 5 maka banyaknya kemungkinan ada 4, yait u (1, 4), (2, 3), (3, 2), (4, 1) * Jika j umlah angka dadu = 7 maka banyaknya kemungkinan ada 6, yait u (1, 6), (2, 5), (3, 4), (4, 3), (5, 2), (6, 1) * Jika j umlah angka dadu = 11 maka banyaknya kemungkinan ada 2, yait u (5, 6), (6, 5) Banyaknya kemungkinan seluruhnya = 1 + 2 + 4 + 6 + 2 = 15 ∴ Peluang j umlah kedua angka dadu yang muncul adalah bilangan prima = SMA Negeri 5 Bengkulu 15 36 Eddy Hermanto, ST 135 Solusi Olimpiade Matematika Tk Provinsi 2005 Bagian Pert ama 12. Misalkan segit iga t ersebut adalah ∆ABC. Maka AB + AC + BC = p sehingga AB = AC = BC = 1 p 3 2 1 2 1 ⎛1 ⎞ p 3 dan QP + QR + QS = s Luas ∆ABC = ⎜⎜ p ⎟⎟ sin 60° = 36 2 ⎝3 ⎠ Luas ∆ABC = Luas ∆ABQ + Luas ∆ACQ + Luas ∆BCQ = ½ AB QS + ½ AC QR + ½ BC QP 1 ⎛1 ⎞ 1 2 p 3 = ⎜⎜ p ⎟⎟(s ) 2 ⎝3 ⎠ 36 ∴ p = 2s 3 13. ab + bc = 44 dan ac + bc = 23 dengan a, b, c asli dan a ≥ b ≥ c Karena c(a + b) = 23 dengan a, b dan c asli maka c = 1 at au 23 Jika c = 23 maka a + b = 1 (t idak memenuhi sebab a + b ≥ 2). Maka c = 1 a + b = 23 dan ab + b = 44 (23 − b)b + b = 44, maka b 2 − 24b + 44 = 0 sehingga (b − 22)(b − 2) = 0 b = 2 at au b = 22 Jika b = 22 maka a = 1 (t idak memenuhi a ≥ b). Maka b = 2 dan a = 21 ∴ Barisan bilangan asli (a, b, c) yang memenuhi adalah (21, 2, 1). 14. Misal garis t ersebut t erlet ak pada sumbu X. Angap t it ik A adalah t it ik paling kiri, D paling kanan sert a B dan C t erlet ak di ant ara A dan D dengan t it ik t erdekat pada A adalah B. Tanpa mengurangi keumuman misalkan t it ik A berada pada x = 0 dan D pada koordinat x = 10. Karena ada yang berj arak 1 dan 9 maka salah sat u B berada di x = 1 at au C pada x = 9 • Jika B t erlet ak pada x = 1 Jarak B dan D = 9 Karena harus ada dua t it ik yang berj arak 4 maka kemungkinan posisi C ada di x = 4, 5 at au 6. Posisi C t idak mungkin di x = 4 sebab akan membuat j arak ant ara sebarang dua t it ik adalah 1, 3, 4, 6, 9, 10. Posisi C t idak mungkin di x = 5 sebab akan membuat j arak ant ara sebarang dua t it ik adalah 1, 4, 5, 9, 10 (t idak ada nilai k) Maka posisi C di x = 6 yang akan membuat j arak dua t it ik sebarang adalah 1, 4, 5, 6, 9, 10 k=6 • Jika C t erlet ak pada x = 9 Jarak C dan A = 9 SMA Negeri 5 Bengkulu Eddy Hermanto, ST 136 Solusi Olimpiade Matematika Tk Provinsi 2005 Bagian Pert ama Karena harus ada dua t it ik yang berj arak 4 maka kemungkinan posisi B ada di x = 4, 5 at au 6. Posisi B t idak mungkin di x = 6 sebab akan membuat j arak ant ara sebarang dua t it ik adalah 1, 3, 4, 6, 9, 10. Posisi B t idak mungkin di x = 5 sebab akan membuat j arak ant ara sebarang dua t it ik adalah 1, 4, 5, 9, 10 (t idak ada nilai k) Maka posisi B di x = 4 yang akan membuat j arak dua t it ik sebarang adalah 1, 4, 5, 6, 9, 10 k=6 ∴ Maka k = 6 15. Secara umum unt uk kelompok t erdiri dari n anggot a. Orang ke-k akan menerima surat set elah sedikit nya t erj adi k − 2 t elepon. Maka orang t erakhir akan menerima surat yang pert ama sedikit nya set elah t erj adi n − 2 kiriman surat . Set elah orang ke-n menerima surat berart i sedikit nya t elah t erj adi n − 1 kiriman surat . Semua inf ormasi yang didapat oleh orang ke-n akan disebar kepada seluruh orang selain dirinya. Sedikit nya dibut uhkan n − 1 surat . Maka banyaknya surat minimum yang diperlukan sehingga set iap orang akan menget ahui n inf ormasi adalah 2(n − 1) ∴ Banyaknya surat yang diperlukan adalah 4008. 16. 2xy − 5x + y = 55, maka (2x + 1)(2y − 5) = 105 = 3 ⋅ 5 ⋅ 7 Maka 2y − 5 membagi 105 sehingga 2y − 5 = ±1, ±3, ±5, ±7, ±15, ±21, ±35, ±105. Karena 105 merupakan perkalian bilangan ganj il maka semua f akt or 105 adalah bilangan ganj il. Karena penj umlahan dua bilangan ganj il adalah bilangan genap yang past i habis dibagi 2 maka berapa pun f akt or posit if dan f akt or negat if dari 105 akan membuat 2x + 1 dan 2y − 5 keduanya membagi f akt or dari 105 t ersebut . 105 memiliki 8 f akt or posit if dan 8 f akt or negat if . ∴ Maka banyaknya pasangan bilangan bulat (x, y) yang memenuhi adalah 16. 17. Misalkan hasil perkalian semua unsur A = p dan penj umlahan semua unsur B = s, maka p = s Himpunan A dapat t erdiri dari 1 at au lebih unsur. 1 + 2 + 3 + 4 + 5 + 6 + 7 + 8 + 9 = 45 * Andaikan 1 adalah unsur t erkecil B. • Jika A t erdiri dari sedikit nya 4 unsur Karena 1 bukanlah unsur dari A maka p ≥ 2 ⋅ 3 ⋅ 4 ⋅ 5 = 120 > 45 (t idak dapat t ercapai p=s) • Jika A t erdiri dari 3 unsur Misalkan ket iga unsur A t ersebut adalah a, b dan c. Jelas bahwa abc < 45 Kemungkinan unsur-unsur A adalah (2, 3, 4), (2, 3, 5), (2, 3, 6), (2, 3, 7) dan (2, 4, 5) Jika unsur-unsur A adalah (2, 3, 4) maka p = 24 dan s = 45 − 9 = 36 (t idak memenuhi p=s) Jika unsur-unsur A adalah (2, 3, 5) maka p = 30 dan s = 45 − 10 = 35 (t idak memenuhi p=s) Jika unsur-unsur A adalah (2, 3, 6) maka p = 36 dan s = 45 − 11 = 34 (t idak memenuhi p=s) Jika unsur-unsur A adalah (2, 3, 7) maka p = 42 dan s = 45 − 12 = 33 (t idak memenuhi p=s) Jika unsur-unsur A adalah (2, 4, 5) maka p = 40 dan s = 45 − 11 = 34 (t idak memenuhi p=s) Maka j ika A t erdiri dari 3 unsur maka t idak ada yang memenuhi p = s. • Jika A t erdiri dari 2 unsur Misalkan kedua unsur A t ersebut adalah a dan b dengan 1 ≤ a, b ≤ 9. Karena p = s maka ab = 45 − a − b sehingga (a + 1)(b + 1) = 46 = 23 ⋅ 2 Misalkan a > b maka a + 1 = 23 dan b + 1 = 2. Maka a = 22 (t idak memenuhi a ≤ 9) SMA Negeri 5 Bengkulu Eddy Hermanto, ST 137 Olimpiade Matematika Tk Provinsi 2005 Solusi Bagian Pert ama • Jika A t erdiri dari 1 unsur p ≤ 9 sedangkan s ≥ 45 − 9 = 36 (t idak mungkin t ercapai p = s) * Andaikan 2 adalah unsur t erkecil B Jika A = {1, 4, 8} dan B = {2, 3, 5, 6, 7, 9} maka : p = 1 ⋅ 4 ⋅ 8 = 32 dan s = 45 − 1 − 4 − 8 = 32 (t erpenuhi p = s) ∴ Unsur t erkecil dari B adalah 2 . (2 (2 18. Misalkan k 3 −1 = 3 3 )( )( )( )( ) ( ) ( ) ) − 1 33 − 1 4 3 − 1 100 3 − 1 L =X + 1 33 + 1 4 3 + 1 100 3 + 1 ( k − 1)( k 2 + k + 1) + 1 ( k + 1)( k − k + 1) (2 − 1)(3 − 1)(4 − 1) (100 − 1) (2 2 + 2 + 1)(3 2 + 3 + 1)(4 2 + 4 + 1) (100 2 + 100 + 1) X = L L (2 + 1)(3 + 1)(4 + 1) (100 + 1) (2 2 − 2 + 1)(3 2 − 3 + 1)(4 2 − 4 + 1) (100 2 − 100 + 1) k 3 2 Perhat ikan bahwa n 2 + n + 1 = (n + 1) 2 − (n + 1) + 1. Maka 22 + 2 + 1 = 32 − 3 + 1 ; 32 + 3 + 1 = 42 − 4 + 1 dan set erusnya. 1 ⋅ 2 ⋅ 3 ⋅ L 99 100 2 + 100 + 1 ⋅ 3 ⋅ 4 ⋅ 5 ⋅ L101 22 − 2 + 1 2 10101 1 X = ⋅ = ⋅ 3367 100 ⋅ 101 3 50 ⋅ 101 2 3 − 1 33 − 1 4 3 − 1 100 3 − 1 3367 = L ∴ 2 3 + 1 33 + 1 4 3 + 1 100 3 + 1 5050 X = ( ( )( )( )( )( ) ( ) ( ) ) 19. Karena ∆ABD sama sisi dan S pert engahan AB maka DS garis t inggi. DS = AD sin60o = ½ 3 . Dengan cara yang sama CS = ½ 3 . Maka ∆CDS sama kaki. Karena ∆CDS sama kaki dan T pert engahan CD maka ST t egak lurus DT. ST2 = DS2 − DT2 2 ⎛1⎞ ⎛1 ⎞ 3 ⎟⎟ − ⎜⎜ ⎟⎟ ST = ⎜⎜ ⎝2⎠ ⎝2 ⎠ ∴ ST = 2 1 2 2 SMA Negeri 5 Bengkulu Eddy Hermanto, ST 138 Olimpiade Matematika Tk Provinsi 2005 Solusi ⎣ ⎣x Bagian Pert ama ⎦ ⎣ 3⎦ + 3⎦ > x + 3 − 1 ⎣x ⎦ + ⎣ 3 ⎦ > x + 3 − 1 Mengingat ⎣ 3 ⎦ = 1 maka : 20. x + 3 = ⎣x ⎦ + x − ⎣x ⎦ < 2 − 3 ⎣ ⎦= 2− Jika x − x ⎣ ⎦ 3 maka x + 3 = ⎣x ⎦ + ⎣ 3⎦ akan menj adi kesamaan t idak mungkin t erj adi. ⎦ kurang sedikit dari 2 − ⎣x ⎦ + 1 = ⎣x ⎦ + 1 sehingga kesamaan t erj adi. ∴ Maka x − ⎣x ⎦ t idak akan lebih besar dari Jika x − ⎣x 3 maka ⎣x ⎦ ⎣x ⎦ + 2 = ⎣x ⎦ + 1 + 3 = ⎣x ⎦ + ⎣ 3⎦ akan sehingga menj adi 2− 3. SMA Negeri 5 Bengkulu Eddy Hermanto, ST 139 SELEKSI OLIMPIADE MATEMATIKA INDONESIA 2006 TINGKAT PROVINSI TAHUN 2005 Prestasi itu diraih bukan didapat !!! SOLUSI SOAL Bidang Mat emat ika Bagian Kedua Disusun oleh : Eddy Her mant o, ST 140 Olimpiade Matematika Tk Provinsi 2005 Solusi Bagian Kedua BAGIAN KEDUA 1. Misalkan ABCD adalah segiempat t ali busur t ersebut dan O adalah pusat lingkaran. Karena lingkaran t ersebut j uga merupakan lingkaran luar ∆ABC maka sesuai dalil sinus : AB sin ∠ACB = 2 R = 2 dengan R menyat akan j ari-j ari lingkaran luar ∆ABC Karena ∠AOB = 2∠ACB maka : ⎛ ∠AOB ⎞ ⎟ ⎝ 2 ⎠ AB = 2 sin ⎜ Dengan cara yang sama didapat : ⎛ ∠BOC ⎞ ⎟ ⎝ 2 ⎠ ⎛ ∠COD ⎞ CD = 2 sin ⎜ ⎟ ⎝ 2 ⎠ ⎛ ∠AOD ⎞ AD = 2 sin ⎜ ⎟ ⎝ 2 ⎠ BC = 2 sin ⎜ ∠AOB + ∠BOC + ∠COD + ∠AOD = 360o Maka min( ∠AOB, ∠BOC, ∠COD, ∠AOD) ≤ 90o Diket ahui bahwa a = maks (AB, BC, CD, AD) Karena unt uk 0o ≤ x ≤ 90o nilai sin x naik maka : ⎛ 90° ⎞ ⎟⎟ a = maks (AB, BC, CD, AD) ≥ 2 sin ⎜⎜ ⎝ 2 ⎠ a≥ 2 Maka nilai minimal a = 2 Karena maks( ∠AOB, ∠BOC, ∠COD, ∠AOD) = min( ∠AOB, ∠BOC, ∠COD, ∠AOD) = 90o maka : ∠AOB = ∠BOC = ∠COD = ∠AOD = 90o yang berart i AB = BC = CD = AD. Karena ∠AOD = 90o sedangkan ∆AOD sama kaki maka ∠DOA = 45o. Dengan cara yang sama didapat ∠COD = 45o yang berart i segiempat ABCD adalah persegi. ∴ Maka nilai a terkecil adalah 2 yang membuat segiempat ABCD adalah persegi. SMA Negeri 5 Bengkulu Eddy Hermanto, ST 141 Olimpiade Matematika Tk Provinsi 2005 Solusi Bagian Kedua 2. Kemungkinan empat j enis bola yang t erambil adalah : • Keempat bola t ersebut adalah (1, 3, 4, 4) Karena ada 4 obyek dan t erdapat 2 yang sama maka banyaknya kemungkinan = • Semua kemungkinannya adalah (1, 3, 4, 4); (1, 4, 3, 4); (1, 4, 4, 3); (3, 1, 4, 4); (3, 4, 1, 4); (3, 4, 4, 1); (4, 1, 3, 4); (4, 1, 4, 3); (4, 3, 1, 4); (4, 3, 4, 1); (4, 4, 1, 3); (4, 4, 3, 1). Keempat bola t ersebut adalah (2, 3, 3, 4) Banyaknya kemungkinan = • 4! = 12 2! Semua kemungkinannya adalah (2, 3, 3, 4); (2, 3, 4, 3); (2, 4, 3, 3); (3, 2, 3, 4); (3, 2, 4, 3); (3, 3, 2, 4); (3, 3, 4, 2); (3, 4, 2, 3); (3, 4, 3, 2); (4, 2, 3, 3); (4, 3, 2, 3); (4, 3, 3, 2). Keempat bola t ersebut adalah (2, 2, 4, 4) Banyaknya kemungkinan = • 4! = 12 2! 4! =6 2!⋅2! Semua kemungkinannya adalah (2, 2, 4, 4); (2, 4, 2, 4); (2, 4, 4, 2); (4, 2, 2, 4); (4, 2, 4, 2); (4, 4, 2, 2). Keempat bola t ersebut adalah (3, 3, 3, 3) Banyaknya kemungkinan = 4! =1 4! Semua kemungkinannya adalah (3, 3, 3, 3) Tot al banyaknya kemungkinan adalah 12 + 12 + 6 + 1 = 31 Hanya ada sat u cara kemungkinan angka yang muncul selalu 3. ∴ Peluang bola yang t erambil selalu bernomor 3 adalah = 1 31 3. Dari x 3 − x − 1 = 0 sert a Ax 3 + Bx 2 + Cx + D = 0 didapat A = 1, B = 0, C = −1 dan D = −1 B =0 A −1 C = = −1 αβ + αγ + βγ = A 1 (−1) D = − =1 αβγ = − A 1 (1 + α )(1 − β )(1 − γ ) + (1 + β )(1 − α )(1 − γ ) + (1 + γ )(1 − α )(1 − γ ) 1+α 1+ β 1+ γ = + + (1 − α )(1 − β )(1 − γ ) 1−α 1− β 1−γ 3 − (α + β + γ ) − (αβ + αγ + βγ ) + 3αβγ = 1 − (α + β + γ ) + (αβ + αγ + βγ ) − αβγ 3 − ( 0) − ( −1) + 3(1) = 1 − ( 0) + ( −1) − (1) α +β +γ = − = ∴ −7 1+α 1+ β 1+ γ = −7 + + 1−α 1− β 1−γ SMA Negeri 5 Bengkulu Eddy Hermanto, ST 142 Olimpiade Matematika Tk Provinsi 2005 Solusi 4. Bagian Kedua b = c2 − a2 ⋅⋅⋅⋅⋅⋅⋅⋅⋅⋅⋅⋅⋅⋅⋅⋅⋅⋅⋅⋅⋅⋅⋅⋅⋅⋅⋅⋅⋅⋅⋅ (1) Luas ∆ABC = ½ab = a + b + c, maka ab = 2(a + b + c) ⋅⋅⋅⋅⋅⋅⋅⋅⋅⋅⋅⋅⋅⋅⋅⋅⋅⋅⋅⋅⋅⋅⋅⋅⋅⋅⋅⋅⋅ (2) Karena a, b dan c adalah bilangan bulat maka sekurang-kurangnya salah sat u di ant ara a at au b adalah kelipat an 2. Jika a = 2k dengan k ∈ bilangan asli maka : 2k c k c 2 2 − 4k − 4k 2 2 = 2(2k + = (2k + c 2 − 4k c − 4k 2 2 2 + c) + c) (k − 1) c − 4k = c + 2k (k − 1) (c + 2k )(c − 2k ) = (c + 2k ) (k − 1) (c − 2k ) = (c + 2k ) (k − 1) (c − 2k ) = c − 2k + 4k (c − 2k )(k − 2k ) = 4k Karena k ≠ 0 maka (c − 2k )(k − 2 ) = 4 2 2 2 2 2 2 2 ⋅⋅⋅⋅⋅⋅⋅⋅⋅⋅⋅⋅⋅⋅⋅⋅⋅⋅⋅⋅⋅⋅⋅⋅⋅⋅⋅⋅⋅⋅⋅⋅⋅⋅⋅⋅⋅⋅⋅⋅ (3) Karena c, k ∈ bilangan asli maka (k − 2) past i membagi 4 dan karena c > 2k maka (k − 2) > 0 Nilai k yang memenuhi adalah 3; 4; 6 Unt uk k = 3 maka a = 6 sehingga c = 10 dan b = 8 ⋅⋅⋅⋅⋅⋅⋅⋅⋅⋅⋅⋅⋅⋅⋅⋅⋅⋅⋅⋅⋅⋅⋅⋅ (4) Unt uk k = 4 maka a = 8 sehingga c = 10 dan b = 6 ⋅⋅⋅⋅⋅⋅⋅⋅⋅⋅⋅⋅⋅⋅⋅⋅⋅⋅⋅⋅⋅⋅⋅⋅ (5) Unt uk k = 6 maka a = 12 sehingga c = 13 dan b = 5 ⋅⋅⋅⋅⋅⋅⋅⋅⋅⋅⋅⋅⋅⋅⋅⋅⋅⋅⋅⋅⋅⋅⋅⋅ (6) Karena a dan b simet ris maka j ika b = 2k akan didapat Unt uk k = 3 maka b = 6 sehingga c = 10 dan a = 8 ⋅⋅⋅⋅⋅⋅⋅⋅⋅⋅⋅⋅⋅⋅⋅⋅⋅⋅⋅⋅⋅⋅⋅⋅ (7) Unt uk k = 4 maka b = 8 sehingga c = 10 dan a = 6 ⋅⋅⋅⋅⋅⋅⋅⋅⋅⋅⋅⋅⋅⋅⋅⋅⋅⋅⋅⋅⋅⋅⋅⋅ (8) Unt uk k = 6 maka b = 12 sehingga c = 13 dan a = 5 ⋅⋅⋅⋅⋅⋅⋅⋅⋅⋅⋅⋅⋅⋅⋅⋅⋅⋅⋅⋅⋅⋅⋅⋅ (9) Tripel (a, b, c) yang memenuhi a ≤ b ≤ c adalah (6, 8, 10) dan (5, 12, 13). Set elah dicek ke persamaan a + b + c = ½ab maka kedua t ripel ini memenuhi. ∴ Maka tripel (a, b, c) yang memenuhi adalah (6, 8, 10) dan (5, 12, 13) 5. Karena A dan B masing-masing beranggot akan bilangan asli berurut an sedangkan A ∩ B = {2005} maka 2005 adalah anggot a t erbesar dari A dan anggot a t erkecil dari B. A = {x, x + 1, x + 2, ⋅⋅⋅, 2005) dan B = {2005, 2006, ⋅⋅⋅, y − 1, y} A ∪ B = {x, x + 1, ⋅⋅⋅, y − 1, y} Maka unsur yang t erbesar dari A ∪ B adalah y. x + x + 1 + L + 2005 2005 + 2006 + L + y = 5002 + y − 2004 2006 − x x + 2005 2005 + y + = 5002 2 2 x + y + 4010 = 10004 x + y = 5994 Karena x bilangan asli maka x t erkecil = 1 sehingga maksimum y = 5994 − 1 = 5993. ∴ Unsur terbesar yang mungkin dari A ∪ B adalah 5993. SMA Negeri 5 Bengkulu Eddy Hermanto, ST 143 SELEKSI TIM OLIMPIADE MATEMATIKA INDONESIA 2006 OLIMPIADE SAINS NASIONAL 2005 JAKARTA, 4 – 9 SEPTEMBER 2005 Bidang Mat emat ika Hari Pert ama Waktu : 180 Menit DEPARTEMEN PENDIDIKAN NASIONAL DIREKTORAT JENDERAL PENDIDIKAN DASAR DAN MENENGAH DIREKTORAT PENDIDIKAN MENENGAH UMUM TAHUN 2005 144 OLIMPIADE SAINS NASIONAL 2005 4 – 9 SEPTEMBER 2005 JAKARTA BI DAN G : MATEMATI KA HARI PERTAMA WAKTU : 180 MENIT 1. Misalkan n bilangan bulat posit if . Tent ukan banyaknya segit iga (t idak saling kongruen) yang panj ang set iap sisinya adalah bilangan bulat dan panj ang sisi t erpanj angnya adalah n. 2. Unt uk sebarang bilangan asli n, didef inisikan p(n) sebagai hasil kali digit -digit n (dalam represent asi basis 10). Tent ukan semua bilangan asli n sehingga 11 ⋅ p(n) = n2 − 2005. 3. Misalkan k dan m bilangan-bilangan asli sehingga a. Bukt ikan bahwa b. Bukt ikan bahwa 4. 1⎛ ⎜ k + 4 m − k ⎞⎟ adalah bilangan bulat . ⎠ 2⎝ k bilangan rasional k bilangan asli Misalkan M suat u t it ik di dalam segit iga ABC sedemikian rupa hingga ∠AMC = 90o, ∠AMB = 150o dan ∠BMC = 120o. Tit ik pusat lingkaran luar dari segit iga-segit iga AMC, AMB dan BMC bert urut t urut adalah P, Q dan R. Bukt ikan bahwa luas segit iga PQR lebih besar dari luas segit iga ABC. 145 SELEKSI TIM OLIMPIADE MATEMATIKA INDONESIA 2006 OLIMPIADE SAINS NASIONAL 2005 JAKARTA, 4 – 9 SEPTEMBER 2005 Bidang Mat emat ika Hari Kedua Waktu : 180 Menit DEPARTEMEN PENDIDIKAN NASIONAL DIREKTORAT JENDERAL PENDIDIKAN DASAR DAN MENENGAH DIREKTORAT PENDIDIKAN MENENGAH UMUM TAHUN 2005 146 OLIMPIADE SAINS NASIONAL 2005 4 – 9 SEPTEMBER 2005 JAKARTA BI DAN G : MATEMATI KA HARI KEDUA WAKTU : 180 MENIT 5. Unt uk sebarang bilangan real x, not asi ⎣x ⎦ menyat akan bilangan bulat t erbesar yang lebih kecil at au sama dengan x. Bukt ikan bahwa ada t epat sat u bilangan bulat m yang memenuhi persamaan ⎢ m ⎥ m−⎢ ⎥ = 2005 ⎣ 2005 ⎦ 6. Tent ukan semua t ripel bilangan bulat (x, y, z) yang memenuhi sist em persamaan x(y + z) = y2 + z2 − 2 y(z + x) = z2 + x 2 − 2 z(x + y) = x 2 + y2 − 2 7. Misalkan ABCD sebuah segiempat konveks. Persegi AB1A2B dibuat sehingga kedua t it ik A2, B1 t erlet ak di luar segiempat ABCD. Dengan cara serupa diperoleh persegi-persegi BC1B2C, CD1C2D dan DA1D2A. Misalkan K adalah t it ik pot ong AA2 dengan BB1, L adalah t it ik pot ong BB2 dengan CC1, M adalah t it ik Pot ong CC2 dengan DD1, dan N adalah t it ik pot ong DD2 dengan AA1. Bukt ikan bahwa KM t egak lurus LN. 8. Sebuah kompet isi mat emat ika diikut i oleh 90 pesert a. Set iap pesert a berkenalan dengan paling sedikit 60 pesert a lainnya. Salah seorang pesert a, Amin, menyat akan bahwa set idaknya t erdapat empat orang pesert a yang banyak t eman barunya sama. Periksa kebenaran pernyat aan Amin. 147 SELEKSI TIM OLIMPIADE MATEMATIKA INDONESIA 2006 OLIMPIADE SAINS NASIONAL 2005 JAKARTA, 4 – 9 SEPTEMBER 2005 Prestasi itu diraih bukan didapat !!! SOLUSI SOAL Bidang Mat emat ika Disusun oleh : Eddy Hermant o, ST 148 Olimpiade Sains Nasional 2005 Solusi Bidang : Mat emat ika 1. Misalkan panj ang sisi-sisi segit iga adalah a, b dan c dengan a adalah sisi t erpanj ang, maka a = n Karena panj ang salah sat u sisi segit iga selalu kurang dari j umlah kedua sisi yang lain dan karena b ≤ n dan c ≤ n maka a < b + c maka b + c = n + 1, n + 2, n + 3, ⋅⋅⋅, 2n. Jika c = k unt uk k bilangan asli maka b = n − k + i unt uk suat u nilai i = 1, 2, 3, ⋅⋅⋅, k. Jika k = 1 maka nilai (b, c) ada 1 yait u (n, 1) Jika k = 2 maka nilai (b, c) ada 2 yait u (n − 1, 2) dan (n, 2) Jika k = 3 maka nilai (b, c) ada 3 yait u (n − 2, 3), (n − 1, 3) dan (n, 3). Jika k = 4 maka nilai (b, c) ada 4 yait u (n − 3, 4), (n − 2, 4), (n − 1, 4) dan (n, 4). M Jika k = n − 1 maka nilai (b, c) ada n − 1 yait u (2, n − 1), (3, n − 1), (4, n − 1), ⋅⋅⋅, (n, n − 1) Jika k = n maka nilai (b, c) ada n yait u (1, n), (2, n), (3, n), ⋅⋅⋅, (n, n) Banyaknya seluruh segit iga adalah 1 + 2 + 3 + ⋅⋅⋅ + n = ½n(n + 1) Tet api segit iga-segit iga sama kaki dengan sisi-sisi a = n, b = k unt uk 1 ≤ k < n dan c = n kongruen dengan segit iga-segit iga sama kaki dengan sisi-sisi a = n, b = n dan c = k unt uk 1 ≤ k < n. Segit iga-segit iga yang sepert i it u banyaknya ada n − 1 yang t erhit ung dua kali di dalam perhit ungan ½n(n + 1). Perlu diingat pula bahwa segit iga-segit iga yang bukan sama kaki dengan a = n, b = n − p dan c = p + r ≤ n t idak kongruen dengan segit iga-segit iga yang panj ang sisinya a = n, b = p + r ≤ n dan c = n − p walaupun ket iga sisinya sama. Maka j umlah segit iga yang dicari = ½n(n + 1) − (n − 1) ∴ Banyaknya segitiga = n2 −n + 2 2 2. Alt ernat if 1 : Akan dit unj ukkan bahwa unt uk set iap n bilangan asli maka p(n) ≤ n Misalkan n = 10kak + 10k-1ak-1 + ⋅⋅⋅ + 10a1 + ao dengan a1, a2, ⋅⋅⋅, ak ∈ {0, 1, 2, ⋅⋅⋅, 9} Karena a0 maks = a1 maks = a2 maks = ⋅⋅⋅ = ak-1 maks = 9 maka p(n) = ao ⋅ a1 ⋅ a2 ⋅ ⋅⋅⋅ ⋅ ak ≤ 9k ⋅ ak ≤ 10k ⋅ ak ≤ n Maka n2 − 2005 = 11 p(n) ≤ 11n 2 ⎛ 11 ⎞ 8141 ⎜⎜ n − ⎟⎟ ≤ ≤ 45 2 2 4 ⎠ ⎝ 1 ≤ n ≤ 50 ⋅⋅⋅⋅⋅⋅⋅⋅⋅⋅⋅⋅⋅⋅⋅⋅⋅⋅⋅⋅⋅⋅⋅⋅⋅⋅⋅⋅⋅⋅⋅ (1) Selain it u n2 − 2005 = 11 p(n) ≥ 0 n ≥ 45 ⋅⋅⋅⋅⋅⋅⋅⋅⋅⋅⋅⋅⋅⋅⋅⋅⋅⋅⋅⋅⋅⋅⋅⋅⋅⋅⋅⋅⋅⋅⋅⋅⋅⋅⋅⋅⋅⋅⋅⋅⋅⋅⋅⋅⋅⋅⋅⋅ (2) Dari (1) dan (2) didapat n = 45, 46, 47, 48, 49 at au 50 Dengan menguj i ke persamaan n 2 − 2005 = 11 p(n) didapat hanya n = 49 yang memenuhi. ∴ Nilai n yang memenuhi hanya n = 49 Alt ernat if 2 : • Jika n t erdiri dari k digit dengan k ≥ 4 n2 merupakan bilangan dengan sedikit nya 2k − 1 digit . Maka n 2 − 2005 merupakan bilangan dengan sedikit nya 2k − 2 digit . 11 p(n) ≤ 11 ⋅ 9 ⋅ 9 ⋅ ⋅⋅⋅ ⋅ 9 < 10k+1. Maka 11 p(n) merupakan bilangan dengan sebanyak-banyaknya t erdiri dari k + 1 digit . Unt uk k ≥ 4 maka 2k ≥ k + 4 sehingga 2k − 2 ≥ k + 2 maka 2k − 2 > k + 1 sehingga t idak ada yang memenuhi 11 ⋅ p(n) = n2 − 2005 SMA Negeri 5 Bengkulu Eddy Hermanto, ST 149 Olimpiade Sains Nasional 2005 Solusi Bidang : Mat emat ika • Jika n t erdiri dari 3 digit Jika angka rat usan n lebih dari 1 maka n 2 − 2005 ≥ 2002 − 2005 = 37995 11 ⋅ p(n) ≤ 11 ⋅ 93 = 8019 < n 2 − 2005 (t anda kesamaan t idak akan t erj adi) Jika angka rat usan n sama dengan 1 maka n 2 − 2005 ≥ 1002 − 2005 = 7995 11 ⋅ p(n) ≤ 11 ⋅ 1 ⋅ 92 = 891 < n2 − 2005 (t anda kesamaan t idak akan t erj adi) • Jika n t erdiri dari 2 digit Misalkan n = 10a + b n t idak mungkin genap sebab ruas kanan akan ganj il sedangkan ruas kiri genap. Karena n ganj il dan 2005 ≡ 1 (mod 4) maka n 2 − 2005 ≡ 0 (mod 4) Akibat nya salah sat u a at au b habis dibagi 4. Karena n ganj il maka a = 4 at au 8. n2 ≡ 0, 1, 4 (mod 8) 2005 ≡ 5 (mod 8) Ruas kanan t idak habis dibagi 8, maka a = 4 11ab = (10a + b) 2 − 2005 44b = 1600 + 80b + b 2 − 2005 b 2 − 36b − 405 = 0. Maka (b − 9)(b + 45) = 0 sehingga b = 9 Bilangan t ersebut adalah n = 49 • Jika n t erdiri dari 1 digit Ruas kanan akan bernilai negat if (t idak memenuhi) ∴ Nilai n yang memenuhi hanya n = 49 3. Alt ernat if 1 : Perhat ikan bahwa a. Misalkan 1⎛ ⎞ ⎜ k + 4 m − k ⎟ merupakan akar persamaan x ⎠ 2⎝ 1⎛ ⎞ ⎜ k + 4 m − k ⎟ = n adalah akar bulat dari x ⎠ 2⎝ n 2 + n k − m = 0 , maka 2 2 + x k − m = 0. + x k − m = 0 maka : m = n2 + n k Karena m bilangan asli maka n ≠ 0 m = n 4 + 2n 3 k + kn m − n 4 − kn 2 k = 2n 3 2 Karena m dan k adalah bilangan asli dan n bilangan bulat t ak nol maka bilangan rasional (t erbukt i). b. Misalkan k = k = k merupakan p unt uk suat u bilangan asli p dan q dengan FPB(p, q) = 1. q p2 . Karena FPB(p, q) = 1 maka FPB(p 2, q 2 ) = 1 2 q Karena k adalah bilangan asli maka q 2 = 1. k = p 2, maka k = p dengan p bilangan asli. Terbukt i bahwa k adalah bilangan asli. SMA Negeri 5 Bengkulu Eddy Hermanto, ST 150 Olimpiade Sains Nasional 2005 Solusi Bidang : Mat emat ika Alt ernat if 2 : a. Karena 1⎛ ⎞ ⎜ k + 4 m − k ⎟ bulat dan m asli maka ⎠ 2⎝ asli p. ( 2 ⎛ k +4 m ⎞ = p + k ⎜ ⎟ ⎝ ⎠ k + 4 m − k = p unt uk bilangan ) 2 4 m = p 2 + 2p k Karena m asli maka t idak mungkin p = 0. 16m = p 4 + 4 p 2 k + 4 p 3 k 16m − p 4 − 4p 2 k k = 4p 3 Karena m dan k asli sedangkan p bulat t ak nol maka (t erbukt i). b. Misalkan k = k = k merupakan bilangan rasional p unt uk suat u bilangan asli p dan q dengan FPB(p, q) = 1. q p2 . Karena FPB(p, q) = 1 maka FPB(p 2, q 2 ) = 1 q2 Karena k adalah bilangan asli maka q 2 = 1. k = p 2, maka k = p dengan p bilangan asli. Terbukt i bahwa k adalah bilangan asli. 4. Perhat ikan gambar berikut Misalkan AB = c, BC = a, CA = b, ∠BAC = α, ∠ABC = β dan ∠ACB = γ Diket ahui bahwa ∠AMC = 90o, ∠AMB = 150o dan ∠BMC = 120o Misalkan j uga not asi [ ] menyat akan luas suat u segit iga. [ ABC] = ½ ab sin γ SMA Negeri 5 Bengkulu Eddy Hermanto, ST 151 Olimpiade Sains Nasional 2005 Solusi Bidang : Mat emat ika Karena ∠AMC = 90o sedangkan P pusat lingkaran luar ∆AMC maka P adalah pert engahan AC. Karena P, Q dan R pusat lingkaran dan AM, BM sert a CM adalah t ali busur persekut uan dua lingkaran maka PR ⊥ CM, PQ ⊥ AM dan QR ⊥ BM. Karena ∠AMC = 90o sedangkan PR ⊥ CM sert a PQ ⊥ AM maka ∠RPQ = 90o. Karena ∠BMC = 120o sedangkan PR ⊥ CM sert a QR ⊥ BM maka ∠PRQ = 60o. ∠PQR = 180o − 90o − 60o = 30o Karena PR t egak lurus CM dan RC = RM maka ∠CRP = ∠PRM = θ Karena RQ ⊥ MB dan RM = RB maka ∠MRQ = ∠QRB = φ sehingga ∠PRM + ∠MRQ = θ + φ = 60o ∠CRB = 2( θ + φ) = 120o Karena RC = RB sedangkan ∠CRB = 120o maka ∠RCB = 30o Misalkan R1 adalah j ari-j ari lingkaran luar ∆BMC Æ a = 2R1 sin ∠BMC a R 1 = CR = 3 Pada ∆CPR berlaku : 2 ⎛ a ⎞ ⎛b ⎞ ⎛ b ⎞⎛ a ⎞ ⎟ cos (γ + 30°) ⎟ − 2⎜⎜ ⎟⎟⎜ = ⎜⎜ ⎟⎟ + ⎜⎜ ⎟ ⎟ ⎜ ⎝2⎠ ⎝ 2 ⎠⎝ 3 ⎠ ⎝ 3⎠ b 2 a 2 ab (cos γ cos 30° − sin γ sin 30°) = + − 4 3 3 2 PR 2 PR 2 PR 2 PR 2 b 2 a 2 ab ⎛⎜ a 2 + b 2 − c 2 3 2[ABC ] 1 ⎞⎟ = + − ⋅ − ⋅ ab 4 3 2ab 2 2 ⎟⎠ 3 ⎜⎝ a2 c 2 ABC = + + 12 4 3 [ ] PQ = PR t an 60o = PR √3 [ PQR] = ½ ⋅ PQ ⋅ PR ⎛ a 2 c 2 [ABC ] ⎞ ⎟ ⋅ ⎜⎜ + + ⎟ 4 3 ⎠ ⎝ 12 2 [PQR ] − [ABC ] = 83 ⎛⎜⎜ a3 + c 2 ⎞⎟⎟ − [ABC 2 ⎝ ⎠ [PQR ] = 3 2 ] Dengan ket aksamaan AM-GM didapat : [ABC ] 3 a2 ⋅2 ⋅c 2 − 8 3 2 [PQR ] − [ABC ] ≥ ac − [ABC ] = ac − ac sin β 4 2 4 4 [PQR ] − [ABC ] ≥ ac (1 − sin β ) > 0 sebab β ≠ 90o 4 [PQR ] > [ABC ] [PQR ] − [ABC ] ≥ ∴ Terbukti bahwa luas segitiga PQR lebih besar dari luas segitiga ABC. SMA Negeri 5 Bengkulu Eddy Hermanto, ST 152 Olimpiade Sains Nasional 2005 Solusi Bidang : Mat emat ika 5. Alt ernat if 1 : ⎢ m ⎥ ⎢ m ⎥ = 2005 , maka ⎢ m −⎢ ⎥ ⎥ = m − 2005 ⎣ 2005 ⎦ ⎣ 2005 ⎦ m m − 1 < m − 2005 ≤ 2005 2005 m − 2005 < 2005(m − 2005) ≤ m −2005 < 2004m − 20052 ≤ 0 20052 − 2005 < 2004m ≤ 20052 2005 < m ≤ 2005 2 2004 2005 < m ≤ 2006 Nilai m yang memenuhi hanya m = 2006 Jika m = 2006 diuj i ke persamaan semula maka ini akan memenuhi. ⎢ m ⎥ ⎥ = 2005 mempunyai tepat satu penyelesaian. ⎣ 2005 ⎦ ∴ Terbukti bahwa m − ⎢ Alt ernat if 2 : Bilangan bulat dapat dibuat ke bent uk m = 2005k + n unt uk k bulat dan n ∈ {0, 1, 2, ⋅⋅⋅, 2004}. ⎢ m ⎥ ⎢ 2005k + n ⎥ = 2005 , maka 2005k + n − ⎢ m −⎢ ⎥ ⎥ = 2005 ⎣ 2005 ⎦ ⎣ 2005 ⎦ Karena 0 ≤ n < 2004 maka 2005k + n − k = 2005 2004k + n = 2005 Karena 0 ≤ n < 2004 maka 2004k > 0 sehingga k > 0 Karena 0 ≤ n < 2004 maka 2004k ≤ 2005 0 < k ≤ 1, maka k = 1 2004 (1) + n = 2005, maka n = 1 Karena nilai k yang memenuhi hanya ada 1 maka kemungkinan nilai m yang memenuhi j uga hanya ada 1 yait u m = 2005 ⋅ 1 + 1 = 2006. Jika m = 2006 diuj i ke persamaan semula maka ini akan memenuhi. ⎢ m ⎥ ⎥ = 2005 mempunyai tepat satu penyelesaian ⎣ 2005 ⎦ ∴ Terbukti bahwa m − ⎢ 6. x(y + z) = y2 + z2 − 2 ⋅⋅⋅⋅⋅⋅⋅⋅⋅⋅⋅⋅⋅⋅⋅⋅⋅⋅⋅⋅⋅⋅⋅⋅⋅⋅⋅⋅⋅⋅⋅⋅⋅⋅⋅⋅⋅⋅⋅ (1) y(z + x) = z2 + x 2 − 2 ⋅⋅⋅⋅⋅⋅⋅⋅⋅⋅⋅⋅⋅⋅⋅⋅⋅⋅⋅⋅⋅⋅⋅⋅⋅⋅⋅⋅⋅⋅⋅⋅⋅⋅⋅⋅⋅⋅⋅ (2) z(x + y) = x 2 + y2 − 2 ⋅⋅⋅⋅⋅⋅⋅⋅⋅⋅⋅⋅⋅⋅⋅⋅⋅⋅⋅⋅⋅⋅⋅⋅⋅⋅⋅⋅⋅⋅⋅⋅⋅⋅⋅⋅⋅⋅⋅ (3) Kurangkan (1) dengan (2), z(x − y) = y2 − x 2 = (x + y)(y − x), maka (x − y)(x + y + z) = 0 Kurangkan (1) dengan (3), y(x − z) = z2 − x 2 = (x + z)(z − x), maka (x − z)(x + y + z) = 0 Kurangkan (2) dengan (3), x(y − z) = z2 − y2 = (y + z)(z − y), maka (y − z)(x + y + z) = 0 ⋅ • Kasus I : x+y+z=0 Subt it usikan ke persamaan (1), (2) at au (3) x( −x) = y2 + z2 − 2, maka x 2 + y2 + z2 = 2 ⋅⋅⋅⋅⋅⋅⋅⋅⋅⋅⋅⋅⋅⋅⋅⋅⋅⋅⋅⋅⋅⋅⋅⋅⋅⋅⋅⋅ (5) SMA Negeri 5 Bengkulu Eddy Hermanto, ST 153 ⋅⋅⋅⋅⋅⋅⋅⋅⋅⋅⋅⋅⋅⋅⋅⋅⋅⋅⋅⋅⋅⋅⋅⋅⋅⋅⋅⋅ (4) ⋅⋅⋅⋅⋅⋅⋅⋅⋅⋅⋅⋅⋅⋅⋅⋅⋅⋅⋅⋅⋅⋅⋅⋅⋅⋅⋅ (6) Solusi Olimpiade Sains Nasional 2005 Bidang : Mat emat ika Maka penyelesaiannya adalah x 2 = 1 ; y 2 = 1 dan z2 = 0 sert a permut asinya. Tripel (x, y, z) yang memenuhi adalah (x, y, z) = (1, −1, 0), (1, 0, −1), ( −1, 1, 0), ( −1, 0, 1), (0, 1, −1) dan (0, −1, 1) • Kasus II : x+y+z≠0 Berdasarkan persamaan (4), (5) dan (6) maka x = y = z Subt it usikan ke persamaan (1) didapat 2x 2 = 2x 2 − 2, maka t idak ada nilai (x, y, z) yang memenuhi. ∴ (x, y, z) = (1, −1, 0), (1, 0, −1), ( −1, 1, 0), ( −1, 0, 1), (0, 1, −1) dan (0, −1, 1) 7. Alt ernat if 1 : Akan dibut ikan bahwa pada segiempat konveks PQRS berlaku PQ2 + RS2 = QR2 + PS2 j ika dan hanya j ika PR t egak lurus QS. Jika PR t egak lurus QS at au α = 90o. PQ2 + RS2 = (PO2 + OQ2) + (OR2 + OS) 2 = (PO2 + OS2) + (OR2 + OQ2) PQ2 + RS2 = PS2 + QR2 * Jika PQ2 + RS2 = QR2 + PS2 Dengan dalil cosinus didapat : PQ2 + RS2 = QR2 + PS2 OP2 + OQ2 − 2 OP ⋅ OQ cos α + OR2 + OS2 − 2 OR ⋅ OS cos α = OQ2 + OR2 − 2 OQ ⋅ OR cos (180o−α) + OP2 + OS2 − 2 OP ⋅ OS cos (180o−α) (2 OQ OR + 2 OP OS + 2 OP OQ + 2 OR OS) cos α = 0 α = 90o Terbukt i bahwa pada segiempat konveks PQRS berlaku PQ2 + RS2 = QR2 + PS2 j ika dan hanya j ika PR t egak lurus QS. * Perhat ikan ∆KAN. SMA Negeri 5 Bengkulu Eddy Hermanto, ST 154 Solusi Olimpiade Sains Nasional 2005 Bidang : Mat emat ika AA1 dan AA2 keduanya diagonal bidang persegi maka ∠KAB = ∠KAB1 = ∠NAD2 = ∠NAD = 45o. Dengan dalil cosinus didapat : KN2 = AK2 + AN2 − 2 AK ⋅ AN cos ∠KAN. Jika ∠BAD ≥ 90o maka ∠KAN = 270o − ∠BAD dan j ika ∠BAD < 90o maka ∠KAN = 90o + ∠BAD Akibat nya cos ∠KAN akan t et ap bernilai −sin ∠BAD. KN2 = AK2 + AN2 + 2 AK ⋅ AN sin ∠BAD. 1 1 1 1 KN 2 = AB 2 + AD 2 + 2 ⋅ 2 AD ⋅ sin ∠BAD 2 AB ⋅ 2 2 2 2 Dengan mengingat luas ∆ABD = [ ABD] = ½ AB ⋅ AD sin ∠BAD maka KN2 = ½AB2 + ½AD2 + 2[ ABD] Dengan cara yang sama unt uk ∆KBL, ∆LCM dan ∆MDN didapat : KL2 = ½AB2 + ½BC2 + 2[ ABC] LM2 = ½BC2 + ½CD2 + 2[ BCD] MN2 = ½CD2 + ½AD2 + 2[ ACD] Sehingga mengingat [ ABD] + [ BCD] = [ ABC] + [ ACD] maka KN2 + LM2 = KL2 + MN2 Mengingat pembukt ian yang t elah dibuat di awal maka KM t egak lurus LN (t erbukt i) Alt ernat if 2 : Jika t it ik (x, y) dirot asi sebesar θ berlawanan arah j arum j am dengan t it ik pusat (a, b) sehingga diperoleh bayangan (x’ , y’ ) maka berlaku : x’ = a + (x − a) cos θ − (y − b) sin θ ⋅⋅⋅⋅⋅⋅⋅⋅⋅⋅⋅⋅⋅⋅⋅⋅⋅⋅⋅⋅⋅⋅⋅⋅⋅⋅⋅⋅⋅⋅⋅⋅⋅⋅⋅⋅⋅⋅⋅⋅⋅⋅⋅⋅⋅⋅⋅⋅⋅⋅⋅ (1) y’ = b + (x − a) sin θ + (y − b) cos θ ⋅⋅⋅⋅⋅⋅⋅⋅⋅⋅⋅⋅⋅⋅⋅⋅⋅⋅⋅⋅⋅⋅⋅⋅⋅⋅⋅⋅⋅⋅⋅⋅⋅⋅⋅⋅⋅⋅⋅⋅⋅⋅⋅⋅⋅⋅⋅⋅⋅⋅⋅ (2) Pembukt ian persamaan di at as dapat dilihat di Buku Mat emat ika SMA Bab Transf ormasi Geomet ri. Misalkan koordinat P(x 1, y1) dan Q(x 2, y2). Karena PQRS adalah persegi maka koordinat t it ik S didapat dengan merot asi t it ik Q sej auh 90o berlawanan arah j arum j am dengan pusat di P. Maka koordinat S(x 1 + y1 − y2, x 2 − x 1 + y1) Karena T adalah pert engahan S dan Q maka koordinat T ⎛⎜ x 1 + x 2 + y 1 − y 2 , x 2 − x 1 + y 1 + y 2 ⎞⎟ . ⎟ ⎜ 2 2 ⎝ SMA Negeri 5 Bengkulu ⎠ Eddy Hermanto, ST 155 Olimpiade Sains Nasional 2005 Solusi Bidang : Mat emat ika Tanpa mengurangi keumuman soal misalkan t it ik A t erlet ak pada (0, 0) sedangkan koordinat B(x B, yB), C(x C, yC) dan D(x D, yD). Dari penj elasan sebelumnya didapat koordinat ⎛x + y B −x B + y B ⎞ ⎛x C + x B + y C −y B x B −x C + y B + y C ⎞ ⎟⎟ , L ⎜⎜ ⎟⎟ , K ⎜⎜ B , , 2 2 2 2 ⎝ ⎠ ⎝ ⎠ ⎛x + x C + y D − y C x C − x D + y C + y D ⎞ ⎛x − y D x D + y D ⎟⎟ dan N ⎜⎜ D M ⎜⎜ D , , 2 2 2 2 ⎠ ⎝ ⎝ x +xC −x B +y D −yC −y B ˆ xC +x B −x D +yC +y D −y B ˆ KM = D i + j 2 2 x −xC −x B −y D −yC +y B ˆ x D −x B +xC +y D −y B −yC ˆ LN = D i + j 2 2 KM ⋅ LN = ⎞ ⎟⎟ . ⎠ 1 (x D + xC − x B + y D − yC − y B )(x D − xC − x B − y D − yC + y B ) + 1 (xC + x B − x D + yC + y D − y B )(x D − x B + xC + y D − y B − y C ) 4 4 Mengingat (a + b)(a − b) = a2 − b 2 maka : KM ⋅ LN = ( 1 (x 4 D −x B −yC ) 2 − (x C + y D −y ) 2 B ) + 14 ((x C +y D −y ) 2 B − (x D −x B −yC ) 2 )= 0 Karena KM ⋅ LN = 0 maka KM t egak lurus LN (t erbukt i) 90 8. Misalkan k i adalah banyaknya kenalan pesert a i dan K = ∑k i =1 i adalah penj umlahan banyaknya kenalan masing-masing pesert a. Jika pesert a A berkenalan dengan B maka banyaknya kenalan A bert ambah 1 begit u j uga dengan B. Jelas bahwa K akan bernilai genap. Andaikan bahwa paling banyak t iga orang siswa akan memiliki j umlah kenalan sama banyaknya. Karena k i ≥ 60 maka k i ∈ {60, 61, 62, 63, ⋅⋅⋅, 89}. Banyaknya kemungkinan nilai k i ada 30. Karena 90/ 3 = 30 maka t erdapat t epat masing-masing 3 pesert a memiliki kenalan sebanyak 60 orang, 61 orang, 62 orang, ⋅⋅⋅, 89 orang. Maka K = 3 ⋅ 60 + 3 ⋅ 61 + 3 ⋅ 62 + ⋅⋅⋅ + 3 ⋅ 89 Di ant ara 60, 61, 62, 63, ⋅⋅⋅, 89 t erdapat 15 bilangan ganj il dan 15 bilangan genap. Mengingat bahwa penj umlahan sej umlah ganj il dari bilangan ganj il menghasilkan bilangan ganj il maka : K = 3 ⋅ 60 + 3 ⋅ 61 + 3 ⋅ 62 + ⋅⋅⋅ + 3 ⋅ 89 merupakan bilangan ganj il (kont radiksi dengan kenyat an semula bahwa K bernilai genap). Maka pengandaian bahwa paling banyak t iga orang siswa akan memiliki j umlah kenalan sama banyaknya t idak t erbukt i. ∴ Terbukti bahwa setidaknya terdapat 4 pesert a yang banyak kenalannya sama. SMA Negeri 5 Bengkulu Eddy Hermanto, ST 156 SELEKSI OLIMPIADE TINGKAT KABUPATEN/ KOTA TAHUN 2006 TIM OLIMPIADE MATEMATIKA INDONESIA TAHUN 2007 Bidang Mat emat ika Waktu : 3, 5 Jam DEPARTEMEN PENDIDIKAN NASIONAL DIREKTORAT JENDERAL MANAJEMEN PENDIDIKAN DASAR DAN MENENGAH DIREKTORAT PEMBINAAN SEKOLAH MENENGAH ATAS TAHUN 2006 157 OLIMPIADE MATEMATIKA NASIONAL SELEKSI TINGKAT KOTA/ KABUPATEN TAHUN 2006 Bagian Pertama Pilih sat u j awaban yang benar. Dalam hal t erdapat lebih dari sat u j awaban yang benar, pilih j awaban yang paling baik. 1. Jumlah t iga bilangan prima pert ama yang lebih dari 50 adalah A. 169 B. 171 C. 173 D. 175 E. 177 2. Dalam sebuah kot ak t erdapat 5 bola merah dan 10 bola put ih. Jika diambil dua bola secara bersamaan, peluang memperoleh dua bola berwarna sama adalah A. 1 2 B. 1 3. Jika X = 2+ A. 2 9 C. 2 21 D. 10 21 E. 11 21 C. 4 9 D. 9 4 E. 12 5 , maka X = 1 2+ 1 4 1 2 B. 5 12 4. Pada segit iga ABC, t it ik F membagi sisi AC dalam perbandingan 1 : 2. Misalkan G t it ik t engah BF dan E t it ik perpot ongan ant ara sisi BC dengan AG. Maka t it ik E membagi sisi BC dalam perbandingan A. 1 : 4 B. 1 : 3 C. 2 : 5 D. 4 : 11 E. 3 : 8 5. Dalam suat u pert emuan t erj adi 28 j abat t angan (salaman). Set iap dua orang salaing berj abat t angan paling banyak sekali. Banyaknya orang yang hadir dalam pert emuan t ersebut paling sedikit adalah A. 28 B. 27 C. 14 D. 8 E. 7 6. Gaj i David lebih banyak 20% daripada gaj i Andika. Ket ika Andika memperoleh kenaikan gaj i, gaj inya menj adi lebih banyak 20% daripada gaj i David. Persent ase kenaikan gaj i Andika adalah A. 0, 44 B. 20 C. 44 D. 144 E. t idak bisa dipast ikan 7. Misalkan T adalah himpunan semua t it ik pada bidang-xy yang memenuhi ⏐x ⏐ + ⏐y⏐ ≤ 4. Luas daerah T adalah A. 4 B. 8 C. 12 D. 16 E. 32 8. Def inisikan a*b = a + b + 1 unt uk semua bilangan bulat a, b. Jika p memenuhi a*p = a, unt uk set iap bilangan bulat a, maka p = A. −1 B. 0 C. 1 D. 2 E. t idak ada yang memenuhi 158 9. Set iap dong adalah ding, dan beberapa dung j uga dong. X : Terdapat dong yang j uga ding sekaligus dung Y : Beberapa ding adalah dung Z : Terdapat dong yang bukan dung A. Hanya X yang benar C. Hanya Z yang benar B. Hanya Y yang benar D. X dan Y keduanya benar E. X, Y dan Z semuanya salah 10. Banyaknya solusi pasangan bilangan bulat posit if persamaan 3x + 5y = 501 adalah A. 33 B. 34 C. 35 D. 36 E. 37 Bagian Kedua Isikan hanya j awaban saj a pada t empat yang disediakan 11. Diket ahui a + (a + 1) + (a + 2) + ⋅⋅⋅ + 50 = 1139. Jika a bilangan posit if , maka a = ⋅⋅⋅⋅⋅ 12. Di ant ara lima orang gadis, Arint a, Elsi, Put ri, Rit a, dan Venny, dua orang memakai rok dan t iga orang memakai celana panj ang. Arint a dan Put ri mengenakan j enis pakaian yang sama. Jenis pakaian Put ri dan Esi berbeda, demikian pula dengan Elsi dan Rit a. Kedua gadis yang memakai rok adalah ⋅⋅⋅⋅⋅⋅ 13. Barisan 2, 3, 5, 6, 7, 10, 11, ⋅⋅⋅ t erdiri dari semua bilangan asli yang bukan kuadrat at au pangkat t iga bilangan bulat . Suku ke-250 barisan adalah ⋅⋅⋅⋅⋅ 14. Jika f (xy) = f (x + y) dan f (7) = 7, maka f (49) = ⋅⋅⋅⋅ 15. Pada sebuah barisan arit mat ika, nilai suku ke-25 t iga kali nilai suku ke-5. Suku yang bernilai dua kali nilai suku pert ama adalah suku ke ⋅⋅⋅⋅⋅⋅ 16. Dimas membeli maj alah set iap 5 hari sekali, sedangkan Andre membeli maj alah set iap 8 hari sekali. Kemarin Dimas membeli maj alah. Andre membeli maj alah hari ini. Keduanya paling cepat akan membeli maj alah pada hari yang sama ⋅⋅⋅⋅⋅ hari lagi. 17. Nanang mencari semua bilangan empat -angka yang selisihnya dengan j umlah keempat angkanya adalah 2007. Banyaknya bilangan yang dit emukan Nanang t idak akan lebih dari ⋅⋅⋅⋅⋅⋅ 18. Parabola y = ax 2 + bx + c memiliki puncak dengan koordinat (4, 2). Jika t it ik (2, 0) t erlet ak pada parabola, maka abc = ⋅⋅⋅⋅⋅⋅ 19. Sebuah garis l1 mempunyai kemiringan −2 dan melalui t it ik (p, −3). Sebuah garis lainnya l2, t egaklurus t erhadap l1 di t it ik (a, b) dan melalui t it ik (6, p). Bila dinyat akan dalam p, maka a = 20. Pada segit iga ABC yang t umpul di C, t it ik M adalah t it ik t engah AB. Melalui C dibuat garis t egak lurus pada BC yang memot ong AB di t it ik E. Dari M t arik garis memot ong BC t egak lurus di D. Jika luas segit iga ABC adalah 54 sat uan luas, maka luas segit iga BED adalah ⋅⋅⋅⋅⋅ 159 SELEKSI OLIMPIADE TINGKAT KABUPATEN/ KOTA 2006 TIM OLIMPIADE MATEMATIKA INDONESIA 2007 Prestasi itu diraih bukan didapat !!! SOLUSI SOAL Bidang Mat emat ika Disusun oleh : Eddy Hermant o, ST 160 Olimpiade Matematika Tk Kabupaten/ Kota 2006 BAGIAN PERTAMA 1. (Jawaban : C) Tiga bilangan prima pert ama yang lebih besar dari 50 adalah 53, 59 dan 61. 53 + 59 + 61 = 173 ∴ Jumlah t iga bilangan prima pert ama yang lebih besar dari 50 = 173 2. (Jawaban : E) Kemungkinan kedua bola t ersebut adalah keduanya berwarna merah at au keduanya berwarna put ih. C2 + 15C 2 11 ∴ Peluang = 21 5 Peluang = C2 15C 2 10 3. (Jawaban : B) 1 X = 2+ 1 2+ ∴ X= = 1 2 1 2+ 2 5 = 5 12 5 12 4. (Jawaban : B) Misalkan t anda [ KML] menyat akan luas ∆KML Misalkan [ ABC] = X. Karena AF : FC = 1 : 2 maka [ ABF] = Karena G pert engahan BF maka [ ABG] = ½ [ ABF] = Karena AF : FC = 1 : 2 maka [ CGF] = 2 [ AFG] = 1 1 [ ABC] = X 3 3 1 X = [ AFG] 6 1 1 X sehingga [ CGB] = X 3 3 Misalkan [ CGE] = P dan [ EGB] = Q SMA Negeri 5 Bengkulu Eddy Hermanto, ST 161 Olimpiade Matematika Tk Kabupaten/ Kota 2006 BE Q Q +X /6 = = EC P P +X / 3+X / 6 6PQ + 3XQ = 6PQ + PX Q 1 = sehingga BE : EC = 1 : 3 P 3 ∴ Tit ik E membagi BC dalam perbandingan = 1 : 3 5. (Jawaban : D) Misalkan banyaknya orang = n nC2 = 28, maka n (n − 1) = 28 2 n2 − n − 56 = 0, maka (n − 8)(n + 7) = 0 ∴ Banyaknya orang yang hadir = 8 6. (Jawaban : C) Misal gaj i Andika sebelum kenaikan = A dan set elah memperoleh kenaikan gaj i gaj inya menj adi Ax. Gaj i David sebelum kenaikan = 1, 2A . Ax = 1, 2 ⋅ (1, 2A) = 1, 44A Kenaikan gaj i Andika = 1, 44A − A = 0, 44A ∴ Kenaikan gaj i Andika adalah 44 % 7. (Jawaban : E) ⏐x ⏐ + ⏐y⏐ ≤ 4 • Jika x dan y di kuadran I maka ⏐x ⏐ = x dan ⏐y ⏐ = y. Persamaannya adalah x + y ≤ 4 • Jika x dan y di kuadran II maka ⏐x ⏐ = −x dan ⏐y ⏐ = y. Persamaannya adalah −x + y ≤ 4 • Jika x dan y di kuadran III maka ⏐x ⏐ = −x dan ⏐y ⏐ = −y. Persamaannya adalah −x − y ≤ 4 • Jika x dan y di kuadran IV maka ⏐x ⏐ = x dan ⏐y ⏐ = −y. Persamaannya adalah x − y ≤ 4 Gambar persamaan-persamaan t ersebut adalah : Karena panj ang sisi-sisinya sama yait u 4 2 sedangkan kedua diagonalnya saling t egak lurus maka luasan berupa persegi. Luas daerah T = 4 2 ⋅ 4 2 ∴ Luas daerah T = 32 SMA Negeri 5 Bengkulu Eddy Hermanto, ST 162 Olimpiade Matematika Tk Kabupaten/ Kota 2006 8. (Jawaban : A) a*b=a+b+1 a*p=a+p+1 a=a+p+1 ∴ p = −1 9. (Jawaban : D) Karena set iap dong adalah ding maka dong merupakan himpunan bagian dari ding. Karena beberapa dung j uga dong maka dung dan dong memiliki irisan. Maka a past i ada. Karena a past i ada maka a merupakan dong yang ding sekaligus dung (pernyat aan X benar) Karena a past i ada maka a adalah merupakan ding yang sekaligus dung (pernyat aan Y benar). Dong yang bukan dung adalah b. Karena b belum past i ada maka pernyat aan Z belum dapat dibukt ikan kebenarannya. ∴ X dan Y keduanya benar. 10. (Jawaban : A) 3x + 5y = 501 5y = 3(167 − x) Karena 3 dan 5 relat if prima maka y = 3k dan 167 − x = 5k unt uk suat u k bulat posit if . Jelas bahwa 0 < 5y ≤ 501 dan 0 < 3x ≤ 501, maka 0 < y ≤ 100 dan 0 < x ≤ 167 Karena t erdapat 100 nilai y yang memenuhi dan 167 nilai x yang memenuhi maka banyaknya ⎢100 ⎥ ⎢167 ⎥ = 33 at au ⎢ ⎥ ⎥ = 33 yait u 1 ≤ k ≤ 33. ⎣ 3 ⎦ ⎣ 5 ⎦ nilai k yang memenuhi adalah ⎢ Cont oh : Jika k = 1 maka y = 3 dan x = 162 memenuhi. (k, x, y) = (2, 157, 6) ; (3, 152, 9) ; ⋅⋅⋅ j uga memenuhi. ∴ Banyaknya pasangan (x, y) yang memenuhi adalah 33 SMA Negeri 5 Bengkulu Eddy Hermanto, ST 163 Olimpiade Matematika Tk Kabupaten/ Kota 2006 BAGIAN KEDUA 11. a + (a + 1) + (a + 2) + ⋅⋅⋅ + 50 = 1139. Banyaknya bilangan a, (a + 1), (a + 2), ⋅⋅⋅, 50 adalah 50 − a + 1 = 51 − a ½(51 − a) ⋅ (a + 50) = 1139, maka a2 − a − 272 = 0 sehingga (a − 17)(a + 16) = 0 ∴ a = 17 12. Karena pakaian Elsi baik dengan Put ri maupun Rit a berbeda maka Put ri dan Rit a memakai pakaian yang sama. Karena Arint a, Put ri dan Rit a memakai pakaian yang sama maka ket iganya t idak mungkin mamakai rok. Maka Arint a, Put ri dan Rit a memakai celana panj ang sedangkan Elsi dan Venny memakai rok. ∴ Kedua gadis yang memakai rok adalah Elsi dan Venny. 13. Bilangan kuadrat yang sekaligus j uga bilangan pangkat t iga adalah bilangan pangkat enam. Bilangan kuadrat ≤ 265 adalah 12, 22, ⋅⋅⋅, 162 ada sebanyak 16 bilangan. Bilangan pangkat t iga ≤ 265 adalah 13, 23, ⋅⋅⋅, 63 ada sebanyak 6 bilangan. Bilangan pangkat enam ≤ 265 adalah 16 dan 26 ada sebanyak 2 bilangan. Banyaknya bilangan yang bukan pangkat dua at au pangkat t iga yang ≤ 265 = 16 + 6 − 2 = 20. Maka 265 adalah suku ke 265 − 20 = 245. Lima bilangan set elah 265 yang bukan bilangan kuadrat at au pangkat t iga adalah 266, 267, 268, 269 dan 270. ∴ Suku ke-250 dari barisan t ersebut adalah 270 14. f (xy) = f (x + y) Jika x = n dan y = 1 maka f (n) = f (n + 1) Maka f (49 ) = f (48) = f (47) = f (46) = ⋅⋅⋅ = f (7) ∴ f (49) = 7 15. u25 = 3(u 5), maka a + 24b = 3(a + 4b) sehingga a = 6b un = a + (n − 1)b = 2u 1 = 2a 6b + (n − 1)b = 2(6b), maka n = 7 ∴ Suku t ersebut adalah suku ke- 7 16. Misalkan hari ini adalah hari ke-0 Karena kemarin Dimas membeli maj alah sedangkan Dimas membeli set iap 5 hari sekali maka Dimas akan membeli maj alah pada hari h1 = 5k + 4 dengan k bilangan asli. Karena hari ini Andre membeli maj alah sedangkan Andre membeli set iap 8 hari sekali maka Andre akan membeli maj alah pada hari h 2 = 8n dengan n bilangan asl i. Mereka akan membeli maj alah pada hari yang sama j ika 5k + 4 = 8n. Karena 4 dan 8 keduanya habis dibagi 4 maka k harus habis dibagi 4. Nilai k t erkecil adalah 4. h1 = h 2 = 5(4) + 4 = 24 ∴ Maka mereka akan membeli maj alah pada hari yang sama pal ing cepat 24 hari lagi. SMA Negeri 5 Bengkulu Eddy Hermanto, ST 164 Olimpiade Matematika Tk Kabupaten/ Kota 2006 17. Misalkan bilangan t ersebut adalah 1000a + 100b + 10c + d Maka 1000a + 100b + 10c + d − a − b − c − d = 2007 999a + 99b + 9c = 2007, maka 111a + 11b + c = 223 Karena a > 0 dan 111a < 223 maka a = 1 at au 2. Jika a = 1 maka 11b + c = 112 > 11(9) + 9 = 108 (t idak ada nilai b dan c yang memenuhi). Jika a = 2 maka 11b + c = 1. Nilai b dan c yang memenuhi hanya b = 0 dan c = 1. Tripel (a, b, c) yang memenuhi hanya ada 1 kemungkinan yait u (2, 0, 1). Nilai d yang memenuhi ada 10 kemungkinan yait u 0, 1, 2, ⋅⋅, 9. Bilangan 4 angka t ersebut yang memenuhi ada 10 yait u 2010, 2011, 2012, 2013, 2014, ⋅⋅⋅, 2019. ∴ Banyaknya bilangan yang dit emukan Nanang t idak akan lebih dari 10. 18. Persamaan parabola yang berpuncak di (x p, yp ) adalah y = a(x − x p) 2 + yp . Karena t it ik puncak parabola di (4, 2) maka y = a(x − 4) 2 + 2 Karena t it ik (2, 0) t erlet ak pada parabola maka : 0 = a(2 − 4) 2 + 2, maka a = −½ Persamaan parabola t ersebut adalah y = −½(x − 4) 2 + 2 = −½x 2 + 4x − 6 a = −½ ; b = 4 dan c = −6 ∴ abc = 12 19. Persamaan garis l1 adalah y + 3 = −2(x − p) Karena l2 t egak lurus l1 maka gradien garis l2 adalah ½. Persamaan garis l2 adalah y − p = ½(x − 6) Kedua garis melalui (a, b) maka : b + 3 = −2(a − p) dan b − p = ½(a − 6) 3 + p = −2(a − p) − ½(a − 6) 6 + 2p = −4a + 4p − a + 6 ∴ a= 2 p 5 20. Misalkan ∠ABC = β Luas ∆ABC = ½ ⋅ BA ⋅ BC sin β = 54 Karena MD sej aj ar EC maka ∆BMD sebangun dengan ∆BEC BM BE = BD BC BM ⋅ BC = BD ⋅ BE Luas ∆BED = ½ ⋅ BE ⋅ BD sin β = ½ ⋅ BM ⋅ BC sin β Luas ∆BED = ½ (½ ⋅ BA ⋅ BC sin β) Luas ∆BED = ½ Luas ∆ABC ∴ Luas segit iga BED adalah 27 satuan luas. SMA Negeri 5 Bengkulu Eddy Hermanto, ST 165 SELEKSI OLIMPIADE TINGKAT PROVINSI 2006 TIM OLIMPIADE MATEMATIKA INDONESIA 2007 Bidang Mat emat ika Bagian Per t ama Waktu : 90 Menit DEPARTEMEN PENDIDIKAN NASIONAL DIREKTORAT JENDERAL MANAJEMEN PENDIDIKAN DASAR DAN MENENGAH DIREKTORAT PEMBINAAN SEKOLAH MENENGAH ATAS TAHUN 2006 166 OLIMPIADE MATEMATIKA TINGKAT PROVINSI TAHUN 2006 BAGIAN PERTAMA 1. Hasil penj umlahan semua bilangan bulat di ant ara 3 2006 dan 2006 adalah ⋅⋅⋅⋅ 2. Pada t rapesium ABCD, sisi AB sej aj ar dengan DC. Sebuah lingkaran yang menyinggung keempat sisi t rapesium dapat dibuat . Jika AB = 75 dan DC = 40, maka keliling t rapesium ABCD = ⋅⋅⋅⋅⋅⋅⋅⋅ 3. Himpunan semua x yang memenuhi (x − 1) 3 + (x − 2) 2 = 1 adalah ⋅⋅⋅⋅⋅ 4. Bilangan prima dua angka t erbesar yang merupakan j umlah dua bilangan prima lainnya adalah ⋅⋅⋅⋅ 5. Af kar memilih suku-suku barisan geomet ri t akhingga 1, geomet ri t akhingga baru yang j umlahnya 1 1 1 , , , ⋅⋅⋅ unt uk membuat barisan 2 4 8 1 . Tiga suku pert ama pilihan Af kar adalah ⋅⋅⋅⋅⋅ 7 6. Luas sisi-sisi sebuah balok adalah 486, 486, 243, 243, 162, 162. Volume balok t ersebut adalah ⋅⋅⋅⋅⋅ ⎛1⎞ 7. Nilai maksimum f ungsi f (x) = ⎜⎜ ⎟⎟ ⎝3⎠ x 2 − 4x +3 adalah ⋅⋅⋅⋅ 8. Diberikan f ungsi f (x) = ⏐⏐x − 2⏐ − a⏐ − 3. Jika graf ik f memot ong sumbu-x t epat di t iga t it ik, maka a = ⋅⋅⋅⋅ 9. Unt uk bilangan asli n, t uliskan s(n) = 1 + 2 + ⋅⋅⋅ + n dan p(n) = 1 x 2 x ⋅⋅⋅ x n. Bilangan genap n t erkecil yang memenuhi p(n) habis dibagi s(n) adalah ⋅⋅⋅⋅ 10. Jika ⏐x ⏐+ x + y = 10 dan x + ⏐y⏐ − y = 12, maka x + y = ⋅⋅⋅⋅⋅⋅ 11. Sebuah himpunan t iga bilangan asli disebut himpunan arit mat ika j ika salah sat u unsurnya merupakan rat a-rat a dari dua unsur lainnya. Banyaknya subhimpunan arit mat ika dari {1, 2, 3, ⋅⋅⋅, 8} adalah ⋅⋅⋅⋅ 12. Dari set iap bilangan sat u-angka a, bilangan N dibuat dengan menyandingkan ket iga bilangan a + 2, a + 1, a yait u N = (a + 2)(a + 1)a . Sebagai cont oh, unt uk a = 8, N = 1098. Kesepuluh bilangan N semacam it u memiliki f akt or persekut uan t erbesar ⋅⋅⋅⋅⋅ 13. Jika x 2 + 1 = 47 , maka x2 x + 1 = ⋅⋅⋅⋅⋅⋅⋅⋅⋅⋅⋅⋅⋅ x 167 14. Sebuah kelas akan memilih seorang murid di ant ara mereka unt uk mewakili kelas t ersebut . Set iap murid mempunyai kesempat an yang sama unt uk dipilih. Peluang seorang murid laki-laki t erpilih sama dengan 23 kali peluang t erpilihnya seorang murid perempuan. Persent ase murid laki-laki di kelas t ersebut adalah ⋅⋅⋅⋅ 15. Pada segit iga ABC, garis bagi sudut A memot ong sisi BC di t it ik D. Jika AB = AD = 2 dan BD = 1, maka CD = ⋅⋅⋅⋅⋅ 16. Jika (x − 1) 2 membagi ax 4 + bx 3 + 1, maka ab = ⋅⋅⋅⋅ 17. Dari t it ik O dit arik dua set engah-garis (sinar) l1 dan l2 yang membent uk sudut lancip α. Tit ik – t it ik berbeda A1, A3, A5 t erlet ak pada garis l2, sedangkan t it ik-t it ik A2, A4, A6 t erlet ak di l1. Jika A1A2 = A2A3 = A3A4 = A4O = OA5 = A5A6 = A6A1, maka α = ⋅⋅⋅⋅⋅ 18. Banyaknya bilangan 7-angka berbeda yang dapat dibent uk dengan cara mengubah susunan angka 2504224 adalah ⋅⋅⋅⋅ 19. Evan membuat sebuah barisan bilangan asli a1, a2, a3, ⋅⋅⋅ yang memenuhi ak+1 − ak = 2(ak − ak-1) −1, unt uk k = 2, 3, ⋅⋅⋅, dan a2 − a1 = 2. Jika 2006 muncul dalam barisan, nilai a1 t erkecil yang mungkin adalah ⋅⋅⋅⋅⋅ 20. Pada segit iga ABC, garis-garis berat dari t it ik sudut B dan t it ik sudut C saling berpot ongan t egak lurus. Nilai minimum ct g B + ct g C adalah ⋅⋅⋅⋅⋅ 168 SELEKSI OLIMPIADE TINGKAT PROVINSI 2006 TIM OLIMPIADE MATEMATIKA INDONESIA 2007 Bidang Mat emat ika Bagian Kedua Waktu : 120 Menit DEPARTEMEN PENDIDIKAN NASIONAL DIREKTORAT JENDERAL MANAJEMEN PENDIDIKAN DASAR DAN MENENGAH DIREKTORAT PEMBINAAN SEKOLAH MENENGAH ATAS TAHUN 2006 169 OLIMPIADE MATEMATIKA TINGKAT PROVINSI TAHUN 2006 BAGIAN KEDUA 1. Misalkan segit iga ABC siku-siku di B. Garis t inggi dari B memot ong sisi AC di t it ik D. Jika t it ik E dan F bert urut -t urut adalah t it ik t engah BD dan CD, bukt ikan bahwa AE ⊥ BF. 2. Misalkan m bilangan asli yang memenuhi 1003 < m < 2006. Diberikan himpunan bilangan asli S = {1, 2, 3, ⋅⋅⋅, m}, berapa banyak anggot a S harus dipilih agar selalu t erdapat paling sedikit sat u pasang anggot a t erpilih yang hasil t ambahnya 2006 ? 3. Misalkan d = FPB(7n + 5, 5n + 4), dimana n adalah bilangan asli. (a) Bukt ikan bahwa unt uk set iap bilangan asli n berlaku d = 1 at au 3. (b) Bukt ikan bahwa d = 3 j ika dan hanya j ika n = 3k + 1, unt uk suat u bilangan asli k. 4. Win memiliki dua koin. Ia akan melakukan prosedur berikut berulang-ulang selama ia masih memiliki koin : lempar semua koin yang dimilikinya secara bersamaan; set iap koin yang muncul dengan sisi angka akan diberikannya kepada Albert . Tent ukan peluang bahwa Win akan mengulangi prosedur ini lebih dari t iga kali. 5. Misalkan a, b, c bilangan-bilangan asli. Jika semua akar ket iga persamaan x 2 − 2ax + b = 0 x 2 − 2bx + c = 0 x 2 − 2cx + a = 0 adalah bilangan asli, t ent ukan a, b dan c. 170 SELEKSI OLIMPIADE MATEMATIKA INDONESIA 2007 TINGKAT PROVINSI TAHUN 2006 Prestasi itu diraih bukan didapat !!! SOLUSI SOAL Bidang Mat emat ika Bagian Pertama Disusun oleh : Eddy Hermant o, ST 171 Solusi Olimpiade Matematika Tk Provinsi 2006 Bagian Pert ama BAGIAN PERTAMA 1. 123 = 1728 ; 133 = 2197 ; 442 = 1936 ; 452 = 2025 2006 < m < 2006 dapat disederhanakan menj adi 13 ≤ m ≤ 44 unt uk m bulat Himpunan m yang memenuhi = {13, 14, 15, ⋅⋅⋅, 44} 13 + 14 + 15 + ⋅⋅⋅ + 44 = 912 ∴ Penj umlahan semua bilangan yang memenuhi sama dengan 912. 3 2. Jika t it ik P di luar lingkaran dan garis yang dit arik dari t it ik P menyinggung lingkaran t ersebut di t it ik Q dan R maka PQ = PR Dari gambar di at as didapat DG = DH ; CG = CF ; BF = BE ; AE = AH Keliling = AE + AH + BE + BF + CF + CG + DG + DH = 2 (DG + CG + AE + BE) Keliling = 2(DC + AB) = 2(40 + 75) ∴ Keliling t rapesium = 230 − 1) 3 + (x − 2) 2 = 1 − 1) 3 = 1 − (x − 2) 2 = (1 − (x − 2))(1 + (x − 2)) = (3 − x)(x − 1) − 1)((x − 1) 2 − (3 − x)) = 0 − 1)(x 2 − x − 2) = 0 − 1)(x + 1)(x − 2) = 0 ∴ Himpunan semua nilai x yang memenuhi adalah { −1, 1, 2} 3. (x (x (x (x (x 4. Misalkan a, b dan c adalah ket iga bilangan prima t ersebut dengan a = b + c Bilangan prima genap hanya ada sat u yait u 2. Karena a > 2 maka a past i ganj il yang menyebabkan parit as b dan c harus berbeda. Tanpa mengurangi keumuman misalkan c ≤ b maka c = 2 a = b + 2 sehingga a − b = 2 Karena a − b = 2 maka t erdapat t epat 1 bilangan asli di ant ara a dan b. Misalkan bilangan t ersebut adalah k. Maka b, k dan a adalah 3 bilangan asli berurut an. Salah sat unya harus habis dibagi 3. Karena b dan a bilangan prima lebih dari 3 maka k habis dibagi 3. Karena k j uga genap maka k habis dibagi 6. Jika k = 16 ⋅ 6 = 96 maka b = 95 bukan prima. Jika k = 15 ⋅ 6 = 90 maka a = 91 bukan prima. Jika k = 14 ⋅ 6 = 84 maka a = 85 bukan prima. Jika k = 13 ⋅ 6 = 78 maka b = 77 bukan prima. Jika k = 12 ⋅ 6 = 72 maka a = 73 dan b = 71 yang memenuhi keduanya prima ∴ Bilangan prima dua angka t erbesar yang memenuhi adalah 73. SMA Negeri 5 Bengkulu Eddy Hermanto, ST 172 Olimpiade Matematika Tk Provinsi 2006 Solusi 5. S ∞ = Bagian Pert ama a 1− r Misalkan bilangan pert ama yang dipilih Af kar adalah (½) a unt uk a bilangan bulat t ak negat if dan rasio, r = (½) b unt uk b bilangan asli maka : ⎛1⎞ ⎜⎜ ⎟⎟ ⎝2⎠ a ⎛1⎞ 1 − ⎜⎜ ⎟⎟ ⎝2⎠ = b 1 7 Karena b asli maka ½ ≤ 1 − (½) b < 1 a 1 ⎛1⎞ 1 ≤ ⎜⎜ ⎟⎟ < 14 ⎝ 2 ⎠ 7 Nilai a yang memenuhi hanya a = 3 sehingga b = 3 Maka 3 suku pert ama yang dipilih Af kar adalah (½) 3, (½) 6 dan (½) 9 ∴ Tiga suku pert ama yang dipilih Af kar adalah 1 1 1 , , . 8 64 512 6. Misalkan panj ang sisi-sisi balok t ersebut adalah a, b dan c. Tanpa mengurangi keumuman misalkan ab = 486 = 2 ⋅ 35 ; (ab)(ac)(bc) = (abc) 2 = 22 ⋅ 314 abc = 2 ⋅ 37 = 4374 ∴ Volume balok = 4374 ⎛1⎞ 7. f (x ) = ⎜⎜ ⎟⎟ ⎝3⎠ ac = 243 = 35 ; bc = 162 = 2 ⋅ 34 x 2 − 4x +3 , maka f ( x ) = 3 −x 2 + 4x −3 Agar f (x) maksimum maka y = −x 2 + 4x − 3 harus maksimum. y = −x 2 + 4x − 3 = −(x − 2) 2 + 1 y maksimum = 1 saat x = 2 f (x) maksimum = 3 ∴ f (x) maksimum = 3 8. f (x) = ⏐⏐x − 2⏐ − a⏐ − 3 f memot ong sumbu x maka ⏐⏐x − 2⏐ − a⏐ − 3 = 0 ⏐⏐x − 2⏐ − a⏐ = 3 ⏐x − 2⏐ − a = 3 at au ⏐x − 2⏐ − a = −3 ⏐x − 2⏐ = a + 3 at au ⏐x − 2⏐ = a − 3 Jika a + 3 = 0 maka ⏐x − 2⏐ = 0 hanya ada 1 penyelesaian. Sebaliknya j ika a + 3 ≠ 0 maka penyelesaian ⏐x − 2⏐ = a + 3 ada 2 penyelesaian yait u x − 2 = a + 3 at au x − 2 = −(a + 3) Hal yang sama unt uk persamaan ⏐x − 2 ⏐ = a − 3 Maka j ika a = −3 akan menyebabkan hanya ada 1 penyelesaian x unt uk persamaan ⏐x − 2⏐ = a + 3 namun ada dua nilai x unt uk penyelesaian ⏐x − 2⏐ = a − 3 SMA Negeri 5 Bengkulu Eddy Hermanto, ST 173 Olimpiade Matematika Tk Provinsi 2006 Solusi Bagian Pert ama Sedangkan j ika a = 3 akan menyebabkan hanya ada 1 penyelesaian x unt uk persamaan ⏐x − 2⏐ = a − 3 namun ada dua nilai x unt uk penyelesaian ⏐x − 2⏐ = a + 3 ∴ Nilai a yang membuat graf ik f memot ong sumbu x t epat di 3 t it ik adalah a = 3 atau a = −3. 9. s(n) = 1 + 2 + ⋅⋅⋅ + n = ½n(n + 1) p(n) = 1 x 2 x ⋅⋅⋅ x n Karena n genap maka ½n bilangan bulat . Karena n + 1 > 1 ; n + 1 > 2 ; ⋅⋅⋅ ; n + 1 > n maka agar p(n) habis dibagi s(n) maka n + 1 t idak boleh prima. Bilangan genap t erkecil yang menyebabkan n + 1 bukan prima adalah 8. ∴ Bilangan genap t erkecil yang memenuhi p(n) habis dibagi s(n) adalah 8. 10. ⏐x ⏐ + x + y = 10 dan x + ⏐y ⏐ − y = 12 * Jika x dan y di kuadran I maka ⏐x ⏐ = x dan ⏐y ⏐ = y 2x + y = 10 dan x = 12 sehingga y = −14 (t idak memenuhi (x, y) di kuadran I) * Jika x dan y di kuadran II maka ⏐x ⏐ = −x dan ⏐y ⏐ = y y = 10 dan x = 12 (t idak memenuhi (x, y) di kuadran II) * Jika x dan y di kuadran III maka ⏐x ⏐ = −x dan ⏐y ⏐ = −y y = 10 dan x − 2y = 12 sehingga x = 32 (t idak memenuhi (x, y) di kuadran III) * Jika x dan y di kuadran IV maka ⏐x ⏐ = x dan ⏐y ⏐ = −y 2x + y = 10 dan x − 2y = 12 Nilai (x, y) yang memenuhi adalah ( ∴ x+y= 32 14 ) (memenuhi (x, y) di kuadran IV) , − 5 5 32 14 18 = − 5 5 5 11. Jika a, b dan c adalah himpunan arit mat ika maka 2b = a + c dengan a < c. • Jika b = 2 maka a + c = 4. Ada 1 pasangan (a, c) yang memenuhi yait u (1, 3) • Jika b = 3 maka a + c = 6. Ada 2 pasangan (a, c) yang memenuhi yait u (1, 5), (2, 4) • Jika b = 4 maka a + c = 8. Ada 3 pasangan (a, c) yang memenuhi yait u (1, 7), (2, 6), (3, 5) • Jika b = 5 maka a + c = 10. Ada 3 pasangan (a, c) yang memenuhi yait u (2, 8), (3, 7), (4, 6) • Jika b = 6 maka a + c = 12. Ada 2 pasangan (a, c) yang memenuhi yait u (4, 8), (5, 7) • Jika b = 7 maka a + c = 14. Ada 1 pasangan (a, c) yang memenuhi yait u (6, 8) ∴ Banyaknya himpunan arit mat ika = 1 + 2 + 3 + 3 + 2 + 1 = 12 12. (a + 2) + (a + 1) + a = 3(a + 1) Maka semua bilangan yang berbent uk N = (a + 2)(a + 1)a habis dibagi 3 sebab penj umlahan digit nya habis dibagi 3. 321 = 3 ⋅ 107 dengan 3 dan 107 adalah bilangan prima. Tet api 432/ 107 bukan bilangan bulat at au 107 t idak membagi 432. FPB (321, 432) = 3 ∴ Maka kesepuluh bilangan N semacam it u memiliki f akt or persekut uan t erbesar = 3 SMA Negeri 5 Bengkulu Eddy Hermanto, ST 174 Olimpiade Matematika Tk Provinsi 2006 Solusi 13. x 2 + 1 x 2 Bagian Pert ama 2 ⎛ 1⎞ 1 = 47 , maka ⎜⎜ x + ⎟⎟ − 2 = 47 sehingga x + = 7 x ⎠ x ⎝ 2 ⎛ 1 ⎞ ⎟ −2=7 ⎜ x + ⎟ ⎜ x ⎠ ⎝ 1 x + =3 ∴ x 14. Misalkan j umlah murid laki-laki = m dan j umlah murid perempuan = n (m : (m + n)) : (n : (m + n)) = 2 : 3 m : n = 2 : 3, maka 3m = 2n m 2m 2 = = m +n 2m + 3m 5 ∴ Persent ase murid laki-laki di kelas t ersebut adalah 40 % 15. Karena α < 45 maka AC > AD sehingga AC > 2 Karena AD adalah garis bagi ∆ABC maka berlaku AB BD sehingga AC = 2 CD = AC CD Misalkan panj ang CD = x maka AC = 2x 2 2 + 2 2 − 12 7 = 2⋅2⋅2 8 ( 2x ) 2 + 2 2 − (1 + x ) 2 2 Pada ∆ABC berlaku cos 2α = 2 cos α − 1 = 2 ⋅ ( 2x ) ⋅ 2 2 2 34 4x + 4 − (1 + 2x + x ) = 64 8x Pada ∆ABD berlaku cos α = 17x = 12x 2 − 8x + 12 (4x − 3)(3x − 4) = 0 Karena AC > 2 maka x > 1 Nilai x yang memenuhi hanya x = ∴ CD = 4 3 4 3 SMA Negeri 5 Bengkulu Eddy Hermanto, ST 175 Olimpiade Matematika Tk Provinsi 2006 Solusi Bagian Pert ama 16. ax 4 + bx 3 + 1 = q(x) ⋅ (x − 1) 2 Jelas bahwa q(x) harus merupakan f ungsi kuadrat . Karena koef isien x 4 adalah a dan konst ant a ruas kiri = 1 maka q(x) = ax 2 + px + 1 ax 4 + bx 3 + 1 = (ax 2 + px + 1) ⋅ (x 2 − 2x + 1) ax 4 + bx 3 + 1 = ax 4 + ( −2a + p)x 3 + (a − 2p + 1)x 2 + (p − 2)x + 1 Dari persamaan di at as didapat : Berdasarkan koef isien x maka p − 2 = 0 sehingga p = 2 Berdasarkan koef isien x 2 maka a − 2p + 1 = 0 sehingga a = 3 Berdasarkan koef isien x 3 maka b = −2a + p sehingga b = −4 ∴ ab = −12 17. Karena A4O = A3A4 maka ∆OA4A3 sama kaki sehingga ∠OA3A4 = α dan ∠A3A4A6 = 2α Pada ∆A4A3A2 sama kaki berlaku ∠A3A2A4 = 2α, maka ∠A4A3A2 = 180o − 4α sehingga ∠A2A3A1 = 3α Pada ∆A1A2A3 sama kaki berlaku ∠A2A1A3 = 3α, maka ∠A1A2A3 = 180o − 6α ∠A1A2A6 = ∠A3A2A4 + ∠A1A2A3 = 180o − 4α Pada ∆A1A2A6 sama kaki berlaku ∠A1A6A2 = ∠A1A2A6 = 180o − 4α, maka ∠A6A1A2 = 8α − 180o ∠A5A1A6 = ∠A2A1A3 − ∠A6A1A2 = 3α − (8α − 180o) = 180o − 5α Pada ∆A1A6A5 sama kaki berlaku ∠A6A5A1 = ∠A5A1A6 = 180o − 5α, maka ∠A6A5O = 5α Pada ∆OA5A6 sama kaki berlaku ∠OA6A5 = ∠A5OA6 = α Pada ∆OA5A6 berlaku ∠A5OA6 + ∠OA5A6 + ∠OA6A5 = 180o α + 5α + α = 180o ∴ α = 180° 7 18. Banyaknya susunan 7 angka dengan 3 buah angka 2 yang sama dan 2 buah angka 4 yang sama adalah 7! = 420. Tet api 420 bilangan t ersebut t ermasuk bilangan dengan angka 0 pada angka 3!⋅2! pert ama. Banyaknya bilangan dengan 0 pada angka pert ama adalah 6! = 60 3!⋅2! ∴ Banyaknya bilangan yang dapat dibent uk adalah 420 − 60 = 360. SMA Negeri 5 Bengkulu Eddy Hermanto, ST 176 Olimpiade Matematika Tk Provinsi 2006 Solusi Bagian Pert ama 19. ak+1 − ak = 2(ak − ak-1) − 1 Misalkan a2 − a1 = 2 = u1 a3 − a2 = 2(a2 − a1) − 1 = 2u1 − 1 = u 2 a4 − a3 = 2(a3 − a2) − 1 = 2u2 − 1 = u 3 M ak+1 − ak = 2(ak − ak-1) − 1 = 2uk-1 − 1 = uk Jumlahkan seluruh persamaan di at as didapat : ak+1 − a1 = u1 + u2 + u3 + ⋅⋅⋅ + uk Karena a1, a2, a3, ⋅⋅⋅ semuanya asli maka ak+1 > u1 + u2 + u3 + ⋅⋅⋅ + uk Misalkan ak+1 = 2006 Agar didapat (a1) minimal maka u1 + u 2 + u3 + ⋅⋅⋅ + uk harus paling dekat dengan 2006 namun kurang dari 2006 u1 = 2 ; u2 = 3 ; u3 = 5 ; u4 = 9 ; u5 = 17 ; u 6 = 33 ; u7 = 65 ; u8 = 129 ; u9 = 257 ; u10 = 513 dan u 11 = 1025. u1 + u2 + u3 + ⋅⋅⋅ + u10 = 1033 sedangkan u 1 + u2 + u3 + ⋅⋅⋅ + u11 = 2058 > 2006 maka 2006 = a11 a11 − a1 = u1 + u2 + u 3 + ⋅⋅⋅ + u10 = 1033 (a1) minimum = 2006 − 1033 ∴ (a1) minimum = 973 20. CF dan BE adalah garis berat yang berpot ongan di t it ik D. Maka CD : DF = 2 : 1 dan BD : DE = 2 : 1 Misalkan DF = x maka CD = 2x dan j ika DE = y maka BD = 2y t an B = t an ( ∠CBD + ∠FBD) = tan ∠CBD + tan ∠FBD 1 − tan ∠CBD ⋅ tan ∠FBD 2x + 2y tan B = 2x 1− 2y x 3xy 2y x 2y = , maka ctgB = − 2 2 x 3x 3y 2y − x ⋅ 2y tan ∠BCD + tan ∠ECD t an C = t an ( ∠BCD + ∠ECD) = 1 − tan ∠BCD ⋅ tan ∠ECD y 2y + y 2x 3xy 2x = , maka ctgC = − tan C = 2x 2 2 2y y 3y 3x 2x − y 1− ⋅ 2x 2x SMA Negeri 5 Bengkulu Eddy Hermanto, ST 177 Olimpiade Matematika Tk Provinsi 2006 Solusi ctgB + ctgC = Bagian Pert ama x y + 3y 3x Berdasarkan ket aksamaan AM-GM maka : ctgB + ctgC ≥ 2 x y 2 ⋅ = 3y 3x 3 ∴ Maka nilai minimum ct g B + ct g C adalah 2 3 SMA Negeri 5 Bengkulu Eddy Hermanto, ST 178 SELEKSI OLIMPIADE MATEMATIKA INDONESIA 2007 TINGKAT PROVINSI TAHUN 2006 Prestasi itu diraih bukan didapat !!! SOLUSI SOAL Bidang Mat emat ika Bagian Kedua Disusun oleh : Eddy Her mant o, ST 179 Solusi Olimpiade Matematika Tk Provinsi 2006 Bagian Kedua BAGIAN KEDUA 1. Alt ernat if 1 : Misalkan ∠GAF = α dan ∠GFA = γ BD DE BD tan A sedangkan tan α = = = AD AD 2AD 2 BD BD 2BD sedangkan tan γ = tan C = = = 2 tan C CD FD CD tan A = ⋅⋅⋅⋅⋅⋅⋅⋅⋅⋅⋅⋅⋅⋅⋅⋅ (1) ⋅⋅⋅⋅⋅⋅⋅⋅⋅⋅⋅⋅⋅⋅⋅⋅ (2) A + C = 90o, maka t an A = t an (90o − C) = ct g C sehingga t an A t an C = 1 t an α ⋅ t an γ = t an A ⋅ t an C = 1 tan (α + γ ) = tan α + tan γ 1 − tan α tan γ Karena t an α ⋅ t an γ = 1 maka α + γ = 90o Pada ∆AGF berlaku ∠AGF = 180o − ( α + γ) = 90o Karena ∠AGF = 90o maka AG t egak lurus FG ∴ Terbukti bahwa AE ⊥ BF Alt ernat if 2 : Misalkan ∠BAC = θ maka ∠ABD = 90o − θ Jelas bahwa ∠DBC = θ. Karena ∆BCD siku-siku di D maka ∠BCD = 90o − θ. Akibat nya ∆ABD sebangun dengan ∆BCD. Karena E pert engahan BD dan F pert engahan CD maka ∆EAD sebangun dengan ∆BDF. Misalkan ∠GAF = α. Karena ∆EAD sebangun dengan ∆BDF, maka ∠FBD = α. Karena ∆AED siku-siku di D maka ∠DEA = ∠GEB = 90o − α. Pada ∆BEG berlaku : ∠BEG + ∠FBD + ∠EGB = 180o ( α) + (90o − α) + ∠EGB = 180o ∠EGB = 90o Karena ∠EGB = 90o maka garis AG t egak lurus BF. Jadi garis AE t egak lurus BF (t erbukt i). ∴ Terbukti bahwa AE ⊥ BF 2. Dibuat subhimpunan {1, 2005}, {2, 2004}, {3, 2003}, ⋅⋅⋅, {1002, 1004}, {1003} Jika diambil sat u bilangan dari masing-masing subhimpunan t ersebut maka t erdapat 1003 bilangan yang t idak ada sepasang di ant aranya yang berj umlah 2006. Jika dit ambahkan sat u bilangan lagi selain 1003 bilangan t ersebut maka dapat dipast ikan t erdapat sepasang bilangan yang berj umlah 2006. ∴ Banyaknya anggot a S harus dipilih agar selalu t erdapat paling sedikit sat u pasang anggot a t erpilih yang hasil t ambahnya 2006 adalah 1004. SMA Negeri 5 Bengkulu Eddy Hermanto, ST 180 Olimpiade Matematika Tk Provinsi 2006 Solusi Bagian Kedua 3. d = FPB(7n + 5, 5n + 4) a. Maka d ⏐7n + 5 dan d ⏐5n + 4 Karena d membagi 7n + 5 maka d j uga membagi 5(7n + 5) Karena d membagi 5n + 4 maka d j uga membagi 7(5n + 4) Akibat nya d j uga membagi 7(5n + 4) − 5(7n + 5) = 3 Karena d⏐3 maka d = 1 atau 3 (terbukti) b. Sebuah bilangan akan t ermasuk ke dalam salah sat u bent uk dari 3k, 3k + 1 at au 3k + 2 Jika n = 3k maka 7n + 5 = 21k + 5 ≡ 2 (mod 3) dan 5n + 4 = 15k + 4 ≡ 1 (mod 3) Jika n = 3k + 1 maka 7n + 5 = 21k + 12 ≡ 0 (mod 3) dan 5n + 4 = 15k + 9 ≡ 0 (mod 3) Jika n = 3k + 2 maka 7n + 5 = 21k + 19 ≡ 1 (mod 3) dan 5n + 4 = 15k + 14 ≡ 2 (mod 3) ∴ Terbukti bahwa hanya bentuk n = 3k + 1 yang menyebabkan kedua bilangan 7n + 5 dan 5n + 4 habis dibagi 3 untuk n bilangan asli. 4. Agar Win akan mengulangi prosedur pelemparan koin lebih dari t iga kali maka pada lemparan yang ket iga masih t erdapat sedikit nya sat u koin yang muncul dengan sisinya bukan angka. Pada lemparan pert ama agar hal t ersebut t erj adi maka sisi koin yang muncul haruslah t erdapat t epat sat u sisi angka dan sat u sisi bukan angka at au kedua sisi bukan angka. ¾ Jika pada lemparan pert ama yang muncul adalah sat u sisi angka dan sat u bukan angka. Peluang t ersebut adalah ½. Pada lemparan kedua dan ket iga sisi sat u-sat unya koin yang ia lempar harus bukan angka. Peluang pada masing-masing kej adian adalah ½ . Peluang Win akan mengulangai prosedur lebih dari t iga kali adalah ½ ⋅ ½ ⋅ ½ = ¾ Jika pada lemparan pert ama kedua koin muncul dengan sisi bukan angka Peluang kej adian t ersebut adalah ½ ⋅ ½ = ¼ 1 8 Agar Win akan mengulangi prosedur maka pada lemparan kedua sisi koin yang muncul haruslah t erdapat t epat sat u sisi angka dan sat u sisi bukan angka at au kedua sisi bukan angka. ¾ Jika pada lemparan kedua yang muncul adalah sat u sisi angka dan sat u bukan angka Peluang t ersebut adalah ½. Pada lemparan ket iga sisi sat u-sat unya koin yang ia lempar t ersebut harus bukan angka. Peluang kej adian t ersebut adalah ½ . 1 16 Peluang Win akan mengulangi prosedur lebih dari t iga kali adalah ¼ ⋅ ½ ⋅ ½ = ¾ Jika pada lemparan kedua, kedua koin muncul dengan sisi bukan angka Peluang kej adian t ersebut adalah ½ ⋅ ½ = ¼ Agar Win akan mengulangai prosedur maka pada lemparan ket iga sisi koin yang muncul haruslah t erdapat t epat sat u sisi angka dan sat u sisi bukan angka at au kedua sisi bukan angka. Peluang kej adian ini adalah ¾. Peluang Win akan mengulangi prosedur lebih dari t iga kali adalah ¼ ⋅ ¼ ⋅ ¾ = ∴ Maka peluang Win akan mengulangi prosedur t ersebut lebih dari 3 3 64 kali adalah 1 1 3 15 + + = 8 16 64 64 SMA Negeri 5 Bengkulu Eddy Hermanto, ST 181 Solusi Olimpiade Matematika Tk Provinsi 2006 Bagian Kedua 5. x 2 − 2ax + b = 0 ⋅⋅⋅⋅⋅⋅⋅⋅⋅⋅⋅⋅⋅⋅⋅⋅⋅⋅⋅⋅⋅⋅⋅⋅⋅⋅⋅⋅⋅⋅⋅⋅⋅⋅⋅⋅⋅⋅⋅⋅⋅⋅⋅⋅⋅⋅⋅⋅ (1) x 2 − 2bx + c = 0 ⋅⋅⋅⋅⋅⋅⋅⋅⋅⋅⋅⋅⋅⋅⋅⋅⋅⋅⋅⋅⋅⋅⋅⋅⋅⋅⋅⋅⋅⋅⋅⋅⋅⋅⋅⋅⋅⋅⋅⋅⋅⋅⋅⋅⋅⋅⋅⋅ (2) x 2 − 2cx + a = 0 ⋅⋅⋅⋅⋅⋅⋅⋅⋅⋅⋅⋅⋅⋅⋅⋅⋅⋅⋅⋅⋅⋅⋅⋅⋅⋅⋅⋅⋅⋅⋅⋅⋅⋅⋅⋅⋅⋅⋅⋅⋅⋅⋅⋅⋅⋅⋅⋅ (3) Karena akar-akar persamaan kuadrat di at as adalah bilangan asli maka diskriminannya harus merupakan kuadrat sempurna. Dari pers (1) didapat f akt a bahwa 4a2 − 4b merupakan kuadrat sempurna Maka a2 − b merupakan kuadrat sempurna ⋅⋅⋅⋅⋅⋅⋅⋅⋅⋅⋅⋅⋅⋅⋅⋅⋅⋅⋅⋅⋅⋅⋅⋅⋅⋅⋅⋅⋅⋅⋅⋅⋅⋅⋅⋅⋅⋅⋅⋅⋅⋅⋅⋅⋅⋅⋅⋅⋅⋅⋅⋅⋅⋅⋅⋅⋅⋅⋅⋅⋅⋅⋅⋅⋅⋅⋅⋅⋅⋅ (4) Dengan cara yang sama unt uk persamaan (2) dan (3) didapat : b 2 − c j uga kuadrat sempurna ⋅⋅⋅⋅⋅⋅⋅⋅⋅⋅⋅⋅⋅⋅⋅⋅⋅⋅⋅⋅⋅⋅⋅⋅⋅⋅⋅⋅⋅⋅⋅⋅⋅⋅⋅⋅⋅⋅⋅⋅⋅⋅⋅⋅⋅⋅⋅⋅⋅⋅⋅⋅⋅⋅⋅⋅⋅⋅⋅⋅⋅⋅⋅⋅⋅⋅⋅⋅⋅⋅ (5) c 2 − a j uga kuadrat sempurna ⋅⋅⋅⋅⋅⋅⋅⋅⋅⋅⋅⋅⋅⋅⋅⋅⋅⋅⋅⋅⋅⋅⋅⋅⋅⋅⋅⋅⋅⋅⋅⋅⋅⋅⋅⋅⋅⋅⋅⋅⋅⋅⋅⋅⋅⋅⋅⋅⋅⋅⋅⋅⋅⋅⋅⋅⋅⋅⋅⋅⋅⋅⋅⋅⋅⋅⋅⋅⋅⋅ (6) Pada persamaan (4) karena a dan b bilangan asli maka a2 − b < a2 at au a2 − b ≤ (a − 1) 2 −b ≤ −2a + 1, maka b ≥ 2a − 1 ⋅⋅⋅⋅⋅⋅⋅⋅⋅⋅⋅⋅⋅⋅⋅⋅⋅⋅⋅⋅⋅⋅⋅⋅⋅⋅⋅⋅⋅⋅⋅⋅⋅⋅⋅⋅⋅⋅⋅⋅⋅⋅⋅⋅⋅⋅ (7) Dengan cara yang sama unt uk persamaan (5) dan (6) didapat : c ≥ 2b − 1 ⋅⋅⋅⋅⋅⋅⋅⋅⋅⋅⋅⋅⋅⋅⋅⋅⋅⋅⋅⋅⋅⋅⋅⋅⋅⋅⋅⋅⋅⋅⋅⋅⋅⋅⋅⋅⋅⋅⋅⋅⋅⋅⋅⋅⋅⋅ (8) a ≥ 2c − 1 ⋅⋅⋅⋅⋅⋅⋅⋅⋅⋅⋅⋅⋅⋅⋅⋅⋅⋅⋅⋅⋅⋅⋅⋅⋅⋅⋅⋅⋅⋅⋅⋅⋅⋅⋅⋅⋅⋅⋅⋅⋅⋅⋅⋅⋅⋅ (9) Maka : a ≥ 2c − 1 ≥ 2(2b − 1) − 1 ≥ 2(2(2a − 1) − 1) − 1. a ≥ 8a − 7, maka a ≤ 1 sehingga a = 1 Dari persamaan (9) didapat 1 ≥ 2c − 1 maka c ≤ 1 sehingga c = 1 Dari persamaan (8) didapat 1 ≥ 2b − 1 maka b ≤ 1 sehingga b = 1 ∴ a, b dan c yang memenuhi persamaan t ersebut hanya a = b = c = 1 SMA Negeri 5 Bengkulu Eddy Hermanto, ST 182 SELEKSI TIM OLIMPIADE MATEMATIKA INDONESIA 2007 OLIMPIADE SAINS NASIONAL 2006 SEMARANG (JAWA TENGAH), 4 – 9 SEPTEMBER 2006 Bidang Mat emat ika Hari Pert ama Waktu : 180 Menit DEPARTEMEN PENDIDIKAN NASIONAL DIREKTORAT JENDERAL MANAJEMEN PENDIDIKAN DASAR DAN MENENGAH DIREKTORAT PEMBINAAN SEKOLAH MENENGAH ATAS TAHUN 2006 183 OLIMPIADE SAINS NASIONAL 2006 4 – 9 SEPTEMBER 2006 SEMARANG, JAWA TENGAH BI DAN G : MATEMATI KA HARI PERTAMA WAKTU : 180 MENIT 1. Tent ukan semua pasangan bilangan real (x, y) yang memenuhi x 3 − y3 = 4(x − y) x 3 + y3 = 2(x + y) 2. Misalkan a, b, c adalah bilangan-bilangan asli. Jika 30⏐(a + b + c), bukt ikan bahwa 30⏐(a5 + b 5 + c 5) [ Cat at an : x ⏐y menyat akan x habis membagi y. ] 3. Misalkan S adalah himpunan semua segit iga ABC yang memenuhi sif at : t an A, t an B dan t an C adalah bilangan-bilangan asli. Bukt ikan bahwa semua segit iga anggot a S sebangun. 4. Misalkan n > 2 sebuah bilangan asli t et ap. Sebuah bidak hit am dit empat kan pada pet ak pert ama dan sebuah bidak put ih dit empat kan pada pet ak t erakhir sebuah papan ‘ cat ur’ berukuran 1 x n. Wiwit dan Sit i lalu melangkah bergant ian. Wiwit memulai permainan dengan bidak put ih. Pada set iap langkah, pemain memindahkan bidaknya sendiri sat u at au dua pet ak ke kanan at au ke kiri t anpa melompat i bidak lawan. Pemain yang t idak bisa melangkah dinyat akan kalah. Pemain manakah yang memiliki cara (st rat egi) unt uk selalu memenangkan permainan, apa pun yang dilakukan lawannya ? Jelaskan st rat egi pemain t ersebut ? 184 SELEKSI TIM OLIMPIADE MATEMATIKA INDONESIA 2007 OLIMPIADE SAINS NASIONAL 2006 SEMARANG (JAWA TENGAH), 4 – 9 SEPTEMBER 2006 Bidang Mat emat ika Hari Kedua Waktu : 180 Menit DEPARTEMEN PENDIDIKAN NASIONAL DIREKTORAT JENDERAL MANAJEMEN PENDIDIKAN DASAR DAN MENENGAH DIREKTORAT PEMBINAAN SEKOLAH MENENGAH ATAS TAHUN 2006 185 OLIMPIADE SAINS NASIONAL 2006 4 – 9 SEPTEMBER 2006 SEMARANG, JAWA TENGAH BI DAN G : MATEMATI KA HARI KEDUA WAKTU : 180 MENIT 5. Pada segit iga ABC, M adalah t it ik t engah BC dan G adalah t it ik berat segit iga ABC. Sebuah garis l melalui G memot ong ruas garis AB di P dan ruas garis AC di Q, dimana P ≠ B dan Q ≠ C. Jika [ XYZ] menyat akan luas segit iga XYZ, t unj ukkan bahwa [BGM ] + [CMG ] = 3 [PAG ] [QGA] 2 6. Set iap nomor t elepon di suat u daerah t erdiri dari 8 angka dan diawali dengan angka 8. Pak Edy, yang baru pindah ke daerah it u, mengaj ukan pemasangan sebuah t elepon baru. Berapakah peluang pak Edy mendapat kan nomor t elepon yang memuat t idak lebih dari 5 angka berbeda ? 7. Misalkan a, b, c bilangan-bilangan real sehingga ab, bc, ca bilangan-bilangan rasional. Bukt ikan bahwa ada bilangan-bilangan bulat x, y, z yang t idak semuanya nol, sehingga ax + by + cz = 0. 8. Tent ukan bilangan bulat 85-angka t erbesar yang memenuhi sif at ; j umlah semua angkanya sama dengan hasilkali semua angkanya. 186 SELEKSI TIM OLIMPIADE MATEMATIKA INDONESIA 2007 OLIMPIADE SAINS NASIONAL 2006 SEMARANG (JAWA TENGAH), 4 – 9 SEPTEMBER 2006 Prestasi itu diraih bukan didapat !!! SOLUSI SOAL Bidang Mat emat ika Disusun ol eh : Eddy Hermant o, ST 187 Olimpiade Sains Nasional 2006 Sol usi 1. x 3 − y 3 = 4(x − y), maka (x − y)(x 2 + xy + y 2) = 4(x − y) x 3 + y 3 = 2(x + y), maka (x + y)(x 2 − xy + y 2) = 2(x + y) ¾ Jika x = y Subt it usikan ke per samaan (2). Bidang : Mat emat ika ⋅⋅⋅⋅⋅⋅⋅⋅⋅⋅⋅⋅⋅⋅⋅⋅⋅⋅⋅⋅⋅⋅⋅⋅⋅⋅⋅⋅⋅⋅⋅⋅⋅⋅⋅⋅⋅⋅ (1) ⋅⋅⋅⋅⋅⋅⋅⋅⋅⋅⋅⋅⋅⋅⋅⋅⋅⋅⋅⋅⋅⋅⋅⋅⋅⋅⋅⋅⋅⋅⋅⋅⋅⋅⋅⋅⋅⋅ (2) ± 2 Pasangan (x, y) yang memenuhi adal ah (0, 0), ( 2 , 2 ), ( − 2 , − 2 ) (2x)(x 2) = 4x, maka x(x 2 − 2) = 0 sehingga x = 0 at au x = ¾ Jika x = −y Subt it usikan ke per samaan (1). (2x)(x 2) = 8x, maka x(x 2 − 4) = 0 x = 0 , x = 2 at au x = −2 Pasangan (x, y) yang memenuhi adal ah (0, 0), (2, −2), ( −2, 2) ¾ Jika x ≠ y dan x ≠ −y x 2 + xy + y 2 = 4 ⋅⋅⋅⋅⋅⋅⋅⋅⋅⋅⋅⋅⋅⋅⋅⋅⋅⋅⋅⋅⋅⋅⋅⋅⋅⋅⋅⋅ (3) x 2 − xy + y 2 = 2 ⋅⋅⋅⋅⋅⋅⋅⋅⋅⋅⋅⋅⋅⋅⋅⋅⋅⋅⋅⋅⋅⋅⋅⋅⋅⋅⋅⋅ (4) Kurangkan (3) dengan (4), maka xy = 1 dan x 2 + y 2 = 3 (x + y) 2 − 2xy = 3, maka (x + y) 2 = 5 * Jika x + y = ( ) 5 x 5 − x =1 x − x 5 +1 = 0 2 * x= 5 +1 1 , maka y = = 2 x x= 5 −1 1 , maka y = = 2 x Jika x + y = − ( ) ⎛ 5 +1 5 −1⎞ 5 −1 ⎟ sehingga (x y) = ⎜ ⎜ 2 , 2 ⎟ 2 ⎠ ⎝ ⎛ 5 −1 5 +1⎞ 5 +1 ⎟ sehingga (x y) = ⎜ ⎜ 2 , 2 ⎟ 2 ⎠ ⎝ 5 x − 5 − x =1 x2 + x 5 +1 = 0 ⎛− − 5 +1 1 5 +1 sehingga (x y) = ⎜ , maka y = = − ⎜ 2 x 2 ⎝ ⎛− − 5 −1 1 5 −1 sehingga (x y) = ⎜ , maka y = = − x= ⎜ 2 x 2 ⎝ x= 5 +1 5 +1⎞ ⎟ ,− 2 2 ⎟⎠ 5 −1 5 −1⎞ ⎟ ,− 2 2 ⎟⎠ Set el ah dicek ke persamaan semul a, semua pasangan (x, y) t ersebut memenuhi. 2 ), ( − 2 , − 2 ), (2, −2), ( −2, 2), 5 −1 5 +1⎞ ⎛ − 5 +1 5 +1⎞ ⎛ − 5 −1 5 −1⎞ ⎟, ⎜ ⎟, ⎜ ⎟ , ,− ,− ⎜ ⎜ ⎟ ⎟ 2 2 2 2 ⎠ ⎝ 2 ⎠ ⎝ 2 ⎟⎠ ∴ Pasangan (x, y) yang memenuhi adalah (0, 0), ( 2 , ⎛ 5 +1 5 −1⎞ ⎜ ⎟ ⎜ 2 , 2 ⎟, ⎝ ⎠ ⎛ ⎜ ⎜ ⎝ SMA Negeri 5 Bengkulu Eddy Hermanto, ST 188 Sol usi Olimpiade Sains Nasional 2006 Bidang : Mat emat ika 2. Akan dibukt ikan bahwa 30⏐n 5 − n unt uk n bil angan asl i. n 5 − n = n(n 2 − 1)(n 2 + 1) = n(n − 1)(n + 1)(n 2 − 4 + 5) n 5 − n = (n − 2)(n − 1)n(n + 1)(n + 2) + 5(n − 1)n(n + 1) Karena n −2, n −1, n, n+1, n+2 adal ah 5 bil . bul at berurut an maka (n − 2)(n − 1)n(n + 1)(n + 2) habis dibagi 5! = 120 sehingga 30 ⏐(n − 2)(n − 1)n(n + 1)(n + 2) n − 1, n dan n + 1 adal ah 3 bil angan bul at berurut an maka 3! = 6 membagi (n − 1)n(n + 1). Akibat nya 30⏐5(n − 1)n(n + 1) Maka 30⏐ n 5 − n unt uk n bil angan asl i 30⏐a5 − a + b 5 − b + c 5 − c unt uk a, b, c bil angan asl i, maka30 ⏐a5 + b 5 + c 5 − (a + b + c) ∴ Karena 30 ⏐(a + b + c) maka 30 ⏐( a5 + b5 + c5 ) (terbukti) 3. Pada segit iga ABC berl aku A + B = 180 o − C t an (A + B) = t an (180o − C) tan A + tan B = − tan C 1 − tan A ⋅ tan B t an A + t an B + t an C = t an A ⋅ t an B ⋅ t an C Misal kan t an A = x, t an B = y dan t an C = z unt uk x, y, z ∈ Bil angan Asl i Tanpa mengurangi keumuman misal kan x ≤ y ≤ z x + y + z = xyz, maka 3z ≥ xyz sehingga xy ≤ 3 Nil ai (x, y) yang memenuhi adal ah (1, 1), (1, 2) dan (1, 3) Jika x = 1 dan y = 1 maka 1 + 1 + z = z sehingga 2 + z = z (t idak ada z yang memenuhi) Jika x = 1 dan y = 2 maka 1 + 2 + z = 2z sehingga z = 3 (memenuhi) Jika x = 1 dan y = 3 maka 1 + 3 + z = 3z sehingga z = 2 (t idak memenuhi bahwa z ≥ y) Maka t r ipel bil angan asl i (t an A, t an B, t an C) yang memenuhi adal ah (1, 2, 3) dan permut asinya. Akibat nya nil ai (A, B, C) yang memenuhi hanya ada sat u kemungkinan. ∴ Karena hanya ada satu tripel (A, B, C) yang memenuhi maka semua segitiga anggota S sebangun (terbukti) 4. Misal kan kej adian (a) adal ah kej adian dengan posisi sebagai berikut : Pemain yang mel angkah t erl ebih dahul u set el ah kej adian (a) t erj adi akan kal ah sebab pemain pert ama t ersebut hanya bisa mel angkah mundur. Jika pemain pert ama mundur dua l angkah maka pemain kedua akan mel angkah maj u dua l angkah sedangkan j ika pemain pert ama mundur sat u l angkah maka pemain kedua akan mel angkah maj u sat u l angkah sehingga kej adian (a) akan sel al u t erj aga sampai suat u saat pemain pert ama t ersebut t idak dapat mel angkah l agi. Misal kan kej adian (b) adal ah kej adian dengan j arak ant ara dua bidak sama dengan 3 pet ak sebagaimana posisi sebagai berikut : Jika pemain pert ama set el ah posi si (b) t erj adi, mel angkah mundur sat u l angkah maka pemain kedua akan maj u sat u l angkah sedangkan j ika pemain pert ama mundur dua l angkah maka pemain kedua akan maj u dua l angkah sehingga posisi (b) akan t erj aga sampai pemain pert ama SMA Negeri 5 Bengkulu Eddy Hermanto, ST 189 Sol usi Olimpiade Sains Nasional 2006 Bidang : Mat emat ika maj u at au ia t idak dapat l agi mundur sehingga harus maj u. Jika pemain pert ama maj u sat u l angkah maka pemain kedua akan maj u dua l angkah sedangkan j ika pemain pert ama maj u dua l angkah maka pemain kedua akan maj u sat u l angkah sehingga posisi akan menj adi posisi (a) sehingga sesuai dengan penj el asan sebel umnya maka pemain pert ama akan kal ah. Misal kan kej adian (c) adal ah kej adian dengan banyaknya pet ak di ant ara dua bidak sama dengan 3k pet ak dengan k bil angan asl i : Jika pemain pert ama set el ah posisi (c) t erj adi mel angkah mundur sat u l angkah maka pemain kedua akan maj u sat u l angkah sedangkan j ika pemain pert ama mundur dua l angkah maka pemain kedua akan maj u dua l angkah sehingga posisi (c) akan t erj aga sampai pemain pert ama maj u at au ia t idak dapat l agi mundur sehingga harus maj u. Jika pemain pert ama maj u sat u l angkah maka pemain kedua akan maj u dua l angkah sedangkan j ika pemain pert ama maj u dua l angkah maka pemain kedua akan maj u sat u l angkah sehingga banyaknya pet ak diant ara kedua bidak kedua pemain akan menj adi 3(k − 1) pet ak. Demikian set erusnya sehingga nil ai k akan semakin kecil sampai suat u saat nil ai k akan menj adi 1 dan sebagaimana penj el asan pada posisi (b) pemain pert ama akan kal ah. Jika banyaknya pet ak di ant ara kedua bidak t i dak habis dibagi 3 maka pemain pert ama akan memenangkan permainan sebab ia punya kesempat an unt uk membuat banyaknya pet ak di ant ara kedua bidak akan habis dibagi 3. ∴ Maka dapat disimpulkan bahwa : Jika n dibagi 3 bersisa 2 maka pemain kedua (Siti) akan memenangkan permainan sedangkan j ika n dibagi 3 bersisa 0 at au 1 maka pemain pertama (Wiwit) akan memenangkan permainan. 5. Karena G adal ah t it ik berat dan AM adal ah garis berat maka AG : GM = 2 : 1. ∆BGM dan ∆BAG adal ah dua segit iga dengan al as yang sama sehingga perbandingan l uas dapat dinyat akan sebagai perbandingan t inggi. Karena AG : GM = 2 : 1 maka [ BAG] = 2[ BGM] . Dengan cara yang sama maka [ CMG] = 2[ CGA] . [BGM ] + [CMG ] = 1 ⎛⎜ [BAG ] + [CGA ] ⎞⎟ [PAG ] [QGA ] 2 ⎜⎝ [PAG ] [QGA ]⎟⎠ Segit iga BAG dan segit iga PAG adal ah dua segit iga dengan t inggi yang sama maka perbandingan l uas dapat dinyat akan sebagai perbandingan al as. [ BAG] : [ PAG] = AB : AP. Dengan cara yang sama maka [ CGA] : [ QGA] = AC : AQ SMA Negeri 5 Bengkulu Eddy Hermanto, ST 190 Olimpiade Sains Nasional 2006 Sol usi [BGM ] + [CMG ] = 1 ⎛⎜ [BAG ] + [CGA ] ⎞⎟ = 1 ⎛⎜ AB [PAG ] [QGA ] 2 ⎜⎝ [PAG ] [QGA ]⎟⎠ 2 ⎜⎝ AP + AC AQ Bidang : Mat emat ika ⎞ ⎟⎟ ⎠ Al t ernat if 1 : [BGM ] + [CMG ] = 1 ⎛⎜ AP + BP [PAG ] [QGA ] 2 ⎜⎝ AP AQ + CQ AQ + ⎞ 1 ⎛ BP CQ ⎞ ⎟⎟ = 1 + ⎜⎜ ⎟ + 2 ⎝ AP AQ ⎟⎠ ⎠ Buat garis mel al ui B dan C masing-masing sej aj ar AM. Misal kan garis yang mel al ui B memot ong garis di R dan garis yang mel al ui C memot ong garis l di S. (Lihat gambar). BP BR = AP AG CQ CS Karena AG sej aj ar CS maka ∆AGQ sebangun dengan ∆CSQ sehingga = AQ AG Karena AG sej aj ar BR maka ∆AGP sebangun dengan ∆BRP sehingga [BGM ] + [CMG ] = 1 + 1 ⎛⎜ BP [PAG ] [QGA ] 2 ⎜⎝ AP + CQ AQ ⎞ 1 ⎛ BR CS ⎟⎟ = 1 + ⎜⎜ + 2 ⎝ AG AG ⎠ Mengingat bahwa BR + CS = 2 ⋅ GM maka : ⎞ ⎟⎟ ⎠ [BGM ] + [CMG ] = 1 + 1 ⎛⎜ BR + CS ⎞⎟ = 1 + 1 ⎛⎜ 2GM ⎞⎟ ⎟ [PAG ] [QGA ] 2 ⎜⎝ AG 2 ⎜⎝ AG ⎟⎠ ⎠ Mengingat bahwa AG : GM = 2 : 1 maka : ∴ [BGM ] + [CMG ] = 3 [PAG ] [QGA ] 2 (terbukti) Al t ernat if 2 : ⎛a + b c ⎞ , ⎟⎟ . 3⎠ ⎝ 3 Tanpa mengurangi keumuman misal kan koordinat A(0, 0), B(b, 0), C(a, c) sehingga G ⎜ ⎜ Tit ik M adal ah pert engahan BC sehingga M ⎛⎜ a + b , c ⎞⎟ . ⎜ 2 2 ⎟⎠ ⎝ Garis AC mel al ui (0, 0) dan gradien c c sehingga persamaan garis AC, y = x . a a Misal kan persamaan garis l adal ah y = mx + k. Garis l mel al ui t it ik G maka : c − ma − mb c − ma − mb ⎞ . Per samaan gar is l adal ah y = mx + ⎟⎟ + k , maka k = 3 3 ⎠ ⎛ ma − mb − c ⎞ ma − mb − c sehingga P ⎜⎜ Garis l memot ong AB di t it ik P maka x p = ,0 ⎟⎟ . 3m 3 m ⎝ ⎠ ⎛a + b c = m ⎜⎜ 3 ⎝ 3 c c − ma − mb x Q = mx Q + a 3 a (c − ma − mb ) c (c − ma − mb ) . maka y Q = xQ = 3(c − ma ) 3(c − ma ) ⎛ a (c − ma − mb ) c (c − ma − mb ) ⎞ Sehingga Q ⎜ ⎟. ⎜ 3(c − ma ) , 3(c − ma ) ⎟⎠ ⎝ Garis l memot ong AC di Q, maka ma + mb − c (c − ma − mb ) 2 2 ; AC = a + c ; AQ = a2 +c 2 3m 3(c − ma ) [BGM ] + [CMG ] = 1 ⎛⎜ AB + AC ⎞⎟ = 1 ⎛ 3mb + 3(c − ma) ⎞ ⎜ ⎟ [PAG ] [QGA] 2 ⎜⎝ AP AQ ⎟⎠ 2 ⎝ ma + mb − c c − ma − mb ⎠ AB = b ; AP = SMA Negeri 5 Bengkulu Eddy Hermanto, ST 191 Olimpiade Sains Nasional 2006 Sol usi Bidang : Mat emat ika [BGM ] + [CMG ] = 1 ⎛⎜ 3mb − 3(c − ma ) ⎞⎟ = 1 ⎛⎜ 3(ma + mb − c ) ⎞⎟ [PAG ] [QGA ] 2 ⎜⎝ ma + mb − c ⎟⎠ 2 ⎜⎝ ma + mb − c ⎟⎠ [BGM ] + [CMG ] = 3 (terbukti) ∴ [PAG ] [QGA ] 2 6. Akan dicari nomor t el epon t ersebut t erdiri dari sedikit nya 6 angka berbeda. ¾ Jika nomor t el epon t ersebut t erdiri dari 6 angka berbeda Banyaknya cara memil ih 5 angka t er sisa dari 9 angka t ersisa adal ah 9C5. * Jika nomor t el epon t ersebut t erdiri dari 1 angka muncul 3 kal i dan 5 angka l ainnya muncul 1 kal i • Jika angka yang muncul t iga kal i adal ah angka 8 Banyaknya nomor t el epon = • C5 ⋅ 7! = 317. 520 2! Jika angka yang muncul t iga kal i adal ah bukan angka 8 Banyaknya nomor t el epon = * 9 9 C 5 ⋅5 C1 ⋅ 7! = 529. 200 3! Jika nomor t el epon t ersebut t erdiri dari 2 angka masing-masing muncul 2 kal i dan 4 angka l ainnya muncul 1 kal i • Jika sal ah sat u angka yang muncul dua kal i t ersebut adal ah angka 8 Banyaknya nomor t el epon = • 9 C 5 ⋅5 C1 ⋅ 7! = 1. 587. 600 2! Jika kedua angka yang muncul dua kal i t ersebut keduanya bukan angka 8 Banyaknya nomor t el epon = 9 C 5 ⋅5 C 2 ⋅ 7! = 1. 587. 600 2!2! ¾ Jika nomor t el epon t ersebut t erdiri dari 7 angka berbeda Banyaknya cara memil ih 6 angka t er sisa dari 9 angka t ersisa adal ah 9C6. Sat u angka akan muncul 2 kal i sedangkan 6 angka l ain akan muncul sat u kal i. * Jika angka yang muncul dua kal i t ersebut adal ah angka 8 Banyaknya nomor t el epon = 9C6 ⋅ 7! = 423. 360 * Jika angka yang muncul dua kal i t ersebut adal ah bukan angka 8 Banyaknya nomor t el epon = 9 C 6 ⋅ 6 C1 ⋅ 7! = 1. 270. 080 2! ¾ Jika nomor t el epon t ersebut t erdiri dari 8 angka berbeda Banyaknya cara memil ih 7 angka t er sisa dari 9 angka t ersisa adal ah 9C7. Banyaknya nomor t el epon = 9C7 ⋅ 7! = 181. 440 Misal kan A adal ah kej adian banyaknya nomor t el epon dengan sedikit nya 6 angka ber beda. A = 317. 520 + 529. 200 + 1. 587. 600 + 1. 587. 600 + 423. 360 + 1. 270. 080 + 181. 440 = 5. 896. 800 p(A) = A = 0, 58968 10 7 ∴ Peluang bahwa pak Edy mendapatkan nomor telepon yang memuat tidak lebih dari 5 angka berbeda = 1 − p(A) = 0, 41032 SMA Negeri 5 Bengkulu Eddy Hermanto, ST 192 Olimpiade Sains Nasional 2006 Sol usi 7. Karena ab, ac dan bc adal ah bil angan r asional maka Bidang : Mat emat ika ab b ab a = , = adal ah j uga bil angan ac c bc c rasional . Misal kan • • • • ab b m ab a p = = , unt uk m, n, p, q ∈ Bil . Bul at dan n, q ≠ 0 = = c q ac c n bc Jika a, b, c ket iganya sama dengan 0 Jel as bahwa berapa pun nil ai (x, y, z) bul at akan memenuhi ax + by + cz = 0 Jika dua di ant ara a, b, c sama dengan 0 Tanpa mengurangi keumuman misal kan a = b = 0 dan c ≠ 0 Nil ai z = 0 dan x, y ≠ 0 akan memenuhi ax + by + cz = 0 Jika sal ah sat u a, b at au c sama dengan 0 Tanpa mengurangi keumuman misal kan a = 0 dan b, c ≠ 0 Maka unt uk x ≠ 0, y = z = 0 akan memenuhi ax + by + cz = 0. Jika t idak ada a, b dan c bernil ai 0 ⎛a ⎞ ⎛p ⎞ m b ax + by + cz = c ⎜⎜ x + y + z ⎟⎟ = c ⎜⎜ x + y + z ⎟⎟ n c ⎝c ⎠ ⎝q ⎠ Maka unt uk x = kq, y = wn dan z = −(kp + wm) dengan k, w ∈ Bil angan Bul at akan memenuhi ax + by + cz = 0 ∴ Terbukti bahwa ada bilangan-bilangan bulat x, y, z yang tidak semuanya nol, sehingga ax + by + cz = 0 8. Misal kan bil angan 85-angka t ersebut adal ah n. Misal kan j uga S(n) adal ah j uml ah semua angkaangka n dan P(n) adal ah hasil kal i semua angka-angka n. Jika 0 adal ah sal ah sat u digit dari n maka P(n) = 0 sehingga S(n) = 0. Akibat nya semua digit dari n harus 0 yang t idak memenuhi n adal ah bil angan bul at 85-angka. Maka 0 bukanl ah sal ah sat u angka n. S(n) maksimal = 9 ⋅ 85 = 765 j ika semua angka n adal ah 9. Karena 29 = 512 < 765 dan 210 = 1024 > 765 maka pal ing banyak 9 angka n bukan 1 dan pal ing sedikit 76 angka t erakhir n harus sama dengan 1. Maka S(n) maksimal = 9 ⋅ 9 + 1 ⋅ 76 = 157. Karena 27 = 128 < 157 dan 28 = 256 > 157 maka pal ing banyak 7 angka n bukan 1 dan pal ing sedikit 78 angka t erakhir n harus sama dengan 1. P(n) t erkecil saat n = 2222222111 ⋅⋅⋅⋅⋅⋅⋅⋅111, maka P(n) = 128 dan S(n) = 92 sehingga P(n) ≠ S(n) Jika 7 angka pert ama n t idak semuanya 2 maka P(n) minimal saat n = 3222222111 ⋅⋅⋅⋅111 yait u 192. Karena S(n) maksimal = 9 ⋅ 7 + 1 ⋅ 78 = 141 < P(n) minimal . Maka t idak ada nil ai n yang memenuhi. Akibat nya pal ing banyak 6 angka n bukan 1. Jika pal ing banyak dua angka dari n bukan 1 maka P(n) maksimal = 9 ⋅ 9 ⋅ 1 ⋅ 1 ⋅ ⋅⋅⋅ ⋅ 1 = 81 sedangkan S(n) minimal = 1 ⋅ 85 = 85. Maka sedikit nya t iga angka n bukan 1. Maka banyaknya angka n yang bukan 1 pal ing sedikit 3 dan pal ing banyak 6. • Jika angka pert ama n adal ah 9 S(n) minimal = 9 ⋅ 1 + 2 ⋅ 5 + 1 ⋅ 79 = 98 dan S(n) maksimal = 9 ⋅ 6 + 1 ⋅ 79 = 133 Karena P(n) habis dibagi 9 maka S(n) j uga harus habis dibagi 9. Maka kemungkinan nil ai P(n) adal ah 99, 108, 117, 126. 99 = 9 ⋅ 11 dan 117 = 9 ⋅ 13. Karena 11 dan 13 adal ah bil angan prima dua angka maka t idak mungkin P(n) = 99 at au 117. Jika P(n) = 126 = 9 ⋅ 7 ⋅ 2 maka ke-85 angka n adal ah 9, 7, 2 dan 1 sebanyak 82 kal i. SMA Negeri 5 Bengkulu Eddy Hermanto, ST 193 Sol usi Olimpiade Sains Nasional 2006 Bidang : Mat emat ika S(n) = 9 + 7 + 2 + 82 = 100 (t idak memenuhi P(n) = S(n)). Jika P(n) = 108 = 9 ⋅ 22 ⋅ 3 maka kemungkinan angka-angka n adal ah : * Ke-85 angka n adal ah 9, 4, 3 dan 1 sebanyak 82 kal i. S(n) = 9 + 4 + 3 + 82 = 98 (t idak memenuhi P(n) = S(n)) * Ke-85 angka n adal ah 9, 6, 2 dan 1 sebanyak 82 kal i. S(n) = 9 + 6 + 2 + 82 = 99 (t idak memenuhi P(n) = S(n)) * Ke-85 angka n adal ah 9, 3, 2, 2 dan 1 sebanyak 81 kal i. S(n) = 9 + 3 + 2 + 2 + 81 = 97 (t idak memenuhi P(n) = S(n)) Maka t idak ada nil ai n dengan angka pert ama adal ah 9 yang memenuhi. • Jika angka pert ama n adal ah 8 S(n) minimal = 8 + 1 ⋅ 84 = 92 dan S(n) maksimal = 8 ⋅ 6 + 1 ⋅ 79 = 127 Karena P(n) habis dibagi 9 maka S(n) j uga harus habis dibagi 8. Maka kemungkinan nil ai P(n) adal ah 96, 104, 112, 120. Jika P(n) = 120 = 8 ⋅ 5 ⋅ 3 maka maka ke-85 angka n adal ah 8, 5, 3 dan 1 sebanyak 82 kal i. S(n) = 8 + 5 + 3 + 82 = 98 (t idak memenuhi P(n) = S(n)). Jika P(n) = 112 = 8 ⋅ 7 ⋅ 2 maka maka ke-85 angka n adal ah 8, 7, 2 dan 1 sebanyak 82 kal i. S(n) = 8 + 7 + 2 + 82 = 99 (t idak memenuhi P(n) = S(n)). Jika P(n) = 104 = 8 ⋅ 13. Karena 13 adal ah bil angan pr ima dua angka maka t idak ada n yang memenuhi. Jika P(n) = 96 = 8 ⋅ 22 ⋅ 3 maka kemungkinan angka-angka n adal ah : * Ke-85 angka n adal ah 8, 4, 3 dan 1 sebanyak 82 kal i. S(n) = 8 + 4 + 3 + 82 = 97 (t idak memenuhi P(n) = S(n)) * Ke-85 angka n adal ah 8, 6, 2 dan 1 sebanyak 82 kal i. S(n) = 8 + 6 + 2 + 82 = 98 (t idak memenuhi P(n) = S(n)) * Ke-85 angka n adal ah 8, 3, 2, 2 dan 1 sebanyak 81 kal i. S(n) = 8 + 3 + 2 + 2 + 81 = 96 (memenuhi) Karena angka-angka n adal ah 8, 3, 2, 2 dan 1 sebanyak 81 kal i maka n t erbesar yang memenuhi adal ah 83221111⋅⋅⋅⋅111. ∴ Dapat disimpul kan bahwa nil ai n t erbesar yang memenuhi adal ah 8322111 ⋅⋅⋅⋅⋅⋅111 . SMA Negeri 5 Bengkulu Eddy Hermanto, ST 194 SELEKSI OLIMPIADE TINGKAT KABUPATEN/ KOTA TAHUN 2007 TIM OLIMPIADE MATEMATIKA INDONESIA TAHUN 2008 Bidang Mat emat ika Waktu : 3, 5 Jam DEPARTEMEN PENDIDIKAN NASIONAL DIREKTORAT JENDERAL MANAJEMEN PENDIDIKAN DASAR DAN MENENGAH DIREKTORAT PEMBINAAN SEKOLAH MENENGAH ATAS TAHUN 2007 195 OLIMPIADE MATEMATIKA NASIONAL SELEKSI TINGKAT KOTA/ KABUPATEN TAHUN 2007 Bagian Pertama Pilih sat u j awaban yang benar. Dalam hal t erdapat lebih dari sat u j awaban yang benar, pilih j awaban yang paling baik. 1. Jika ⎣x ⎦ menyat akan bilangan bulat t erbesar yang lebih kecil dari at au sama dengan bilangan ⎣ 3 − 5⎦ 2 real x, maka A. −1 = B. 0 5+2− 2. Bilangan A. bulat negat if D. irrasional posit if 3 3 C. 1 D. 9 E. 81 5 − 2 merupakan bilangan B. bulat posit if E. irrasional negat if C. pecahan 3. Banyaknya soal yang dikerj akan Amin hari ini bert ambah t epat 40% dibandingkan dengan yang dikerj akannya kemarin. Banyaknya soal yang dikerj akan Amin hari ini paling sedikit ada A. 5 B. 6 C. 7 D. 8 E. t idak bisa dit ent ukan 4. Misalkan H adalah himpunan semua f akt or posit if dari 2007. Banyaknya himpunan bagian dari H yang t idak kosong adalah A. 6 B. 31 C. 32 D. 63 E. 64 5. Misalkan N sebuah bilangan asli dua-angka dan M adalah bilangan asli yang diperoleh dengan mempert ukarkan kedua angka N. Bilangan prima yang selalu habis membagi N − M adalah A. 2 B. 3 C. 7 D. 9 E. 11 6. Sebuah sampel diperoleh dari lima pengamat an. Jika rat aan hit ung(mean) sampel sama dengan 10 dan median sampel sama dengan 12, maka nilai t erkecil j angkauan sampel sama dengan A. 2 B. 3 C. 5 D. 7 E. 10 7. Peluang menemukan di ant ara t iga orang ada paling sedikit dua orang yang lahir dalam bulan yang sama adalah A. 17 72 B. 33 72 C. 39 72 D. 48 72 E. 55 72 8. Keliling sebuah segit iga adalah 8. Jika panj ang sisi-sisinya adalah bilangan bulat , maka luas segit iga t ersebut sama dengan A. 2 2 B. 16 3 9 C. 2 3 196 D. 4 E. 4 2 9. Sepot ong kawat dipot ong menj adi 2 bagian, dengan perbandingan panj ang 3: 2. Masing-masing bagian kemudian dibent uk menj adi sebuah persegi. Perbandingan luas kedua persegi adalah A. 4 : 3 B. 3 : 2 C. 5 : 3 D. 9 : 4 E. 5 : 2 tan 2 x + cos 2 x 10. Unt uk set iap bilangan real x berlaku = sin x + sec x B. sec x − sin x E. cos x + sin x A. sec x + sin x D. cos x − csc x C. cos x + csc x Bagian Kedua Isikan hanya j awaban saj a pada t empat yang disediakan 11. Misalkan f (x) = 2x - 1, dan g(x) = x . Jika f (g(x)) = 3, maka x = ⋅⋅⋅⋅⋅ 12. Pengepakan buah “ Drosophila” akan mengemas 44 apel ke dalam beberapa kot ak. Ada dua j enis kot ak yang t ersedia, yait u kot ak unt uk 10 apel dan kot ak unt uk 6 apel. Banyak kot ak yang diperlukan adalah ⋅⋅⋅⋅⋅⋅ 13. Semua pasangan bilangan bulat (x, y) yang memenuhi x + y = xy − 1 dan x ≤ y, adalah ⋅⋅⋅⋅ 14. Jika n adalah bilangan asli sehingga 3n adalah f akt or dari 33! , maka nilai n t erbesar yang mungkin adalah ⋅⋅⋅⋅ ⎛ 15. Sebuah ruas garis mulai dari t it ik ⎜⎜ 3,2 ⎝ ⎛ 1⎞ 3⎞ ⎟⎟ dan berakhir di ⎜⎜ 99,68 ⎟⎟ . Banyaknya t it ik dengan 5⎠ 5⎠ ⎝ koordinat bilangan bulat yang dilalui garis t ersebut adalah ⋅⋅⋅⋅⋅⋅ 16. Pada segit iga PQR samasisi diberikan t it ik-t it ik S dan T yang t erl et ak bert urut -t urut pada sisi QR dan PR demikian rupa, sehingga ∠SPR = 40o dan ∠TQR = 35o. Jika t it ik X adalah perpot ongan garis-garis PS dan QT, maka ∠SXT = ⋅⋅⋅⋅⋅⋅⋅⋅⋅ 17. Pada segit iga ABC yang siku-siku di C, AE dan BF adalah garis-garis berat (median). AE 2 + BF AB 2 Maka 2 = ⋅⋅⋅⋅⋅⋅ 18. Diket ahui empat t it ik pada bidang dengan koordinat A(1, 0), B(2008, 2007), C(2007, 2007), D(0, 0). Luas j aj aran genj ang ABCD sama dengan ⋅⋅⋅⋅⋅⋅ 19. Sebuah lingkaran berj ari-j ari 1. Luas maksimal segit iga samasisi yang dapat dimuat di dalam lingkaran adalah ⋅⋅⋅⋅⋅⋅ 20. Sebuah daerah persegi dibagi menj adi 2007 daerah kecil dengan menarik garis-garis lurus yang menghubungkan dua sisi berbeda pada persegi. Banyak garis lurus yang harus dit arik paling sedikit ada ⋅⋅⋅⋅⋅ 197 SELEKSI OLIMPIADE TINGKAT KABUPATEN/ KOTA 2007 TIM OLIMPIADE MATEMATIKA INDONESIA 2008 Prestasi itu diraih bukan didapat !!! SOLUSI SOAL Bidang Mat emat ika Disusun oleh : Eddy Hermant o, ST 198 Olimpiade Matematika Tk Kabupaten/ Kota 2007 BAGIAN PERTAMA 1. (Jawaban : C) 3 ≈ 1,7 5 ≈ 2,2 ; 3 − 5 ≈ −0,5 sehingga ⎣ 3 − 5⎦ ⎣ 3 − 5 ⎦ = −1 . Maka ⎣ 3 − 5 ⎦ 2 =1 2 ∴ =1 2. (Jawaban : B) Misalkan X 3 X 3 5+2 −3 3 5−2 =X ( 5 + 2) − ( 5 − 2) − 3⎛⎜⎝ = 4 − 3( 5 − 4 )(X ) = 3 5 + 2 ⎞⎟⎛⎜ 3 ⎠⎝ 5 − 2 ⎞⎟⎛⎜ 3 ⎠⎝ 5+2+3 5 − 2 ⎞⎟ ⎠ 3 X3 + 3X − 4 = 0 (X − 1)(X2 + X + 4) = 0 Akar-akar persamaan X2 + X + 4 = 0 t idak real. Maka X = 1 3 5+2 −3 ∴ 3 5 −2 =1 5+2 −3 5 − 2 merupakan bilangan bulat positif . 3. (Jawaban : C) Misalkan banyaknya soal yang dikerj akan Amin kemarin y maka banyaknya soal yang dikerj akan hari ini adalah n = 75 y dengan y dan n keduanya asli. 7 n = 5 y Maka y = 5k dan n = 7k unt uk suat u bilangan asli k. Nilai n t erkecil adalah saat k = 1 sehingga n = 7 ∴ Banyaknya soal yang dikerj akan Amin hari ini paling sedikit ada 7. 4. (Jawaban : D) 2007 = 32 ⋅ 2231 Banyaknya f akt or posit if dari 2007 adalah (2 + 1)(1 + 1) = 6 Banyaknya himpunan bagian t ak kosong dari H adalah 26 − 1 = 63. ∴ Banyaknya himpunan bagian t ak kosong dari H adalah 26 − 1 = 63. 5. (Jawaban : B) Misalkan N = 10a + b maka M = 10b + a N − M = 9(a − b) sehingga 9⏐(N − M) ∴ Maka bilangan prima yang selalu membagi N − M adalah 3. SMA Negeri 5 Bengkulu Eddy Hermanto, ST 199 Olimpiade Matematika Tk Kabupaten/ Kota 2007 6. (Jawaban : C) Misalkan bilangan t ersebut adalah x 1 ≤ x 2 ≤ x 3 ≤ x 4 ≤ x 5. Maka x 1 + x 2 + x 3 + x 4 + x 5 = 50 dan x 3 = 12. Agar j angkauan minimal maka x 5 harus sekecil mungkin dan x 1 harus sebesar mungkin. Jelas bahwa x 1 < 10 dan x 5 ≥ 12. Jika x 1 = x 2 = 9 maka x 4 + x 5 = 20. Tidak mungkin x 5 ≥ x 4 ≥ 12. Jika x 5 = x 4 = 12 maka x 1 + x 2 = 36. Nilai t erbesar x 1 adalah saat x 1 = x 2 = 7. Kelima bilangan t ersebut adalah 7, 7, 12, 12, 12. ∴ Jangkauan = 12 − 7 = 5. 7. (Jawaban : A) Alt ernat if 1 : Misalkan A adalah kej adian sedikit nya 2 dari 3 orang lahir pada bulan yang sama. Maka A’ adalah kej adian 3 orang lahir pada bulan yang berbeda. Banyaknya kemungkinan t ripel 3 orang lahir adalah 12x12x12 kemungkinan. Banyaknya 3 orang lahir pada bulan yang berbeda adalah 12x11x10 12 ⋅ 11 ⋅ 10 55 = 12 ⋅ 12 ⋅ 12 72 55 17 Maka p (A ) = 1 − = 72 72 p (A ' ) = Alt ernat if 2 : Banyaknya kemungkinan t ripel 3 orang lahir adalah 12x12x12 kemungkinan. Banyaknya kemungkinan t epat 2 dari 3 orang lahir pada bulan yang sama = 3C2 x 12x1x11 = 3x12x1x11 Banyaknya kemungkinan t epat 3 dari 3 orang lahir pada bulan yang sama = 3C3 x 12x1x1 = 1x12x1x1 3x 12x 1x 11 + 1x 12x 1x 1 17 = 12x 12x 12 72 17 ∴ Peluang di ant ara t iga orang ada paling sedikit dua orang lahir dalam bulan yang sama = 72 Peluang paling sedikit dua orang lahir dalam bulan yang sama = 8. (Jawaban : A) a + b + c = 8 dengan a, b, dan c semuanya bilangan asli. Syarat : panj ang salah sat u sisi selalu kurang dari j umlah kedua sisi yang lain, Dengan memperhat ikan syarat t ersebut maka panj ang sisi-sisi segit iga yang memenuhi adalah 2, 3, 3. s = ½ (a + b + c) = 4 Dengan rumus Heron, Luas ∆ = ∴ Luas ∆ = 2 √2 s (s − a )(s − b )(s − c ) = 2√2 9. (Jawaban : D) Misalkan panj ang kawat semula 20a maka kawat akan t erbagi dua dengan panj ang 12a dan 8a. Panj ang sisi persegi pert ama = 3a dan panj ang sisi persegi kedua = 2a. Perbandingan luas = 32 : 22 = 9 : 4. ∴ Perbandingan luas kedua persegi adalah 9 : 4 . SMA Negeri 5 Bengkulu Eddy Hermanto, ST 200 Olimpiade Matematika Tk Kabupaten/ Kota 2007 10. (Jawaban : B) tan 2 x + cos 2 x sec 2 x − 1 + 1 − sin 2 x sec 2 x − sin 2 x = sec x − sin x = = sin x + sec x sin x + sec x sin x + sec x tan 2 x + cos 2 x ∴ = sec x − sin x sin x + sec x BAGIAN KEDUA 11. f ( x )= 3 2 x −1 = 3 ∴ x=4 12. Misalkan banyaknya keranj ang berisi 10 apel = x dan keranj ang berisi 6 apel = y dengan x dan y keduanya bulat t ak negat if . Maka 10x + 6y = 44 sehingga 5x + 3y = 22 5x ≤ 22 Nilai x yang mungkin adalah 0, 1, 2, 3 at au 4. Set elah dicek sat u-sat u, nilai x yang memenuhi hanya x = 2 yang membuat y = 4 Maka x + y = 6 ∴ Banyak kot ak yang diperlukan adalah 6 13. xy − x − y − 1 = 0 sehingga (x − 1)(y − 1) = 2 Maka (x − 1) ⏐ 2. Nilai x − 1 yang mungkin adalah −1, 1, −2, 2 Unt uk x − 1 = −1 maka x = 0 dan y = −1 Unt uk x − 1 = 1 maka x = 2 dan y = 3 Unt uk x − 1 = −2 maka x = −1 dan y = 0 Unt uk x − 1 = 2 maka x = 3 dan y = 2 Set elah dicek sat u-sat u pasangan (x, y) yang memenuhi persamaan dan berlaku x ≤ y adalah ( −1, 0) dan (2, 3) ∴ Semua pasangan bilangan bulat (x, y) yang memenuhi adalah ( −1, 0), (2, 3) 14. Alt ernat if 1 : ⎢ 33 ⎥ ⎢ 33 ⎥ ⎢ 33 ⎥ ⎢ 33 ⎥ ⎥ + ⎢ 2 ⎥ + ⎢ 3⎥ + ⎢ 4⎥ +L ⎣ 3 ⎦ ⎣3 ⎦ ⎣3 ⎦ ⎣3 ⎦ n t erbesar = ⎢ n t erbesar = 11 + 3 + 1 + 0 + 0 + ⋅⋅⋅ n t erbesar = 15 Alt ernat if 2 : Bilangan dari 1 sampai dengan 33 yang memiliki f akt or 3 ada 11 yait u 3, 6, 9, 12, 15, 18, ⋅⋅⋅, 33 Di ant ara 11 bilangan t ersebut yang habis dibagi 33 = 27 ada 1 yait u 27. SMA Negeri 5 Bengkulu Eddy Hermanto, ST 201 Olimpiade Matematika Tk Kabupaten/ Kota 2007 Di ant ara 11 bilangan t ersebut yang habis dibagi 32 = 9 t et api t idak habis dibagi 33 = 27 ada 2 yait u 9, 18. Sisanya adalah 8 bilangan yang habis dibagi 3 t et api t idak habis dibagi 9 maupun 27. Maka nilai n t erbesar yang membagi 33! = 3x1 + 2x2 + 1x8 = 15. ∴ Nilai n t erbesar yang mungkin adalah 15 . ⎛ 1⎞ ⎛ 3⎞ ⎝ ⎠ ⎝ ⎠ 15. Persamaan garis yang melalui ⎜⎜ 3,2 ⎟⎟ dan ⎜⎜ 99,68 ⎟⎟ adalah 120y = 83x + 15. 5 5 15(8y − 1) = 83x Karena 15 t idak membagi 83 maka 15 membagi x. Nilai x yang mungkin adalah 15, 30, 45, 60, 75 at au 90. Set elah dicek sat u-sat u maka nilai x bulat yang memenuhi y j uga bulat hanyalah x = 75 yang membuat y = 52 ∴ Banyaknya t it ik dengan koordinat bilangan bulat yang dilalui garis t ersebut adalah 1. 16. Pada ∆QRT berlaku ∠RTQ = 180o − 60o − 35o = 85o Pada ∆PRS berlaku ∠PSR = 180o − 60o − 40o = 80o Pada segiempat RSXT berlaku 360o = 60o + ∠RTQ + ∠PSR + ∠SXT ∠SXT = 135o. ∴ ∠SXT = 135 o. 17. Misalkan AC = b dan BC = a maka AB2 = a2 + b 2 AE2 = (0, 5a) 2 + b 2 dan BF2 = a2 + (0, 5b) 2 AE2 + BF2 = 1, 25(a2 + b 2) ∴ AE + BF AB 2 2 2 = 5 4 18. Diket ahui A = (1, 0), B(2008, 2007), C(2007, 2007) dan D(0, 0) Alt ernat if 1 : Misalkan E(0, 2007) dan F(2008, 0) Luas j aj aran genj ang = Luas persegi panj ang DFBE − Luas ∆DCE − Luas ∆AFB. Luas j aj aran genj ang = 2008 ⋅ 2007 − ½ ⋅ 2007 ⋅ 2007 − ½ ⋅ 2007 ⋅ 2007 = 2007 Alt ernat if 2 : Panj ang alas = ⏐DA⏐ = 1 Tinggi = 2007 − 0 = 2007 Luas j aj aran ganj ang = alas x t inggi Luas j aj aran genj ang = 2007 ∴ Luas j aj aran genj ang = 2007 19. Misalkan segit iga t ersebut adalah ∆ABC. Agar luas segit iga maksimum maka ket iga t it ik sudut segit iga sama sisi t ersebut harus t erlet ak pada lingkaran. SMA Negeri 5 Bengkulu Eddy Hermanto, ST 202 Olimpiade Matematika Tk Kabupaten/ Kota 2007 R = abc 4[ABC ] dengan [ ABC] menyat akan luas segit iga ABC. Karena ∆ABC sama sisi maka abc = a3 a3 2a 2 sin 60° a = 3 1= Luas ∆ABC = ½ a2 sin 60o ∴ Luas ∆ABC = 3 3 4 20. Misalkan r n adalah banyaknya region maksimal yang t erj adi akibat t erdapat n buah garis lurus. Banyaknya region akan maksimal apabila t idak ada sedikit nya dua garis sej aj ar dan t idak ada sedikit nya t iga garis yang bert emu di sat u t it ik. Jelas bahwa r 0 = 1, r 1 = 2, r 2 = 4, r 3 = 7, r 4 = 11 dan set erusnya. Ini dirumuskan dengan r n = r n-1 + n r2 − r1 = 2 r3 − r2 = 3 r4 − r3 = 4 M rn − r4 = n Jumlahkan semua persamaan didapat : rn − r1 = n −1 (2 + n ) 2 n2 +n + 2 Karena r 1 = 2 maka r n = 2 Jika n = 62 maka r n = 1954 < 2007 Jika n = 63 maka r n = 2017 > 2007 Maka banyaknya garis minimal adalah 63. ∴ Banyaknya garis lurus yang harus dit arik paling sedikit ada 63. SMA Negeri 5 Bengkulu Eddy Hermanto, ST 203 SELEKSI OLIMPIADE TINGKAT PROVINSI 2007 TIM OLIMPIADE MATEMATIKA INDONESIA 2008 Bidang Mat emat ika Bagian Per t ama Waktu : 90 Menit DEPARTEMEN PENDIDIKAN NASIONAL DIREKTORAT JENDERAL MANAJEMEN PENDIDIKAN DASAR DAN MENENGAH DIREKTORAT PEMBINAAN SEKOLAH MENENGAH ATAS TAHUN 2007 204 OLIMPIADE MATEMATIKA TINGKAT PROVINSI TAHUN 2007 BAGIAN PERTAMA 1. Bilangan ganj il 4-angka t erbesar yang hasil penj umlahan semua angkanya bilangan prima adalah 2. Sej umlah uang t erdiri dari koin pecahan Rp. 500, Rp. 200, dan Rp. 100 dengan nilai t ot al Rp. 100. 000. Jika nilai uang pecahan 500-an set engah dari nilai uang pecahan 200-an, t et api t iga kali nilai uang pecahan 100-an, maka banyaknya koin adalah ⋅⋅⋅⋅ 3. Panj ang sisi miring sebuah segit iga siku-siku sama dengan dua kali panj ang sisi t erpendeknya, sedangkan panj ang sisi ket iga 1 sat uan panj ang lebih panj ang dari panj ang sisi t erpendeknya. Luas segit iga it u adalah ⋅⋅⋅⋅⋅⋅⋅⋅⋅⋅ sat uan luas. 4. Di ant ara bilangan-bilangan 2006, 2007 dan 2008, bilangan yang memiliki f akt or prima berbeda t erbanyak adalah ⋅⋅⋅⋅⋅⋅⋅ 5. Seorang pedagang mobil bekas menj ual dua buah mobil dengan harga sama. Ia merugi 10% unt uk mobil pert ama, t et api impas (kembali modal) unt uk kedua mobil. Persent ase keunt ungan pedagang it u unt uk mobil kedua adalah ⋅⋅⋅⋅⋅ 6. Dona menyusun lima buah persegi yang kongruen menj adi sebuah bangun dat ar. Tidak ada persegi yang menindih persegi lainnya. Jika luas bangun yang diperoleh Dona adalah 245 cm 2, keliling bangun t ersebut paling sedikit adalah ⋅⋅⋅⋅ cm. 7. Empat t im sepakbola mengikut i sebuah t urnamen. Set iap t im bert anding melawan masingmasing t im lainnya sekali. Set iap kali bert anding, sebuah t im memperoleh nilai 3 j ika menang, 0 j ika kalah dan 1 j ika pert andingan berakhir seri. Di akhir t urnamen salah sat u t im memperoleh nilai t ot al 4. Jumlah nilai t ot al ket iga t im lainnya paling sedikit adalah ⋅⋅⋅⋅ 8. Unt uk bilangan asli n, didef inisikan n! = 1 x 2 x 3 x ⋅⋅⋅ x n. Dalam bent uk sederhana, 1! 1 + 2! 2 + 3! 3 + ⋅⋅⋅ + n! n = ⋅⋅⋅⋅⋅⋅⋅ 9. Tit ik P t erlet ak di kuadran I pada garis y = x. Tit ik Q t erlet ak pada garis y = 2x demikian sehingga PQ t egak lurus t erhadap garis y = x dan PQ = 2. Maka koordinat Q adalah ⋅⋅⋅⋅⋅ 10. Himpunan semua bilangan asli n sehingga 6n + 30 adalah kelipat an 2n + 1 adalah ⋅⋅⋅⋅⋅ ⎛ 11. Suku konst ant a pada ekspansi ⎜⎜ 2x ⎝ 9 2 1⎞ − ⎟⎟ adalah ⋅⋅⋅⋅⋅⋅⋅⋅ x ⎠ 12. Absis t it ik pot ong garis l dengan sumbu-x dan ordinat t it ik pot ong l dengan sumbu-y adalah bilangan-bilangan prima. Jika l j uga melalui t it ik (3, 4), persamaan l adalah ⋅⋅⋅⋅ 205 13. Tuj uh belas permen dikemas ke dalam kant ong-kant ong sehingga banyak permen dalam set iap dua kant ong berselisih paling banyak 1. Banyaknya cara mengemas permen t ersebut ke dalam paling sedikit dua kant ong adalah ⋅⋅⋅⋅ 14. Jika nilai maksimum x + y pada himpunan {(x, y) ⏐x ≥ 0, y ≥ 0, x + 3y ≤ 6, 3x + y ≤ a} adalah 4, haruslah a = ⋅⋅⋅⋅⋅ 15. Sebuah kubus berukuran 5 x 5 x 5 disusun dari 125 kubus sat uan. Permukaan kubus besar lalu dicat . Rasio sisi (permukaan) ke-125 kubus sat uan yang dicat t erhadap yang t idak dicat adalah ⋅⋅⋅ 16. Sebuah papan persegi dibagi ke dalam 4 x 4 pet ak dan diwarnai sepert i papan cat ur. Set iap pet ak diberi nomor dari 1 hingga 16. Andi ingin menut up pet ak-pet ak pada papan dengan 7 kart u seukuran 2 x 1 pet ak. Agar ke-7 kart unya dapat menut upi papan, ia harus membuang dua pet ak. Banyak cara ia membuang dua pet ak adalah ⋅⋅⋅⋅ 17. Bilangan-bilangan asli 1, 2, ⋅⋅⋅, n dit uliskan di papan t ulis, kemudian salah sat u bilangan dihapus. Rat a-rat a arit mat ika bilangan yang t ert inggal adalah 35 t erj adi adalah ⋅⋅⋅⋅⋅ 7 . Bilangan n yang memungkinkan ini 17 18. Diberikan segit iga ABC siku-siku di A, t it ik D pada AC dan t it ik F pada BC. Jika AF ⊥ BC dan BD = DC = FC = 1, maka AC = ⋅⋅⋅⋅ 19. Di ant ara semua solusi bilangan asli (x, y) persamaan t erbesar adalah (x, y) = ⋅⋅⋅⋅⋅⋅ x +y + xy = 54 , solusi dengan x 2 20. Misalkan V adalah himpunan t it ik-t it ik pada bidang dengan koordinat bilangan bulat dan X adalah himpunan t it ik t engah dari semua pasangan t it ik pada himpunan V. Unt uk memast ikan bahwa ada angot a X yang j uga memiliki koordinat bilangan bulat , banyak anggot a V paling sedikit harus ⋅⋅⋅⋅⋅ 206 SELEKSI OLIMPIADE TINGKAT PROVINSI 2007 TIM OLIMPIADE MATEMATIKA INDONESIA 2008 Bidang Mat emat ika Bagian Kedua Waktu : 120 Menit DEPARTEMEN PENDIDIKAN NASIONAL DIREKTORAT JENDERAL MANAJEMEN PENDIDIKAN DASAR DAN MENENGAH DIREKTORAT PEMBINAAN SEKOLAH MENENGAH ATAS TAHUN 2007 207 OLIMPIADE MATEMATIKA TINGKAT PROVINSI TAHUN 2007 BAGIAN KEDUA 1. Misalkan ABCD sebuah segiempat dengan AB = BC = CD = DA. (a) Bukt ikan bahwa t it ik A harus berada di luar segit iga BCD. (b) Bukt ikan bahwa set iap pasangan sisi berhadapan pada ABCD selalu sej aj ar. 2. Misalkan a dan b dua bilangan asli, yang sat u bukan kelipat an yang lainnya. Misalkan pula KPK(a, b) adalah bilangan 2-angka, sedangkan FPB(a, b) dapat diperoleh dengan membalik urut an angka pada KPK(a, b). Tent ukan b t erbesar yang mungkin. [ KPK : Kelipat an Persekut uan t erKecil; FPB : Fakt or (pembagi) Persekut uan t erBesar] 3. Tent ukan semua bilangan real x yang memenuhi x 4 − 4x 3 + 5x 2 − 4x + 1 = 0 4. Pada segit iga lancip ABC, AD, BE dan CF adalah garis-garis t inggi, dengan D, E, F bert urut -t urut pada sisi BC, CA, dan AB. Bukt ikan bahwa DE + DF ≤ BC 5. Bilangan-bilangan 1, 2, 3, ⋅⋅⋅, 15, 16 disusun pada persegi 4 x 4. Unt uk i = 1, 2, 3, 4, misalkan b i adalah j umlah bilangan-bilangan pada baris ke-i dan k i adalah j umlah bilangan-bilangan pada kolom ke-i. Misalkan pula d 1 dan d 2 adalah j umlah bilangan-bilangan pada kedua diagonal. Susunan t ersebut dapat disebut ant imagic j ika b 1, b 2, b 3, b 4, k 1, k 2, k 3, k 4, d 1, d 2 dapat disusun menj adi sepuluh bilangan berurut an. Tent ukan bilangan t erbesar di ant ara sepuluh bilangan berurut an ini dapat diperoleh dari sebuah ant imagic. 208 SELEKSI OLIMPIADE TINGKAT PROVINSI 2007 TIM OLIMPIADE MATEMATIKA INDONESIA 2008 Prestasi itu diraih bukan didapat !!! SOLUSI SOAL Bidang Mat emat ika Bagian Pertama Disusun oleh : Eddy Hermant o, ST 209 Solusi Olimpiade Matematika Tk Provinsi 2007 Bagian Pert ama BAGIAN PERTAMA 1. Penj umlahan semua angkanya maksimal = 36. Tet api 36, 35, 34, 33 dan 32 bukan bilangan prima. Maka penj umlahan maksimal semua angkanya = 31. Dua angka pert ama harus sebesar mungkin, yait u 99. Jika angka ke-3 j uga 9 maka angka ke-4 harus 4, t et api 9994 bukanlah bilangan ganj il. Maka angka ket iga haruslah 8 dengan angka keempat adalah 5 yang merupakan bilangan ganj il. ∴ Bilangan ganj il 4-angka yang memenuhi adalah 9985. 2. Misalkan nilai uang pecahan 100-an = x Maka nilai uang pecahan 500-an = 3x dan nilai uang pecahan 200-an = 6x Karena (x) + (3x) + (6x) = 100. 000 maka x = 10. 000 Banyaknya koin 100-an = 10000 : 100 = 100 Banyaknya koin 200-an = (6 ⋅ 10000) : 200 = 300 Banyaknya koin 500-an = (3 ⋅ 10000) : 500 = 60 ∴ Banyaknya koin = 100 + 300 + 60 = 460. 3. Misalkan panj ang sisi miring segit iga t ersebut = r, sisi t erpendek = x dan sisi lainnya = y Diket ahui bahwa r = 2x dan y = x + 1 x 2 + y2 = r 2 maka x 2 + (x + 1) 2 = (2x) 2 2x 2 − 2x − 1 = 0 maka x 1, 2 = 2 ± 2 2 − 4( 2)( −1) 1 ± 3 = 2⋅2 2 Ambil nilai x yang posit if maka x = Luas segit iga = ½ ⋅ x ⋅ y ∴ Luas segit iga = 1+ 3 3+ 3 sehingga y = 2 2 3+ 2 3 4 4. 2006 = 2 ⋅ 17 ⋅ 59 ; 2007 = 32 ⋅ 223 ; 2008 = 23 ⋅ 251 Banyaknya f akor prima dari 2006 = 3 Banyaknya f akor prima dari 2007= 2 Banyaknya f akor prima dari 2008= 2 ∴ Maka bilangan yang memiliki f akt or prima berbeda t erbanyak adalah 2006. 5. Misalkan ia menj ual mobil masing-masing seharga y. Misalkan j uga modal mobil pert ama adalah x. Maka agar impas modal mobil kedua haruslah 2y − x. 1 sehingga 10y = 9x x −y = x 10 Keunt ungan mobil kedua = y − ( 2y − x ) x −y 10x − 9x 1 = = = 2y − x 2y − x 18x − 10x 8 ∴ Persent ase keunt ungan pedagang unt uk mobil kedua = 12, 5 % SMA Negeri 5 Bengkulu Eddy Hermanto, ST 210 Olimpiade Matematika Tk Provinsi 2007 Solusi Bagian Pert ama 6. Karena t idak ada yang t umpang t indih maka luas persegi = 245 : 5 = 49 cm 2. Panj ang sisi persegi = 7. Agar kelilingnya kecil maka harus semakin banyak sisi-sisi persegi yang menempel dengan sisi-sisi yang lain. ∴ Keliling persegi = 10 x panj ang sisi persegi = 70 cm 7. Apabila pert andingan dua t im berakhir seri maka t ot al nilai yang didapat kedua t im adalah 2 sedangkan apabila pert andingan dua buah t im berakhir dengan kemenangan salah sat u t im maka t ot al nilai kedua t im sama dengan 3. Tot al pert andingan = 4C2 = 6. Nilai 4 hanya didapat j ika t im t ersebut menang sat u kali, seri sat u kali dan kalah sat u kali. Agar j umlah nilai ket iga t im lainnya paling sedikit maka haruslah t iga pert andingan lainnya berakhir seri. Maka dari 6 pert andingan t erdapat 4 pert andingan yang berakhir seri. Nilai t ot al keempat t im = 2 ⋅ 4 + 3 ⋅ 2 = 14 Maka t ot al nilai ket iga t im lainnya = 14 − 4 = 10. ∴ Maka t ot al nilai ket iga t im lainnya paling sedikit = 14 − 4 = 10 . 8. 1! 1 + 2! 2 + 3! 3 + ⋅⋅⋅ + n! n = 1! (2 − 1) + 2! (3 − 1) + 3! (4 − 1) + ⋅⋅⋅ + n! ((n + 1) − 1) 1! 1 + 2! 2 + 3! 3 + ⋅⋅⋅ + n! n = (2! − 1! ) + (3! − 2! ) + (4! − 3! ) + ⋅⋅⋅ + (n + 1)! − n! 1! 1 + 2! 2 + 3! 3 + ⋅⋅⋅ + n! n = (n + 1)! − 1! ∴ 1! 1 + 2! 2 + 3! 3 + ⋅⋅⋅ + n! n = (n + 1)! − 1 9. Misalkan koordinat Q(x Q, yQ) dan P(x P, yP) Alt ernat if 1 : Karena P di kuadran I maka Q pun akan di kuadran I. Karena y Q = 2x Q maka yQ ≥ x Q Jarak Q ke garis y = x adalah PQ = 2. Jarak Q(x Q, yQ) ke garis Ax + By + C = 0 dirumuskan dengan : d = Ax Q + By Q A2 +B +C 2 Maka : d = yQ −xQ 12 + 12 = 2 . Maka yQ − x Q = 2√2 ⋅⋅⋅⋅⋅⋅⋅⋅⋅⋅⋅⋅⋅⋅⋅⋅⋅⋅⋅⋅⋅⋅⋅ (1) Karena garis y = 2x melalui Q maka yQ = 2x Q ⋅⋅⋅⋅⋅⋅ (2) Dari persamaan (1) dan (2) didapat x Q = 2√2 dan yQ = 4√2 SMA Negeri 5 Bengkulu Eddy Hermanto, ST 211 Olimpiade Matematika Tk Provinsi 2007 Solusi Bagian Pert ama Alt ernat if 2 : Gradien garis y = x adalah m = 1. Maka gradien garis yang melalui PQ adalah m PQ = −1 yQ −y P = −1 . Maka 2x Q − x P = x P − x Q sehingga 3x Q = 2x P ⋅⋅⋅⋅⋅⋅⋅⋅⋅⋅⋅⋅⋅⋅⋅⋅⋅⋅⋅⋅⋅ (3) xQ −x P (x Q −x ) + (y 2 P −y Q 2 2x P = ) 2 P = 2 sehingga (x Q2 − 2x Qx P + x P2) + (4x Q2 − 4x Qx P + x P2) = 4 5x Q2 − 6x Qx P + 4 ⋅⋅⋅⋅⋅⋅⋅⋅⋅⋅⋅⋅⋅⋅ (4) Subt it usikan persamaan 3x Q = 2x P ke persamaan (4) 10x Q2 − 18x Q2 + 9x Q2 = 8 Karena Q di kuadran I m aka x Q = 2√2 dan yQ = 4√2 ( ) ∴ Koordinat Q adalah 2 2,4 2 . 10. 6n + 30 = k(2n + 1) unt uk suat u k dan n bilangan asli. (k − 3)(2n + 1) = 27 = 33 Nilai 2n + 1 yang memenuhi hanya j ika 2n + 1 = 3, 9 at au 27 Jika 2n + 1 = 3 maka k − 3 = 9 sehingga n = 1 dan k = 12 Jika 2n + 1 = 9 maka k − 3 = 3 sehingga n = 4 dan k = 6 Jika 2n + 1 = 27 maka k − 3 = 1 sehingga n = 13 dan k = 4 ∴ Nilai n asli yang memenuhi 6n + 30 adalah kelipat an 2n + 1 adalah n = 1, 4, 13. ⎛ 11. ⎜⎜ 2x ⎝ 9 2 1⎞ 1 1 − ⎟⎟ = 9 C 0 ( 2x 2 ) 9 ( − ) 0 + L+ 9 C k ( 2x 2 ) k ( − ) 9−k + L x ⎠ x x Unt uk mencari suku konst ant a maka harus dipenuhi x 2k ⋅ x k-9 = x 0 sehingga k = 3 2 3 3-9 = 672 9C3(2x ) (x) ⎛ ∴ Maka konst ant a pada ekspansi ⎜⎜ 2x ⎝ 9 2 1⎞ − ⎟⎟ adalah 672 . x ⎠ 12. Misalkan persamaan garis l adalah y = mx + c Karena t it ik pot ongnya dengan sumbu y bilangan prima maka c adalah bilangan prima. Tit ik pot ong dengan sumbu x j ika y = 0. Maka mx + c = 0 sehingga x = − Karena c prima maka − c adalah bilangan prima. m c akan prima hanya j ika m = −1. Maka y = −x + c m Karena garis melalui t it ik (3, 4) maka 4 = −3 + c. Akibat nya c = 7 dan y = −x + 7 ∴ Persamaan garis l adalah y = −x + 7. 13. Karena dalam set iap dua kant ong berselisih paling bayak 1, maka banyaknya permen hanya akan ada 2 j enis, yait u m dan m + 1. Pendapat 1 : Karena 17 ≡ 1 (mod 2) maka unt uk dua kant ong akan t erdapat dua kemungkinan yait u sat u kant ong berisi 8 permen sedangkan kant ong lainnya 9 permen dan sebaliknya. SMA Negeri 5 Bengkulu Eddy Hermanto, ST 212 Solusi Olimpiade Matematika Tk Provinsi 2007 Bagian Pert ama Karena 17 ≡ 2 (mod 3) maka unt uk t iga kant ong, kemungkinan mengemas permen akan t erdiri dari dua kant ong berisi 6 permen dan sat u kant ong lagi berisi 5 permen. Banyaknya cara mengemas permen pada kasus ini adalah 3! = 3 cara. 2!⋅1! Karena 17 ≡ 1 (mod 4) maka unt uk empat kant ong, kemungkinan mengemas permen akan t erdiri dari sat u kant ong berisi 5 permen dan t iga kant ong lagi berisi 4 permen. Banyaknya cara mengemas permen pada kasus ini adalah 4! = 4 cara. Demikian set erusnya. 1!⋅3! Misalkan Banyaknya cara = N maka : N = 2! + 3! + 4! + 5! + 6! + 7! + 8! + 9! + 10! + 11! + 12! + 13! + 14! + 15! + 16! + 17! 1!⋅1! 2!⋅1! 1!⋅3! 2!⋅3! 5!⋅1! 3!⋅4! 1!⋅7! 8!⋅1! 7!⋅3! 6!⋅5! 5!⋅7! 4!⋅9! 3!⋅11! 2!⋅13! 1!⋅15! 0!⋅17! Banyaknya cara = 2 + 3 + 4 + 10 + 6 + 35 + 8 + 9 + 120 + 462 + 792 + 715 + 364 + 105 + 16 + 1 ∴ Banyaknya cara mengemas permen = 2652 Pendapat 2 : Karena banyaknya permen pada masing-masing kant ong adalah m at au m + 1 maka pada masing-masing banyaknya kant ong hanya akan ada 1 kemungkinan cara mengemas permen. ∴ Karena kemungkinan banyaknya kant ong ada 16, maka banyaknya cara mengemas permen ada 16. Cat at an : Pendapat 1 didasarkan asumsi bahwa kant ong-kant ong t ersebut semuanya berbeda sehingga 8 permen dimasukkan ke kant ong pert ama dan 9 permen dimasukkan ke kant ong kedua akan berbeda dengan bila 9 permen dimasukkan ke kant ong pert ama dan 8 permen dimasukkan ke kant ong kedua. Sedangkan Pendapat 2 didasarkan asumsi bahwa kant ong-kant ong t ersebut semuanya ident ik sehingga 8 permen dimasukkan ke kant ong pert ama dan 9 permen dimasukkan ke kant ong kedua akan sama dengan bila 9 permen dimasukkan ke kant ong pert ama dan 8 permen dimasukkan ke kant ong kedua. Solusi Panit ia Pusat adalah sesuai dengan pendapat 2. 14. Alt ernat if 1 : Digambar daerah x ≥ 0, y ≥ 0 dan x + 3y ≤ 6. SMA Negeri 5 Bengkulu Eddy Hermanto, ST 213 Solusi Olimpiade Matematika Tk Provinsi 2007 Bagian Pert ama Daerah yang memenuhi x ≥ 0, y ≥ 0 dan x + 3y ≤ 6 adalah OCF. Dibuat garis x + y = 4 Perpot ongan garis x + 3y = 6 dengan x + y = 4 adalah di E(3, 1). Perpot ongan garis y = 0 dengan x + y = 4 adalah di B(4, 0). Dibuat garis 3x + y = p yang melalui E(3, 1) dan 3x + y = q yang melalui B(4, 0). Maka akan didapat nilai p = 10 dan q = 12. ⎛ 15 3 ⎞ Garis 3x + y = 12 memot ong garis x + 3y = 6 di D ⎜⎜ , ⎟⎟ yang membuat x + y > 4. ⎝ 4 4⎠ ⎛ 10 ⎞ ,0 ⎟⎟ yang membuat x + y < 4. Garis 3x + y = 10 memot ong garis y = 0 di A ⎜⎜ ⎝ 3 ⎠ Karena nilai x + y yang dimint a dalam soal adalah nilai maksimum maka persamaan 3x + y ≤ a yang memenuhi adalah 3x + y ≤ 10. ∴ Maka nilai a yang memenuhi adalah a = 10. Alt ernat if 2 : Karena x + 3y ≤ 6 dan 3x + y ≤ a maka 4(x + y) ≤ 6 + a Karena maks(x + y) = 4 maka haruslah dipenuhi 4 ⋅ 4 = 6 + a a = 10 ∴ Maka nilai a yang memenuhi adalah a = 10. 15. Banyaknya sisi dapat dinyat akan dalam luasan. Luasan yang dicat = 6 x 5 x 5 = 150. Luasan keseluruhan = 125 buah x 6 x 1 x 1 = 750 Luasan yang t idak dicat = 750 − 150 = 600 ∴ Rasio sisi yang dicat t erhadap yang t idak dicat = 150 : 600 = 1 : 4. 16. Jika sat u kart u dit aruh pada papan maka kart u t ersebut akan menut upi sat u pet ak warna hit am dan sat u pet ak warna put ih. Maka j elas bahwa dua pet ak yang dibuang agar dipenuhi bahwa sisa pet ak dapat dit ut upi oleh 7 buah kart u harus memenuhi bahwa kedua pet ak t ersebut berbeda warna. Akan dibukt ikan bahwa bagaimana pun cara memilih pet ak asalkan berbeda warna maka sisanya akan dapat dit ut upi oleh 7 buah kart u. Dalam sat u baris 4 x 1 pet ak maupun dalam sat u kolom 1 x 4 pet ak, j elas dapat dit ut upi oleh dua buah kart u. • Jika dua pet ak yang dibuang berada pada sat u baris Jika 2 pet ak t ersebut berada pada kolom ke-1 dan 2 at au kolom 3 dan 4 maka sisanya dapat dit ut upi oleh 1 buah kart u. Tiga baris sisa akan dapat dit ut upi oleh 6 buah kart u. Jika 2 pet ak t ersebut berada pada kolom ke-2 dan 3 maka j elas baris t ersebut dan baris didekat nya dapat dit ut upi oleh 3 buah kart u. Sedangkan 2 baris sisanya dapat dit ut upi oleh 4 buah kart u lagi. • Jika dua pet ak yang dibuang, salah sat unya di baris ke-n sedangkan sat u lagi di baris ke-(n+1) Jelas j uga bahwa dua baris t ersebut dapat dit ut upi oleh t iga buah kart u. Dua baris lainnya sesuai dengan ket erangan sebelumnya dapat dit ut upi oleh 4 buah kart u. SMA Negeri 5 Bengkulu Eddy Hermanto, ST 214 Olimpiade Matematika Tk Provinsi 2007 Solusi Bagian Pert ama • Jika dua pet ak yang dibuang, salah sat unya di baris ke-n sedangkan sat u lagi di baris ke-(n+2) Taruh sebuah kart u dalam arah vert ikal sedemikian sehingga t erdapat sat u baris berisi sat u pet ak yang dibuang dan sat u pet ak lagi merupakan salah sat u pet ak dari kart u yang dit aruh dengan warna kedua pet ak t ersebut berbeda. Maka akan t erbent uk 2 bagian. Sat u bagian t erdiri dari 2 baris dengan 2 pet ak “ dibuang” dan sat u baris sisanya t erdiri 2 pet ak yang ‘ dibuang’ . Sesuai dengan ket erangan sebelumnya maka sisa pet ak akan dapat dit ut upi oleh 6 buah kart u. • Jika dua pet ak yang dibuang, salah sat unya di baris ke-1 sedangkan sat u lagi di baris ke-4 Taruh sebuah kart u vert ikal dengan kedua pet aknya t erlet ak pada baris ke-2 dan ke-3 sedemikian sehingga dua pet ak pada baris ke-1 dan ke-2 yang t idak dapat dit aruh kart u lagi akan berbeda warna. Maka sesuai dengan ket erangan sebelumnya pada baris ke-1 dan ke-2 dapat dit ut upi oleh t iga buah kart u lagi. Demikian j uga dengan baris ke-3 dan ke-4. Terbukt i bahwa bagaimana pun cara memilih pet ak asalkan berbeda warna maka sisanya akan dapat dit ut upi oleh 7 buah kart u. . Banyaknya pet ak hit am dan put ih masing-masing ada 8. Maka banyaknya cara memilih dua pet ak agar dapat dipenuhi adalah 8 x 8 = 64. ∴ Banyaknya cara memilih dua pet ak = 64. (Cat at an : Persoalan persegi panj ang dengan ukuran yang lebih umum pernah dibahas di www. olimpiade. org. Pembukt ian dapat dilakukan dengan induksi mat emat ika) 17. Misalkan bilangan yang dihapus adalah k. 1+ 2 +L+ n − k 602 maka n + n − k = 35 7 = 17 n −1 17 2 n −1 n −k Karena k ≥ 1 maka n − k ≤ n − 1 sehingga 0 ≤ ≤1 n −1 7 n 7 sehingga 68 < n ≤ 70 34 ≤ ≤ 35 17 2 17 602 Jika n = 70 maka k = (1 + 2 + 3 + L + 70) − ⋅ 69 . Karena 17 t idak membagi 69 maka t idak ada 17 nilai k asli yang memenuhi. ( ) Jika n = 69 maka k = 1 + 2 + 3 + L + 69 − ∴ Maka n = 69 602 ⋅ 68 = 7 17 SMA Negeri 5 Bengkulu Eddy Hermanto, ST 215 Olimpiade Matematika Tk Provinsi 2007 Solusi Bagian Pert ama 18. Misalkan panj ang AC = x maka AD = x − 1 Pada ∆AFC berlaku AC cos C = FC maka cos C = 1 x Karena DC = DB maka ∆CDB sama kaki sehingga ∠DBC = C. Akibat nya ∠BDA = 2C Pada ∆BDA berlaku : BD cos ∠BDA = AD. Maka 1 ⋅ cos 2C = x − 1 sehingga 2cos 2 C − 1 = x − 1 2 =x x2 x =32 ∴ Maka AC = 19. 3 2 x +y + xy = 54 . Maka 2 x + y =6 3 (x + y ) 2 = 108 Karena x dan y keduanya bilangan asli maka x dan y keduanya harus berbent uk 3k 2. Agar didapat solusi x t erbesar maka y haruslah minimal. Nilai t erkecil y adalah y = 3. Maka didapat x = 5 3 sehingga x = 75. ∴ Maka solusi dengan x t erbesar adalah (x, y) = (75, 3) . 20. Misal koordinat V1(x 1, y1) dan V2(x 2, y2) dengan t it ik t engah V1 dan V2 adalah X12. Maka koordinat X12 adalah (½(x 1 + x 2), ½(y2 + y2)). Jika X12 memiliki koordinat bilangan bulat maka haruslah x 1 + x 2 dan y1 + y2 genap. Syarat it u t erj adi haruslah x 1 dan x 2 memiliki parit as yang sama dan y1 dan y2 j uga memiliki parit as yang sama. Kemungkinan j enis koordinat (dalam bahasa lain disebut parit as) suat u t it ik let is pada bidang hanya ada 4 kemungkinan yait u (genap, genap), (genap, ganj il), (ganj il, ganj il) dan (ganj il, genap). Agar dapat dipast ikan bahwa ada anggot a X yang memiliki koordinat bilangan bulat maka sesuai Pigeon Hol e Principl e (PHP) maka haruslah t erdapat sekurang-kurangnya 5 buah t it ik let is. ∴ Maka anggot a V paling sedikit harus 5 . SMA Negeri 5 Bengkulu Eddy Hermanto, ST 216 SELEKSI OLIMPIADE TINGKAT PROVINSI 2007 TIM OLIMPIADE MATEMATIKA INDONESIA 2008 Prestasi itu diraih bukan didapat !!! SOLUSI SOAL Bidang Mat emat ika Bagian Kedua Disusun oleh : Eddy Her mant o, ST 217 Olimpiade Matematika Tk Provinsi 2007 Solusi Bagian Kedua BAGIAN KEDUA 1. (a) Andaikan A berada di dalam segit iga BCD. Karena panj ang sisi-sisi ∆BAD dan ∆BDC sama maka ∆BAD dan ∆BCD kongruen dan karena sisi BD berhimpit sert a AB = AD = BC = CD maka t it ik A haruslah berhimpit dengan C. Kont radiksi dengan kenyat aan bahwa ABCD adalah segiempat . ∴ Terbukti bahwa A haruslah berada di luar segitiga BDC. (b) Misalkan ∠BAD = α dan ∠ABC = β Pada ∆BAD dan ∆BDC berlaku : BD2 = AD2 + AB2 − 2 ⋅ AD ⋅ AB cos ∠BAD BD2 = CD2 + CB2 − 2 ⋅ CD ⋅ CB cos ∠BCD Karena AD = AB = CD = CB maka ∠BAD = ∠BCD = α Pada ∆ABC dan ∆ADC berlaku : AC2 = BA2 + BC2 − 2 ⋅ BA ⋅ BC cos ∠ABC AC2 = DA2 + DC2 − 2 ⋅ DA ⋅ DC cos ∠ADC Karena BA = BC = DA = DC maka ∠ABC = ∠ADC= β Akibat nya α + β = 180o Karena α + β = 180o dan ∠ABC = β maka ∠CBE = α Karena ∠DAE = ∠CBE = α maka haruslah AD sej aj ar BC Karena α + β = 180o dan ∠ADC = β maka ∠CDF = α Karena ∠BAF = ∠CDF = α maka haruslah AB sej aj ar DC ∴ Terbukti bahwa setiap pasangan sisi berhadapan pada ABCD selalu sej aj ar. SMA Negeri 5 Bengkulu Eddy Hermanto, ST 218 Solusi Olimpiade Matematika Tk Provinsi 2007 Bagian Kedua 2. Misalkan FPB(a, b) = d = 10p + q maka KPK(a, b) = 10q + p Pendapat 1 : a = dx dan b = dy unt uk suat u bilangan asli d, x, y sert a FPB(x, y) = 1 dan x, y ≠ 1 Karena a dan b simet ri dan diinginkan b maksimum maka b > a sehingga y > x Jelas bahwa KPK(a, b) = dxy (10p + q)xy = (10q + p) Karena 10p + q dan 10q + p keduanya bilangan asli dua angka maka xy < 10 Karena x, y ≠ 1 dan FPB(x, y) = 1 maka pasangan (x, y) yang memenuhi hanya x = 2 dan y = 3. 6(10p + q) = 10q + p sehingga 59p = 4q Karena 59 adalah bilangan prima maka q haruslah kelipat an 59. Tet api q bilangan asli sat u angka. Maka t idak ada nilai q yang memenuhi. ∴ Tidak ada nilai b yang memenuhi. Pendapat 2 : Jika p ≥ 1. Karena 10p + q ⏐10q + p maka 10p + q ⏐10(q + 10p) − 99p sehingga 10p + q ⏐99p Jika 11 t idak membagi q + 10p maka 10p + q ⏐9p t et api 0 < 9p < q + 10p. Kont radiksi. Jika 11⏐10p + q maka 11⏐11p + q − p sehingga 11⏐q − p Karena 0 ≤ q − p ≤ 9 maka q − p = 0 sehingga KPK (a, b) = FPB (a, b). Akibat nya a = b. Kont radiksi. Jadi p = 0 KPK (a, b) = 10q dan FPB (a, b) = q. Misalkan a = xq dan b = yq dengan FPB (x, y) = 1. Karena KPK (a, b) = 10q maka KPK (x, y) = 10. Maka y = 10, 5, 2, 1. Jika y = 10 maka x = 1. Akibat nya x ⏐y sehingga a⏐b. Kont radiksi. Jika y = 5 maka x = 2. Akibat nya karena 1 ≤ q ≤ 9 maka b maks = yq = 45 dan a = 18. b = 45 dan a = 18 memenuhi syarat pada soal. Unt uk 1 ≤ y ≤ 5 maka karena 1 ≤ q < 9 maka b = yq ≤ 45 ∴ Nilai maksimum b yang memenuhi adalah b maks = 45. Pendapat 1 memiliki pendapat sebagai berikut . Pada soal diket ahui bahwa KPK(a, b) adalah bilangan 2-angka, sedangkan FPB(a, b) dapat diperoleh dengan membalik urut an angka pada KPK(a, b). Art inya j ika KPK(a, b) = xy maka FPB(a, b) = yx. Jika KPK(a, b) = 90 maka FPB(a, b) = 09. Dari pengert ian ini maka KPK(a, b) t idak boleh berakhiran dengan angka 0 sebab akan menyebabkan penulisan FPB(a, b) akan dimulai dengan angka 0 sehingga hal ini t idak lazim. Pendapat 2 memiliki pendapat sebagai berikut . Berdasarkan soal maka hanya KPK (a, b) yang harus merupakan bilangan dua angka sedangkan FPB(a, b) t idak harus merupakan bilangan dua angka. Membalik urut an angka bukan merupakan f ungsi sat u-sat u. Ekspresi mat emat ika haruslah kit a t uliskan dalam bent uk yang paling sederhana walaupun dari mana asalnya. 009, 09, dan 9 mempunyai art i yang sama kalau mereka kit a art ikan sebagai bilangan. Dengan demikian mereka merupakan sat u kelas dan cukup diwakili oleh 9 (sebagai ekspresi yang paling sederhana). Solusi dari Panit ia Pusat adalah sesuai dengan pendapat 2. SMA Negeri 5 Bengkulu Eddy Hermanto, ST 219 Solusi Olimpiade Matematika Tk Provinsi 2007 Bagian Kedua 3. x 4 − 4x 3 + 5x 2 − 4x + 1 = 0 (x 4 − 4x 3 + 6x 2 − 4x + 1) − x 2 = 0 ((x − 1) 2) 2 − x 2 = 0 Mengingat a2 − b 2 = (a − b)(a + b) maka : (x 2 − 2x + 1 − x)(x 2 − 2x + 1 + x) = 0 (x 2 − 3x + 1)(x 2 − x + 1) = 0 Karena ( −1) 2 − 4(1)(1) < 0 maka t idak ada x real yang memenuhi x 2 − x + 1 = 0. Unt uk x 2 − 3x + 1 = 0 dipenuhi oleh x 1, 2 = 3± 5 3 ± 3 2 − 4(1)(1) sehingga x 1, 2 = 2 ⋅1 2 ∴ Maka nilai x real yang memenuhi adalah x = 3+ 5 3− 5 atau x = 2 2 4. Misalkan ∠BAC = α, ∠ABC = β dan ∠ACB = γ Karena ∆AEB siku-siku di E dan ∆ADB siku-siku di D maka dengan AB sebagai diamet er, dapat dibuat sebuah lingkaran yang melalui A, E, D dan B. Maka AEDB adalah segiempat t alibusur. Karena AEDB adalah segiempat t alibusur maka ∠ABD + ∠AED = 180o ( β) + (90o + ∠BED) = 180o sehingga ∠BED = 90o − β. Maka ∠DEC = β Dengan cara yang sama ACDF adalah segiempat t alibusur. Karena ACDF segiempat t alibusur maka ∠ACD + ∠DFA = 180o ( γ) + (90o + ∠CFD) = 180o sehingga ∠CFD = 90o − γ. Maka ∠BFD = γ Karena ∆BFC siku-siku di F maka ∠BCF = 90o − β Karena ∆BEC siku-siku di E maka ∠EBC = 90o − γ DF BD ⋅⋅⋅⋅⋅⋅⋅⋅⋅⋅⋅⋅⋅⋅⋅⋅⋅⋅⋅⋅ = sin β sin γ DF DC Pada ∆CDF berlaku = sin( 90° − β ) sin( 90° − γ ) DE DC Pada ∆CDE berlaku ⋅⋅⋅⋅⋅⋅⋅⋅⋅⋅⋅⋅⋅⋅⋅⋅⋅⋅⋅⋅ = sin γ sin β DE BD Pada ∆BDE berlaku = sin( 90° − γ ) sin( 90° − β ) Pada ∆BDF berlaku SMA Negeri 5 Bengkulu (1) sehingga DF DC ⋅⋅⋅⋅⋅⋅⋅⋅⋅⋅⋅⋅⋅⋅⋅⋅⋅⋅⋅ (2) = cos β cos γ (3) sehingga DE BD ⋅⋅⋅⋅⋅⋅⋅⋅⋅⋅⋅⋅⋅⋅⋅⋅⋅⋅⋅ (4) = cos γ cos β Eddy Hermanto, ST 220 Solusi Olimpiade Matematika Tk Provinsi 2007 Bagian Kedua Dari persamaan (1) dan (4) didapat : ⎛ sin β cos γ ⎞ ⎟⎟ DE + DF = BD ⎜⎜ + ⎝ sin γ cos β ⎠ Mengingat sin 2x = 2 sin x cos x maka : 2 sin γ cos β (DE + DF) = BD (sin 2β + sin 2γ) Dari persamaan (2) dan (3) didapat : ⋅⋅⋅⋅⋅⋅⋅⋅⋅⋅⋅⋅⋅⋅⋅⋅⋅⋅⋅⋅⋅⋅⋅⋅⋅⋅⋅⋅⋅⋅⋅⋅⋅⋅⋅⋅⋅⋅⋅⋅ (5) ⎛ sin γ cos β ⎞ ⎟⎟ DE + DF = DC ⎜⎜ + ⎝ sin β cos γ ⎠ Mengingat sin 2x = 2 sin x cos x maka : 2 sin β cos γ (DE + DF) = DC (sin 2β + sin 2γ) ⋅⋅⋅⋅⋅⋅⋅⋅⋅⋅⋅⋅⋅⋅⋅⋅⋅⋅⋅⋅⋅⋅⋅⋅⋅⋅⋅⋅⋅⋅⋅⋅⋅⋅⋅⋅⋅⋅⋅⋅ (6) Mengingat sin β cos γ + cos β sin γ = sin ( β + γ) dan sin x + sin y = 2 sin ½(x + y) cos ½(x − y) maka : Persamaan (5 ) + Persamaan (6) : 2 sin( β + γ)(DE + DF) = 2 (BD + DC) sin ( β + γ) cos ( β − γ) DE + DF = BC cos ( β − γ) Mengingat cos ( β − γ) ≤ 1 maka DE + DF ≤ BC (t erbukt i) Tanda kesamaan t erj adi apabila β − γ = 0 at au β = γ sehingga ∆ABC sama kaki dengan AB = AC yang berakibat D adalah pert engahan BC. ∴ Terbukt i bahwa DE + DF ≤ BC 5. 1 + 2 + 3 + 4 + ⋅⋅⋅ + 16 = 136. Jelas bahwa b 1 + b 2 + b 3 + b 4 = k 1 + k 2 + k 3 + k 4 = 136. Misalkan m = min (b 1, b 2, b 3, b 4, k 1, k 2, k 3, k 4) dan M = maks (b 1, b 2, b 3, b 4, k 1, k 2, k 3, k 4) Misalkan j uga S = b 1 + b 2 + b 3 + b 4 + k 1 + k 2 + k 3 + k 4 = 2 ⋅ 136 = 272 Alt ernat if 1 : Andaikan bahwa (b 1, b 2, b 3, b 4, k 1, k 2, k 3, k 4) dapat disusun menj adi 8 bilangan berurut an, maka haruslah t erdapat nilai n bulat sehingga memenuhi : (n − 4) + (n − 3) + (n − 2) + (n − 1) + (n) + (n +1) + (n +2) + (n + 3) = 2 ⋅ 136 = 272 Karena 8n − 4 = 272 maka t idak ada n bulat yang memenuhi. Maka harus t erdapat sedikit nya salah sat u d 1 at au d 2 yang memenuhi m < d i < M. • Jika m < d 1 < M dan d 2 > M Maka maks (b 1, b 2, b 3, b 4, k 1, k 2, k 3, k 4, d 1, d 2) = d 2 dan M = m + 8 dan d 2 = m + 9 (m) + (m + 1) + ⋅⋅⋅ + (m + 6) + (m + 8) ≤ S ≤ (m) + (m + 2) + (m + 3) + ⋅⋅⋅ + (m + 8) 8m + 29 ≤ 272 ≤ 8m + 35 sehingga 29 < m ≤ 30. Maka nilai m yang mungkin hanya m = 30 b 1 + b 2 + b 3 + b 4 + k 1 + k 2 + k 3 + k 4 + d 1 = 30 + 31 + 32 + ⋅⋅⋅ + 38 = 306 d 1 = 306 − 272 = 34 sehingga d 2 = m + 9 = 39 Maka maks (b 1, b 2, b 3, b 4, k 1, k 2, k 3, k 4, d 1, d 2) = 39 Cont oh ant imagic yang memenuhi nilai maksimumnya = 39 SMA Negeri 5 Bengkulu Eddy Hermanto, ST 221 Solusi • Olimpiade Matematika Tk Provinsi 2007 Bagian Kedua Jika m < d 1 < d 2 < M Maka maks (b 1, b 2, b 3, b 4, k 1, k 2, k 3, k 4, d 1, d 2) = M dan M = m + 9 (m) + (m + 1) + ⋅⋅⋅ + (m + 6) + (m + 9) ≤ S ≤ (m) + (m + 3) + (m + 4) + ⋅⋅⋅ + (m + 9) 8m + 30 ≤ 272 ≤ 8m + 42 sehingga 28 < m ≤ 30. Maka nilai m yang mungkin hanya m = 29 at au 30 Karena ingin dicapai nilai M maksimum maka dipilih m = 30 sehingga M = m + 9 = 39 Alt ernat if 2 : Nilai maks (b 1, b 2, b 3, b 4, k 1, k 2, k 3, k 4, d 1, d 2) semakin besar apabila m semakin besar. Jika m ≥ 31 maka 31 + 32 + 33 + 34 + 35 + 36 + 37 + 38 ≤ S sehingga 276 ≤ 272 (t idak memenuhi) Maka t idak mungkin min (b 1, b 2, b 3, b 4, k 1, k 2, k 3, k 4) ≥ 31 sehingga m ≤ 30 Jika m = 30 maka 30 + 31 + 32 + ⋅⋅⋅ + 37 ≤ S ≤ 32 + 33 + 34 + ⋅⋅⋅ + 39 sehingga 268 ≤ 272 ≤ 284 Ada kemungkin t erdapat ant imagic yang memenuhi m = 30. Jika ada ant imagic yang memenuhi unt uk m = 30 maka maks (b 1, b 2, b 3, b 4, k 1, k 2, k 3, k 4, d 1, d 2) = 39. Cont oh ant imagic yang memenuhi unt uk m = 30. Unt uk m < 30 t idak perlu dicari sebab nilai maks (b 1, b 2, b 3, b 4, k 1, k 2, k 3, k 4, d 1, d 2) akan semakin kecil. ∴ Nilai maks (b 1, b 2, b 3, b 4, k 1, k 2, k 3, k 4, d 1, d 2) = 39. SMA Negeri 5 Bengkulu Eddy Hermanto, ST 222 SELEKSI TIM OLIMPIADE MATEMATIKA INDONESIA 2008 OLIMPIADE SAINS NASIONAL 2007 SURABAYA (JAWA TIMUR), 3 – 8 SEPTEMBER 2007 Bidang Mat emat ika Hari Pert ama Waktu : 180 Menit DEPARTEMEN PENDIDIKAN NASIONAL DIREKTORAT JENDERAL MANAJEMEN PENDIDIKAN DASAR DAN MENENGAH DIREKTORAT PEMBINAAN SEKOLAH MENENGAH ATAS TAHUN 2007 223 OLIMPIADE SAINS NASIONAL 2007 3 – 8 SEPTEMBER 2007 SURABAYA, JAWA TIMUR BI DAN G : MATEMATI KA HARI PERTAMA WAKTU : 180 MENIT 1. Misalkan ABC segit iga dengan ∠ABC = ∠ACB = 70o. Misalkan t it ik D pada sisi BC sehingga AD garis t inggi, t it ik E pada sisi AB sehingga ∠ACE = 10o, dan t it ik F adalah perpot ongan AD dan CE. Bukt ikan bahwa CF = BC. 2. Unt uk set iap bilangan asli n, b(n) menyat akan banyaknya f akt or posit if n dan p(n) menyat akan hasil penj umlahan semua f akt or posit if n. Sebagai cont oh, b(14) = 4 dan p(14) = 24. Misalkan k sebuah bilangan asli yang lebih besar dari 1. a. Bukt ikan bahwa ada t ak berhingga banyaknya bilangan asli n yang memenuhi b(n) = k 2 − k + 1. b. Bukt ikan bahwa ada berhingga banyaknya bilangan asli n yang memenuhi p(n) = k 2 − k + 1. 3. Misalkan a, b, c bilangan-bilangan real posit if yang memenuhi ket aksamaan 5(a2 + b 2 + c 2) < 6(ab+bc+ca). Bukt ikan bahwa ket iga ket aksamaan berikut berlaku : a + b > c, b + c > a, dan c + a > b 4. Suat u susunan 10-angka 0, 1, 2, 3, 4, 5, 6, 7, 8, 9 dikat akan cant ik j ika (i) saat dibaca dari kiri ke kanan, 0, 1, 2, 3, 4 membent uk barisan naik, sedangkan 5, 6, 7, 8, 9 membent uk barisan t urun, dan (ii) angka 0 t idak berada pada uj ung kiri. Sebagai cont oh, 9807123654 adalah susunan cant ik. Tent ukan banyaknya susunan cant ik. 224 SELEKSI TIM OLIMPIADE MATEMATIKA INDONESIA 2008 OLIMPIADE SAINS NASIONAL 2007 SURABAYA (JAWA TIMUR), 3 – 8 SEPTEMBER 2007 Bidang Mat emat ika Hari Kedua Waktu : 180 Menit DEPARTEMEN PENDIDIKAN NASIONAL DIREKTORAT JENDERAL MANAJEMEN PENDIDIKAN DASAR DAN MENENGAH DIREKTORAT PEMBINAAN SEKOLAH MENENGAH ATAS TAHUN 2007 225 OLIMPIADE SAINS NASIONAL 2007 3 – 8 SEPTEMBER 2007 SURABAYA, JAWA TIMUR BI DAN G : MATEMATI KA HARI KEDUA WAKTU : 180 MENIT 5. Misalkan r, s dua bilangan asli dan P sebuah ` papan cat ur' dengan r baris dan s laj ur. Misalkan M menyat akan banyak maksimal bent eng yang dapat dilet akkan pada P sehingga t idak ada dua bent eng yang saling menyerang. a. Tent ukan M. b. Ada berapa cara melet akkan M buah bent eng pada pada P sehingga t idak ada dua bent eng yang saling menyerang ? 6. Tent ukan semua t ripel bilangan real (x, y, z) yang memenuhi ket iga persamaan berikut sekaligus x = y3 + y − 8 y = z3 + z − 8 z = x3 + x − 8 7. Tit ik-t it ik A, B, C, D t erlet ak pada lingkaran S demikian rupa, sehingga AB merupakan garis t engah S, t et api CD bukan garis t engah S. Diket ahui pula bahwa C dan D berada pada sisi yang berbeda t erhadap AB. Garis singgung t erhadap S di C dan D berpot ongan di t it ik P. Tit ik-t it ik Q dan R bert urut -t urut adalah perpot ongan garis AC dengan garis BD dan garis AD dengan garis BC. a. Bukt ikan bahwa P, Q dan R segaris. b. Bukt ikan bahwa garis QR t egak lurus t erhadap garis AB. 8. Misalkan m dan n dua bilangan asli. Jika ada t ak berhingga banyaknya bilangan bulat k sehingga k 2 + 2kn + m 2 adalah bilangan kuadrat sempurna, bukt ikan bahwa m = n. 226 SELEKSI TIM OLIMPIADE MATEMATIKA INDONESIA 2008 OLIMPIADE SAINS NASIONAL 2007 SURABAYA (JAWA TIMUR), 3 – 8 SEPTEMBER 2007 Prestasi itu diraih bukan didapat !!! SOLUSI SOAL Bidang Mat emat ika Disusun oleh : Eddy Hermant o, ST 227 Olimpiade Sains Nasional 2007 Solusi Bidang : Mat emat ika 1. Karena ∠ABC = ∠ACB = 70o maka ∆ABC sama kaki. Alt ernat if 1 : Karena AD adalah garis t inggi dan ∆ABC sama kaki maka AD akan memot ong pert engahan BC. Karena FD memot ong pert engahan BC dan FD garis t inggi ∆BFC maka ∆BFC sama kaki dengan FB = FC. ∠FCB = 70o − 10o = 60o. Maka ∠FBC = ∠FCB = 60o. Akibat nya ∠BFC = 60o. Maka ∆FBC adalah segit iga sama sisi. ∴ Jadi CF = BC (t erbukt i) Alt ernat if 2 : ∠BAC = 180o − 70o − 70o = 40o. Karena AD garis t inggi maka ∠FAC = 20o sehingga ∠AFC = 150o. Karena ∠ACF = 10o maka pada ∆AFC berlaku CF AC = sin 20° sin 150° Sehingga CF = 2 AC sin 20o ⋅⋅⋅⋅⋅⋅⋅⋅⋅⋅⋅⋅⋅⋅⋅⋅⋅⋅⋅⋅⋅⋅⋅⋅⋅⋅⋅⋅⋅⋅⋅⋅⋅⋅⋅ (1) Pada ∆ABC berlaku BC AC = sin 40° sin 70° Mengingat sin 70o = cos 20o dan sin 40o = 2 sin 20o cos 20o maka BC = 2 AC sin 20o ⋅⋅⋅⋅⋅⋅⋅⋅⋅⋅⋅⋅⋅⋅⋅⋅⋅⋅⋅⋅⋅⋅⋅⋅⋅⋅⋅⋅⋅⋅⋅⋅⋅⋅⋅ (2) ∴ Dari (1) dan (2) dapat disimpulkan bahwa CF = BC (t erbukt i) 2. Misalkan n = p 1d1 ⋅ p 2d2 ⋅ p 3d3 ⋅ ⋅⋅⋅ ⋅ p ndn dengan p 1, p 2, p 3, ⋅⋅⋅, p n adalah bilangan prima maka banyaknya f akt or posit if dari n adalah (d 1 + 1)(d 2 + 1)(d 3 + 1) ⋅⋅⋅ (d n + 1) k 2 −k a. Dengan mengambil n = p maka banyaknya f akt or posit if n = b(n) = k 2 − k + 1 ∴ Karena ada t ak berhingga bilangan prima maka ada t ak berhingga n yang memenuhi banyaknya f akt or posit if n = b(n) = k 2 − k + 1 (t erbukt i). b. Unt uk n > 1 maka n memiliki sedikit nya 2 f akt or posit if . Unt uk n > 1 maka p(n) ≥ 1 + n > 2 k2 − k + 1 ≥ 1 + n > 2 1 < n ≤ k2 − k Karena n t erlet ak pada suat u selang t ert ent u maka t idak mungkin ada t ak berhingga banyaknya bilangan asli n yang memenuhi p(n) = k 2 − k + 1. ∴ Maka berhingga banyaknya bilangan asli n yang memenuhi p(n) = k 2 − k + 1. SMA Negeri 5 Bengkulu Eddy Hermanto, ST 228 Solusi Olimpiade Sains Nasional 2007 Bidang : Mat emat ika 3. 5(a2 + b 2 + c 2) < 6(ab+bc+ca) ⋅⋅⋅⋅⋅⋅⋅⋅⋅⋅⋅⋅⋅⋅⋅⋅⋅⋅⋅⋅⋅⋅⋅⋅⋅⋅⋅⋅⋅⋅⋅ (1) Alt ernat if 1 : Tanpa mengurangi keumuman misalkan maks (a, b, c) = c 5ac + 5bc − 5c2 − a2 − ab + ac − ab − b 2 + bc > 4a2 − 8ab + 4b 2 (5c − a − b)(a + b − c) > (2a − 2b) 2 Karena bilangan kudrat t idak mungkin negat if maka (5c − a − b)(a + b − c) > 0 ⋅⋅⋅⋅⋅⋅⋅⋅⋅⋅⋅⋅⋅⋅⋅⋅⋅⋅⋅⋅⋅⋅⋅⋅⋅⋅⋅⋅⋅⋅⋅ (2) Karena maks (a, b, c) = c maka 5c − a − b > 0 Maka agar ket aksamaan (2) t erpenuhi maka haruslah a + b > c Karena c > a maka b + c > a dan karena c > b maka a + c > b ∴ Terbukt i bahwa a + b > c, b + c > a dan a + c > b Alt ernat if 2 : Tanpa mengurangi keumuman misalkan maks (a, b, c) = c Ket aksamaan (1) ekuivalen dengan 4(a + b)(a + b − c) > 4(a − b) 2 + 5(a + b − c) 2 Karena bilangan kudrat t idak mungkin negat if maka 4(a + b)(a + b − c) > 0 ⋅⋅⋅⋅⋅⋅⋅⋅⋅⋅⋅⋅⋅⋅⋅⋅⋅⋅⋅⋅⋅⋅⋅⋅⋅⋅⋅⋅⋅⋅⋅ (3) Karena a dan b posit if maka ket aksamaan (3) akan t erpenuhi bila a + b > c Karena c > a maka b + c > a dan karena c > b maka a + c > b ∴ Terbukt i bahwa a + b > c, b + c > a dan a + c > b Alt ernat if 3 : Tanpa mengurangi keumuman misalkan maks (a, b, c) = c 5(a2 + b 2 + c 2) < 6(ab+bc+ca) (b + c − a) 2 + (a +c − b) 2 + (a + b − c) 2 < 4(ab + ac + bc) − 2(a2 + b 2 + c2) = 2((b + c − a)(c + a − b) + (b + c − a)(a + b − c) + (c + a − b)(a + b − c)) Misalkan x = b + c − a, y = c + a − b dan z = a + b − c. x 2 + y2 + z2 < 2(xy + xz + yz) ⋅⋅⋅⋅⋅⋅⋅⋅⋅⋅⋅⋅⋅⋅⋅⋅⋅⋅⋅⋅⋅⋅⋅⋅⋅⋅⋅⋅⋅⋅⋅ (4) Jelas bahwa karena maks (a, b, c) = c maka x > 0 dan y > 0. Akan dibukt ikan bahwa z > 0 Jika z ≤ 0 maka x 2 + y2 + z2 − 2(xy + xz + yz) = (x − y) 2 + z2 − 2z(x + y) Karena z ≤ 0 maka 2z(x + y) ≤ 0 maka x 2 + y2 + z2 − 2(xy + xz + yz) ≥ 0. Kont radiksi dengan ket aksamaan (4). Maka haruslah t erpenuhi z > 0. ∴ Terbukt i bahwa a + b > c, b + c > a dan a + c > b Alt ernat if 4 : Tanpa mengurangi keumuman misalkan c = maks (a, b, c) Ket aksamaan (1) ekuivalen dengan 5(a + b + c)(a + b − c) > 10a2 + 10b 2 + 4ab − 6ac − 6bc ⋅⋅⋅⋅⋅⋅⋅⋅⋅⋅⋅⋅⋅⋅⋅⋅⋅⋅⋅⋅⋅⋅⋅⋅⋅⋅⋅⋅⋅⋅⋅⋅⋅⋅⋅⋅⋅⋅⋅⋅⋅⋅⋅⋅⋅⋅⋅⋅⋅⋅⋅⋅⋅⋅⋅⋅⋅⋅⋅ (5) Misalkan x = a + b − c, y = a + c − b, z = b + c − a maka 2a = x + y, 2b = x + z dan 2c = y + z Ket aksamaan (5) akan menj adi 10(x + y + z)x > 5(x + y) 2 + 5(x + z) 2 + 2(x + y)(x + z) − 3(x + y)(y + z) − 3(x + z)(y + z) 10(x + y + z)x > 12x 2 + 2y2 + 2z2 + 6xy + 6xz − 4yz 10(x + y + z)x > 6x 2 + 6x(x + y + z) + 2(y − z) 2 4(x + y + z)x > 6x 2 + 2(y − z) 2 Karena bilangan kuadrat t idak mungkin negat if maka 4(x + y + z)x > 0 Karena a + b + c > 0 maka x + y + z > 0 sehingga x > 0. Jadi a + b − c > 0 Karena c > a maka b + c > a dan karena c > b maka a + c > b. ∴ Terbukt i bahwa a + b > c, b + c > a dan a + c > b SMA Negeri 5 Bengkulu Eddy Hermanto, ST 229 Solusi Olimpiade Sains Nasional 2007 Bidang : Mat emat ika 4. Karena saat dibaca dari kiri ke kanan, 0, 1, 2, 3, 4 membent uk barisan naik, sedangkan 5, 6, 7, 8, 9 membent uk barisan t urun maka j elas bahwa angka paling kiri haruslah 0 at au 9. Tet api karena 0 t idak berada di uj ung kiri maka angka paling kiri haruslah 9. Alt ernat if 1 : Misalkan uj ung paling kiri adalah t empat pert ama. Maka angka-angka 5, 6, 7 dan 8 akan berada di 9 t empat lainnya. Banyaknya cara memilih 4 dari 9 t empat lainnya t ersebut = 9C4 = 126. Angka-angka 5, 6, 7 dan 8 akan dit empat kan di 4 t empat t ersebut . Karena angka-angka 5, 6, 7, 8 t ersebut t elah dit ent ukan urut annya maka hanya ada 1 cara menempat kan angkaangka 5, 6, 7, dan 8 pada ke-4 t empat t ersebut . Lima t empat t ersisa akan diisi oleh angka-angka 0, 1, 2, 3 dan 4. Karena angka-angka 0, 1, 2, 3, dan 4 t ersebut t elah dit ent ukan urut annya maka hanya ada 1 cara menempat kan angkaangka 0, 1, 2, 3 dan 4 pada 5 t empat t ersisa t ersebut . ∴ Maka banyaknya susunan cant ik = 9C4 x 1 x 1 = 126 susunan. Alt ernat if 2 : Angka-angka 5, 6, 7 dan 8 akan dit empat kan pada 4 dari 9 t empat . Misalkan x 1 menyat akan banyaknya angka-angka yang t erlet ak di ant ara angka 9 dan 8. x 2 menyat akan banyaknya angka-angka yang t erlet ak di ant ara angka 8 dan 7. x 3 menyat akan banyaknya angka-angka yang t erlet ak di ant ara angka 7 dan 6. x 4 menyat akan banyaknya angka-angka yang t erlet ak di ant ara angka 6 dan 5. x 5 menyat akan banyaknya angka-angka yang t erlet ak di sebelah kanan angka 5. Jelas bahwa x 1, x 2, x 3, x 4 dan x 5 adalah bilangan bulat t ak negat if dan kurang dari 6 sert a memenuhi persamaan x 1 + x 2 + x 3 + x 4 + x 5 = 5. Banyaknya pasangan bulat t ak negat if (x i , x j ) yang memenuhi x i + x j = m adalah m + 1, yait u (0, m), (1, m − 1), (2, m − 2), ⋅⋅⋅, (m, 0). Misalkan x i + x j + x k = n. Karena x k adalah bilangan bulat t ak negat if maka banyaknya t ripel bilangan bulat t ak negat if (x i , x j , x k) yang memenuhi adalah merupakan penj umlahan banyaknya pasangan (x i , x j ) yang memenuhi x i + x j = p dengan p = 0, 1, 2, ⋅⋅⋅, n. Banyaknya pasangan (x 1, x 2) yang memenuhi x 1 + x 2 = 0 ada 1. Banyaknya pasangan (x 1, x 2) yang memenuhi x 1 + x 2 = 1 ada 2. Banyaknya pasangan (x 1, x 2) yang memenuhi x 1 + x 2 = 2 ada 3. Banyaknya pasangan (x 1, x 2) yang memenuhi x 1 + x 2 = 3 ada 4. Banyaknya pasangan (x 1, x 2) yang memenuhi x 1 + x 2 = 4 ada 5. Banyaknya pasangan (x 1, x 2) yang memenuhi x 1 + x 2 = 5 ada 6. Banyaknya t ripel (x 3, x 4, x 5) yang memenuhi x 3 + x 4 + x 5 = 0 ada 1. Banyaknya t ripel (x 3, x 4, x 5) yang memenuhi x 3 + x 4 + x 5 = 1 ada 1 + 2 = 3 Banyaknya t ripel (x 3, x 4, x 5) yang memenuhi x 3 + x 4 + x 5 = 2 ada 1 + 2 + 3 = 6 Banyaknya t ripel (x 3, x 4, x 5) yang memenuhi x 3 + x 4 + x 5 = 3 ada 1 + 2 + 3 + 4 = 10 Banyaknya t ripel (x 3, x 4, x 5) yang memenuhi x 3 + x 4 + x 5 = 4 ada 1 + 2 + 3 + 4 + 5 = 15 Banyaknya t ripel (x 3, x 4, x 5) yang memenuhi x 3 + x 4 + x 5 = 5 ada 1 + 2 + 3 + 4 + 5 + 6 = 21 Maka banyaknya susunan cant ik = 1 ⋅ 21 + 2 ⋅ 15 + 3 ⋅ 10 + 4 ⋅ 6 + 5 ⋅ 3 + 6 ⋅ 1 ∴ Maka banyaknya susunan cant ik = 126. 5. Misalkan p = min (r, s) dan q = maks (r, s) Agar t idak ada dua bent eng yang saling menyerang maka t idak boleh ada sedikit nya dua bent eng t erlet ak pada baris maupun laj ur yang sama. SMA Negeri 5 Bengkulu Eddy Hermanto, ST 230 Olimpiade Sains Nasional 2007 Solusi Bidang : Mat emat ika a. Jika M > min (r, s) Jika min (r, s) = r maka sesuai dengan Pigeon Hol e Principl e maka akan ada sedikit nya dua bent eng t erlet ak pada sat u baris yang sama. Maka akan t erdapat dua bent eng yang saling menyerang. Jika min (r, s) = s maka sesuai dengan Pigeon Hol e Principl e maka akan ada sedikit nya dua bent eng t erlet ak pada sat u laj ur yang sama. Maka akan t erdapat dua bent eng yang saling menyerang. Jadi, j elas bahwa M ≤ min (r, s). Misalkan x i, j menyat akan pet ak cat ur pada baris ke-i dan laj ur ke-j . Jika p = min (r, s) buah bent eng dilet akkan pada pet ak x k, k dengan k = 1, 2, 3, ⋅⋅⋅, p maka p buah bent eng ini t idak akan saling menyerang. ∴ Maka M = min (r, s). b. Jika min (r, s) = r Cara melet akkan bent eng pada baris ke-1 ada s cara. Karena bent eng pada baris ke-1 dan ke-2 t idak boleh pada sat u laj ur maka banyaknya cara melet akkan bent eng pada baris ke-2 ada s − 1 cara. Cara melet akkan bent eng pada baris ke-3 ada s − 2 cara. Dan set erusnya. Cara melet akkan bent eng pada baris ke-r ada s − r + 1 Maka banyaknya cara melet akkan r bent eng t ersebut = s ⋅ (s − 1) ⋅ (s − 2) ⋅ ⋅⋅⋅ ⋅ (s − r + 1) Banyaknya cara melet akkan r bent eng t ersebut = s! (maks ( r , s ))! = (s − r )! ( maks ( r , s ) − min( r , s ))! Jika min (r, s) = s Cara melet akkan bent eng pada laj ur ke-1 ada r cara. Karena bent eng pada laj ur ke-1 dan ke-2 t idak boleh pada sat u baris maka banyaknya cara melet akkan bent eng pada laj ur ke-2 ada r − 1 cara. Cara melet akkan bent eng pada laj ur ke-3 ada r − 2 cara. Dan set erusnya. Cara melet akkan bent eng pada laj ur ke-s ada r − s + 1 Maka banyaknya cara melet akkan s bent eng t ersebut = r ⋅ (r − 1) ⋅ (r − 2) ⋅ ⋅⋅⋅ ⋅ (r − s + 1) Banyaknya cara melet akkan r bent eng t ersebut = r! (maks ( r , s ))! = ( r − s )! ( maks ( r , s ) − min( r , s ))! ∴ Dapat disimpulkan bahwa banyaknya cara melet akkan M buah bent eng pada pada P sehingga t idak ada dua bent eng yang saling menyerang = 6. x = y3 + y − 8 ⋅⋅⋅⋅⋅⋅⋅⋅⋅⋅⋅⋅⋅⋅⋅⋅⋅⋅⋅⋅⋅⋅⋅⋅⋅⋅⋅⋅⋅⋅⋅⋅⋅⋅⋅⋅⋅⋅⋅⋅⋅⋅⋅⋅⋅⋅ y = z3 + z − 8 ⋅⋅⋅⋅⋅⋅⋅⋅⋅⋅⋅⋅⋅⋅⋅⋅⋅⋅⋅⋅⋅⋅⋅⋅⋅⋅⋅⋅⋅⋅⋅⋅⋅⋅⋅⋅⋅⋅⋅⋅⋅⋅⋅⋅⋅⋅ 3 z=x +x−8 ⋅⋅⋅⋅⋅⋅⋅⋅⋅⋅⋅⋅⋅⋅⋅⋅⋅⋅⋅⋅⋅⋅⋅⋅⋅⋅⋅⋅⋅⋅⋅⋅⋅⋅⋅⋅⋅⋅⋅⋅⋅⋅⋅⋅⋅⋅ Alt ernat if 1 : Jika x < 0, berdasarkan persamaan (3) persamaan (2) maka y < 0 sert a j ika y < 0, Maka dapat disimpulkan bahwa j ika salah negat if . • Jika y > x y − x = (z3 − y3) + (z − y) Maka (z − y)(z2 + yz + y2 + 1) > 0 (maks ( r , s ))! ( maks ( r , s ) − min( r , s ))! (1) (2) (3) maka z < 0. Sedangkan j ika z < 0, berdasarkan berdasarkan persamaan (1) maka x < 0. sat u x, y at au z negat if maka x, y dan z semuanya SMA Negeri 5 Bengkulu Eddy Hermanto, ST 231 Solusi Olimpiade Sains Nasional 2007 Bidang : Mat emat ika Karena j ika salah sat u dari y at au z negat if akan menyebabkan y dan z keduanya negat if maka j elas bahwa z2 + yz + y 2 + 1 > 0. Akibat nya z > y. Karena z > y maka z − y = (x 3 − z3) + (x − z) > 0 Maka (x − z)(x 2 + xz + z2 + 1) > 0 Karena j ika salah sat u dari x at au z negat if akan menyebabkan x dan z keduanya negat if maka j elas bahwa x 2 + xz + z2 + 1 > 0. Akibat nya x > z. Maka dapat disimpulkan bahwa x > z > y > x. Kont radiksi. • Jika y < x y − x = (z3 − y3) + (z − y) Maka (z − y)(z2 + yz + y2 + 1) < 0 Karena j ika salah sat u dari y at au z negat if akan menyebabkan y dan z keduanya negat if maka j elas bahwa z2 + yz + y 2 + 1 > 0. Akibat nya z < y. Karena z < y maka z − y = (x 3 − z3) + (x − z) < 0 Maka (x − z)(x 2 + xz + z2 + 1) < 0 Karena j ika salah sat u dari x at au z negat if akan menyebabkan x dan z keduanya negat if maka j elas bahwa x 2 + xz + z2 + 1 < 0. Akibat nya x < z. Maka dapat disimpulkan bahwa x < z < y < x. Kont radiksi. • Jika y = x Berdasarkan persamaan (1) maka x 3 − 8 = 0 sehingga x = y = 2. z = 23 + 2 − 8 = 2 ∴ Maka x = y = z = 2 adalah sat u-sat unya penyelesaian. Alt ernat if 2 : Perhat ikan bahwa f ungsi f (t ) = t 3 + t − 8 adalah f ungsi monot on naik. Andaikan bahwa x ≠ y. Tanpa mengurangi keumuman misalkan x < y. Karena f (t ) merupakan f ungsi monot on naik maka z = f (x) < f (y) = x. Karena z < x maka y = f (z) < f (x) = z sehingga y < z < x yang merupakan kont radiksi dengan pengandaian bahwa x < y. Jadi x = y. Dengan cara yang sama akan diperoleh bahwa y = z. Dengan demikian x = y = z. Maka x = x 3 + x − 8 sehingga x 3 = 8. x = y = z = 2. Set elah diuj i ke persamaan (1), (2) dan (3) t ernyat a t ripel (2, 2, 2) memenuhi ket iga persamaan t ersebut . Jadi (2, 2, 2) merupakan penyelesaian persamaan pada soal. ∴ Maka x = y = z = 2 adalah sat u-sat unya penyelesaian. Alt ernat if 3 : Jumlahkan persamaan (1), (2) dan (3) diperoleh x 3 + y3 + z3 = 24 ⋅⋅⋅⋅⋅⋅⋅⋅⋅⋅⋅⋅⋅⋅⋅⋅⋅⋅⋅⋅⋅⋅⋅⋅⋅⋅⋅⋅⋅⋅ (4) Tanpa mengurangi keumuman misalkan x ≥ y, z. Jadi 3x 3 ≥ x 3 + y3 + z3 = 24 sehingga x ≥ 2. Akibat nya z = x 3 + x − 8 ≥ 2 dan y = z3 + z − 8 ≥ 2. Karena x ≥ 2, y ≥ 2 dan z ≥ 2 maka x 3 + y3 + z3 ≥ 24. Agar memenuhi persamaan (4) maka haruslah x = y = z = 2. Set elah diuj i ke persamaan (1), (2) dan (3) t ernyat a x = y = z = 2 memenuhi ket iga persamaan t ersebut . ∴ Maka x = y = z = 2 adalah sat u-sat unya penyelesaian. SMA Negeri 5 Bengkulu Eddy Hermanto, ST 232 Solusi Olimpiade Sains Nasional 2007 Bidang : Mat emat ika 7. Alt ernat if 1 : b. Karena AB diamet er dan C t erlet ak pada lingkaran S maka ∠ACB = 90o sehingga QC t egak lurus BR. Karena AB diamet er dan D t erlet ak pada lingkaran S maka ∠ADB = 90o sehingga RD t egak lurus BQ. Karena RD dan QC merupakan garis t inggi dan keduanya melalui t it ik A maka perpanj angan BA haruslah merupakan garis t inggi ∆BRQ. ∴ Terbukt i bahwa QR t egak lurus t erhadap garis AB. a. Misalkan ∠CBA = α dan ∠ABD = β sert a perpot ongan AB dengan RQ adalah t it ik T. Karena BT t egak lurus RQ maka ∠ERB = 90o − α Karena O pusat lingkaran maka ∠OCB = α sedangkan CQ ⊥ BC maka ∠QCO = 90o − α EC adalah garis singgung di t it ik C maka FC t egak lurus CO sehingga ∠ECQ = α Karena QC t egak lurus CR maka ∠ECR = 90o − α. Karena ∠ERC = ∠ECR maka ER = EC. Karena BT t egak lurus RQ maka ∠FQD = 90o − β Karena O pusat lingkaran maka ∠ODB = β sedangkan DR ⊥ BD maka ∠RDO = 90o − β FD adalah garis singgung di t it ik D maka ED t egak lurus DO sehingga ∠EDA = β Karena RD t egak lurus DQ maka ∠FDQ = 90o − β. Karena ∠FQD = ∠FDQ maka FD = FQ. Karena RC t egak lurus CQ dan ∠QRC = 90o − α maka ∠EQC = α sehingga ∆EQC sama kaki dengan EQ = EC = ER. Karena ∠QCR = 90o maka E adalah pusat lingkaran yang melalui t it ik Q, C dan R dengan QR adalah diamet er sehingga E adalah pert engahan QR. Karena QD t egak lurus DR dan ∠RQD = 90o − β maka ∠FRD = B sehingga ∆FRD sama kaki dengan FR = FD = FQ. Karena ∠RDQ = 90o maka F adalah pusat lingkaran yang melalui t it ik Q, D dan R dengan QR adalah diamet er sehingga F adalah pert engahan QR. Akibat nya haruslah E dan F berhimpit . Tet api garis singgung t erhadap S di C dan D berpot ongan di t it ik P, maka P = E = S sehingga P adalah pert engahan QR. ∴ Terbukt i bahwa P, Q dan R segaris. Alt ernat if 2 : a. Karena AB diamet er maka ∠ACB = ∠ADB = 90o sehingga ∠CQB = ∠BRD = 90o − ∠DBC. Misalkan O adalah pusat lingkaran S. ∠CPD = 180o − ∠DOC = 180o − 2∠CAD = 2∠CQB = 2∠CQD. SMA Negeri 5 Bengkulu Eddy Hermanto, ST 233 Olimpiade Sains Nasional 2007 Solusi Bidang : Mat emat ika Karena ∠CPD = 2∠CQD maka t it ik C, D dan Q t erlet ak pada sat u lingkaran dengan P adalah pusat lingkaran t ersebut . Dengan cara yang sama dapat dit unj ukkan bahwa P j uga merupakan pusat lingkaran yang melalui t it ik C, D dan R. Karena P adalah pusat sebuah lingkaran yang melalui t it ik-t it ik Q, D, C dan R maka ∆PCR dan ∆PQD keduanya adalah segit iga sama kaki. ∠DPQ + ∠RPC = (180o − 2∠PDQ) + (180o − 2∠PCR) = 2((90o − ∠PDQ) + (90o − ∠PCR)) Karena RD t egak lurus BQ dan QC t egak lurus BR maka ∠DPQ + ∠RPC = 2( ∠RDP + ∠PCQ) Karena ∠ADB = ∠ACB = 90o maka ∠RDP = ∠ODB dan ∠PCQ = ∠OCB. Karena ∆OCB dan ∆ODB keduanya sama kaki maka ∠ODB = ∠OBD dan ∠OCB = ∠OBC. ∠DPQ + ∠RPC = 2( ∠OBD + ∠OBC) = 2∠DBC = ∠DOC = 180o − ∠CPD Sehingga didapat ∠DPQ + ∠RPC + ∠CPD = 180o. ∴ Akibat nya haruslah t it ik-t it ik R, P dan Q segaris (t erbukt i). b. Karena AB diamet er dan C t erlet ak pada lingkaran S maka ∠ACB = 90o sehingga QC t egak lurus BR. Karena AB diamet er dan D t erlet ak pada lingkaran S maka ∠ADB = 90o sehingga RD t egak lurus BQ. Karena RD dan QC merupakan garis t inggi dan keduanya melalui t it ik A maka perpanj angan BA haruslah merupakan garis t inggi ∆BRQ. ∴ Terbukt i bahwa QR t egak lurus t erhadap garis AB. 8. Alt ernat if 1 : * Jika m 2 ≠ n2 Unt uk suat u bilangan bulat a > ⏐m 2 − n2⏐ t idak mungkin merupakan f akt or dari m 2 − n2. Misalkan t adalah f akt or dari m 2 − n2 maka j elas bahwa −⏐m 2 − n2⏐ ≤ t ≤ ⏐m 2 − n2⏐. Maka berhingga banyaknya kemungkinan t yang memenuhi. Misalkan k 2 + 2kn + m 2 = p 2 unt uk suat u bilangan asli p. (k + n) 2 + m 2 − n2 = p 2 m 2 − n2 = (p + k + n)(p − (k + n)) Maka p + k + n adalah f akt or dari m 2 − n2. m 2 −n2 . t t m 2 −n2 Jika kedua persamaan t ersebut dikurangkan akan didapat k = −n + − 2 2t Misalkan p + k + n = t unt uk suat u bilangan bulat t ak nol t maka p − (k + n) = Karena berhingga banyaknya kemungkinan nilai t maka akan berhingga j uga banyaknya bilangan bulat k yang memenuhi. Kont radiksi karena dinyat akan bahwa ada t ak berhingga banyaknya bilangan bulat k yang memenuhi. * Jika m 2 = n2 k 2 + 2kn + m 2 = (k + n) 2 Maka berapapun bilangan bulat k akan menyebabkan k 2 + 2kn + m 2 merupakan bilangan kuadrat sempurna unt uk suat u bilangan asli m dan n. Sehingga ada t ak berhingga banyaknya bilangan bulat k yang memenuhi. Maka didapat bahwa k2 + 2kn + m 2 akan merupakan bilangan kuadrat sempurna j ika m 2 = n2. ∴ Karena m dan n keduanya bilangan asli maka k 2 + 2kn + m 2 akan merupakan bilangan kuadrat sempurna j ika m = n (t erbukt i). SMA Negeri 5 Bengkulu Eddy Hermanto, ST 234 Solusi Olimpiade Sains Nasional 2007 Bidang : Mat emat ika Alt ernat if 2 : Andaikan m ≠ n sehingga m 2 − n2 ≠ 0. Misalkan k 1, k 2, ⋅⋅⋅ dan s1, s2, ⋅⋅⋅ adalah barisan-barisan bilangan bulat demikian sehingga k i 2 + 2k i n + m 2 = si 2 unt uk semua i = 1, 2, ⋅⋅⋅ Maka m 2 − n2 = si 2 − k i 2 + 2k i n − n2 m 2 − n2 = si 2 − (k i + n) 2 m 2 − n2 = (si − k i − n)(si + k i + n) Akibat nya ⏐m 2 − n2⏐ ≥ si + k i + n dan ⏐m 2 − n2⏐ ≥ −(si − k i − n) Sehingga 2⏐m 2 − n2⏐ ≥ (si + k i + n) − (si − k i − n) = 2k i + 2n ⏐m 2 − n2⏐ ≥ k i + n ⏐m 2 − n2⏐ − n ≥ k i unt uk semua I = 1, 2, ⋅⋅⋅ Karena m dan n adalah suat u nilai t et ap maka hanya ada 1 nilai k i unt uk semua i = 1, 2, ⋅⋅⋅ kont radiksi dengan kenyat aan bahwa ada t ak berhingga banyaknya bilangan bulat k yang memenuhi. Jadi haruslah m = n. SMA Negeri 5 Bengkulu Eddy Hermanto, ST 235 SELEKSI OLIMPIADE TINGKAT KABUPATEN/ KOTA TAHUN 2008 TIM OLIMPIADE MATEMATIKA INDONESIA TAHUN 2009 Bidang Mat emat ika Waktu : 3, 5 Jam DEPARTEMEN PENDIDIKAN NASIONAL DIREKTORAT JENDERAL MANAJEMEN PENDIDIKAN DASAR DAN MENENGAH DIREKTORAT PEMBINAAN SEKOLAH MENENGAH ATAS TAHUN 2008 236 OLIMPIADE MATEMATIKA NASIONAL SELEKSI TINGKAT KOTA/ KABUPATEN TAHUN 2008 Bagian Pertama Pilih sat u j awaban yang benar. Dalam hal t erdapat lebih dari sat u j awaban yang benar, pilih j awaban yang paling baik. 1. Jika a bilangan real, maka A. −⏐a⏐ B. −a a2 = C. ±a 2. Banyaknya f akt or posit if dari 5! adalah A. 4 B. 5 C. 16 D. a E. ⏐a⏐ D. 24 E. 120 3. Banyaknya susunan huruf B, I, O, L, A sehingga t idak ada dua huruf hidup (vowel) yang bert urut an adalah A. 8 B. 10 C. 12 D. 14 E. 16 4. Lingkaran T merupakan lingkaran luar bagi segit iga ABC dan lingkaran dalam bagi segit iga PQR. Jika ABC dan PQR keduanya segit iga samasisi, maka rasio keliling ∆ABC t erhadap keliling ∆PQR adalah A. 1 6 B. 1 4 C. 1 2 D. 2 E. 4 5. Jumlah empat bilangan asli bert urut an senant iasa habis dibagi p. Maka nilai p t erbesar adalah A. 1 B. 2 C. 4 D. 5 E. 7 6. Banyaknya himpunan X yang memenuhi {1, 2} ⊆ X ⊆ {1, 2, 3, 4, 5} adalah A. 3 B. 4 C. 8 D. 16 E. 32 7. Segit iga ABC sama kaki, yait u AB = AC, dan memiliki keliling 32. Jika panj ang garis t inggi dari A adalah 8, maka panj ang AC adalah A. 9 1 3 8. Jika f (x) = A. 1 f (− x) B. 10 C. 10 2 3 D. 11 1 3 E. 12 x +1 , maka unt uk x 2 ≠ 1, f ( −x) = x −1 B. −f ( −x) C. −f (x) 237 D. f (x) E. 1 f ( x) 9. Pada t rapesium ABCD, sisi AB sej aj ar sisi DC dan rasio luas segit iga ABC t erhadap luas segit iga ACD adalah 1/ 3. Jika E dan F bert urut -t urut adal ah t it ik t engah BC dan DA, maka rasio luas ABEF t erhadap luas EFDC adalah A. 1 3 B. 3 5 C. 1 D. 5 3 E. 3 10. Diket ahui bahwa a, b, c dan d adalah bilangan-bilangan asli yang memenuhi Jika b ≠ 1 dan c ≠ d, maka a b−a < c d −c b(d − 1) a D. < d (b − 1) c A. b−a a < d −c c a+b a E. < c+d c B. C. a c dan c < a. < b d a b(d − 1) < c d (b − 1) Bagian Kedua Isikan hanya j awaban saj a pada t empat yang disediakan 11. Suat u pert uj ukan dihadiri oleh sej umlah penont on. Set iap penont on dewasa membayar t iket seharga 40 ribu rupiah, sedangkan set iap penont on anak-anak membayar t iket 15 ribu rupiah. Jika j umlah uang penj ualan t iket adalah 5 j ut a rupiah, dan banyaknya penont on dewasa adalah 40 % dari seluruh penont on, maka banyaknya penont on anak-anak adalah ⋅⋅⋅⋅⋅⋅⋅⋅⋅⋅ 12. Diket ahui FPB (a, 2008) = 251. Jika a > 2008 maka nilai t erkecil yang mungkin bagi a adalah ⋅⋅⋅⋅⋅ 13. Set iap dung adalah ding. Ada lima ding yang j uga dong. Tidak ada dung yang dong. Jika banyaknya ding adalah 15, dan t iga di ant aranya t idak dung dan t idak dong, maka banyaknya dung adalah ⋅⋅⋅⋅⋅⋅⋅⋅⋅⋅⋅ 14. Dua buah dadu ident ik (sama persis) dilemparkan bersamaan. Angka yang muncul adalah a dan b. Peluang a dan b t erlet ak pada sisi-sisi yang bert olak belakang (di dadu yang sama) adalah ⋅⋅⋅⋅ 15. Bilangan 4-angka dibent uk dari 1, 4, 7 dan 8 dimana masing-masing angka digunakan t epat sat u kali. Jika semua bilangan 4-angka yang diperoleh dengan cara ini dij umlahkan, maka j umlah ini mempunyai angka sat uan ⋅⋅⋅⋅⋅⋅⋅ 16. Tit ik A dan B t erlet ak pada parabola y = 4 + x − x 2. Jika t it ik asal O merupakan t it ik t engah ruas garis AB, maka panj ang AB adalah ⋅⋅⋅⋅⋅ 17. Jika a dan b adalah bilangan-bilangan bulat dan x 2 − x − 1 merupakan f akt or dari ax 3 + bx 2 + 1, maka b = ⋅⋅⋅⋅⋅⋅⋅⋅⋅⋅ 18. Kubus ABCDEFGH dipot ong oleh bidang yang melalui diagonal HF, membent uk sudut 30o t erhadap diagonal EG dan memot ong rusuk AE di P. Jika panj ang rusuk kubus adalah 1 sat uan, maka panj ang ruas AP adalah ⋅⋅⋅⋅⋅⋅⋅⋅⋅⋅⋅⋅ 238 19. Himpunan semua bilangan asli yang sama dengan enam kali j umlah angka-angkanya adalah ⋅⋅⋅⋅⋅⋅ 20. Diket ahui bahwa a dan b adalah besar dua sudut pada sebuah segit iga. Jika sin a + sin b = 1 1 2 dan cos a + cos b = 6 , maka sin (a + b) = ⋅⋅⋅⋅⋅⋅⋅⋅⋅⋅ 2 2 239 SELEKSI OLIMPIADE TINGKAT KABUPATEN/ KOTA 2008 TIM OLIMPIADE MATEMATIKA INDONESIA 2009 Prestasi itu diraih bukan didapat !!! SOLUSI SOAL Bidang Mat emat ika Disusun oleh : Eddy Hermant o, ST 240 Olimpiade Matematika Tk Kabupaten/ Kota 2008 BAGIAN PERTAMA 1. (Jawaban : E) Akar dari suat u bilangan posit if adalah j uga bilangan posit if , maka a 2 = a j ika a bilangan real posit if a 2 = −a j ika a bilangan real negat ive ∴ Karena a bilangan real maka a 2 = ⏐a ⏐ 2. (Jawaban : C) 5! = 120 = 23 ⋅ 3 ⋅ 5 Banyaknya f akt or posit if = (3 + 1)(1 + 1)(1 + 1) = 16 ∴ Banyaknya f akt or posit if dari 5! adalah 16 . 3. (Jawaban : C) Agar huruf hidup t idak berdekat an maka ket iga huruf hidup t ersebut harus berada pada urut an ke-1, ke-3 dan ke-5. Sisanya harus diisi oleh huruf konsonan. Maka banyaknya susunan = 3! ⋅ 2! = 12 ∴ Banyaknya susunan = 12 . 4. (Jawaban : C) Misalkan j ari-j ari lingkaran t ersebut adalah R, sisi ∆ABC = x dan sisi ∆PQR = y. x = 2R sehingga 3x = 3R√3 sin 60° Luas ∆PQR = ½ ⋅ R ⋅ (3y) ½ y2 sin 60o = ½ ⋅ R ⋅ 3y sehingga 3y = 6R√3 Keliling ∆ABC : Keliling ∆PQR = 3x : 3y = 1 : 2 ∴ Rasio keliling ∆ABC t erhadap keliling ∆PQR adalah SMA Negeri 5 Bengkulu 1 . 2 Eddy Hermanto, ST 241 Olimpiade Matematika Tk Kabupaten/ Kota 2008 5. (Jawaban : B) (n) + (n + 1) + (n + 2) + (n + 3) = 2(2n + 3) 2n + 3 adalah bilangan ganj il. ∴ Maka nilai p t erbesar adalah 2 . 6. (Jawaban : C) {1, 2} ⊆ X ⊆ {1, 2, 3, 4, 5} X t erdiri dari sedikit nya 2 unsur dan maksimal 5 unsur dengan 2 unsur di ant aranya haruslah 1 dan 2. Sedangkan sisanya dipilih dari unsur-unsur 3, 4 at au 5. Jika X t erdiri dari 2 unsur maka banyaknya himpunan X = 3C0 = 1 Jika X t erdiri dari 3 unsur maka banyaknya himpunan X = 3C1 = 3 Jika X t erdiri dari 4 unsur maka banyaknya himpunan X = 3C2 = 3 Jika X t erdiri dari 5 unsur maka banyaknya himpunan X = 3C3 = 1 Banyaknya himpunan X = 1 + 3 + 3 + 1 = 8. ∴ Banyaknya himpunan X yang memenuhi adalah 8 . 7. (Jawaban : B) Misalkan panj ang AB = AC = x maka panj ang BC = 2 x − 64 maka 2 x + x 2 − 64 = 16 x 2 − 64 = (16 − x) 2 = x 2 − 32x + 256 32x = 320 x = 10 Panj ang AC = 10 ∴ Panj ang AC adalah 10 . 8. (Jawaban : E) x +1 x −1 1 − x +1 x −1 f ( −x) = = = − x −1 x +1 f ( x) 1 ∴ f ( −x) = f ( x) f (x) = 9. (Jawaban : D) SMA Negeri 5 Bengkulu Eddy Hermanto, ST 242 Olimpiade Matematika Tk Kabupaten/ Kota 2008 ∆ABC dan ∆ACD memiliki t inggi yang sama maka perbandingan luas keduanya dapat dinyat akan sebagai perbandingan alas. AB : DC = 1 : 3 Misalkan panj ang sisi AB = x maka panj ang sisi DC = 3x. E adalah pert engahan BC dan F pert engahan DA sehingga FE sej aj ar AB dan DC. Maka FE = ½ (x + 3x) = 2x Misalkan t inggi t rapesium = t . ( AB +FE ) t 3tx ⋅ = 2 2 4 ( FE + DC ) t 5tx Luas EFDC = ⋅ = 2 2 4 Luas ABEF = Rasio luas ABEF t erhadap luas EFDC = 3 : 5. ∴ Rasio luas ABEF t erhadap luas EFDC adalah 3 . 5 10. (Jawaban : A) a c b d < maka > . b d a c b−a d −c a b−a sehingga > < a c c d −c a b−a ∴ < c d −c Karena BAGIAN KEDUA 11. Misal penont on dewasa = x dan penont on anak-anak = y maka 40. 000x + 15. 000y = 5. 000. 000 8x + 3y = 1000 ⋅⋅⋅⋅⋅⋅⋅⋅⋅⋅⋅⋅⋅⋅⋅⋅⋅⋅⋅ (1) x = 40% (x + y) 3x = 2y ⋅⋅⋅⋅⋅⋅⋅⋅⋅⋅⋅⋅⋅⋅⋅⋅⋅⋅⋅⋅⋅⋅⋅⋅⋅ (2) Subt it usikan persamaan (2) ke (1) 16y + 9y = 3000 y = 120 ∴ Banyaknya penont on anak-anak adalah 120 12. 2008 = 8 ⋅ 251 dan a = 251 ⋅ k dengan k dan 8 relat if prima sert a k bilangan asli. Karena k > 8 dan dua bilangan asli berurut an akan relat if prima maka k min = 9. a minimum = k min ⋅ 251 a minimum = 9 ⋅ 251 = 2259. ∴ Nilai a t erkecil yang mungkin adalah 2259 . SMA Negeri 5 Bengkulu Eddy Hermanto, ST 243 Olimpiade Matematika Tk Kabupaten/ Kota 2008 13. Kalau persoalan t ersebut digambarkan dalam diagram venn maka ∴ Maka banyaknya dung adalah 7 . 14. Pasangan bilangan yang muncul adalah 1 dan 6 at au 2 dan 5 at au 3 dan 4. Banyaknya pasangan yang mungkin ada 6. ∴ Peluang = 6 36 15. Banyaknya bilangan yang mungkin ada 4! = 24. Masing-masing angka 1, 4, 7 dan 8 akan muncul 6 kali sebagai angka sat uan. Angka sat uan bilangan t ersebut = angka sat uan 6⋅ 1 + 6 ⋅ 4 + 6 ⋅ 7 + 6 ⋅ 8 ∴ Angka sat uan bilangan t ersebut adalah 0 . 16. Misalkan koordinat A adalah (p, q) maka karena pert engahan AB adalah t it ik (0, 0) maka koordinat B adalah ( −p, −q). Tit ik A dan B t erlet ak pada parabola maka q = 4 + p − p 2 ⋅⋅⋅⋅⋅⋅⋅⋅⋅⋅⋅⋅⋅⋅⋅ (1) −q = 4 − p − p 2 ⋅⋅⋅⋅⋅⋅⋅⋅⋅⋅⋅⋅⋅⋅⋅ (2) Jumlahkan persamaan (1) dan (2) didapat 0 = 8 − 2p 2 sehingga p = ±2 Jika p = 2 maka q = 4 + 2 − 22 = 2 Jika p = −2 maka q = 4 − 2 − 22 = −2 Koordinat A dan B adalah (2, 2) dan ( −2, −2) Panj ang AB = (2 − ( −2)) 2 + ( 2 − ( −2)) 2 ∴ Panj ang AB = 4 √2. 17. Karena koef isien x 3 adalah a dan konst ant anya adalah 1 maka haruslah (ax 3 + bx 2 + 1) = (x 2 − x − 1)(ax − 1) (ax 3 + bx 2 + 1) = ax 3 − (a + 1)x 2 + (1 − a)x + 1 Maka 1 − a = 0 sehingga a = 1 b = − (a + 1) sehingga b = −(1 + 1) = −2 ∴ Nilai b yang memenuhi adalah b = 2 . SMA Negeri 5 Bengkulu Eddy Hermanto, ST 244 Olimpiade Matematika Tk Kabupaten/ Kota 2008 18. Perhat ikan gambar. Perpot ongan bidang yang melalui HF t ersebut dengan kubus adalah segit iga PFH. Misalkan panj ang AP = x maka PE = 1 − x. E. PFH adalah bangunan prisma dengan alas berbent uk segit iga sama kaki. Karena PF = PH dan FE = HE maka proyeksi E pada bidang PFH akan berada pada garis t inggi PK. Sudut ant ara garis EG dengan bidang PFH adalah ∠EKP. EK = 1 2 2 Pada ∆KEP siku-siku di E. t an ∠EKP = 1 EP = EK 3 1 1− x = 1 3 2 2 6− 6 AP = 6 ∴ Panj ang ruas AP adalah 6− 6 . 6 19. Misalkan bilangan t ersebut adalah N. Misalkan N adalah bilangan n angka dengan angka-angka N adalah x 1, x 2, x 3, ⋅⋅⋅, x n. N ≥ 10n−1 dan N = 6(x 1 + x 2 + ⋅⋅⋅ + x n) ≤ 54n Lemma : Akan dibukt ikan bahwa j ika t erbukt i 54k < 10k−1 maka 54(k + 1) < 10k unt uk k bilangan asli ≥ 3. Andaikan bahwa 54k < 10k−1. Karena k ≥ 3 maka 54 < 9 ⋅ 10k−1 sehingga 54k + 54 < 10k−1 + 9 ⋅ 10k−1 54(k + 1) < 10k Terbukt i bahwa unt uk k asli ≥ 3 maka j ika 54k < 10k−1 maka 54(k + 1) < 10k. Pembuktian di atas sama saj a dengan membuktikan bahwa untuk k ≥ 3 maka j ika tidak ada N yang terdiri dari k angka yang memenuhi nilainya sama dengan 6 kali j umlah angka-angkanya maka tidak akan ada j uga N terdiri dari k + 1 angka yang memenuhi nilainya sama dengan 6 kali j umlah angka-angkanya. SMA Negeri 5 Bengkulu Eddy Hermanto, ST 245 Olimpiade Matematika Tk Kabupaten/ Kota 2008 • Jika N t erdiri dari 1 angka N = x 1 = 6(x 1) sehingga t idak ada N asli yang memenuhi. • Jika bilangan t ersebut adalah bilangan dua angka N = 10x 1 + x 2 = 6(x 1 + x 2) 4x 1 = 5x 2 Karena x 1 dan x 2 asli maka pasangan (x 1, x 2) yang memenuhi hanya (5, 4). Bilangan yang memenuhi hanya 54. • Jika N t erdiri dari 3 angka Misalkan N = 100x 1 + 10x 2 + x 3 = 6(x 1 + x 2 + x 3) 94x 1 + 4x 2 = 5x 3 Karena x 1 ≥ 1 maka t idak ada t ripel (x 1, x 2, x 3) yang memenuhi. Sesuai dengan lemma maka unt uk n ≥ 3 maka t idak ada N yang memenuhi nilainya sama dengan 6 angka j umlah angka-angkanya. Himpunan semua bilangan yang memenuhi hanya {54}. ∴ Himpunan semua bilangan yang memenuhi adalah {54} . 2 1 ⎛1 ⎞ 20. (sin a + sin b) = ⎜ 2⎟ = 2 ⎝2 ⎠ 1 sin2a + sin 2b + 2 sin a sin b = 2 2 ⋅⋅⋅⋅⋅⋅⋅⋅⋅⋅⋅⋅⋅⋅⋅⋅ (1) 2 3 ⎞ ⎛1 6⎟ = 2 ⎠ ⎝2 3 cos2a + cos2b + 2 cos a cos b = 2 (cos a + cos b) 2 = ⎜ ⋅⋅⋅⋅⋅⋅⋅⋅⋅⋅⋅⋅⋅⋅⋅⋅ (2) Jumlahkan (1) dan (2) dan dengan mengingat sin 2α + cos2α = 1 maka 2 + 2 (sin a sin b + cos a cos b) = 2 sin a sin b + cos a cos b = 0 cos (a − b) = 0 ⋅⋅⋅⋅⋅⋅⋅⋅⋅⋅⋅⋅⋅⋅⋅⋅⋅⋅⋅⋅⋅⋅⋅⋅⋅⋅⋅⋅⋅ (3) ⎛1 ⎞⎛ 1 ⎞ 1 2 ⎟⎜ 6⎟ = 3 ⎝2 ⎠⎝ 2 ⎠ 2 1 sin a cos a + sin b cos b + sin a cos b + cos a sin b = 3 2 1 ½ (sin 2a + sin 2b) + sin (a + b) = 3 2 1 sin (a + b) cos (a − b) + sin (a + b) = 3 2 1 Mengingat cos (a − b) = 0 maka sin (a + b) = 3. 2 1 ∴ sin (a + b) = 3. 2 (sin a + sin b )(cos a + cos b) = ⎜ Cat at an : Jika yang dicari adalah nilai a dan b. Tanpa mengurangi keumuman misalkan a ≥ b. SMA Negeri 5 Bengkulu Eddy Hermanto, ST 246 Olimpiade Matematika Tk Kabupaten/ Kota 2008 Berdasarkan cos (a − b) = 0 maka a − b = 90o ⋅⋅⋅⋅⋅⋅⋅⋅⋅⋅⋅⋅⋅⋅⋅⋅⋅⋅⋅⋅⋅⋅⋅⋅⋅⋅ (4) Karena sin (a + b) = • • 1 3 maka : 2 a + b = 60o ⋅⋅⋅⋅⋅⋅⋅⋅⋅⋅⋅⋅⋅⋅ (5) Berdasarkan (4) dan (5) maka didapat a = 75o dan b = −15o yang t idak memenuhi bahwa a dan b adalah besar dua sudut pada sebuah segit iga. a + b = 120o Berdasarkan (4) dan (6) maka didapat a = 115o dan b = 15o. Tet api bila a = 115o dan b = 15o disubt it usikan ke persamaan sin a + sin b = persamaan cos a + cos b = 1 2 dan 2 1 6 t ernyat a t idak memenuhi keduanya. 2 Dapat disimpulkan bahwa t idak ada pasangan (a, b) yang memenuhi. SMA Negeri 5 Bengkulu Eddy Hermanto, ST 247 SELEKSI OLIMPIADE TINGKAT PROVINSI 2008 TIM OLIMPIADE MATEMATIKA INDONESIA 2009 Bidang Mat emat ika Bagian Per t ama Waktu : 90 Menit DEPARTEMEN PENDIDIKAN NASIONAL DIREKTORAT JENDERAL MANAJEMEN PENDIDIKAN DASAR DAN MENENGAH DIREKTORAT PEMBINAAN SEKOLAH MENENGAH ATAS TAHUN 2008 248 SELEKSI TINGKAT PROVINSI CALON PESERTA OLIMPIADE SAINS NASIONAL 2008 MATEMATIKA SMA BAGIAN PERTAMA 1. Banyaknya pembagi posit if dari 2008 adalah ⋅⋅⋅⋅⋅⋅ 2. Cara menyusun huruf -huruf MATEMATIKA dengan kedua T t idak berdekat an ada sebanyak ⋅⋅⋅⋅⋅⋅⋅⋅⋅⋅ 3. Jika 0 < b < a dan a2 + b 2 = 6ab, maka a+b = ⋅⋅⋅⋅⋅⋅⋅ a−b 4. Dua dari panj ang garis t inggi segit iga ABC lancip, bert urut -t urut sama dengan 4 dan 12. Jika panj ang garis t inggi yang ket iga dari segit iga t ersebut merupakan bilangan bulat , maka panj ang maksimum garis t inggi segit iga t ersebut adalah ⋅⋅⋅⋅⋅⋅ 5. Dalam bidang XOY, banyaknya garis yang memot ong sumbu X di t it ik dengan absis bilangan prima dan memot ong sumbu Y di t it ik dengan ordinat bilangan bulat posit if sert a melalui t it ik (4, 3) adalah ⋅⋅⋅⋅⋅ 6. Diberikan segit iga ABC, AD t egak lurus BC sedemikian rupa sehingga DC = 2 dan BD = 3. Jika ∠BAC = 45o, maka luas segit iga ABC adalah ⋅⋅⋅⋅⋅⋅ 7. Jika x dan y bilangan bulat yang memenuhi y2 + 3x 2y2 = 30x 2 + 517, maka 3x 2y2 = ⋅⋅⋅⋅⋅⋅⋅ 8. Diberikan segit iga ABC, dengan BC = a, AC = b dan ∠C = 60o. Jika sudut B adalah ⋅⋅⋅⋅⋅ a = 2 + 3 , maka besarnya b 9. Serat us siswa suat u Provinsi di Pulau Jawa mengikut i seleksi t ingkat Provinsi dan skor rat arat anya adalah 100. Banyaknya siswa kelas II yang mengikut i seleksi t ersebut 50% lebih banyak dari siswa kelas III, dan skor rat a-rat a siswa kelas III 50% lebih t inggi dari skor rat a-rat a siswa kelas II. Skor rat a-rat a siswa kelas III adalah ⋅⋅⋅⋅⋅⋅ 10. Diberikan segit iga ABC, dengan BC = 5, AC = 12, dan AB = 13. Tit ik D dan E bert urut -t urut pada AB dan AC sedemikian rupa sehingga DE membagi segit iga ABC menj adi dua bagian dengan luas yang sama. Panj ang minimum DE adalah ⋅⋅⋅⋅⋅ 11. Misalkan a, b, c dan d bilangan rasional. Jika diket ahui persamaan x 4 + ax 3 + bx 2 + cx + d = 0 mempunyai 4 akar real, dua di ant aranya adalah ⋅⋅ 249 2 dan 2008 . Nilai dari a + b + c + d adalah 12. Diberikan segit iga ABC dengan sisi-sisi a, b, dan c. Nilai a2 + b 2 + c 2 sama dengan 16 kali luas segit iga ABC. Besarnya nilai ct g A + ct g B + ct g C adalah ⋅⋅⋅⋅⋅ 13. Diberikan f (x) = x 2 + 4. Misalkan x dan y adalah bilangan-bilangan real posit if yang memenuhi f (xy) + f (y − x) = f (y + x). Nilai minimum dari x + y adalah ⋅⋅⋅⋅⋅⋅ 14. Banyak bilangan bulat posit if n kurang dari 2008 yang mempunyai t epat n dan relat if prima t erhadap n adalah ⋅⋅⋅⋅⋅⋅⋅ n bilangan kurang dari 2 15. Suat u polinom f (x) memenuhi persamaan f (x 2) − x 3f (x) = 2(x 3 − 1) unt uk set iap x bilangan real. Deraj at (pangkat t ert inggi x) f (x) adalah ⋅⋅⋅⋅⋅⋅ 16. Anggap sat u t ahun 365 hari. Peluang dari 20 orang yang dipilih secara acak ada dua orang yang berulang t ahun pada hari yang sama adalah ⋅⋅⋅⋅⋅⋅⋅ 17. Tiga bilangan dipilih secara acak dari {1, 2, 3, ⋅⋅⋅, 2008}. Peluang j umlah ket iganya genap adalah ⋅⋅⋅ 18. Misalkan ⏐X⏐ menyat akan banyaknya anggot a himpunan X. Jika ⏐A ∪ B⏐ = 10 dan ⏐A⏐ = 4, maka nilai yang mungkin unt uk ⏐B⏐ adalah ⋅⋅⋅⋅⋅⋅ 19. Diket ahui AD adalah garis t inggi dari segit iga ABC, ∠DAB = ∠ACD, AD = 6, BD = 8. Luas segit iga ABC adalah ⋅⋅⋅⋅⋅ 1004 20. Nilai dari ∑3 k =0 k ⎛1004 ⎞ ⎜⎜ ⎟⎟ = ⋅⋅⋅⋅⋅⋅⋅ ⎝ k ⎠ 250 SELEKSI OLIMPIADE TINGKAT PROVINSI 2008 TIM OLIMPIADE MATEMATIKA INDONESIA 2009 Bidang Mat emat ika Bagian Kedua Waktu : 120 Menit DEPARTEMEN PENDIDIKAN NASIONAL DIREKTORAT JENDERAL MANAJEMEN PENDIDIKAN DASAR DAN MENENGAH DIREKTORAT PEMBINAAN SEKOLAH MENENGAH ATAS TAHUN 2008 251 SELEKSI TINGKAT PROVINSI CALON PESERTA OLIMPIADE SAINS NASIONAL 2008 MATEMATIKA SMA BAGIAN KEDUA 1. Carilah semua pasangan bilangan asli (x, n) yang memenuhi 1 + x + x 2 + ⋅⋅⋅ + x n = 40 2. Diberikan polinom real P(x) = x 2008 + a1x 2007 + a2x 2006 + ⋅⋅⋅ + a2007x + a2008 dan Q(x) = x 2 + 2x + 2008. Misalkan persamaan P(x) = 0 mempunyai 2008 selesaian real dan P(2008) ≤ 1. Tunj ukkan bahwa persamaan P(Q(x)) = 0 mempunyai selesaian real. 3. Lingkaran dalam dari segit iga ABC, menyinggung sisi-sisi BC, CA, dan AB bert urut -t urut di D, E, dan F. Melalui D, dit arik garis t egak lurus EF yang memot ong EF di G. Bukt ikan bahwa FG BF = EG CE 4. Bilangan 1, 2, 3, ⋅⋅⋅, 9 disusun melingkar secara acak. Bukt ikan bahwa ada t iga bilangan berdekat an yang j umlahnya lebih besar dari 15. 5. Tent ukan banyaknya bilangan posit if 5-angka palindrom yang habis dibagi 3. Palindrom adalah bilangan/ kat a yang sama j ika dibaca dari kiri ke kanan at au sebaliknya. Sebagai cont oh 35353 adalah bilangan palindrom, sedangkan 14242 bukan. 252 SELEKSI OLIMPIADE TINGKAT PROVINSI 2008 TIM OLIMPIADE MATEMATIKA INDONESIA 2009 Prestasi itu diraih bukan didapat !!! SOLUSI SOAL Bidang Mat emat ika Bagian Pertama Disusun oleh : Eddy Hermant o, ST 253 Olimpiade Matematika Tk Provinsi 2008 Solusi Bagian Pert ama BAGIAN PERTAMA 1. 2008 = 23 ⋅ 251 Banyaknya pembagi posit if dari 2008 = (3 + 1)(1 + 1) ∴ Banyaknya pembagi posit if dari 2008 = 8 . 2. Alt ernat if 1 : Banyaknya cara menyusun huruf -huruf MATEMATIKA adalah 10! = 151200 3!⋅2!⋅2! Banyaknya cara menyusun huruf -huruf MATEMATIKA dengan syarat kedua T berdekat an adalah sama dengan banyaknya cara menyusun huruf -huruf MATEMAIKA, yait u 9! = 30240 3!⋅2! Banyaknya cara menyusun huruf -huruf MATEMATIKA dengan kedua T t idak berdekat an adalah = 151200 − 30240 = 120960. ∴ Banyaknya cara menyusun = 120960 . Alt ernat if 2 : Karena T t idak boleh berdekat an maka kedua huruf T hanya dapat dit empat kan ke dalam 9 dari 10 t empat . Banyaknya cara memilih 9 t empat = 9C2 = 36 cara Ke-8 t empat yang lain akan diisi oleh ke-8 uruf t ersisa yang t erdiri dari 2 huruf M, 3 huruf A dan masing-masing sat u huruf yait u E, I dan K. Banyaknya cara = 8! = 3360 cara. 2!⋅3! Banyaknya cara menyusun huruf -huruf MATEMATIKA dengan kedua T t idak berdekat an adalah = 36 x 3360 = 120960. ∴ Banyaknya cara menyusun = 120960 . 3. Karena 0 < b < a maka a+b akan bernilai posit if . a−b 2 a 2 + b 2 + 2ab 6ab + 2ab ⎛a+b⎞ =2 = ⎜ ⎟ = 2 a + b 2 − 2ab 6ab − 2ab ⎝a −b⎠ a+b ∴ = 2 a −b 4. Misalkan panj ang sisi-sisi segit iga ABC adalah BC = a, AC = b dan AB = c. SMA Negeri 5 Bengkulu Eddy Hermanto, ST 254 Olimpiade Matematika Tk Provinsi 2008 Solusi Bagian Pert ama Misalkan j uga panj ang garis t inggi dari A adalah x dengan x bilangan asli. Ada dua kemungkinan pemahaman t erhadap pert anyaan pada soal. i) Yang dit anyakan adalah maks (x, 4, 12). [ ABC] = ½ ⋅ AC ⋅ 12 = ½ ⋅ AB ⋅ 4 b ⋅ 12 = AB ⋅ 4 AB = 3b [ ABC] = ½ ⋅ a ⋅ x = ½ ⋅ 4 ⋅ 3b a x = 12b ⋅⋅⋅⋅⋅⋅⋅⋅⋅⋅⋅⋅⋅⋅⋅⋅⋅⋅⋅⋅⋅⋅⋅⋅⋅⋅⋅⋅⋅⋅⋅⋅ (1) Akan dibukt ikan bahwa x ≤ 12 sehingga panj ang maksimum dari garis t inggi segit iga ABC adalah 12. Andaikan bahwa x > 12. Dari persamaan (1) akan didapat bahwa a < b ⋅⋅⋅⋅⋅⋅⋅⋅⋅⋅⋅⋅⋅⋅⋅⋅⋅⋅⋅⋅ (2) Pada segit iga siku-siku ACF j elas bahwa AC = b > AF Karena AB = 3b maka FB > 2b Pada segit iga siku-siku BCF berlaku bahwa BC > FB Karena BC = a < b sedangkan FB > 2b maka ket aksamamaan t idak mungkin t erj adi. Kont radiksi dengan pengandaian awal. Jadi, x ≤ 12. Maka panj ang maksimum garis t inggi segit iga ABC adalah 12 . ii) Yang dit anyakan adalah panj ang maksimum dari garis t inggi yang ket iga dari segit iga ABC Panj ang garis t inggi-garis t inggi yang bert urut -t urut sepadan dengan sisi-sisi a, b dan c adalah x, 12 dan 4. Dengan rumus luas segit iga ABC didapat hubungan xa = 12b = 4c Dengan ket aksamaan segit iga didapat c < a + b sehingga 1 < 1< a b + c c 4 1 + sehingga didapat x < 6. x 3 Jika x = 5 maka 5a = 12b = 4c a: b: c= 1 1 1 : : = 12 : 5 : 15 5 12 4 Karena 122 + 52 < 152 maka segit iga t ersebut t umpul. Kont radiksi. Jika x = 4 maka 4a = 12b = 4c a: b: c= 1 1 1 : : =3: 1: 3 4 12 4 Segit iga t ersebut adalah segit iga lancip sebab 32 + 12 > 32. Jadi, panj ang maksimum garis t inggi yang ket iga dari segit iga ABC adalah 4 . ∴ Dari dua kemungkinan ini Penulis lebih cenderung pada kemungkinan pert ama yang sesua dengan kat a-kat a pada soal. Panj ang maksimum garis t inggi dari segit iga ABC adalah 12 . 5. Misalkan persamaan garis t ersebut adalah y = mx + c Misalkan j uga garis memot ong sumbu X di (p, 0) dan sumbu Y di (0, q) dengan p adalah bilangan prima dan q adalah bilangan bulat posit if . Karena garis memot ong sumbu X di (p, 0) dan sumbu Y di (0, q) maka persamaan garis t ersebut adalah y = − q x+c. p SMA Negeri 5 Bengkulu Eddy Hermanto, ST 255 Olimpiade Matematika Tk Provinsi 2008 Solusi Garis melalui (0, q) maka c = q. Jadi persamaan garis t ersebut adalah y = − Bagian Pert ama q x+q p Karena garis melalui (4, 3) maka berlaku 3p = −4q + pq (p − 4)(q − 3) = 12 * Jika p genap maka p = 2 sehingga q = −3. Tidak memenuhi q bulat posit if . * Jika p ganj il maka p − 4 ganj il. Nilai p − 4 yang mungkin memenuhi adalah ±1 at au ±3. - Jika p − 4 = −1 maka p = 3 dan q = −9. Tidak memenuhi q bulat posit if . - Jika p − 4 = 1 maka p = 5 dan q = 15. Jadi persamaan garis adalah y = −3x + 15 yang melalui t it ik (4, 3) - Jika p − 4 = −3 maka p = 1 yang t idak memenuhi bahwa p adalah bilangan prima. - Jika p − 4 = 3 maka p = 7 dan q = 7. Jadi persamaan garis adalah y = −x + 7 yang melalui t it ik (4, 3) Persamaan garis yang memenuhi adalah y = −3x + 15 dan y = −x + 7. ∴ Banyaknya garis yang memenuhi ada 2 . 6. Perhat ikan gambar. Diket ahui dari soal ∠BAC = 45o. Alt ernat if 1 : Misalkan luas segit iga ABC = [ ABC] Dengan dalil pit agoras didapat : AC2 = AD2 + 4 ⋅⋅⋅⋅⋅⋅⋅⋅⋅⋅⋅⋅⋅⋅⋅⋅⋅⋅ (1) AB2 = AD2 + 9 ⋅⋅⋅⋅⋅⋅⋅⋅⋅⋅⋅⋅⋅⋅⋅⋅⋅⋅ (2) Persamaan (2) j umlahkan dengan (1) didapat AB2 + AC2 = 2AD2 + 13 ⋅⋅⋅⋅⋅⋅⋅⋅⋅⋅⋅⋅⋅⋅⋅⋅⋅⋅⋅⋅ (3) [ ABC] = ½ BC ⋅ AD Karena BC = 5 maka AD = 2[ABC ] ⋅⋅⋅⋅⋅⋅⋅⋅⋅⋅⋅⋅⋅⋅⋅⋅⋅⋅⋅⋅⋅ (4) 5 Pada segit iga ABC berlaku BC2 = AB2 + AC2 − 2 AB AC cos 45o = AB2 + AC2 − 2 AB AC sin 45o 25 = 2 AD2 + 13 − 4[ ABC] ⋅⋅⋅⋅⋅⋅⋅⋅⋅⋅⋅⋅⋅⋅⋅⋅⋅⋅⋅⋅⋅ (5) Subt it usikan persamaan (4) ke (5) 8[ABC ] − 4[ ABC ] 25 2 12 = (2[ ABC] + 5)([ ABC] − 15) = 0 Maka [ ABC] = 15 ∴ Luas segit iga ABC adalah 15 . SMA Negeri 5 Bengkulu Eddy Hermanto, ST 256 Olimpiade Matematika Tk Provinsi 2008 Solusi Bagian Pert ama Alt ernat if 2 : Misalkan AD = t ∠BAD + ∠CAD = 45o 1 = t an 45o = tan ∠BAD + tan ∠CAD 1 − tan ∠BAD ⋅ tan ∠CAD 3 2 + 1 = t t yang ekivalen dengan 3 2 1− ⋅ t t (t − 6)(t + 1) = 0 [ ABC] = ½ ⋅ BC ⋅ AD = ½ ⋅ 5 ⋅ 6 ∴ Luas segit iga ABC adalah 15 . 7. Persamaan t ersebut dapat diubah menj adi (3x 2 + 1)(y2 − 10) = 507 = 3 ⋅ 132 Karena 3x 2 + 1 bulat posit if maka y2 − 10 j uga bilangan bulat posit if . Fakt or posit if dari 507 ada 6 yait u 1, 3, 13, 39, 169 dan 507. y2 − 10 adalah f akt or dari 507 maka y 2 = 11, 13, 23, 49, 179 at au 517 dan yang merupakan bilangan kuadrat sempurna hanya 49. Maka y 2 = 49. Sehingga 3x 2 + 1 = 13. ∴ 3x 2y2 = 12 x 49 = 588. 8. Alt ernat if 1 : tan 45° − tan 30° = tan 15° = tan (45° − 30°) = 1 + tan 45° tan 30° tan 15° = 3 3+ 2 3 1 3 3− 3 3+ 3 3 ⋅ = 1 + 3 3 3+ 3 1 + 1⋅ 3 3 1− ⋅⋅⋅⋅⋅⋅⋅⋅⋅⋅⋅⋅⋅⋅⋅⋅⋅⋅⋅⋅⋅⋅ (1) Dengan dalil cosinus a b sin ∠A a sehingga = = = 2+ 3 sin ∠A sin ∠B sin ∠B b sin ∠A = 2 + 3 sin ∠B ⋅⋅⋅⋅⋅⋅⋅⋅⋅⋅⋅⋅⋅⋅⋅⋅⋅⋅⋅⋅⋅⋅⋅⋅⋅⋅ (2) ( ) Karena ∠C = 60o maka ∠A = 120o − ∠B sin ∠A = sin (120o − ∠B) = sin 120o cos ∠B − cos 120o sin ∠B (2 + 3 )sin ∠B = 12 1 3 cos ∠B + sin ∠B 2 1 ⎛3 ⎞ 3 cos ∠B ⎜ + 3 ⎟ sin ∠B = 2 ⎝2 ⎠ 3 tan ∠B = = tan 15 o 3+ 2 3 ∴ Besarnya sudut B adalah 15 o. SMA Negeri 5 Bengkulu Eddy Hermanto, ST 257 Olimpiade Matematika Tk Provinsi 2008 Solusi Bagian Pert ama Alt ernat if 2 : Misal AB = c dan dengan dalil cosinus didapat c 2 = a 2 + b 2 − 2ab cos 60° Subt it usikan a = 2 + 3 b yang didapat dari soal. 2 1 c2 = 2 + 3 b2 + b2 − 2 2 + 3 b ⋅ b ⋅ 2 2 2 c = 6+3 3 b ( ( ) ( ) ( ) ) Dengan dalil sinus didapat c2 b2 = (sin ∠C )2 (sin ∠B )2 (6 + 3 3 )b 2 b2 = sin 2 ∠B 3 4 sin 2 ∠B = ( 1 4 2+ 3 ) = 1 1 − 3 2 4 ∠B = 15 ∴ Besarnya sudut B adalah 15 o. o 9. Karena banyaknya siswa = 100 orang sedangkan banyaknya siswa kelas II 50% lebih banyak dari siswa kelas III maka banyaknya siswa kelas II yang mengikut i seleksi = 60 orang sedangkan siswa kelas III = 40 orang. Misalkan skor rat a-rat a kelas III adalah x maka skor rat a-rat a kelas II adalah 2 x. 3 2 60 ⋅ x + 40 ⋅ x 3 100 = 100 x = 125 ∴ Skor rat a-rat a siswa kelas III adalah 125 . 10. Misalkan panj ang AD = x dan panj ang AE = y 1 5 12 (5)(12) = 30 dan sin A = sert a cos A = 2 13 13 1 Luas ∆ADE = xy sin A = 15. Maka xy = 78. 2 Luas ∆ABC = Sesuai dalil cosinus pada ∆ADE maka : DE2 = x 2 + y 2 − 2xy cos A = x 2 + y2 − 144 Dengan AM-GM maka DE2 ≥ 2xy − 144 = 12 DE2 akan minimum sama dengan 12 j ika x = y = 78 ∴ DEminimum = 2 3 SMA Negeri 5 Bengkulu Eddy Hermanto, ST 258 Olimpiade Matematika Tk Provinsi 2008 Solusi Bagian Pert ama 11. Misalkan ke-4 akar t ersebut adalah x 1, x 2, x 3 dan x 4 dengan x 1 = 2 dan x 2 = 2008 = 2 502 . Alt ernat if 1 : x 4 + ax 3 + bx 2 + cx + d = (x − x 1) (x − x 2) (x − x 3) (x − x 4) = 0 x 1 + x 2 + x 3 + x 4 = −a yang merupakan bilangan rasional. Maka ada 2 kemungkinan nilai x 3 dan x 4. • x 3 = p − 2 − 2 502 dan x 4 = q unt uk p dan q bilangan rasional. x 1x 2x 3x 4 = d yang merupakan bilangan rasional. ( 2 )(2 )( ) 502 p − 2 − 2 502 (q ) = bilangan rasional unt uk p, q rasional 4 p 251 − 4 251 − 2008 2 = bilangan rasional. Maka t idak ada p rasional yang memenuhi • x 3 = p − 2 dan x 4 = q − 2 502 unt uk p dan q bilangan rasional. x 1x 2x 3x 4 = d yang merupakan bilangan rasional. ( 2 )(2 )( )( ) 502 p − 2 q − 2 502 = bilangan rasional 4 pq 251 − 2008 p 2 − 4q 502 + 4016 = bilangan rasional Kesamaan di at as akan t erpenuhi hanya j ika p = q = 0 sehingga x 3 = − 2 dan x 4 = − 2008 x 4 + ax 3 + bx 2 + cx + d = (x − 2 ) (x − 2008 ) (x + 2 ) (x + 2008 ) x 4 + ax 3 + bx 2 + cx + d = (x 2 − 2)(x 2 − 2008) = x 4 − 2010x 2 + 4016 Maka a = 0, b = −2010, c = 0 dan d = 4016 a + b + c + d = 0 − 2010 + 0 + 4016 ∴ Nilai a + b + c + d adalah 2006 . Alt ernat if 2 : 2 disubt it usikan ke persamaan x 4 + ax 3 + bx 2 + cx + d = 0 didapat (2a + c ) 2 = −(2b + d + 4) Jika Karena a, b, c dan d rasional maka kesamaan hanya mungkin t erj adi j ika 2a + c = 0 ⋅⋅⋅⋅⋅⋅⋅⋅⋅⋅ (1) Sehingga 2b + d + 4 = 0 ⋅⋅⋅⋅⋅⋅⋅⋅⋅⋅⋅⋅⋅⋅⋅ (2) 2008 = 2 502 disubt it usikan ke persamaan x 4 + ax 3 + bx 2 + cx + d = 0 didapat (2008a + c ) 2 = −(2008b + d + 4032064 ) Jika Karena a, b, c dan d rasional maka kesamaan hanya mungkin t erj adi j ika 2008a + c = 0 ⋅⋅⋅⋅⋅⋅⋅⋅⋅ (3) Sehingga 2008b + d + 4032064 = 0 ⋅⋅⋅⋅⋅⋅⋅⋅⋅⋅⋅⋅⋅⋅⋅ (4) Dari persamaan (1) dan (3) didapat a = 0 dan c = 0 Dari persamaan (2) dan (4) didapat b = −2010 dan d = 4016 a + b + c + d = 0 − 2010 + 0 + 4016 = 2006 ∴ Nilai a + b + c + d adalah 2006 . 12. Misalkan [ ABC] menyat akan luas ∆ABC. AB 2 + AC 2 − BC 2 . Berdasarkan dalil cosinus, cos ∠A = 2 ⋅ AB ⋅ AC AB 2 + AC 2 − BC 2 AB 2 + AC 2 − BC 2 cos ∠A Maka ct g ∠A = = = 4[ ABC ] 2 ⋅ AB ⋅ AC ⋅ sin ∠A sin ∠A SMA Negeri 5 Bengkulu Eddy Hermanto, ST 259 Solusi Olimpiade Matematika Tk Provinsi 2008 Bagian Pert ama Dengan cara yang sama didapat : ct g ∠B = AB 2 + BC 2 − AC 2 4[ ABC ] ct g ∠C = AC 2 + BC 2 − AB 2 4[ ABC ] AB 2 + AC 2 + BC 2 16 = ct g ∠A + ct g ∠B + ct g ∠C = 4[ ABC ] 4 ∴ ct g ∠A + ct g ∠B + ct g ∠C = 4. 13. f (x) = x 2 + 4 f (xy) = x 2y2 + 4 f (y − x) = (y − x) 2 + 4 f (y + x) = (y + x) 2 + 4 f (xy) + f (y − x) = f (y + x) x 2y2 + 4 + (y − x) 2 + 4 = (y + x) 2 + 4 x 2y2 + y2 + x 2 − 2xy + 4 = y 2 + x 2 + 2xy x 2y2 + 4 = 4xy (xy − 2) 2 = 0 Jadi xy = 2 Dengan ket aksamaan AM-GM maka x + y ≥ 2 xy = 2 2 Dengan memanf aat kan bilangan kuadrat t ak mungkin negat if 2 2 ⎛ 2⎞ ⎟ +2 2 x + y = x + = ⎜⎜ x − x ⎝ x ⎟⎠ t anda kesamaan t erj adi j ika x = y = 2 ∴ Nilai minimum dari x + y adalah 2 2 14. Jelas bahwa n harus genap. Misalkan n = 2y ⋅ p 1x1 ⋅ p 2x2 ⋅ ⋅⋅⋅ ⋅ p kxk dengan p i unt uk i = 1, 2, ⋅⋅⋅, k semuanya bilangan prima ganj il dan x i unt uk i = i, 2, ⋅⋅⋅, k semuanya bilangan bulat t ak negat if sert a y asli. Karena salah sat u f akt or dari n adalah 2 maka semua bilangan genap ≤ n t idak akan relat if prima dengan n. Banyaknya bilangan genap ≤ n ada t epat sebanyak kurang dari n j uga ada sebanyak n dan banyaknya bilangan ganj il 2 n . 2 Tet api unt uk semua 1 < p i < n dengan i = 1, 2, ⋅⋅⋅, k j uga merupakan f akt or dari n yang mengakibat kan semua 1 < p i < n dengan i = 1, 2, ⋅⋅⋅, k t idak akan relat if prima dengan n. Maka agar t erpenuhi ada t epat n bilangan kurang dari n dan relat if prima t erhadap n maka n 2 t idak boleh memiliki f akt or ganj il selain 1. Jadi p i = 1 unt uk semua i = 1, 2, ⋅⋅⋅, k. Maka n = 2y unt uk suat u bilangan asli y. SMA Negeri 5 Bengkulu Eddy Hermanto, ST 260 Solusi Olimpiade Matematika Tk Provinsi 2008 Bagian Pert ama Karena n < 2008 maka 2y < 2008. Jadi y ≤ 10. Maka nilai n yang memenuhi adalah 2, 4, 8, 16, 32, 64, 128, 256, 512, 1024. ∴ Banyaknya bilangan bulat posit if n yang memenuhi ada 10 . 15. Misalkan f (x) berderaj at n maka f (x 2) akan berderaj at 2n. x 3f (x) akan berderaj at n + 3. • Jika n > 3 maka 2n > n + 3 sehingga f (x 2) − x 3f (x) akan berderaj at 2n > 6. Jadi, t anda kesamaan t idak mungkin t erj adi. • Jika n = 3 maka f (x 2) dan x 3f (x) akan berderaj at sama yait u 6 sehingga masih dimungkinkan f (x 2) − x 3f (x) akan berderaj at 3. Jika f (x) = x 3 − 2 maka f (x 2) − x 3f (x) = (x 6 − 2) − x 3(x 3 − 2) = 2(x 3 − 1) yang memenuhi. • Jika n < 3 maka 2n < n + 3 sehingga f (x 2) − x 3f (x) akan berderaj at n + 3. Karena ruas kanan berderaj at 3 maka n = 0. ∴ Deraj at f (x) adalah 3 . 16. Banyaknya kemungkinan t anggal lahir dari 20 orang = 36520. Banyaknya kemungkinan dari 20 orang t ersebut t idak ada sat upun yang berulang t ahun di hari yang sama = 365 ⋅ 364 ⋅ 363 ⋅ ⋅⋅⋅ ⋅ 346 = 365P20. Peluang yang dit anyakan pada soal dapat dicari dengan cara komplemen. Peluang dari 20 orang yang dipilih secara acak ada dua orang yang berulang t ahun pada hari yang sama adalah 1− ∴ Peluang dari soal = 1 − P20 365 20 365 P20 365 20 365 17. Ada dua kemungkinan j umlah ket iga bilangan t ersebut genap • Ket iga bilangan t ersebut semuanya genap • 1004 ⋅ 1003 ⋅ 1002 C 167 6 = Peluang = 1004 3 = 2008 ⋅ 2007 ⋅ 2006 1338 2008 C 3 6 Ada sat u bilangan genap dan dua lainnya ganj il 1004 ⋅ 1003 502 1004 C1 ⋅1004 C 2 2 = = 2008 ⋅ 2007 ⋅ 2006 1338 2008 C 3 6 1004 ⋅ 167 502 + 1338 1338 1 ∴ Peluang j umlah ket iga bilangan t ersebut genap = 2 Peluang j umlah ket iga bilangan t ersebut genap = SMA Negeri 5 Bengkulu Eddy Hermanto, ST 261 Olimpiade Matematika Tk Provinsi 2008 Solusi Bagian Pert ama 18. ⏐A ∪ B⏐ = ⏐A⏐ + ⏐B⏐ − ⏐A ∩ B⏐ 10 = 4 + ⏐B⏐ − ⏐A ∩ B⏐ ⏐B⏐ − ⏐A ∩ B⏐ = 6 Jelas bahwa 0 ≤ ⏐A ∩ B⏐ ≤ ⏐A⏐ sehingga 0 ≤ ⏐A ∩ B⏐ ≤ 4. Jadi 6 ≤ ⏐B⏐ ≤ 10 Karena ⏐B⏐ bulat t ak negat if maka ⏐B⏐ = 6, 7, 8, 9 at au 10. ∴ ⏐B⏐ = 6, 7, 8, 9 at au 10 . 19. Misalkan ∠DAB = ∠ACD = α ct g α = AD CD = BD AD 6 CD 9 sehingga CD = = 8 6 2 Luas segit iga ABC = ½ ⋅ (BD + CD) ⋅ AD = ∴ Luas segit iga ABC = 75 2 75 2 20. Dengan binom Newt on didapat 4 1004 ∴ = (3 + 1) 1004 1004 ∑3 k =0 k ⎛1004 ⎞ 1004 1004 k ⎛1004 ⎞ ⎛1004 ⎞ 0 ⎛1004 ⎞ 1 ⎛1004 ⎞ 2 ⎟⎟ ⎟ ⎜ ⎟ ⎜ ⎟ ⎜ = ∑ 3 ⎜⎜ =⎜ ⎟3 + ⎜ 1 ⎟3 + ⎜ 2 ⎟3 + L + ⎜⎜1004 ⎟⎟3 0 k k 0 = ⎠ ⎝ ⎠ ⎝ ⎠ ⎝ ⎠ ⎝ ⎠ ⎝ ⎛1004 ⎞ ⎟⎟ = 2 2008 . ⎜⎜ ⎝ k ⎠ SMA Negeri 5 Bengkulu Eddy Hermanto, ST 262 SELEKSI OLIMPIADE TINGKAT PROVINSI 2008 TIM OLIMPIADE MATEMATIKA INDONESIA 2009 Prestasi itu diraih bukan didapat !!! SOLUSI SOAL Bidang Mat emat ika Bagian Kedua Disusun oleh : Eddy Her mant o, ST 263 Olimpiade Matematika Tk Provinsi 2008 Solusi Bagian Kedua BAGIAN KEDUA 1. 1 + x + x 2 + ⋅⋅⋅ + x n = 40 x + x 2 + ⋅⋅⋅ + x n = 39 x(1 + x + x 2 + ⋅⋅⋅ + x n−1) = 39 Karena x dan n bilangan asli maka x merupakan f akt or dari 39 Nilai x yang mungkin memenuhi adalah 1, 3, 13 at au 39. • Jika x = 1 maka 1 + 12 + ⋅⋅⋅ + 1n = 39. Jadi, n = 39 • Jika x = 3 x n +1 − 1 Karena x ≠ 1 maka 1 + x + x + ⋅⋅⋅ + x = = 40 x −1 2 • n Unt uk x = 3 maka 3n+1 − 1 = 80 Nilai n yang memenuhi adalah n = 3. Jika x = 13 Karena x ≠ 1 maka 1 + x + x 2 + ⋅⋅⋅ + x n = • x n +1 − 1 = 40 x −1 Unt uk x = 13 maka 13n+1 − 1 = 480 13n+1 = 481 = 13 ⋅ 37 Karena 37 t idak habis dibagi 13 maka t idak ada n asli yang memenuhi. Jika x = 39 Karena x ≠ 1 maka 1 + x + x 2 + ⋅⋅⋅ + x n = x n +1 − 1 = 40 x −1 Unt uk x = 39 maka 39n+1 − 1 = 1520 39n+1 = 1521 = 392 Nilai n yang memenuhi adalah n = 1. ∴ Semua pasangan bilangan asli (x, n) yang memenuhi adalah (1, 39), (3, 3), (39, 1) 2. Karena P(x) = 0 mempunyai 2008 selesaian real maka berlaku P(x) = (x − x 1)(x − x 2)(x − x 3) ⋅⋅⋅ (x − x 2008) dengan x i semua real unt uk i = 1, 2, ⋅⋅⋅, 2008. Karena P(2008) ≤ 1 maka t idak mungkin semua x i < 2007. P(Q(x)) = P(x 2 + 2x + 2008) P(Q(x)) = (x 2 + 2x + 2008 − x 1)(x 2 + 2x + 2008 − x 2) ⋅⋅⋅(x 2 + 2x + 2008 − x 2008) = 0 Diskriminan x 2 + 2x + 2008 − x i adalah Diskriminan = 4 − 4(2008 − x i ) Diskriminan = 4(x i − 2007) unt uk i = 1, 2, ⋅⋅⋅, 2008. Karena t idak semua x i < 2007 maka akan t erdapat x k sehingga Diskriminan = 4(x i − 2007) ≥ 0. Karena diskriminan ≥ 0 maka t erbukt i ada sedikit nya 2 bilangan x real yang memenuhi P(Q(x))= 0 ∴ Terbukti bahwa persamaan P(Q(x)) = 0 mempunyai selesaian real. SMA Negeri 5 Bengkulu Eddy Hermanto, ST 264 Solusi Olimpiade Matematika Tk Provinsi 2008 Bagian Kedua 3. Misalkan O adalah pusat lingkaran dalam segit iga ABC. Maka garis bagi dari B dan C akan melalui t it ik O. Karena CO dan BO adalah garis bagi maka ∠ECO = ∠DCO dan ∠DBO = ∠FBO Misalkan ∠ECO = ∠DCO = γ ⋅⋅⋅⋅⋅⋅⋅⋅⋅⋅ (1) dan ∠DBO = ∠FBO = β ⋅⋅⋅⋅⋅⋅⋅⋅⋅⋅ (2) Jelas bahwa ∠CEO = ∠CDO = 90o sehingga ∠EOD = 180o − 2γ ⋅⋅⋅⋅⋅⋅⋅⋅⋅⋅⋅⋅⋅⋅⋅⋅⋅⋅⋅⋅⋅⋅⋅⋅ (3) Jelas j uga bahwa ∠BDO = ∠BFO = 90o sehingga ∠DOF = 180o − 2β ⋅⋅⋅⋅⋅⋅⋅⋅⋅⋅⋅⋅⋅⋅⋅⋅⋅⋅⋅⋅⋅⋅⋅⋅ (4) Maka ∠EOF = 360o − ∠EOD − DOF = 2( γ + β) ⋅⋅⋅⋅⋅⋅⋅⋅⋅⋅⋅⋅⋅⋅⋅⋅⋅⋅⋅⋅⋅⋅⋅⋅ (5) Segit iga EOF adalah segit iga sama kaki sehingga ∠OEF = ∠OFE = 90o − ( γ + β) ⋅⋅⋅⋅⋅⋅⋅⋅⋅⋅⋅⋅⋅⋅⋅⋅⋅⋅⋅⋅⋅⋅⋅ (6) Lingkaran dalam menyinggung segit iga ABC di D, E dan F sehingga CE = CD dan BD = BF. Karena CE = CD dan OE = OD maka segiempat CEOD adalah layang-layang. Jadi, CO ⊥ ED. ED = 2 CE sin γ ⋅⋅⋅⋅⋅⋅⋅⋅⋅⋅⋅⋅⋅⋅⋅⋅⋅⋅⋅⋅⋅⋅ (7) ∠CED = 90o − γ sehingga ∠OED = γ ∠GED = ∠OEF + ∠OED = (90o − ( γ + β)) + ( γ) = 90o − β ⋅⋅⋅⋅⋅⋅⋅⋅⋅⋅⋅⋅⋅⋅⋅⋅⋅⋅⋅⋅⋅⋅⋅⋅⋅⋅⋅ (8) EG = ED cos ∠GED = (2 CE sin γ)(cos (90o − β) EG = 2 sin γ sin β CE ⋅⋅⋅⋅⋅⋅⋅⋅⋅⋅⋅⋅⋅⋅⋅⋅⋅⋅⋅⋅⋅⋅⋅ (9) Karena BD = BF dan OD = OF maka segiempat BDOF adalah layang-layang. Jadi, BO ⊥ DF. DF = 2 BF sin β ⋅⋅⋅⋅⋅⋅⋅⋅⋅⋅⋅⋅⋅⋅⋅⋅⋅⋅⋅⋅⋅⋅ (10) ∠BFD = 90o − β sehingga ∠OFD = β ∠GFD = ∠OFE + ∠OFD = (90o − ( γ + β)) + ( β) = 90o − γ ⋅⋅⋅⋅⋅⋅⋅⋅⋅⋅⋅⋅⋅⋅⋅⋅⋅⋅⋅⋅⋅⋅⋅⋅⋅⋅⋅ (11) FG = DF cos ∠GFD = (2 BF sin β)(cos (90o − γ) FG = 2 sin γ sin β BF ⋅⋅⋅⋅⋅⋅⋅⋅⋅⋅⋅⋅⋅⋅⋅⋅⋅⋅⋅⋅⋅⋅⋅ (12) Dari persamaan (9) dan (12) dapat disimpulkan bahwa ∴ Terbukti bahwa FG EG FG BF sehingga . = = BF CE EG CE FG BF = EG CE SMA Negeri 5 Bengkulu Eddy Hermanto, ST 265 Olimpiade Matematika Tk Provinsi 2008 Solusi Bagian Kedua 4. Alt ernat if 1 : Andaikan bahwa t idak ada t iga bilangan berdekat an yang j umlahnya lebih besar dari 15. Jika t erdapat t iga bilangan dengan dua diant aranya adalah 7, 8 at au 9 maka ket iga bilangan t ersebut akan memiliki j umlah lebih dari 15. Maka haruslah t erdapat dua bilangan di ant ara 7, 8 dan 9. Kemungkinan susunan hanya ada 1, yait u : Rat a-rat a enam bilangan 1, 2, 3, 4, 5 dan 6 adalah 3, 5. Maka maks (A + B, C + D, E + F) ≥ 7. • Jika maks (A + B, C + D, E + F) = 7 maka A + B = C + D = E + F = 7 Maka 9 j ika dipasangkan dengan salah sat u dari pasangan (A, B), (C, D) at au (E, F) akan membent uk t iga bilangan yang j umlahnya lebih dari 15. Kont radiksi dengan anggapan semula. • Jika maks (A + B, C + D, E + F) > 7 maka maks (A + B, C + D, E + F) ≥ 8 Pasangan bilangan yang memiliki nilai maks t ersebut past i akan berdekat an dengan 8 at au 9 yang penj umlahan ket iga bilangan t ersebut akan bernilai lebih besar dari 15. Kont radiksi dengan anggapan semula. ∴ Terbukti bahwa ada tiga bilangan berdekat an yang j umlahnya lebih besar dari 15. Alt ernat if 2 : Misalkan ai ∈ {1, 2, 3, ⋅⋅⋅, 9} unt uk i = 1, 2, ⋅⋅⋅, 9. Ke-9 bilangan ai , i = 1, 2 ⋅⋅⋅, 9 disusun sebagai berikut . Misalkan S1 = a1 + a2 + a3 S2 = a2 + a3 + a4 M S9 = a9 + a1 + a2 Dengan demikian S1 + S2 + ⋅⋅⋅ + S9 = 3(a1 + a2 + ⋅⋅⋅ + a9) = 3 ⋅ 45 = 135 Karena a1 ≠ a4 maka S1 ≠ S2. Andaikan t idak ada 3 bilangan berdekat an yang j umlahnya lebih besar dari 15, yait u Si ≤ 15 unt uk semua i = 1, 2, ⋅⋅⋅, 9. Karena S1 ≠ S2 maka S1 at au S2 kurang dari 15. Akibat nya S1 + S2 + ⋅⋅⋅ + S9 < 9 x 15 = 135. Kont radiksi. ∴ Terbukti bahwa ada tiga bilangan berdekat an yang j umlahnya lebih besar dari 15. SMA Negeri 5 Bengkulu Eddy Hermanto, ST 266 Solusi Olimpiade Matematika Tk Provinsi 2008 Bagian Kedua 5. Sebuah bilangan akan habis dibagi 3 apabila penj umlahan angka-angkanya habis dibagi 3. Ada 4 angka/ digit yang habis dibagi 3 dan masing-masing ada 3 angka/ digit yang bersisa 1 at au 2 j ika dibagi 3. Misalkan bil angan palindrom t ersebut adalah abcba. Penj umlahan angka = 2(a + b) + c. Karena angka pert ama t idak boleh 0 maka banyaknya cara memilih digit a ≡ 0 (mod 3) hanya ada 3 kemungkinan. • Jika c ≡ 0 (mod 3) Maka 2(a + b) ≡ 0 (mod 3) sehingga a + b ≡ 0 (mod 3) Tiga kemungkinan pasangan (a, b) adalah a ≡ 0 (mod 3) dan b ≡ 0 (mod 3), a ≡ 1 (mod 3) dan b ≡ 2 (mod 3) at au a ≡ 2 (mod 3) dan b ≡ 1 (mod 3) Banyaknya cara memilih digit c adalah 4. Maka banyaknya cara memilih bilangan palindrom j ika c ≡ 0 (mod 3) = 4 ⋅ (3 ⋅ 4 + 3 ⋅ 3 + 3 ⋅ 3) Maka banyaknya cara memilih bilangan palindrom j ika c ≡ 0 (mod 3) = 120. • Jika c ≡ 1 (mod 3) Maka 2(a + b) ≡ 2 (mod 3) sehingga a + b ≡ 1 (mod 3) Tiga kemungkinan pasangan (a, b) adalah a ≡ 0 (mod 3) dan b ≡ 1 (mod 3), a ≡ 1 (mod 3) dan b ≡ 0 (mod 3) at au a ≡ 2 (mod 3) dan b ≡ 2 (mod 3) Banyaknya cara memilih digit c adalah 3. Maka banyaknya cara memilih bilangan palindrom j ika c ≡ 1 (mod 3) = 3 ⋅ (3 ⋅ 3 + 3 ⋅ 4 + 3 ⋅ 3) Maka banyaknya cara memilih bilangan palindrom j ika c ≡ 1 (mod 3) = 90. • Jika c ≡ 2 (mod 3) Maka 2(a + b) ≡ 1 (mod 3) sehingga a + b ≡ 2 (mod 3) Tiga kemungkinan pasangan (a, b) adalah a ≡ 0 (mod 3) dan b ≡ 2 (mod 3), a ≡ 1 (mod 3) dan b ≡ 1 (mod 3) at au a ≡ 2 (mod 3) dan b ≡ 0 (mod 3) Banyaknya cara memilih digit c adalah 3. Maka banyaknya cara memilih bilangan palindrom j ika c ≡ 2 (mod 3) = 3 ⋅ (3 ⋅ 3 + 3 ⋅ 3 + 3 ⋅ 4) Maka banyaknya cara memilih bilangan palindrom j ika c ≡ 2 (mod 3) = 90. Banyaknya bilangan palindrom yang memenuhi adalah 120 + 90 + 90 = 300. ∴ Banyaknya bilangan palindrom 5-angka yang habis dibagi 3 adalah 300 . SMA Negeri 5 Bengkulu Eddy Hermanto, ST 267 SELEKSI TIM OLIMPIADE MATEMATIKA INDONESIA 2009 OLIMPIADE SAINS NASIONAL 2008 MAKASSAR (SULAWESI SELATAN), 8 – 14 AGUSTUS 2008 Bidang Mat emat ika Hari Pert ama Waktu : 4 Jam DEPARTEMEN PENDIDIKAN NASIONAL DIREKTORAT JENDERAL MANAJEMEN PENDIDIKAN DASAR DAN MENENGAH DIREKTORAT PEMBINAAN SEKOLAH MENENGAH ATAS TAHUN 2008 268 OLIMPIADE SAINS NASIONAL 2008 8 – 14 AGUSTUS 2008 MAKASSAR, SULAWESI SELATAN BI DAN G : MATEMATI KA HARI PERTAMA WAKTU : 4 JAM 1. Diberikan segit iga ABC. Tit ik-t it ik D, E, dan F di luar segit iga ABC sedemikian sehingga segit iga ABD, segit iga BCE, dan segit iga CAF adalah segit iga sama sisi. Bukt ikan bahwa ket iga lingkaran luar segit iga t ersebut berpot ongan di sat u t it ik. 2. Bukt ikan bahwa unt uk x dan y bilangan real posit if , berlaku 1 + 1 (1 + x ) (1 + y ) 2 2 ≥ 2 x+ y+2 3. Carilah semua bilangan asli yang dapat dinyat akan dalam bent uk a+b b+c c+a + + c a b unt uk suat u a, b, dan c bilangan asli dengan FPB (a, b) = FPB (b, c) = FPB (c, a) = 1 4. Misalkan A = {1, 2, ⋅⋅⋅, 2008} (a) Tent ukan cacah subhimpunan dari A yang hasilkali semua anggot anya habis dibagi 7. (b) Misalkan N(i) menyat akan cacah subhimpunan dari A yang j umlah semua anggot anya bersisa I j ika dibagi 7. Bukt ikan bahwa 7 ∑ (− 1) N (i) = 0 i i =0 269 SELEKSI TIM OLIMPIADE MATEMATIKA INDONESIA 2009 OLIMPIADE SAINS NASIONAL 2008 MAKASSAR (SULAWESI SELATAN), 8 – 14 AGUSTUS 2008 Bidang Mat emat ika Hari Kedua Waktu : 4 Jam DEPARTEMEN PENDIDIKAN NASIONAL DIREKTORAT JENDERAL MANAJEMEN PENDIDIKAN DASAR DAN MENENGAH DIREKTORAT PEMBINAAN SEKOLAH MENENGAH ATAS TAHUN 2008 270 OLIMPIADE SAINS NASIONAL 2008 8 – 14 AGUSTUS 2008 MAKASSAR, SULAWESI SELATAN BI DAN G : MATEMATI KA HARI KEDUA WAKTU : 4 JAM 5. Misalkan m, n > 1 bilangan-bilangan bulat sedemikian hingga n membagi 4m − 1 dan 2m membagi n − 1. Haruskah n = 2m + 1 ? Jelaskan. 6. Ada 21 orang berhubungan secara rahasia dengan menggunakan f rekuensi gelombang radio yang berbeda. Ada pasangan dua orang yang dapat berhubungan, mungkin ada yang t idak dapat . Set iap pasang yang berhubungan hanya menggunakan sat u f rekuensi t ert ent u yang berbeda dengan f rekuensi yang digunakan pasangan lain. Set iap t iga orang selalu ada dua orang di ant aranya yang t idak dapat berhubungan. Tent ukan banyak maksimum f rekuensi berbeda yang diperlukan dan j elaskan. 7. Diberikan segit iga ABC dengan panj ang sisi-sisinya a, b, dan c. Garis-garis singgung lingkaran dalam segit iga ABC yang sej aj ar dengan sisi-sisi segit iga ABC membent uk t iga segit iga kecil. Pada masing-masing segit iga kecil dibuat lingkaran dalam. Bukt ikan bahwa j umlah luas dari lingkaran dalam segit iga ABC dan ket iga lingkaran dalam segit iga kecil adalah π (a 2 + b 2 + c 2 )(b + c − a )(c + a − b )(a + b − c ) (a + b + c )3 8. Tent ukan semua f ungsi f : N Æ N yang memenuhi f (mn) + f (m + n) = f (m)f (n) + 1 unt uk semua m, n ∈ N. 271 SELEKSI TIM OLIMPIADE MATEMATIKA INDONESIA 2009 OLIMPIADE SAINS NASIONAL 2008 MAKASSAR (SULAWESI SELATAN), 8 – 14 AGUSTUS 2008 Prestasi itu diraih bukan didapat !!! SOLUSI SOAL Bidang Mat emat ika Disusun oleh : Eddy Hermant o, ST 272 Olimpiade Sains Nasional 2008 Solusi Bidang : Mat emat ika 1. Misalkan lingkaran luar ∆ACF dan lingkaran luar ∆BCE berpot ongan di t it ik C dan K. Karena t erlet ak pada sat u lingkaran, segi empat AKCF adalah segiempat t ali busur. Maka ∠AKC = 180o − ∠AFC = 120o. Karena t erlet ak pada sat u lingkaran, segi empat BKCE adalah segiempat t ali busur. Maka ∠BKC = 180o − ∠BEC = 120o. Jadi, ∠AKB = 360o − ∠AKC − ∠BKC = 120o. Karena ∠AKB + ∠ADB = 120o + 60o = 180o maka segiempat BKAD adalah segiempat t alibusur. Jadi ada sebuah lingkaran yang melalui t it ik D, B, K dan A. Maka lingkaran luar ∆ACF, lingkaran luar ∆BCE dan lingkaran luar ∆ABD melalui t it ik K. ∴ Terbukt i bahwa ket iga lingkaran luar segit iga ACF, BCE dan ABD berpot ongan di sat u t it ik 2. Alt ernat if 1 : Dengan AM-GM maka x +1 ≥ 2 x y +1 ≥ 2 y ( x + y + 2 + ( x + 1) + ( y + 1) x + y + 2 + 2 x + 2 y 1 + x x+ y+2= ≥ = 2 2 ) + (1 + y ) 2 2 2 Dengan AM-HM maka (1 + x ) + (1 + y ) x+ y+2≥ 2 2 2 ≥ 2 1 ∴ 1 + 1 (1 + x ) (1 + y ) 2 1 + (1 + x ) (1 + y ) 2 2 ≥ 2 2 (t erbukt i) x+ y+2 SMA Negeri 5 Bengkulu Eddy Hermanto, ST 273 Olimpiade Sains Nasional 2008 Solusi Bidang : Mat emat ika Alt ernat if 2 : Dengan ket aksamaan AM-GM didapat x +1 ≥ 2 x ⋅⋅⋅⋅⋅⋅⋅⋅⋅⋅⋅⋅⋅⋅⋅⋅⋅⋅ (1) Tanda kesamaan t erj adi j ika x = 1 (1 + x ) 2 1 (1 + x ) 2 = 1 + 2 x + x ≤ 2( x + 1) maka 1 ⋅⋅⋅⋅⋅⋅⋅⋅⋅⋅⋅⋅⋅⋅⋅⋅⋅⋅⋅⋅⋅⋅⋅⋅ (2) ≥ 2( x + 1) Dengan cara yang sama didapat 1 (1 + y ) ≥ 1 2( y + 1) 1 + 1 2 (1 + x ) (1 + y ) 2 2 ⋅⋅⋅⋅⋅⋅⋅⋅⋅⋅⋅⋅⋅⋅⋅⋅⋅⋅⋅⋅⋅⋅⋅⋅ (3) maka ≥ 1⎛ 1 1 ⎞ ⎜⎜ ⎟ + 2 ⎝ x + 1 y + 1 ⎟⎠ Dengan ket aksamaan AM-HM didapat 1 1 4 4 + ≥ = maka x +1 y +1 x +1+ y +1 x + y + 2 1 1 2 + ≥ ∴ (t erbukt i) 2 2 x+ y+2 1+ x 1+ y ( ) ( ) 3. Tanpa mengurangi keumuman, misalkan a = maks (a, b, c) a+b b+c c+a = k unt uk suat u bilangan asli k. + + c a b a + b b + c c + a ab(a + b ) + ac(a + c ) + bc (b + c ) = k unt uk suat u bilangan asli k. + + = c a b abc Misalkan j uga Karena c⏐ac(a + c) + bc(b + c) maka c ⏐ab(a + b) Karena FPB(a, c) = FPB(b, c) = 1 maka c ⏐(a + b) Dengan cara yang sama didapat b ⏐(a + c) a⏐(b + c) Karena b ≤ a dan c ≤ a maka a ≤ b + c ≤ 2a • Kasus 1, b + c = a Karena b ⏐(a + c) maka b ⏐2a − b Jadi b ⏐2a Karena FPB(a, b) = 1 maka b ⏐2. Jadi, b = 1 at au 2 Karena c⏐(a + b) maka c ⏐2a − c Jadi c ⏐2a Karena FPB(a, c) = 1 maka c ⏐2. Jadi, c = 1 at au 2 SMA Negeri 5 Bengkulu Eddy Hermanto, ST 274 Olimpiade Sains Nasional 2008 Solusi Bidang : Mat emat ika ¾ Jika b = 1 dan c = 1 maka a = 2 a+b b+c c+a + + =7 c a b ¾ Jika b = 1 dan c = 2 at au b = 2 dan c = 1 maka a = 3 a+b b+c c+a + + =8 c a b • ¾ Jika b = 2 dan c = 2 t idak memenuhi bahwa FPB(b, c) = 1 Kasus 2, b + c = 2a Kesamaan hanya t erj adi j ika a = b dan a = c. Karena FPB(a, b) = FPB(a, c) = FPB(b, c) = 1 maka a = b = c = 1 a+b b+c c+a + + =6 c a b a+b b+c c+a ∴ Nilai dari adalah 6, 7 atau 8 . + + c a b 4. A = {1, 2, ⋅⋅⋅, 2008} (a) Karena cacah anggot a A sebanyak 2008 maka cacah subhimpunan A = 22008. Jika suat u himpunan memiliki anggot a yang merupakan kelipat an 7 maka hasil kali semua anggot anya merupakan kelipat an 7. Jadi agar hasil kali semua anggot a dari suat u himpunan t idak habis dibagi 7 maka anggot anya t idak ada yang merupakan kelipat an 7. Banyaknya anggot a A yang merupakan kelipat an 7 ada 286, yait u 7, 14, 28, ⋅⋅⋅, 2002. Jadi cacah subhimpunan dari A yang hasilkali anggot anya habis dibagi 7 adalah 22008 − 22008−286 ∴ Cacah subhimpunan dari A yang hasilkali anggot anya habis dibagi 7 adalah 2 2008 − 2 1722 . (b) Misalkan x i ∈ A maka 2009 − x i ∈ A. Akan dibukt ikan bahwa j ika ada sebanyak k bilangan yang merupakan anggot a A yang memenuhi j umlah k bilangan t ersebut bersisa i j ika dibagi 7 maka ada sebanyak k bilangan yang j uga merupakan anggot a A dan memenuhi j umlah k bilangan t ersebut akan bersisa 7 − i j ika dibagi 7. Misalkan ada sebanyak k bilangan yang merupakan anggot a A dan memenuhi x 1 + x 2 + ⋅⋅⋅ + x k = 7p + i unt uk suat u bilangan asli p. (2009 − x 1) + x 1 + (2009 − x 2) + x 2 + ⋅⋅⋅ + (2009 − x k) + x k = 2009k = 7m unt uk suat u m ∈ asli. (2009 − x 1) + (2009 − x 2) + ⋅⋅⋅ + (2009 − x k) = 7n + 7− i unt uk suat u bilangan asli n. Jadi ada k bilangan 2009 − x i unt uk i = 1, 2, ⋅⋅⋅, k yang memenuhi j umlah k bilangan t ersebut bersisa 7 − i j ika dibagi 7. Terbukt i bahwa N(i) = N(7 − i) 7 ∑ (− 1) N (i) = N (0) − N (1) + N (2) − N (3) + N (4) − N (5) + N (6) − N (7) = 0 i i =0 ∴ Terbukt i bahwa 7 ∑ (− 1) N (i) = 0 i i =0 SMA Negeri 5 Bengkulu Eddy Hermanto, ST 275 Solusi Olimpiade Sains Nasional 2008 Bidang : Mat emat ika 5. Karena 2m⏐n − 1 maka n = k ⋅ 2m + 1 unt uk suat u bilangan asli k. Karena n⏐4m − 1 maka n ≤ 4m − 1 k ⋅ 2m + 1 ≤ 4m − 1 < 4m + 1 Maka k < 2m ⋅⋅⋅⋅⋅⋅⋅⋅⋅⋅⋅⋅⋅⋅⋅⋅⋅⋅⋅⋅⋅⋅⋅⋅⋅⋅⋅⋅⋅⋅⋅ (1) Karena n ⏐4m − 1 maka n⏐(4m − 1) ⋅ k 2 = (k ⋅ 2m ) 2 − k 2. Sehingga n⏐(n − 1) 2 − k 2 = n2 − 2n + 1 − k 2. Karena n⏐n2 − 2n maka n⏐k 2 − 1. Jadi, n ≤ k 2 − 1 unt uk k ≠ 1 ⋅⋅⋅⋅⋅⋅⋅⋅⋅⋅⋅⋅⋅⋅⋅⋅⋅⋅⋅⋅⋅⋅ (2) n ≤ k 2 − 1 < k 2 < k ⋅ 2m < k ⋅ 2m + 1 = n kont radiksi unt uk k ≠ 1. Jika k = 1 maka n = 2m + 1 yang memenuhi 2m ⏐n − 1 dan n ⏐4m − 1. ∴ Jadi, haruslah n = 2 m + 1 . 6. Misalkan t erdapat 3 orang di ant aranya , yait u A, B dan C dan memiliki hubungan t epat ada dua f rekuensi di ant ara mereka dan yang berhubungan adalah AB dan AC. Tent unya kit a dapat membagi ket iga orang ini menj adi dua kelompok, misalkan A kelompok merah sert a B dan C pada kelompok put ih sehingga ket iganya hanya berhubungan j ika berbeda kelompok. Misalkan j uga D berhubungan dengan A maka t ent unya A t idak dapat berhubungan C sehingga A dapat dimasukkan ke dalam kelompok yang berbeda dengan A. Misalkan j uga E berhubungan dengan C maka t ent unya E dapat berhubungan dengan B t et api t idak dapat berhubungan dengan A sehingga E dapat dimasukkan ke dalam kelompok yang sama dengan A. Jadi, kit a dapat membagi 21 orang t ersebut ke dalam dua kelompok sehingga yang dapat berhubungan hanya j ika berbeda kelompok. Misalkan banyaknya anggot a masing-masing kelompok adalah k dan 21 − k. 212 ⎛ 21 ⎞ Banyaknya f rekuensi yang mungkin = k(21 − k) = − ⎜k − ⎟ 4 ⎝ 2⎠ 2 Karena k bilangan asli Maka banyaknya f rekuensi akan maksimal adalah j ika k = 10 at au 11. ∴ Banyaknya maksimum f rekuensi berbeda yang diperlukan = 10 ⋅ 11 = 110 . 7. Perhat ikan gambar ! Karena DE sej aj ar BC maka ∆ADE sebangun dengan ∆ABC. SMA Negeri 5 Bengkulu Eddy Hermanto, ST 276 Olimpiade Sains Nasional 2008 Solusi Bidang : Mat emat ika Misalkan j ari-j ari lingkaran dalam ∆ABC = r, j ari-j ari lingkaran dalam ∆ADE = r 1, j ari-j ari lingkaran dalam ∆BFG = r 2 dan j ari-j ari lingkaran dalam ∆CHJ = r 3. Misalkan j uga j arak dari A ke BC = t A dan j arak dari A ke DE = t 1. Misalkan s = ½ (a + b + c). 1 r (a + b + c ) = s (s − a )(s − b )(s − c ) . 2 (a + b + c )(a + b − c )(a + c − b )(b + c − a ) 2(a + b + c ) Luas ∆ABC = r= 1 t A a = s (s − a )(s − b )(s − c ) 2 (a + b + c )(a + b − c )(a + c − b )(b + c − a ) tA = 2a Karena j arak dari A ke DE = t 1 maka t 1 = t A − 2r Karena ∆ADE sebangun dengan ∆ABC maka perbandingan sisi j uga merupakan perbandingan garis t inggi kedua segit iga. AD AE DE r1 t1 t A − 2r = = = = = c b a r tA tA Dengan rumus luas ∆ABC maka r1 = r 2a b+c−a r a = sehingga 1 = 1 − = tA a + b + c r a+b+c a+b+c b+c−a r a+b+c Dengan cara yang sama didapat a+c−b r a+b+c a+b−c r r3 = a+b+c r2 = Misalkan j umlah luas dari lingkaran dalam segit iga ABC dan ket iga lingkaran dalam segit iga kecil adalah L. 2 2 2 2 ⎛ (a + b − c)(a + c − b)(b + c − a) ⎞⎛ (a + b + c) + (b + c − a) + (a + c − b) + (a + b − c) ⎞ ⎟ ⎟⎟⎜⎜ L = π r 2 + r12 + r22 + r32 = π ⎜⎜ ⎟ 4(a + b + c) (a + b + c)2 ⎝ ⎠⎝ ⎠ ( ) Misalkan P = (a + b + c) 2 + (b + c − a) 2 + (a + c − b) 2 + (a + b − c) 2 P = (a2 + b 2 + c2 + 2ab + 2ac + 2bc) + (a2 + b 2 + c2 − 2ab − 2ac + 2bc) + (a2 + b 2 + c2 − 2ab + 2ac − 2bc) + (a2 + b 2 + c2 + 2ab − 2ac − 2bc) P = 4a2 + 4b 2 + 4c2 (a + b − c )(a + c − b )(b + c − a ) (4a 2 + 4b 2 + 4c 2 ) 3 4(a + b + c ) π (a 2 + b 2 + c 2 )(b + c − a )(c + a − b )(a + b − c ) L= (a + b + c )3 L =π ∴ Terbukt i bahwa j umlah luas dari lingkaran dalam segit iga ABC dan ket iga lingkaran dalam segit iga kecil adalah π (a 2 + b 2 + c 2 )(b + c − a )(c + a − b )(a + b − c ) (a + b + c )3 SMA Negeri 5 Bengkulu Eddy Hermanto, ST 277 Olimpiade Sains Nasional 2008 Solusi Bidang : Mat emat ika 8. Jika m = n = 1 maka f (1) + f (2) = f (1)f (1) + 1 ⋅⋅⋅⋅⋅⋅⋅⋅⋅⋅⋅⋅⋅⋅⋅⋅⋅⋅⋅⋅⋅⋅⋅⋅⋅⋅⋅⋅⋅⋅⋅⋅ (1) Jika m = 1 dan n = 2 maka f (2) + f (3) = f (1)f (2) + 1 ⋅⋅⋅⋅⋅⋅⋅⋅⋅⋅⋅⋅⋅⋅⋅⋅⋅⋅⋅⋅⋅⋅⋅⋅⋅⋅⋅⋅⋅⋅⋅⋅ (2) Jika m = n = 2 maka 2f (4) = f (2)f (2) + 1 ⋅⋅⋅⋅⋅⋅⋅⋅⋅⋅⋅⋅⋅⋅⋅⋅⋅⋅⋅⋅⋅⋅⋅⋅⋅⋅⋅⋅⋅⋅⋅⋅ (3) Jika m = 1 dan n = 3 maka f (3) + f (4) = f (1)f (3) + 1 ⋅⋅⋅⋅⋅⋅⋅⋅⋅⋅⋅⋅⋅⋅⋅⋅⋅⋅⋅⋅⋅⋅⋅⋅⋅⋅⋅⋅⋅⋅⋅⋅ (4) Misalkan f (1) = a, f (2) = b, f (3) = c dan f (4) = d sert a subt it usikan persamaan (2) dan (3) ke (4). ⎛ (ab + 1 − b ) + ⎜⎜ b ⎝ 2 + 1⎞ ⎟ = a(ab + 1 − b ) + 1 2 ⎟⎠ 2ab + 2 − 2b + b 2 + 1 = 2a2b + 2a − 2ab + 2 2a2b − b 2 − 4ab + 2a + 2b − 1 = 0 ⋅⋅⋅⋅⋅⋅⋅⋅⋅⋅⋅⋅⋅⋅⋅⋅⋅⋅⋅⋅⋅⋅⋅⋅⋅⋅ (5) Dari persamaan (1) didapat a + b = a2 + 1 ⋅⋅⋅⋅⋅⋅⋅⋅⋅⋅⋅⋅⋅⋅⋅⋅⋅⋅⋅⋅⋅⋅⋅⋅⋅⋅⋅⋅ (6) 2a2(a2 − a + 1) − (a2 − a + 1) 2 − 4a(a2 − a + 1) + 2a + 2(a2 − a + 1) − 1 = 0 (2a4 − 2a3 + 2a2) − (a4 + a2 + 1 − 2a3 + 2a2 − 2a) − (4a3 − 4a2 + 4a) + 2a + (2a2 − 2a + 2) − 1 = 0 (2a4 − 2a3 + 2a2) − (a4 − 2a3 + 3a2 − 2a + 1) − (4a3 − 4a2 + 4a) + 2a + (2a2 − 2a + 2) − 1 = 0 a4 − 4a3 + 5a2 − 2a = 0 a(a − 1) 2(a − 2) = 0 Karena f : N Æ N maka nilai f (1) yang mungkin memenuhi hanya 1 at au 2. ¾ Jika f (1) = 1 Unt uk n = 1 maka f (m) + f (m + 1) = f (m)f (1) + 1 Karena f (1) = 1 maka f (m + 1) = 1 Jadi, f (x) = 1 unt uk x ∈ N. Jika f (x) = 1 unt uk x ∈ N maka f (mn) + f (m + n) = 2 dan f (m)f (n) + 1 = 2. Memenuhi. ¾ Jika f (1) = 2 Unt uk n = 1 maka f (m) + f (m + 1) = f (m)f (1) + 1 Karena f (1) = 2 maka f (m + 1) = f (m) + 1 Jika m = 1 maka f (2) = f (1) + 1 Jika m = 2 maka f (3) = f (2) + 1 Jika m = 3 maka f (4) = f (3) + 1 M Jika m = x − 1 unt uk x ∈ N maka f (x) = f (x − 1) + 1 Jumlahkan seluruh persamaan didapat : f (x) = f (1) + x − 1 Karena f (1) = 2 maka f (x) = x + 1 Jadi, f (x) = x + 1 unt uk x ∈ N. Jika f (x) = x + 1 unt uk x ∈ N maka f (mn) + f (m + n) = mn + 1 + m + n + 1 = (m + 1)(n + 1) + 1 dan f (m)f (n) + 1 = (m + 1)(n + 1) + 1. Memenuhi. ∴ Semua f ungsi f : N Æ N yang memenuhi adalah f (x) = 1 dan f (x) = x + 1 unt uk x ∈ N. SMA Negeri 5 Bengkulu Eddy Hermanto, ST 278 SELEKSI OLIMPIADE TINGKAT KABUPATEN/ KOTA TAHUN 2009 TIM OLIMPIADE MATEMATIKA INDONESIA TAHUN 2010 Bidang Mat emat ika Waktu : 2 Jam DEPARTEMEN PENDIDIKAN NASIONAL DIREKTORAT JENDERAL MANAJEMEN PENDIDIKAN DASAR DAN MENENGAH DIREKTORAT PEMBINAAN SEKOLAH MENENGAH ATAS TAHUN 2009 279 OLIMPIADE MATEMATIKA NASIONAL SELEKSI TINGKAT KOTA/ KABUPATEN TAHUN 2009 Isikan hanya j awaban saj a pada l embar j awaban yang disediakan. 1. Banyaknya bilangan asli kurang dari 1000 yang dapat dinyat akan dalam bent uk x 2 − y2 unt uk suat u bilangan ganj il x dan y adalah ⋅⋅⋅⋅⋅⋅⋅ 2. Bilangan bulat posit if t erkecil n dengan n > 2009 sehingga 13 + 2 3 + 33 + L + n 3 n merupakan bilangan bulat adalah ⋅⋅⋅⋅⋅⋅⋅⋅⋅ 3. Banyaknya solusi real x dari persamaan 3(1/ 2+log3 (cos x−sin x )) + 2(log2 (cos x+sin x )) = 2 adalah ⋅⋅⋅⋅ 4. Diberikan f ungsi f : R Æ R sedemikian hingga x 2f (x) + f (1 − x) = 2x − x 4 unt uk semua x ∈ R. Nilai f (2009) adalah ⋅⋅⋅⋅⋅⋅ 5. Banyaknya segit iga siku-siku yang kelilingnya 2009 dan sisi-sisinya bilangan bulat sert a j ari-j ari lingkaran dalamnya j uga bilangan bulat adalah ⋅⋅⋅⋅⋅⋅⋅ ⎛ 2009 ⎞ ⎛ 2009 ⎞ ⎛ 2009 ⎞ ⎟⎟ adalah ⋅⋅⋅⋅⋅⋅⋅⋅ ⎟⎟ + L + ⎜⎜ 2 ⎠ 1004 ⎝ ⎠ ⎟⎟ + ⎜⎜ 6. Nilai eksak dari ⎜⎜ 1 ⎝ ⎠ ⎝ 7. Jika t iga pasang suami ist eri akan menempat i t uj uh kursi yang berj aj ar ke samping dengan syarat semua suami ist eri duduk berdekat an dan t idak ada laki-laki dan perempuan bukan suami ist eri yang duduk berdekat an, maka banyak caranya adalah ⋅⋅⋅⋅⋅⋅⋅ 2009 8. Nilai dari ∑ FPB(k ,7 ) adalah ⋅⋅⋅⋅⋅⋅⋅⋅ k =1 280 9. Banyaknya pasangan bilangan asli (x, y) sehingga x 4 + 4y4 merupakan bilangan prima adalah ⋅⋅⋅⋅⋅ 10. Bilangan real x sehingga pernyat aan x 2 = x j ika dan hanya j ika x 3 = x bernilai salah adalah ⋅⋅⋅⋅⋅ 11. Diket ahui ABC adalah segit iga siku-siku di A dengan AB = 30 cm dan AC = 40 cm. Misalkan AD adalah garis t inggi dari dan E adalah t it ik t engah AD. Nilai dari BE + CE adalah ⋅⋅⋅⋅⋅⋅⋅ 12. Suat u t urnamen diikut i 20 t im, dimana lain. Kemenangan memperoleh poin 1, t erat as memperoleh poin yang sama, berbeda-beda. Jumlah semua bilangan pada klasemen akhir adalah ⋅⋅⋅⋅⋅⋅⋅⋅ set iap t im sedangkan sedangkan yang t idak bert emu sat u kekalahan 0. 17 t im yang muncul pada kali dengan semua t im yang Pada klasemen akhir, 3 t im lain memperoleh poin yang poin yang dimiliki suat u t im 13. Tit ik E t erlet ak di dalam persegi ABCD sedemikian rupa sehingga ABE adalah segit iga sama sisi. Jika panj ang AB = 1 + 3 dan F t it ik pot ong ant ara diagonal BD dengan segmen garis AE, maka luas segit iga ABF sama dengan ⋅⋅⋅⋅⋅⋅ 14. Misalkan f (x) = ( ) 3 + 1 sin y + ( ) 3 − 1 cos y . Nilai maksimum unt uk (f (y)) 2 dimana y bilangan real adalah ⋅⋅⋅⋅⋅⋅ 15. Diberikan persegi ABCD dengan panj ang sisi 10. Misalkan E pada AB dan F pada BD dengan AE = FB = 5. Misalkan P adalah t it ik pot ong CE dan AF. Luas DFPC adalah ⋅⋅⋅⋅⋅⋅⋅⋅⋅ 16. Jika x k +1 = x k + 1 unt uk k = 1, 2, ⋅⋅⋅ dan x 1 = 1 maka x 1 + x 2 + ⋅⋅⋅ + x 400 = ⋅⋅⋅⋅⋅⋅⋅ 2 17. Diberikan segit iga ABC t umpul ( ∠ABC > 90o), AD dan AE membagi sudut BAC sama besar. Panj ang segmen garis BD, DE dan EC bert urut -t urut adalah 2, 3, dan 6. Panj ang t erpendek dari sisi segit iga ABC adalah ⋅⋅⋅⋅⋅⋅ 18. Jika 10999999999 dibagi oleh 7, maka sisanya adalah ⋅⋅⋅⋅⋅⋅⋅⋅⋅ 281 19. Diket ahui A adalah himpunan semua bilangan asli yang habis dibagi 3, t idak habis dibagi 5, dan t idak lebih dari 100. Banyaknya f ungsi f dari himpunan semua bilangan real yang t idak nol ke ⎛x⎞ dalam A yang memenuhi f ⎜⎜ ⎟⎟ = f ( x − y ) adalah ⋅⋅⋅⋅⋅⋅ ⎝ y⎠ 20. Delapan bilangan asli memiliki rat a-rat a 6, 5. Empat dari delapan bilangan t ersebut adalah 4, 5, 7, dan 8. Selisih ant ara bilangan t erbesar dan t erkecil adalah 10. Jika ke delapan bilangan diurut kan dari kecil ke besar, maka banyaknya susunan ada ⋅⋅⋅⋅⋅⋅⋅⋅ . 282 SELEKSI OLIMPIADE TINGKAT KABUPATEN/ KOTA 2009 TIM OLIMPIADE MATEMATIKA INDONESIA 2010 Prestasi itu diraih bukan didapat !!! SOLUSI SOAL Bidang Mat emat ika Disusun oleh : Eddy Hermant o, ST 283 Olimpiade Matematika Tk Kabupaten/ Kota 2009 1. Misalkan x = 2m + 1 dan y = 2n + 1 x 2 − y2 = 4m(m + 1) − 4n(n + 1) m(m + 1) dan n(n + 1) keduanya adalah bilangan genap maka x 2 − y2 merupakan kelipat an 8. Selain it u, 8k = (2k + 1) 2 − (2k − 1) 2 sehingga set iap bilangan kelipat an 8 dapat diubah menj adi selisih kuadrat dua bilangan ganj il. Maka banyaknya bilangan asli kurang dari 1000 yang dapat dinyat akan dalam bent uk x 2 − y2 dengan x dan y adalah bilangan ganj il adalah 1000 − 1 = 124. Tanda −1 sebab 1000 t idak 8 t ermasuk ke dalam bagian ini. ∴ Maka banyaknya bilangan asli kurang dari 1000 yang dapat dinyat akan dalam bent uk x 2 − y2 dengan x dan y adalah bilangan ganj il adalah 124 . 13 + 2 3 + 33 + L + n 3 = n 2. ( ( )) n n +1 2 2 n ⎛ n + 1⎞ = n⎜ ⎟ ⎝ 2 ⎠ 2 n( n2+1 ) merupakan bilangan kuadrat maka haruslah n merupakan bilangan kuadrat 2 Agar sempurna. Bilangan kuadrat t erdekat set elah 2009 adalah 452 = 2025. ∴ Nilai n > 2009 yang memenuhi 3. 3(log3 3 (cos x −sin x ) ( ) 13 + 23 + 33 +L+ n 3 n merupakan bilangan kuadrat adalah 2025 . ) + 2(log2 (cos x+sin x )) = 2 3 (cos x − sin x ) + (cos x + sin x ) = 2 3 + 1 cos x − ( ) 3 − 1 sin x = 2 ⎛ 6 ⎛ 6 2⎞ 2⎞ 1 ⎜ ⎟ cos x − ⎜ ⎟ sin x = + − ⎜ 4 ⎟ ⎜ ⎟ 4 ⎠ 4 ⎠ 2 ⎝ ⎝ 4 sin 105o = 6 2 6 2 sedangkan cos 105o = − + + 4 4 4 4 sin 105o cos x + cos 105o sin x = sin 30o sin (105o + x) = sin 30o 105o + x = 30o + k ⋅ 360o at au 105o + x = 180o − 30o + k ⋅ 360o x = 285o + n ⋅ 360o at au x = 45o + k ⋅ 360o Unt uk x = 45o + n ⋅ 360o akan menyebabkan cos x − sin x = 0 sehingga t idak memenuhi persyarat an bahwa cos x − sin x > 0. Maka : Tet api saat x = 285o + n ⋅ 360o maka akan menyebabkan cos x > sin x yang menyebabkan t erpenuhinya syarat cos x − sin x > 0. Karena ada t ak berhingga nilai n yang mungkin maka banyaknya solusi real x yang memenuhi adalah t ak berhingga. ∴ Jadi, banyaknya solusi real x yang memenuhi adalah t ak berhingga. SMA Negeri 5 Bengkulu Eddy Hermanto, ST 284 Olimpiade Matematika Tk Kabupaten/ Kota 2009 4. x 2f (x) + f (1 − x) = 2x − x 4 k 2f (k) + f (1 − k) = 2k − k 4 ⋅⋅⋅⋅⋅⋅⋅⋅⋅⋅⋅⋅⋅⋅⋅⋅⋅⋅⋅⋅⋅⋅⋅⋅⋅⋅⋅ (1) (1 − k) 2f (1 − k) + f (k) = 2(1 − k) − (1 − k) 4 ⋅⋅⋅⋅⋅⋅⋅⋅⋅⋅⋅⋅⋅⋅⋅⋅⋅⋅⋅⋅⋅⋅⋅ (2) Kalikan persamaan (1) dengan (1 − k) 2 lalu kurangkan dengan persamaan (2) didapat (k 2(k − 1) 2 − 1) f (k) = 2k(k − 1) 2 − k 4(k − 1) 2 − 2(1 − k) + (k − 1) 4 (k 2(k − 1) 2 − 1) f (k) = 2k 3 − 4k 2 + 2k − k 4(k − 1) 2 − 2 + 2k + k 2(k − 1) 2 − 2k(k − 1) 2 + (k − 1) 2 (k 2(k − 1) 2 − 1) f (k) = 2k 3 − 4k 2 + 4k − k 4(k − 1) 2 − 2 + k 2(k − 1) 2 − 2k(k − 1) 2 + (k − 1) 2 (k 2(k − 1) 2 − 1) f (k) = 2k 3 − 4k 2 + 4k − k 4(k − 1) 2 − 2 + k 2(k − 1) 2 − 2k 3 + 4k 2 − 2k + k 2 − 2k + 1 (k 2(k − 1) 2 − 1) f (k) = k 2 − 1 − k 4(k − 1) 2 − 1 + k 2(k − 1) 2 (k 2(k − 1) 2 − 1) f (k) = (k 2(k − 1) 2 − 1)(1 − k 2) f (k) = 1 − k 2 ∴ f (2009) = 1 − 2009 2 . 5. Akan dibukt ikan bahwa t idak ada segit iga siku-siku dengan sisi-sisinya bilangan bulat dan memenuhi bahwa kelilingnya merupakan bilangan ganj il. Alt ernat if 1 : Misalkan sisi-sisi segit iga t ersebut adalah a, b dan 2009 − a − b a2 + b 2 = (2009 − a − b) 2 2ab − 4018a − 4018b + 20092 = 0 Karena 2ab − 4018a − 4018b genap sedangkan 20092 ganj il maka t idak ada bilangan bulat a dan b yang memenuhi 2ab − 4018a − 4018b + 20092 = 0. Jadi t idak ada segit iga yang demikian. Alt ernat if 2 : Misalkan sisi-sisi siku-sikunya adalah a dan b sedangkan hipot enusa c. Karena 2009 ganj il maka sisi-sisi segit iga t ersebut haruslah ket iga-t iganya ganj il at au t epat sat u yang ganj il. • Jika ket iga-t iganya ganj il Karena a2 + b 2 ≡ 2 (mod 4) maka t idak mungkin ada hipot enusa yang memenuhi. • Jika t epat sat u yang ganj il Jika yang ganj il t ersebut merupakan hipot enusa maka a2 + b 2 ≡ 0 (mod 4) sehingga hipot enusa haruslah merupakan bilangan genap. Kont radiksi. Jika hipot enusa genap maka a2 + b 2 ≡ 1 (mod 4) sehingga hipot enusa haruslah merupakan bilangan ganj il. Kont radiksi. Maka t idak ada segit iga siku-siku dengan sisi-sisinya bilangan bulat dan memenuhi bahwa kelilingnya sama dengan 2009. ∴ Jadi, banyaknya segit iga yang memenuhi adalah 0 . 6. ⎛ 2009 ⎞ ⎛ 2009 ⎞ ⎜⎜ ⎟⎟ = ⎜⎜ ⎟⎟ ⎝ k ⎠ ⎝ 2009 − k ⎠ ⎛ 2009 ⎞ ⎛ 2009 ⎞ ⎛ 2009 ⎞ ⎟⎟ + L + ⎜⎜ ⎟⎟ = 2 2009 ⎜⎜ ⎟⎟ + ⎜⎜ ⎝ 2009 ⎠ ⎝ 0 ⎠ ⎝ 1 ⎠ ⎛ 2009 ⎞ 2 2009 ⎛ 2009 ⎞ ⎛ 2009 ⎞ ⎟⎟ + L + ⎜⎜ ⎟⎟ = ⎜⎜ ⎟⎟ + ⎜⎜ = 2 2008 2 ⎝ 1004 ⎠ ⎝ 0 ⎠ ⎝ 1 ⎠ SMA Negeri 5 Bengkulu Eddy Hermanto, ST 285 Olimpiade Matematika Tk Kabupaten/ Kota 2009 ⎛ 2009 ⎞ ⎛ 2009 ⎞ ⎛ 2009 ⎞ ⎟⎟ + L + ⎜⎜ ⎟⎟ = 2 2008 − 1 ⎜⎜ ⎟⎟ + ⎜⎜ ⎝ 1004 ⎠ ⎝ 1 ⎠ ⎝ 2 ⎠ ⎛ 2009 ⎞ ⎛ 2009 ⎞ ⎛ 2009 ⎞ ⎟⎟ + L + ⎜⎜ ⎟⎟ = 2 2008 − 1 ⎟⎟ + ⎜⎜ ∴ Jadi, ⎜⎜ ⎝ 1004 ⎠ ⎝ 1 ⎠ ⎝ 2 ⎠ 7. Misalkan penomoran kursi urut dari kiri ke kanan. Ada t iga bent uk susunan yang mungkin. • Susunannya berbent uk SIISSI at au ISSIIS Karena suami ist eri harus berdekat an maka posisi kursi kosong haruslah kursi nomor 1, 3, 5 at au 7. Banyaknya susunan masing-masing bent uk t anpa memperhit ungkan kursi kosong adalah 3! . Jadi, banyaknya susunan yang mungkin adalah 4 ⋅ 2 ⋅ 3! = 48. • Susunannya berbent uk SISIIS at au ISISSI Karena t idak ada laki-laki dan perempuan yang bukan suami ist eri yang duduk berdekat an sert a suami ist eri harus berdekat an maka posisi kursi kosong haruslah kursi nomor 3. Banyaknya susunan yang mungkin adalah 1 ⋅ 2 ⋅ 3! = 12 • Susunannya berbent uk SIISIS at au ISSISI Bent uk di at as adalah percerminan bent uk kedua. Banyaknya susunan yang mungkin adalah 1 ⋅ 2 ⋅ 3! = 12 Maka banyaknya susunan yang mungkin adalah = 48 + 12 + 12 = 72. ∴ Maka banyaknya susunan yang mungkin adalah = 72. 8. FPB (k, 7) = 1 j ika k bukan merupakan kelipat an 7 sedangkan FPB (k, 7) = 7 j ika k kelipat an 7. Bilangan asli dari 1 sampai 2009 yang habis dibagi 7 banyaknya ada ⎣2009/ 7⎦ = 287. Bilangan asli dari 1 sampai 2009 yang t idak habis dibagi 7 banyaknya ada 2009 − 287 = 1722 2009 ∑ FPB(k ,7 ) = 287 ⋅ 7 + 1722 ⋅ 1 = 3731 k =1 ∴ 2009 ∑ FPB(k ,7 ) = 3731 k =1 9. x 4 + 4y4 = (x 2 + 2y2) 2 − (2xy) 2 = (x 2 + 2y2 − 2xy)(x 2 + 2y2 + 2xy) x 4 + 4y4 = ((x − y) 2 + y2)(x 2 + 2y2 + 2xy) Suku kedua persamaan di at as selalu lebih dari sat u. Agar x 4 + 4y4 prima maka (x − y) 2 + y2 = 1 yang t erpenuhi hanya j ika x = y dan y = 1. Maka pasangan (x, y) yang memenuhi hanya (1, 1) ∴ Banyaknya pasangan (x, y) bilangan asli sehingga x 4 + 4y4 adalah bilangan prima ada 1 . 10. Agar bernilai salah maka x 2 = x benar dan x 3 = x salah at au x 2 = x salah dan x 3 = x benar. Jika x 2 = x benar maka nilai x yang memenuhi adalah 0 at au 1. Tet api x = 0 at au x = 1 akan membuat x 3 = x benar. Jika x 3 = x benar maka nilai x yang memenuhi adalah 0 at au 1 at au −1. Tet api x = 0 at au x = 1 akan membuat x 3 = x benar sedangkan x = −1 akan membuat x 2 = x salah. ∴ Jadi bilangan real x yang memenuhi adalah x = −1 . SMA Negeri 5 Bengkulu Eddy Hermanto, ST 286 Olimpiade Matematika Tk Kabupaten/ Kota 2009 11. Jelas bahwa panj ang BC = 50 cm. BD = 30 ⋅ 30 = 18 cm. 50 DC = 50 − 18 = 32 cm. AD = 30 ⋅ 40 = 24 cm 50 DE = 12 cm BE2 = BD2 + DE2 = 182 + 122 = 62 ⋅ 13 CE2 = CD2 + DE2 = 322 + 122 = 42 ⋅ 73 BE + CE = 4 73 + 6 13 ∴ Nilai dari BE + CE adalah 4 73 + 6 13 cm. 12. Nilai yang mungkin bagi pesert a adalah 0, 1, 2, ⋅⋅⋅, 19. Jumlah seluruh pert andingan = 20C2 = 190. Karena dalam sat u pert andingan hanya ada nilai 1 at au 0 maka nilai t ot al seluruh pesert a haruslah sama dengan 190. Misalkan nilai t ot al 17 pesert a t erbawah adalah M. Mmin = 0 + 1 + 2 + 3 + ⋅⋅⋅ + 16 = 136. Tot al nilai t iga t im t erat as maksimum adalah 190 − 136 = 54. Maka t idak mungkin nilai masing-masing t iga t im t erat as sama dengan 19. Nilai t erkecil masing-masing t iga t im t erat as adalah 17 sebab nilai t erendah t im ke-4 adalah 16. • Jika masing-masing t iga t im t erat as sama dengan 17 Nilai peringkat ke-4 haruslah 16. Maka M = 136. Tet api t ot al nilai seluruh pesert a = 136 + 3 ⋅ 17 = 187 ≠ 190. Tidak memenuhi syarat t ot al nilai seluruh pesert a sama dengan 190. • Jika masing-masing t iga t im t erat as sama dengan 18 Maka M = 190 − 3 ⋅ 18 = 136 = Mmin. Maka nilai-nilai t im peringkat ke-4 sampai 20 adalah 16, 15, 14, 13, ⋅⋅⋅, 0. Jadi nilai yang t idak muncul adalah 17 dan 19. ∴ Jumlah semua bilangan yang t idak muncul pada poin yang dimiliki suat u t im = 36 . SMA Negeri 5 Bengkulu Eddy Hermanto, ST 287 Olimpiade Matematika Tk Kabupaten/ Kota 2009 13. ∠AFB = 180o − ∠BAF − ∠FBA = 180o − 60o − 45o = 75o. Dengan dalil sinus pada segit iga AFB maka : 1+ 3 AF = sin 75° sin 45° 1 sin 75° = 2 1 + 3 . Maka 4 2 AF = 1+ 3 ( ) Luas segit iga ABF = ½ AB ⋅ AF sin 60o ∴ Luas segitiga ABF = ( ) 14. f (x) = 3 + 1 sin y + Alt ernat if 1 : ( 3 . 2 ) 3 − 1 cos y a sin x + b cos x = a 2 + b 2 cos( x − α ) dengan tan α = ( f ( y ))2 = 8 cos 2 ( y − α ) a b Alt ernat if 2 : 6 2 6 2 sedangkan cos 105o = − + + 4 4 4 4 ⎞ ⎛ 6 2⎞ 4 ⎛⎜ ⎛ 6 2⎞ ⎜ ⎟ sin y − ⎜ − ⎟ cos y ⎟ f (x ) = + + ⎜ 4 ⎟ 4 ⎟⎠ 4 ⎟⎠ 2 ⎜⎝ ⎜⎝ 4 ⎝ ⎠ 4 (sin 105° sin y − cos105° cos y ) = − 4 cos( y − 105°) f (x ) = 2 2 2 2 ( f ( y )) = 8 cos ( y − 105°) sin 105o = ∴ Nilai maksimum unt uk (f (y)) 2 adalah 8. SMA Negeri 5 Bengkulu Eddy Hermanto, ST 288 Olimpiade Matematika Tk Kabupaten/ Kota 2009 15. Misalkan koordinat A(0, 0), B(10, 0) maka C(10, 10) dan D(0, 10). ⎛ ⎜ ⎝ Panj ang BF = 5 sedangkan ∠DBA = 45o maka koordinat F ⎜10 − Persamaan garis AF adalah y = 5 2 5 2⎞ ⎟. , 2 2 ⎟⎠ 2 x dan persamaan garis EC adalah y = 2x − 10 4− 2 2 10 4 − 2 130 + 20 2 30 + 40 2 x P sehingga x P = 2 x P − 10 = = dan y P = 23 23 4− 2 8−3 2 ( ) Misalkan [ ABCD] menyat akan luas bangunan ABCD. ⎛ 30 + 40 2 ⎞ 75 + 100 2 1 ⎟ = ⋅ 5 ⋅ ⎜⎜ ⎟ 23 2 23 ⎝ ⎠ ⎛ 20 − 5 2 ⎞ 100 − 25 2 1 ⎟ = [ AFD] = ⋅ 10 ⋅ ⎜ ⎜ ⎟ 2 2 2 ⎝ ⎠ [ AEP] = [ EBC] = 25 [ DFPC] = 100 − [ AEP] − [ AFD] − [ EBC] ∴ Luas DFPC adalah 1000 + 375 2 . 46 Cat at an : Jawaban yang dikirim dari pusat menyat akan bahwa j awaban dari soal ini adalah 55 yang didapat j ika penulisan t it ik sudut nya sebagai berikut (bukt ikan). Tet api, penulisan t it ik sudut t ersebut t idak sesuai dengan kesepakat an umum penulisan t it ik sudut . SMA Negeri 5 Bengkulu Eddy Hermanto, ST 289 Olimpiade Matematika Tk Kabupaten/ Kota 2009 16. x k+1 − x k = ½ Karena selisih dua bilangan berurut an konst an maka soal t ersebut merupakan deret arit mat ika dengan beda sama dengan ½ dan suku pert ama sama dengan 1. x 1 + x 2 + ⋅⋅⋅ + x 400 = 400 ⎛ ⎛ 1 ⎞⎞ ⎜⎜ 2(1) + (400 − 1)⎜ ⎟ ⎟⎟ 2 ⎝ ⎝ 2 ⎠⎠ x 1 + x 2 + ⋅⋅⋅ + x 400 = 40300 ∴ x1 + x2 + ⋅⋅⋅ + x400 = 40300. 17. Perhat ikan gambar. Misalkan ∠CAE = ∠EAD = ∠DAB = α dan panj ang AB = x. Pada ∆EAB, ruas AD adalah garis bagi sehingga 3x EA 3 . = . Maka EA = AB 2 2 Misalkan j uga AD = y. Dengan dalil cosinus maka 9 2 x + y 2 − 32 y + x −2 4 = = cos α 2 xy 3xy 2 2 2 6y2 + 6x 2 − 24 = 9x 2 + 4y2 − 36 2y2 = 3x 2 − 12 ⋅⋅⋅⋅⋅⋅⋅⋅⋅⋅⋅⋅⋅⋅⋅⋅⋅⋅⋅⋅⋅⋅⋅⋅⋅⋅⋅⋅⋅ (1) Pada ∆DAC, karena AE adalah garis bagi maka berlaku AC = 2 AD = 2y Sesuai dalil cosinus pada ∆CAE maka 62 = 4 y 2 + 2 2 2 9 2 ⎛ 3 ⎞⎛ y + x − 2 ⎞ ⎟⎟ x − 2(2 y )⎜ x ⎟⎜⎜ 4 2 xy ⎝ 2 ⎠⎝ ⎠ 144 = 16y2 + 9x 2 − 12(y2 + x2 − 4) 96 = 4y2 − 3y2 Subt it usikan persamaan (1) 96 = 6x 2 − 24 − 3x 2 x = 2 10 Karena ∠ABC > 90o maka sisi t erpanj ang ∆ABC adalah sisi AC. Karena x = 2 10 < 2 ⋅ 4 < 2 + 3 + 6 = 11 = BC maka panj ang sisi yang t erpendek adalah AB = x ∴ Panj ang sisi segit iga ABC yang t erpendek adalah 2 10 SMA Negeri 5 Bengkulu Eddy Hermanto, ST 290 Olimpiade Matematika Tk Kabupaten/ Kota 2009 18. 10999999999 = 1000333333333 = (7 ⋅ 143 − 1) 333333333. 10999999999 ≡ ( −1) 333333333 (mod 7) ≡ −1 (mod 7) 10999999999 dibagi 7 maka akan bersisa 6. ∴ 10999999999 dibagi 7 akan bersisa 6 . 19. Banyaknya bilangan asli yang kurang dari 100 dan habis dibagi 3 ada 33. Banyaknya bilangan asli yang habis dibagi 3 dan habis dibagi 5 sert a kurang dari 100 ada 6. Banyaknya anggot a himpunan A adalah 33 − 6 = 27. Fungsi f dari himpunan semua bilangan real yang t idak nol ke dalam A memenuhi f xy = f ( x − y ) . () Alt ernat if 1 : Ambil x = ab b dan y = unt uk a ≠ 1 sert a a dan b t ak nol. Maka a −1 a −1 f (a) = f (b) Jadi f merupakan f ungsi konst an. Karena banyaknya anggot a himpunan A ada 27 maka banyaknya f ungsi yang memenuhi ada 27. ∴ Banyaknya f ungsi yang memenuhi adalah 27 . Alt ernat if 2 : f (x) = f (2x − x) = f ( 2xx ) = f (2) sehingga f merupakan f ungsi konst an. Karena banyaknya anggot a himpunan A ada 27 maka banyaknya f ungsi yang memenuhi ada 27. ∴ Banyaknya f ungsi yang memenuhi adalah 27 . 20. Karena rat a-rat a delapan bilangan sama dengan 6, 5 maka j umlah kedelapan bilangan = 52. Jumlah empat bilangan yang ada adalah 4 + 5 + 7 + 8 = 24 sehingga j umlah keempat bilangan yang lain sama dengan 28. • Jika bilangan yang t erkecil sama dengan 1 maka bilangan t erbesar sama dengan 11 Jumlah dua bilangan t erakhir = 16. Pasangan yang memenuhi adalah (5, 11), (6, 10), (7, 9) dan (8, 8) yang semuanya ada 4. • Jika bilangan yang t erkecil sama dengan 2 maka bilangan t erbesar sama dengan 12 Jumlah dua bilangan t erakhir = 14. Pasangan yang memenuhi adalah (2, 12), (3, 11), (4, 10), (5, 9), (6, 8) dan (7, 7) yang semuanya ada 6. • Jika bilangan yang t erkecil sama dengan 3 maka bilangan t erbesar sama dengan 13 Jumlah dua bilangan t erakhir = 12. Pasangan yang memenuhi adalah (3, 9), (4, 8), (5, 7) dan (6, 6) yang semuanya ada 4. • Jika bilangan yang t erkecil sama dengan 4 maka bilangan t erbesar sama dengan 14 Jumlah dua bilangan t erakhir = 10. Pasangan yang memenuhi adalah (4, 6) dan (5, 5) yang semuanya ada 2. • Jika bilangan yang t erkecil sama dengan 4 maka bilangan t erbesar sama dengan 14 dengan bilangan 4 t ersebut merupakan salah sat u dari 4 bilangan awal. Jumlah t iga bilangan lain haruslah 14 dan t idak ada salah sat u di ant aranya sama dengan 4. Karena yang t erendah sama dengan 5 maka nilai minimal sama dengan 15. Tidak ada yang memenuhi. Banyaknya susunan = 4 + 6 + 4 + 2 + 0 = 16. ∴ Banyaknya susunan yang mungkin ada 16 . SMA Negeri 5 Bengkulu Eddy Hermanto, ST 291 SELEKSI OLIMPIADE TINGKAT PROVINSI 2009 TIM OLIMPIADE MATEMATIKA INDONESIA 2010 Waktu : 210 Menit DEPARTEMEN PENDIDIKAN NASIONAL DIREKTORAT JENDERAL MANAJEMEN PENDIDIKAN DASAR DAN MENENGAH DIREKTORAT PEMBINAAN SEKOLAH MENENGAH ATAS TAHUN 2009 292 SELEKSI TINGKAT PROVINSI CALON PESERTA OLIMPIADE SAINS NASIONAL 2009 MATEMATIKA SMA/ MA Petunj uk untuk peserta : 1. Tes t erdiri dari dua bagian. Tes bagian pert ama t erdiri dari 20 soal isian singkat dan t es bagian kedua t erdiri dari 5 soal uraian. 2. Wakt u yang disediakan unt uk menyelesaikan semua soal adalah 210 menit . 3. Tuliskan nama, kelas dan asal sekolah Anda di sebelah kanan at as pada set iap halaman. 4. Unt uk soal bagian pert ama : (a) Masing-masing soal bagian pert ama bernilai 1 (sat u) angka. (b) Beberapa pert anyaan dapat memiliki lebih dari sat u j awaban yang benar. Anda dimint a memberikan j awaban yang paling t epat at au persis unt uk pert anyaan sepert i ini. Nilai hanya akan diberikan kepada pemberi j awaban paling t epat at au paling persis. (c) Tuliskan hanya j awaban dari soal yang diberikan. Tuliskan j awaban t ersebut pada kot ak di sebelah kanan set iap soal. 5. Unt uk soal bagian kedua : (a) Masing-masing soal bagian kedua bernilai 7 (t uj uh) angka (b) Anda dimint a menyelesaikan soal yang diberikan secara lengkap. Selain j awaban akhir, Anda dimint a menuliskan semua langkah dan argument asi yang Anda gunakan unt uk sampai kepada j awaban akhir t ersebut . (c) Jika halaman muka t idak cukup, gunakan halaman sebaliknya. 6. Jawaban hendaknya Anda t uliskan dengan menggunakan t int a, bukan pensil. 7. Selama t es, Anda t idak diperkenankan menggunakan buku, cat at an dan alat bant u hit ung. Anda j uga t idak diperkenankan bekerj a sama. 8. Mulailah bekerj a hanya set elah pengawas memberi t anda dan berhent ilah bekerj a segera set elah pengawas memberi t anda. 9. Selamat bekerj a. 293 SELEKSI TINGKAT PROVINSI CALON PESERTA OLIMPIADE SAINS NASIONAL 2009 MATEMATIKA SMA/ MA BAGIAN PERTAMA 1. Tiga dadu berwarna hit am, merah, dan put ih dilempar bersama-sama. Macam hasil lemparan sehingga j umlah ket iga mat a dadu adalah 8 sebanyak ⋅⋅⋅⋅⋅⋅⋅⋅⋅ 2. Banyaknya bilangan real x yang memenuhi persamaan x 4 − 2x 3 + 5x 2 − 176x + 2009 = 0 adalah ⋅⋅⋅⋅⋅⋅ 3. Bilangan rasional a < b < c membent uk barisan hit ung (arit mat ika) dan a b c + + =3 b c a Banyaknya bilangan posit if a yang memenuhi adalah ⋅⋅⋅⋅⋅⋅⋅⋅⋅⋅⋅⋅ 4. Misalkan N menyat akan himpunan semua bilangan bulat posit if dan ⎧ ⎫ n 2009 + 2 S = ⎨n ∈ N ∈ N⎬ n +1 ⎩ ⎭ Banyaknya himpunan bagian dari S adalah ⋅⋅⋅⋅⋅⋅⋅⋅⋅ 5. Diberikan segit iga ABC dengan t an ∠CAB = 22 . Melalui t it ik sudut A dit arik garis t inggi 7 sedemikian rupa sehingga membagi sisi BC menj adi segmen-segmen dengan panj ang 3 dan 17. Luas segit iga ABC adalah ⋅⋅⋅⋅⋅⋅⋅⋅ 6. Nilai minimum dari f ( x ) = 9 x 2 sin 2 x + 4 unt uk 0 < x < π adalah ⋅⋅⋅⋅⋅⋅⋅ x sin x 7. Diberikan segit iga dengan panj ang dari ket iga garis t inggi segit iga it u merupakan bilangan bulat . Jika panj ang kedua garis t ingginya adalah 10 dan 6, maka panj ang maksimum garis t inggi ket iga adalah ⋅⋅⋅⋅⋅⋅ 8. Suat u f ungsi f : Z Æ Q mempunyai sif at f ( x + 1) = 1 + f (x ) unt uk set iap x ∈ Z. Jika f (2) = 2, 1 − f (x ) maka nilai f ungsi f (2009) adalah ⋅⋅⋅⋅⋅ 9. Diket ahui segit iga siku-siku ABC dengan panj ang sisi-sisinya a, b, dan c sert a a < b < c. Misalkan r dan R bert urut -t urut menyat akan panj ang j ari-j ari lingkaran dalam dan lingkaran luarnya. Jika r r (a + b + c ) adalah ⋅⋅⋅⋅⋅⋅⋅⋅ = 3 maka nilai dari 2 a+b+c R 294 10. Jika t an x + t an y = 25 dan cot x + cot y = 30, maka nilai t an (x + y) adalah ⋅⋅⋅⋅⋅⋅ 11. Pada bagian kanan 100! t erdapat digit 0 bert urut -t urut sebanyak ⋅⋅⋅⋅⋅⋅⋅⋅ 12. Ada empat pasang sepat u akan diambil empat sepat u secara acak. Peluang bahwa yang t erambil ada yang berpasangan adalah ⋅⋅⋅⋅⋅⋅ 13. Diket ahui k, m, dan n adalah t iga bilangan bulat posit if yang memenuhi k m 1 + = m 4n 6 Bilangan m t erkecil yang memenuhi adalah ⋅⋅⋅⋅⋅⋅ 14. Bilangan prima p yang memenuhi (2p − 1) 3 + (3p) 2 = 6p ada sebanyak ⋅⋅⋅⋅⋅⋅ 15. Jika x 1, x 2, ⋅⋅⋅, x 2009 bilangan real, maka nilai t erkecil dari cos x 1 sin x 2 + cos x2 sin x 3 + ⋅⋅⋅ + cos x 2009 sin x 1 adalah ⋅⋅⋅⋅⋅ 16. Misalkan a, b, c adalah akar-akar polinom x 3 − 8x 2 + 4x − 2. Jika f (x) = x3 + px 2 + qx + r adalah polinom dengan akar-akar a + b − c, b + c − a, c + a − b maka f (1) = ⋅⋅⋅⋅⋅⋅ 17. Banyaknya segit iga t umpul dengan sisi bilangan asli yang memiliki sisi-sisi t erpanj ang 10 adalah ⋅⋅ (Cat at an : dua segit iga kongruen dianggap sama) 18. Misalkan n bilangan asli t erkecil yang mempunyai t epat 2009 f akt or dan n merupakan kelipat an 2009. Fakt or prima t erkeci dari n adalah ⋅⋅⋅⋅⋅⋅ 19. Misalkan p(x) = x 2 − 6 dan A = {x ∈ R⏐p(p(x)) = x}. Nilai maksimal dari { ⏐x ⏐ : x ∈ A} adalah ⋅⋅⋅⋅⋅⋅ 20. Misalkan q = 5 +1 dan ⎣x ⎦ menyat akan bilangan bulat t erbesar yang lebih kecil at au sama 2 dengan x. Nilai ⎣q ⎣qn⎦⎦ − ⎣q 2n⎦ unt uk sebarang n ∈ N adalah ⋅⋅⋅⋅⋅⋅ 295 SELEKSI TINGKAT PROVINSI CALON PESERTA OLIMPIADE SAINS NASIONAL 2009 MATEMATIKA SMA/ MA BAGIAN KEDUA 1. Seekor semut hendak melangkah ke makanan yang berada sej auh 10 langkah di depannya. Semut t ersebut sedang mendapat kan hukuman, ia hanya boleh melangkah ke depan sebanyak kelipat an t iga langkah dan selebihnya harus melangkah ke belakang. Tent ukan banyaknya cara melangkah agar bisa mencapai makanan, j ika ia harus melangkah t idak lebih dari dua puluh langkah. (Cat at an : j ika semut melangkah dua kali dimana masing-masing melangkah sekali ke belakang, maka dianggap sama saj a dengan dua langkah ke belakang. ) 2. Diberikan n adalah bilangan asli. Misalkan x = 6 + 2009 n . Jika x 2009 − x merupakan bilangan x3 − x rasional, t unj ukkan bahwa n merupakan kuadrat dari suat u bilangan asli. 3. Diberikan segit iga ABC dan t it ik D pada sisi AC. Misalkan r 1, r 2 dan r bert urut -t urut menyat akan j ari-j ari lingkaran dalam dari segit iga-segit iga ABD, BCD, dan ABC. Bukt ikan bahwa r 1 + r 2 > r. 4. Diket ahui p adalah bilangan prima sehingga persamaan 7p = 8x 2 − 1 dan p 2 = 2y2 − 1 mempunyai solusi x dan y berupa bilangan bulat . Tent ukan semua nilai p yang memenuhi. 5. Diket ahui himpunan H mempunyai lima anggot a dari {0, 1, 2, 3, ⋅⋅⋅, 9}. Bukt ikan ada dua himpunan bagian dari H, yang t idak kosong dan saling asing, yang j ika semua anggot anya dij umlahkan hasilnya sama. 296 SELEKSI OLIMPIADE TINGKAT PROVINSI 2009 TIM OLIMPIADE MATEMATIKA INDONESIA 2010 Prestasi itu diraih bukan didapat !!! SOLUSI SOAL BAGIAN PERTAMA Disusun oleh : Eddy Hermant o, ST 297 Olimpiade Matematika Tk Provinsi 2009 Solusi Bagian Pert ama BAGIAN PERTAMA 1. Banyaknya macam adalah (1, 1, 6), (1, 2, 5), (1, 3, 4), (2, 2, 4), (2, 3, 3) besert a permut asi yang bert urut -t urut ada sebanyak 3, 6, 6, 3 dan 3. ∴ Banyaknya macam hasil lemparan = 3 + 6 + 6 + 3 + 3 = 21 . 2. x 4 − 2x 3 + 5x 2 − 176x + 2009 = 0 (x 2 − x) 2 + (2x − 44) 2 + 73 = 0 Karena bilangan kuadrat t idak mungkin negat if maka t idak ada x real yang memenuhi. ∴ Banyaknya bilangan real x yang memenuhi adalah 0 . 3. a b c + + =3 b c a Karena a, b dan c posit if maka dengan ket aksamaan AM-GM didapat a b c a b c + + ≥ 3⋅ 3 ⋅ ⋅ = 3 b c a b c a Tanda kesamaan t erj adi j ika a = b = c. Karena a b c + + = 3 maka haruslah a = b = c yang kont radiksi dengan a < b < c. b c a ∴ Banyaknya bilangan posit if a yang memenuhi adalah 0 . 4. ⎧ ⎫ n 2009 + 2 S = ⎨n ∈ N ∈ N⎬ n +1 ⎩ ⎭ 2009 2009 n +1 +2 n 1 + ∈N = n +1 n +1 n +1 Karena n + 1⏐n2009 + 1 maka haruslah n + 1⏐1 Jadi n + 1 ≤ 1, t et api n ∈ N sehingga t idak ada n ∈ N yang memenuhi. Semua himpunan bagian dari S hanya ada sat u yait u { }. ∴ Banyaknya himpunan bagian dari S adalah 1 . 5. Misalkan garis t inggi dari A memot ong sisi BC di D dan AD = x. Tanpa mengurangi keumuman misalkan CD = 3 dan DB = 17. SMA Negeri 5 Bengkulu Eddy Hermanto, ST 298 Olimpiade Matematika Tk Provinsi 2009 Solusi tan ∠CAB = tan (∠CAD + ∠DAB ) = Bagian Pert ama tan ∠CAD + tan ∠DAB 1 − tan ∠CAD ⋅ tan ∠DAB 3 17 + 22 x x yang ekivalen dengan = 3 17 7 1− ⋅ x x 11x 2 − 561 = 70x (x − 11)(11x + 51) = 0 Karena x > 0 maka x = AD = 11 Luas ∆ABC = ½ ⋅ AD ⋅ BC = ½ ⋅ 11 ⋅ (3 + 17) ∴ Luas ∆ABC adalah 110 . 6. f (x ) = 9 x 2 sin 2 x + 4 x sin x Unt uk 0 < x < π maka sin x > 0 Dengan AM-GM didapat 9 x 2 sin 2 x + 4 4 4 = 9 x sin x + ≥ 2 9 x sin x ⋅ = 12 x sin x x sin x x sin x 4 2 at au x sin x = Tanda kesamaan t erj adi j ika 9 x sin x = x sin x 3 2 2 9 x sin x + 4 ∴ Nilai minimum dari f ( x ) = adalah 12 . x sin x f (x ) = 7. Misalkan garis t inggi ket iga = t . Misalkan j uga 6, 10 dan t adalah garis t inggi-garis t inggi yang bert urut -t urut sepadan dengan sisisisi a, b dan c. Dengan rumus luas segit iga ABC didapat hubungan 6a = 10b = t c Dengan ket aksamaan segit iga didapat a<b+c b c + a a 3 6 1< + 5 t 1< t < 15. Jika t = 14 maka 6a = 10b = 14c a :b:c = 1 1 1 : : = 35 : 21 : 15 6 10 14 Karena a = 35k < b + c = 36k unt uk suat u nilai real k maka t = 14 memenuhi. ∴ Panj ang maksimum garis t inggi ket iga adalah 14. SMA Negeri 5 Bengkulu Eddy Hermanto, ST 299 Olimpiade Matematika Tk Provinsi 2009 Solusi 8. Bagian Pert ama 1 + f (x ) dan f (2) = 2 1 − f (x ) 1+ 2 f (3) = = −3 1− 2 1− 3 1 f (4 ) = =− 1+ 3 2 1 1− 2 =1 f (5) = 1 3 1+ 2 1 1+ 3 =2 f (6) = 1 1− 3 f ( x + 1) = Sehingga nilai f (n) unt uk n bulat ≥ 2 akan periodik dengan kala ulang 4. Karena 2009 = 4(502) + 1 maka nilai f (2009) = f (5) ∴ Nilai f ungsi f (2009) adalah 1 . 3 9. r (a + b + c ) = 3 R2 1 1 abc Luas ∆ABC = r (a + b + c ) = ab = 2 2 4R ab = 3 R2 Alt ernat if 1 : Dengan mensubt it usikan bahwa c = 2R, a = c sin A dan b = c cos A maka 4 sin A cos A = 3 1 sin 2 A = 3 2 Karena a < b < c maka A < B < C. SMA Negeri 5 Bengkulu Eddy Hermanto, ST 300 Olimpiade Matematika Tk Provinsi 2009 Solusi Bagian Pert ama Jadi, A = 30o, B = 60o dan C = 90o. r r (a + b + c ) ab c sin 30° ⋅ c cos 30° 3 = = = = 2 2 2 a + b + c (a + b + c ) (a + b + c ) (c sin 30° + c cos 30° + c ) 1 + 3 + 2 ( ∴ ) 2 2 3 −3 r = 6 a+b+c Alt ernat if 2 : Karena R = 2c maka 4ab = c a2 + b 2 = c 2 2 3 3a 2 + 3b 2 = 4ab 3 a − b 3 3a − b 3 = 0 ( )( ) Karena a < b maka b = a 3 dan c = 2a ab a2 3 a 3 r= = = a + b + c a + a 3 + 2a 3 + 3 3 r a 3 = = a + b + c 3 + 3 a + a 3 + 2a 3+ 3 ( ∴ )( ) ( ) 2 2 3 −3 r = 6 a+b+c 10. t an x + t an y = 25 cot x + cot y = 30 1 1 + = 30 tan x tan y tan x + tan y = 30 tan x ⋅ tan y 5 t an x ⋅ t an y = 6 tan x + tan y 25 tan (x + y ) = = 5 1 − tan x ⋅ tan y 1− 6 ∴ t an (x + y) = 150 . ⎢100 ⎥ ⎢100 ⎥ 11. Nilai maksimal k sehingga 5k⏐100! adalah ⎢ + 2 = 24. ⎣ 5 ⎥⎦ ⎢⎣ 5 ⎥⎦ ∴ Bagian kanan 100! t erdapat digit 0 bert urut -t urut sebanyak 24 . SMA Negeri 5 Bengkulu Eddy Hermanto, ST 301 Solusi Olimpiade Matematika Tk Provinsi 2009 Bagian Pert ama 12. Alt ernat if 1 : Akan ada dua kasus 1) Ada t epat sepasang sepat u yang berpasangan dan dua lainnya dipilih dari 3 pasang sepat u t ersisa sehinga keduanya t idak berpasangan. Sepasang sepat u dipilih dari kemungkinan 4 pasangan. Banyaknya cara memilih ada 4. Banyaknya cara memilih dua sepat u dari t iga pasang sepat u sehingga keduanya t idak berpasangan adalah 3C2 ⋅ 2 ⋅ 2 = 12. Banyaknya cara memilih sehingga t epat sepasang sepat u yang berpasangan dan 2 lainnya dipilih dari 3 pasang sepat u t ersisa sehinga keduanya t idak berpasangan = 4 ⋅ 12 = 48. 2) Ada t epat dua pasang sepat u berpasangan yang dipilih dari kemungkinan empat pasang sepat u. Banyaknya cara memilih adalah 4C2 = 6. ∴ Peluang kej adian = 48 + 6 27 = 35 8 C4 Alt ernat if 2 : Komplemen dari kej adian dimaksud adalah t idak ada sepasang sepat u dari keempat sepat u t ersebut yang berpasangan, sehingga masing-masing sat u buah sepat u dipilih dari masingmasing empat pasang sepat u t ersebut . Banyaknya cara adalah 2 ⋅ 2 ⋅ 2 ⋅ 2 = 16. 16 8 C4 27 ∴ Peluang kej adian = . 35 Peluang kej adian = 1 − 13. k m 1 + = dengan k, m dan n adalah t iga bilangan bulat posit if . m 4n 6 3m 2 = 2n(m − 6k) Karena ruas kiri posit if maka haruslah m > 6k > 6. Ruas kanan past i genap sehingga m harus genap. Karena m genap dan m > 6 maka m ≥ 8. Jika m = 8 maka 48 = 4n − 3kn 48 = n(4 − 3k) n = 48 dan k = 1 adalah salah sat u pasangan (n, k) yang memenuhi. ∴ Bilangan m t erkecil yang memenuhi adalah 8 . 14. (2p − 1) 3 + (3p) 2 = 6p unt uk suat u bilangan prima p. Jika p = 2 maka 33 + 62 ≠ 62 sehingga p = 2 t idak memenuhi. Jika p = 3 maka 53 + 92 ≠ 63 sehingga p = 3 t idak memenuhi. Karena p ≠ 2, 3 dan p prima maka p dapat dinyat akan p = 6k + 1 at au 6k + 5 dengan k bulat t aknegat if . • Jika p = 6k + 1 Persamaan semula akan ekivalen dengan (12k + 1) 3 + 9(6k + 1) 2 = 66k+1 (12k) 3 + 3(12k) 2 + 3(12k) 2 + 1 + 9(6k + 1) 2 = 66k+1 Ruas kiri dibagi 9 bersisa 1 sedangkan ruas kanan habis dibagi 9. SMA Negeri 5 Bengkulu Eddy Hermanto, ST 302 Olimpiade Matematika Tk Provinsi 2009 Solusi Bagian Pert ama Maka t idak ada nilai k asli yang memenuhi. Jika p = 6k + 5 Persamaan semula akan ekivalen dengan (12k + 9) 3 + 9(6k + 5) 2 = 66k-1 33(4k + 3) 3 + 324k 2 − 540k + 180 = 66k+5 Karena 180 ≡ 9 (mod 27) maka ruas kiri dibagi 27 bersisa 9 sedangkan 27 membagi ruas kanan. Maka t idak ada nilai k asli yang memenuhi. Jadi, t idak ada bilangan prima p yang memenuhi. ∴ Banyaknya bilangan prima p yang memenuhi adalah 0 . • 15. Misalkan k = cos x 1 sin x 2 + cos x 2 sin x 3 + ⋅⋅⋅ + cos x 2009 sin x 1 maka 2k = 2 cos x 1 sin x 2 + 2 cos x 2 sin x 3 + ⋅⋅⋅ + 2 cos x 2009 sin x 1 Mengingat bahwa sin2α + cos2α = 1 maka 2009+2k = cos2x 1 + 2cosx1sinx 2 + (sin2x 2 + cos2x 2) + 2cosx2sinx3 + (sin2x3 + cos2x 3) + ⋅⋅⋅ + 2cosx 2009sinx1 + sin2x 1 2009+2k=(cos2x1 + 2cosx 1sinx 2 + sin2x 2)+(cos2x 2 + 2cosx2sinx3 + sin2x 3)+⋅⋅⋅+(cos2x 2009 + 2cosx2009sinx1 + sin2x1) 2009 + 2k =(cos x 1 + sin x 2) 2 + (cos x 2 + sin x 1) 2 + ⋅⋅⋅ + (cos x 2009 + sin x 1) 2 + (cos x 1 + sin x 2009) 2 Karena bilangan kuadrat t idak mungkin negat if maka 2009 + 2k min = 0 k min = − 2009 2 Nilai minimum didapat j ika cos x 1 = −sin x 2, cos x 2 = −sin x 1, cos x 2 = −sin x 3, cos x 3 = −sin x 2, ⋅⋅⋅, cos x 2009 = −sin x 1 dan cos x 2009 = −sin x 1 yang dapat dipenuhi oleh x1 = x 2 = L = x 2009 = ∴ Nilai minimum dari cos x 1 sin x 2 + cos x2 sin x 3 + ⋅⋅⋅ + cos x 2009 sin x 1 adalah − 3π rad. 4 2009 . 2 16. x 3 − 8x 2 + 4x − 2 = 0 akar-akarnya a, b dan c. Maka a + b + c = 8. Subt it usi y = 8 − 2x sehingga x = 3 8− y ke persamaan x 3 − 8x 2 + 4x − 2 = 0. Maka 2 2 ⎛8− y ⎞ ⎛8− y ⎞ ⎛8− y ⎞ ⎟ − 2 = 0 memiliki akar-akar 8 − 2a, 8 − 2b dan 8 − 2c ⎟ + 4⎜ ⎟ − 8⎜ ⎜ ⎝ 2 ⎠ ⎝ 2 ⎠ ⎝ 2 ⎠ Polinom f (x) = x 3 + px 2 + qx + r memiliki akar-akar, yait u a + b − c = 8 − 2c, a + c − b = 8 − 2b dan b + c − a = 8 − 2a. Karena koef isien x 3 dari f (x) sama dengan 1 maka 3 2 ⎛8− x⎞ ⎛8− x⎞ ⎛8− x⎞ ⎟ + 16 = 0 j uga memiliki akar-akar 8 − 2a, 8 − 2b ⎟ − 32⎜ ⎟ + 64⎜ ⎝ 2 ⎠ ⎝ 2 ⎠ ⎝ 2 ⎠ Polinom f ( x) = −8⎜ dan 8 − 2c. 3 2 ⎛ 8 − 1⎞ ⎛ 8 − 1⎞ ⎛ 8 − 1⎞ f (1) = −8⎜ ⎟ + 16 = 345 ⎟ − 32⎜ ⎟ + 64⎜ ⎝ 2 ⎠ ⎝ 2 ⎠ ⎝ 2 ⎠ ∴ f (1) = 345 . SMA Negeri 5 Bengkulu Eddy Hermanto, ST 303 Olimpiade Matematika Tk Provinsi 2009 Solusi Bagian Pert ama 17. Tanpa mengurangi keumuman misalkan sisi-sisi segit iga adalah a, b dan 10 dengan a ≤ b ≤ 10. Ket aksamaan segit iga, a + b > 10 Karena segit iga t umpul maka a2 + b 2 < 102 Pasangan (a, b) bilangan asli yang memenuhi kedua ket aksamaan t ersebut adalah (2, 9), (3, 8), (3, 9), (4, 7), (4, 8), (4, 9), (5, 6), (5, 7), (5, 8), (6, 6), (6, 7) dan (7, 7). Banyaknya pasangan (a, b) bilangan asli yang memenuhi ada 12. ∴ Banyaknya segit iga yang memenuhi adalah 12 . 18. 2009 = 72 ⋅ 41 maka 72 dan 41 haruslah merupakan f akt or dari n. nmin = 240 ⋅ 76 ⋅ 416 memenuhi banyaknya f akt or posit if dari n adalah (40 + 1)(6 + 1)(6 + 1) = 2009 ∴ Fakt or prima t erkecil dari n adalah 2 . 19. p(x) = x 2 − 6 p(p(x) = x (x 2 − 6) 2 − 6 = x x 4 − 12x 2 − x + 30 = 0 (x + 2)(x − 3)(x 2 + x − 5) = 0 Nilai x yang memenuhi adalah −2, 3, Karena − 1 − 21 − 1 + 21 , . 2 2 − 1 − 21 1 + 21 1 + 25 = < = 3 maka nilai t erbesar ⏐x⏐ yang memenuhi adalah 3. 2 2 2 ∴ Nilai maksimal dari { ⏐x ⏐ : x ∈ A} adalah 3 . 20. Karena q = q2 = q + 1 q −1 = 5 +1 maka 2 5 −1 2 q 2n = nq + n Karena n bulat maka ⎣q 2n⎦ = ⎣nq + n⎦ = ⎣qn⎦ + n ⋅⋅⋅⋅⋅⋅⋅⋅⋅⋅⋅⋅⋅⋅⋅⋅⋅⋅⋅⋅⋅⋅⋅⋅⋅⋅⋅⋅ (1) ⎣q ⎣qn⎦⎦ = ⎣(q − 1) ⎣qn⎦ + ⎣qn⎦⎦ Karena ⎣qn⎦ bulat maka ⎣q ⎣qn⎦⎦ = ⎣(q − 1) ⎣qn⎦⎦ + ⎣qn⎦ ⋅⋅⋅⋅⋅⋅⋅⋅⋅⋅⋅⋅⋅⋅⋅⋅⋅⋅⋅⋅⋅⋅⋅⋅ (2) ⎢⎛ 5 − 1 ⎞⎛ ⎛ 5 + 1 ⎞ ⎞⎥ ⎢ ⎛ 5 − 1 ⎞⎥ ⎟⎜ ⎜ ⎟ = n −1 ⎟n − 1⎟⎥ = ⎢n − ⎜ ⎟ ⎜ ⎟ ⎟⎥ ⎢ ⎜ 2 ⎟⎥⎥ ⎢⎣⎝ 2 ⎠⎜⎝ ⎝ 2 ⎠ ⎠⎦ ⎠⎦ ⎣ ⎝ ⎣(q − 1)⎣qn⎦⎦ ≥ ⎣(q − 1)(qn − 1)⎦ = ⎢⎜⎜ Karena q − 1 t ak bulat maka ⎛ 5 − 1 ⎞⎛ ⎛ 5 + 1 ⎞ ⎞ ⎟ ⎟ ⎟⎜ ⎜ ⎟⎜ ⎜ 2 ⎟n ⎟ = n 2 ⎠ ⎠ ⎠⎝ ⎝ ⎝ ⎣(q − 1)⎣qn⎦⎦ < (q − 1)⎣qn⎦ < ⎜⎜ SMA Negeri 5 Bengkulu Eddy Hermanto, ST 304 Solusi Olimpiade Matematika Tk Provinsi 2009 Bagian Pert ama Karena n > ⎣(q − 1) ⎣qn⎦⎦ ≥ n − 1 maka ⎣(q − 1) ⎣qn⎦⎦ = n − 1 ⎣q ⎣qn⎦⎦ = ⎣(q − 1) ⎣qn⎦⎦ + ⎣qn⎦ ⎣q ⎣qn⎦⎦ = n − 1 + ⎣qn⎦ ⋅⋅⋅⋅⋅⋅⋅⋅⋅⋅⋅⋅⋅⋅⋅⋅⋅⋅⋅⋅⋅⋅⋅⋅⋅⋅⋅ (3) Kurangkan persamaan (3) dengan persamaan (1) ⎣q ⎣qn⎦⎦ − ⎣q 2n⎦ = (n − 1 + ⎣qn⎦) − ( ⎣qn⎦ + n) ⎣q ⎣qn⎦⎦ − ⎣q 2n⎦ = −1 ∴ Nilai ⎣q ⎣qn⎦⎦ − ⎣q 2n⎦ unt uk sebarang n ∈ N adalah −1 . SMA Negeri 5 Bengkulu Eddy Hermanto, ST 305 SELEKSI OLIMPIADE TINGKAT PROVINSI 2009 TIM OLIMPIADE MATEMATIKA INDONESIA 2010 Prestasi itu diraih bukan didapat !!! SOLUSI SOAL BAGIAN KEDUA Disusun oleh : Eddy Hermant o, ST 306 Olimpiade Matematika Tk Provinsi 2009 Solusi Bagian Kedua BAGIAN KEDUA 1. Jelas bahwa semut harus melangkah ke depan lebih dari 3 kali. Jika semut melangkah ke depan lebih dari 5 kali maka semut t ersebut harus mundur sekurangkurangnya 8 langkah sehingga t ot al langkah lebih dari 20. Jadi, hanya ada 2 kasus : - Semut t ersebut maj u 3 x 4 langkah dan mundur 2 langkah, t ot al langkah 14. Banyaknya cara sama saj a dengan banyaknya susunan 333311 6! = 15 cara. 4!⋅2! Banyaknya cara = - Cara lainnya sama dengan menempat kan 4 angka t iga ke 4 dari 6 t empat . Banyaknya cara = 6C4 = 15 cara. Semut t ersebut maj u 3 x 5 langkah dan mundur 5 langkah, t ot al langkah 20. Banyaknya cara sama saj a dengan banyaknya susunan 3333311111 10! = 252 cara. 5!⋅5! Banyaknya cara = Cara lainnya sama dengan menempat kan 5 angka t iga ke 5 dari 10 t empat . Banyaknya cara = 10C5 = 252 cara. ∴ Banyaknya cara semut t ersebut melangkah agar mencapai makanan adalah 15 + 252 = 267 2. x = 6 + 2009 n x 2009 − x a = dengan a dan b bilangan bulat dan b ≠ 0. b x3 − x Karena p1 + q1 n p 2 + q 2 n = ( p1 p 2 + q1 q 2 n ) + ( p1 q 2 + p 2 q1 ) n ( )( ) yang j uga berbent uk pi + qi n unt uk suat u bilangan asli p i dan q i dengan i adalah bilangan asli maka x j uga akan i berbent uk pi + qi n unt uk suat u bilangan asli i. x 2009 − x a x 2008 − 1 = = 2 b x3 − x x −1 p 2008 + q 2008 n − 1 a = b p2 + q2 n − 1 Karena x ≠ 0 maka b ⋅ p 2008 − a ⋅ p 2 + a − b = (a ⋅ q 2 − b ⋅ q 2008 ) n Karena a, b, p 2, p 2008, q 2 dam q 2008 adalah bilangan bulat maka n haruslah merupakan kuadrat dari suat u bilangan rasional. 2 ⎛k⎞ n = ⎜ ⎟ dengan k, m bilangan asli dan FPB(k, m) = 1 ⎝m⎠ Karena n bilangan asli maka haruslah m = 1 sehingga n merupakan kuadrat dari suat u bilangan asli. ∴ Terbukti bahwa n merupakan kuadrat dari suatu bilangan asli. SMA Negeri 5 Bengkulu Eddy Hermanto, ST 307 Solusi Olimpiade Matematika Tk Provinsi 2009 Bagian Kedua 3. Misalkan [ ABC] menyat akan luas ∆ABC, maka [ ABC] = [ ABD] + [ BCD] 1 1 1 r ( AB + BC + AC ) = r1 ( AB + BD + AD ) + r2 (BC + BD + DC ) 2 2 2 Pada ∆ABD dan ∆BCD bert urut -t urut berlaku BD < AD + AB dan BD < BC + DC sehingga r(AB + BC + AC) = r 1(AB + BD + AD) + r 2(BC + BD + DC) < r 1(AB + BC + DC + AD) + r 2(BC + AD + AB + DC) Karena AD + DC = AC maka r(AB + BC + AC) < r 1(AB + BC + AC) + r 2(BC + AC + AB) r < r1 + r2 ∴ Terbukti bahwa r 1 + r 2 > r 4. 7p = 8x 2 − 1 ⋅⋅⋅⋅⋅⋅⋅⋅⋅⋅⋅⋅⋅⋅⋅⋅⋅⋅⋅⋅⋅⋅⋅⋅⋅⋅⋅⋅⋅⋅ (1) p 2 = 2y2 − 1 ⋅⋅⋅⋅⋅⋅⋅⋅⋅⋅⋅⋅⋅⋅⋅⋅⋅⋅⋅⋅⋅⋅⋅⋅⋅⋅⋅⋅⋅⋅ (2) Jika (x, y) = (x 1, y1) memenuhi persamaan maka ( −x 1, −y1) past i memenuhi sehingga t anpa mengurangi keumuman dapat dimisalkan x, y ≥ 0. p 2 − y2 = y2 − 1. Karena y = 0 dan y = 1 t idak memenuhi persamaan maka y 2 > 1 sehingga p > y ⋅⋅⋅⋅⋅⋅⋅⋅⋅⋅⋅⋅⋅⋅⋅⋅ (3) Jika p = 2 maka 15 = 8x 2 yang t idak akan t erpenuhi unt uk x bilangan bulat . Jika p = 3 maka 22 = 8x 2 yang t idak akan t erpenuhi unt uk x bilangan bulat . Jika p = 5 maka 36 = 8x 2 yang t idak akan t erpenuhi unt uk x bilangan bulat . Jika p = 7 maka 50 = 8x 2 yang t idak akan t erpenuhi unt uk x bilangan bulat . Jadi, p > 7. Kurangkan persamaan (2) dengan (1) didapat p(p − 7) = 2(y + 2x)(y − 2x) Karena p > 7 maka y > 2x sehingga p > y > 2x ⋅⋅⋅⋅⋅⋅⋅⋅⋅⋅⋅⋅⋅⋅⋅⋅⋅⋅⋅⋅⋅⋅⋅⋅⋅⋅⋅ (4) Karena p ≠ 2 maka p ⏐(y + 2x)(y − 2x) Karena p > y ≥ y − 2x dan p bilangan prima maka p ⏐y + 2x Karena p ≤ y + 2x < p + p = 2p maka hanya t erpenuhi j ika p = y + 2x Maka p 2 = 2(p − 2x) 2 − 1 sehingga p 2 − 8xp + 8x 2 − 1 = 0 Subt it usikan persamaan (1) sehingga p 2 − 8xp + 7p = 0 Karena p ≠ 0 maka p = 8x − 7 ⋅⋅⋅⋅⋅⋅⋅⋅⋅⋅⋅⋅⋅⋅⋅⋅⋅⋅⋅⋅⋅⋅⋅⋅⋅⋅⋅⋅ (5) Subt it usikan persamaan (5) ke persamaan (1) 7(8x − 7) = 8x 2 − 1 (x − 6)(x − 1) = 0 * Jika x = 1 dan sesuai persamaan (5) maka p = 1 (t idak memenuhi bahwa p bilangan prima) * Jika x = 6 maka p = 41 dan y = 29 yang memenuhi bahwa p bilangan prima dan y bulat ∴ Semua nilai p yang memenuhi adalah p = 41. SMA Negeri 5 Bengkulu Eddy Hermanto, ST 308 Solusi Olimpiade Matematika Tk Provinsi 2009 Bagian Kedua 5. Misalkan A ⊂ H dan B ⊂ H yang memenuhi A ∩ B = { } sert a A dan B keduanya bukan himpunan kosong. H = {0, 1, 2, 4, 8} merupakan count er example dari soal. Bagaimana pun disusun A ⊂ H dan B ⊂ H sert a A ∩ B = { } t idak akan didapat j ika semua anggot a A dij umlahkan hasilnya akan sama dengan j umlah semua anggot a B. ∴ Tidak dapat dibukt ikan ada dua himpunan bagian dari H, yang t idak kosong dan saling asing, yang j ika semua anggot anya dij umlahkan hasilnya sama. SMA Negeri 5 Bengkulu Eddy Hermanto, ST 309 SELEKSI TIM OLIMPIADE MATEMATIKA INDONESIA 2010 OLIMPIADE SAINS NASIONAL 2009 DKI JAKARTA, 3 – 9 AGUSTUS 2009 Bidang Mat emat ika Hari Pert ama Waktu : 4 Jam DEPARTEMEN PENDIDIKAN NASIONAL DIREKTORAT JENDERAL MANAJEMEN PENDIDIKAN DASAR DAN MENENGAH DIREKTORAT PEMBINAAN SEKOLAH MENENGAH ATAS TAHUN 2009 310 OLIMPIADE SAINS NASIONAL 2009 3 – 9 AGUSTUS 2009 DKI JAKARTA BI DAN G : MATEMATI KA HARI PERTAMA WAKTU : 4 JAM 1. Tent ukan banyaknya bilangan n ∈ {1, 2, 3, ⋅⋅⋅, 2009} sedemikian sehingga 4n6 + n3 + 5 habis dibagi 7. 2. Misalkan unt uk set iap bilangan real x didef inisikan ⎣x ⎦ sebagai bilangan bulat t erbesar yang lebih kecil at au sama dengan x. Diberikan a1, a2, a3, ⋅⋅⋅ suat u barisan bilangan asli yang memenuhi a1 > 1 dan ⎢ a1 + 1⎥ ⎢ a 2 + 1⎥ ⎢ a3 + 1⎥ ⎥=⎢ ⎢ ⎥=⎢ ⎥ = L. ⎣ a 2 ⎦ ⎣ a3 ⎦ ⎣ a 4 ⎦ Bukt ikan bahwa ⎢ a n + 1⎥ ⎥ ≤1 ⎢ ⎣ a n +1 ⎦ unt uk set iap bilangan asli n. 3. Pada segit iga ABC, t it ik-t it ik D, E dan F bert urut -t urut t erlet ak pada segmen BC, CA dan AB. Nyat akan P sebagai t it ik perpot ongan AD dan EF. Tunj ukkan bahwa AB AC AD xDC + xDB = xBC AF AE AP 4. Di suat u pulau t erdapat 7 kot a dan ada j aringan keret a api yang melalui kot a-kot a t ersebut . Set iap segmen rel menghubungkan t epat 2 kot a, dan diket ahui bahwa set iap kot a memiliki paling sedikit 3 segmen ke kot a lain. Bukt ikan bahwa t erdapat rut e perj alanan keret a api yang mengunj ungi 4 kot a yang berbeda masing-masing sekali dan kembali ke kot a asalnya. (Cont oh : rut e A − B − C − D − A) 311 SELEKSI TIM OLIMPIADE MATEMATIKA INDONESIA 2010 OLIMPIADE SAINS NASIONAL 2009 DKI JAKARTA, 3 – 9 AGUSTUS 2009 Bidang Mat emat ika Hari Kedua Waktu : 4 Jam DEPARTEMEN PENDIDIKAN NASIONAL DIREKTORAT JENDERAL MANAJEMEN PENDIDIKAN DASAR DAN MENENGAH DIREKTORAT PEMBINAAN SEKOLAH MENENGAH ATAS TAHUN 2009 312 OLIMPIADE SAINS NASIONAL 2009 3 – 9 AGUSTUS 2009 DKI JAKARTA BI DAN G : MATEMATI KA HARI KEDUA WAKTU : 4 JAM 5. Di dalam suat u laci t erdapat paling banyak 2009 bola yang t erdiri dari bola put ih dan biru yang t ercampur secara acak. Jika dua bola diambil secara acak t anpa pengembalian, maka diket ahui probabilit as bahwa t erambil keduanya bola warna put ih at au keduanya bola warna biru adalah 1 2 . Berapa banyak maksimum bola put ih yang mungkin berada dalam laci sedemikian sehingga pernyat aan t ent ang probabilit as t ersebut t et ap t erpenuhi ? 6. Tent ukan nilai t erkecil yang mungkin dari f ungsi f (x) = x 2008 − 2 x 2007 + 3 x 2006 − 4 x 2005 + 5 x 2004 + ⋅⋅⋅ − 2006 x 3 + 2007 x 2 − 2008x + 2009 unt uk sebarang bilangan real x. 7. Suat u pasangan bilangan bulat (m, n) dikat akan baik bila m ⏐n2 + n dan n ⏐m 2 + m Diberikan sebarang dua bilangan asli a, b > 1 yang relat if prima, bukt ikan bahwa t erdapat pasangan baik (m, n) dengan a⏐m dan b ⏐n t et api a t idak membagi n dan b t idak membagi m. 8. Diberikan segit iga ABC lancip. Lingkaran dalam segit iga ABC menyinggung BC, CA, dan AB bert urut -t urut di D, E, dan F. Garis bagi sudut A memot ong DE dan DF bert urut -t urut di K dan L. Misalkan AA1 adalah garis t inggi dan M t it ik t engah BC. (a) Bukt ikan bahwa BK dan CL t egak lurus garis bagi sudut BAC (b) Tunj ukkan bahwa A1KML adalah segiempat t alibusur 313 SELEKSI TIM OLIMPIADE MATEMATIKA INDONESIA 2010 OLIMPIADE SAINS NASIONAL 2009 DKI JAKARTA, 3 – 9 AGUSTUS 2009 Prestasi itu diraih bukan didapat !!! SOLUSI SOAL Bidang Mat emat ika Disusun oleh : Eddy Hermant o, ST 314 Olimpiade Sains Nasional 2009 Solusi Bidang : Mat emat ika 1. 4n6 + n3 + 5 • Jika n ≡ 0 (mod 7) maka 4n 6 + n3 + 5 ≡ 4(0) 6 + (0) 3 + 5 (mod 7) ≡ 5 (mod 7) • Jika n ≡ 1 (mod 7) maka 4n 6 + n3 + 5 ≡ 4(1) 6 + (1) 3 + 5 (mod 7) ≡ 3 (mod 7) • Jika n ≡ 2 (mod 7) maka 4n 6 + n3 + 5 ≡ 4(2) 6 + (2) 3 + 5 (mod 7) ≡ 3 (mod 7) • Jika n ≡ 3 (mod 7) maka 4n 6 + n3 + 5 ≡ 4(3) 6 + (3) 3 + 5 (mod 7) ≡ 1 (mod 7) • Jika n ≡ −3 (mod 7) maka 4n 6 + n3 + 5 ≡ 4( −3) 6 + ( −3) 3 + 5 (mod 7) ≡ 3 (mod 7) • Jika n ≡ −2 (mod 7) maka 4n 6 + n3 + 5 ≡ 4( −2) 6 + ( −2) 3 + 5 (mod 7) ≡ 1 (mod 7) • Jika n ≡ −1 (mod 7) maka 4n 6 + n3 + 5 ≡ 4( −1) 6 + ( −1) 3 + 5 (mod 7) ≡ 1 (mod 7) Maka t idak ada nilai n asli yang akan menyebabkan 4n 6 + n 3 + 5 habis dibagi 7. ∴ Banyaknya bilangan n yang memenuhi ada 0 . ⎢ a1 + 1⎥ ⎢ a 2 + 1⎥ ⎢ a3 + 1⎥ ⎥=⎢ ⎥=⎢ ⎥ = L = p unt uk suat u bilangan bulat t ak negat if p. ⎣ a 2 ⎦ ⎣ a3 ⎦ ⎣ a 4 ⎦ 2. Misalkan ⎢ Misalkan j uga t erdapat ak = 1 unt uk suat u bilangan asli k. Maka berdasarkan pengert ian f ungsi t angga maka ⎢ a k −1 + 1⎥ ⎥ = a k −1 + 1 = p ≥ 2 sebab ak−1 merupakan bilangan asli. ⎢ ⎣ ak ⎦ ⎢ a k + 1⎥ ⎢ 2 ⎥ ⎥ = p ≤ 2 sebab ak+1 merupakan bilangan asli. ⎥=⎢ ⎢ a a ⎣ k +1 ⎦ ⎣ k +1 ⎦ Akibat nya haruslah ak−1 = ak+1 = ak = 1. Jadi, haruslah a1 = 1. Kont radiksi. Jadi, an ≠ 1 unt uk set iap bilangan asli n. Berdasarkan pengert ian f ungsi t angga maka a a m −1 + 1 1 ≥ p unt uk suat u bilangan asli m > 1 sehingga m −1 ≥ p − . Dengan cara yang sama am am am didapat a m−2 1 ≥ p− . Demikian set erusnya. a m −1 a m −1 Misalkan k < m unt uk suat u bilangan asli k. Maka ⎛ a k a k +1 a a 1 ⎞⎛ 1 ⎞ ⎛ 1 ⎞ ⎟. ⎟⎟ L⎜⎜ p − ⎟⎟⎜⎜ p − ⋅ ⋅ L ⋅ m −1 = k ≥ ⎜⎜ p − a k +1 a k + 2 am am ⎝ a k +1 ⎠⎝ a k +1 ⎠ ⎝ a m ⎟⎠ a Jika p ≥ 2 maka k > 1 sehingga ak > am unt uk set iap bilangan asli k, m dan k < m. am Jadi, j ika p ≥ 2 maka a1, a2, a3, ⋅⋅⋅ merupakan barisan t urun. Karena a1 memiliki suat u nilai t ert ent u maka akan t erdapat an < 1 unt uk suat u bilangan asli n. Kont radiksi karena an merupakan bilangan asli. Jadi, haruslah p ≤ 1. 2, 3, 2, 3, 2, ⋅⋅⋅ adalah cont oh barisan unt uk p = 1 sedangkan 2, 4, 6, ⋅⋅⋅ adalah cont oh barisan unt uk p = 0. ⎢ a n + 1⎥ ⎥ ≤ 1. ⎣ a n +1 ⎦ ∴ Terbukti bahwa ⎢ SMA Negeri 5 Bengkulu Eddy Hermanto, ST 315 Olimpiade Sains Nasional 2009 Solusi Bidang : Mat emat ika 3. Alt ernat if 1 : Misalkan [ XYZ] menyat akan luas segit iga XYZ. ∆ACD dan ∆ABC memiliki t inggi yang sama sehingga luas dapat dinyat akan sebagai perbandingan [ ADC ] alas. Maka [ ABC ] = DC ⋅⋅⋅⋅⋅⋅⋅⋅⋅⋅⋅⋅⋅⋅⋅⋅⋅⋅⋅⋅⋅⋅⋅⋅⋅⋅ (1). BC [ ABD ] ∆ABD dan ∆ABC j uga memiliki t inggi yang sama sehingga [ ABC ] = Misalkan ∠DAC = α dan ∠BAD = β. [ ADC ] [ APE ] = 1 ⋅ AC ⋅ AD⋅sin α 2 1 ⋅ AP⋅ AE ⋅sin α 2 = AC ⋅ AD AP⋅ AE ⋅⋅⋅⋅⋅⋅⋅⋅⋅⋅⋅⋅⋅⋅⋅⋅⋅⋅⋅⋅ (3) [ ABD ] [ AFP ] = 1 ⋅ AD⋅ AB⋅sin β 2 1 ⋅ AP⋅ AF ⋅sin β 2 = AD⋅ AB AP⋅ AF ⋅⋅⋅⋅⋅⋅⋅⋅⋅⋅⋅⋅⋅⋅⋅⋅⋅⋅⋅⋅ (4) AB AF ⋅ DC + AC AE ⋅DB = [ ADC ] ⋅ [ ABC ] ⋅BC + AB AF AC AE [ ABD ] ⋅ [ ABC ] ⋅BC = AB AF ⋅ AD. AC AP. AE ⋅⋅⋅⋅⋅⋅⋅⋅⋅⋅⋅⋅⋅⋅⋅⋅⋅⋅⋅⋅⋅⋅⋅⋅⋅⋅ (2). DB BC [ APE ] ⋅ [ ABC ] BC + AC AE ⋅ AD . AB AP . AF [ AFP ] [ ABC ] ⋅BC Karena [ AFE] = [ APE] + [ AFP] maka AB AF ⋅ DC + AC AE [ AFE ] ⋅DB = Karena [ ABC ] = AB AF ⋅ DC + ∴ AB AF AC AE ⋅ DC + AF . AE AB . AC ⋅DB = AC AE ⋅ AB AF AD. AC AP. AE [ AFE ] ⋅ [ ABC ] BC maka AD AP ⋅BC ⋅DB = AD AP ⋅BC (t erbukt i) Alt ernat if 2 : Tanpa mengurangi keumuman misalkan koordinat A(0, 0), B(a, 0) dan C(b, c) sert a absis t it ik D, E dan F bert urut -t urut d, e dan f , maka koordinat F(f , 0). Persamaan garis AC adalah y = bc x sehingga koordinat E(e, ceb ). Persamaan garis EF adalah Persamaan garis BC adalah y −0 ce −0 b y −0 c −0 cex − cef be −bf = x− f e− f yang set ara dengan y = = x−a b−a yang set ara dengan y = − c ( aa−−db ) ). ac − cd ) Persamaan garis AD adalah y = ( ad −bd x. . ( ax−−ab ) . Maka koordinat t it ik D(d, c Tit ik P adalah perpot ongan ant ara garis EF dan AD maka cexP − cef be −bf = ( adac−−cdbd ) x P adex P − bdex P − adef + bdef = abex P − bdex P − abf x P + bdf x P xP = def ( a −b ) ade + abf −bdf − abe Karena A, F dan B berada pada sat u garis lurus maka SMA Negeri 5 Bengkulu AB AF = xB − x A xF − x A = a f ⋅⋅⋅⋅⋅⋅⋅⋅⋅⋅⋅⋅⋅⋅⋅⋅⋅⋅⋅⋅⋅⋅⋅⋅⋅⋅⋅⋅⋅⋅⋅ (1) Eddy Hermanto, ST 316 Olimpiade Sains Nasional 2009 Solusi Bidang : Mat emat ika Karena A, E dan C berada pada sat u garis lurus maka AC AE = xC − x A xE − x A = Karena C, D dan B berada pada sat u garis lurus maka DC BC = x D − xC x B − xC = d −b a −b ⋅⋅⋅⋅⋅⋅⋅⋅⋅⋅⋅⋅⋅⋅⋅⋅⋅⋅⋅⋅⋅⋅⋅⋅⋅⋅⋅⋅⋅⋅⋅ (3) Karena C, D dan B berada pada sat u garis lurus maka DB BC = xB − xD x B − xC = a−d a −b ⋅⋅⋅⋅⋅⋅⋅⋅⋅⋅⋅⋅⋅⋅⋅⋅⋅⋅⋅⋅⋅⋅⋅⋅⋅⋅⋅⋅⋅⋅⋅ (4) Karena A, D dan P berada pada sat u garis lurus maka AD AP = xD − x A xP − x A = ade + abf − bdf − abe ef ( a − b ) AB AF ⋅ DC + BC AC AE ⋅ DB = BC a f ⋅ ((da −−bb )) + be ⋅ ((aa−−db )) = ade + abf − bdf − abe ef ( a − b ) b e ⋅⋅⋅⋅⋅⋅⋅⋅⋅⋅⋅⋅⋅⋅⋅⋅⋅⋅⋅⋅⋅⋅⋅⋅⋅⋅⋅⋅⋅⋅⋅ (2) ⋅⋅⋅⋅⋅⋅⋅⋅⋅⋅⋅⋅⋅⋅⋅ (5) ⋅⋅⋅⋅⋅⋅⋅⋅⋅⋅⋅⋅⋅⋅⋅⋅⋅⋅⋅⋅⋅⋅⋅⋅⋅⋅⋅ (6) Dari persamaan (5) dan (6) didapat AB DC ⋅ + AC ⋅ DB = AD AP AF BC AE BC ∴ AB AF ⋅ DC + AC AE ⋅DB = AD AP ⋅BC (t erbukt i) 4. Sebuah segmen rel menghubungkan dua kot a. Maka j umlah t ot al ‘ kot a’ yang muncul dari seluruh segmen haruslah genap. Jika masing-masing kot a t epat t erhubung dengan t iga kot a lainnya maka banyaknya kot a yang muncul adalah 3 x 7 = 21 yang ganj il. Maka akan ada sekurang-kurangnya 11 segmen dan ada sat u kot a yang t erhubung dengan sekurang-kurangnya 4 kot a. Tanpa mengurangi keumuman misalkan kot a yang t erhubung dengan sekurang-kurangnya 4 kot a t ersebut adalah kot a A yang t erhubung dengan kot a B, C, D dan E. Karena sebuah kot a t erhubung dengan sekurang-kurangnya 3 kot a lain maka kot a F dan G masing-masing t erhubung dengan sedikit nya salah sat u dari B, C, D dan E. Akan ada 2 kasus : • Kot a F at au G t erhubung dengan sekurang-kurangnya 2 di ant ara B, C, D dan E misalkan B dan C, maka bukt i selesai sebab akan t erdapat rut e A – B – F/ G – C – A. • Kot a F dan G masing-masing t erhubung dengan salah sat u di ant ara B, C, D dan E. Kot a F dan G t erhubung dengan salah sat u di ant ara B, C, D dan E. Selain it u kot a F j uga harus t erhubung dengan kot a A dan G sert a kot a G j uga harus t erhubung dengan A. Tanpa mengurangi keumuman misalkan kot a F t erhubung dengan kot a B. Maka akan t erdapat rut e A – B – F – G – A. ∴ Terbukti bahwa terdapat rute perj alanan kereta api yang mengunj ungi 4 kota yang berbeda masing-masing sekali dan kembali ke kota asalnya. 5. x C2 + y C2 x + y C2 = x ( x −1)+ y ( y −1) ( x + y )( x + y −1) 1 2 = 1 2 2x 2 − 2x + 2y2 − 2y = x 2 + y2 − x − y + 2xy (x − y) 2 = x + y ≤ 2009 x − y ≤ 44 x + y = (x − y) 2 ≤ 442 = 1936 2x ≤ 44 + 1936 x ≤ 990 Jika x = 990 maka y = 1936 − 990 = 946. Peluang = 990 C 2 + 946 C 2 1936 C 2 = 990⋅989 + 946⋅945 1936⋅1935 == 1 2 ∴ Jadi, x t erbesar adalah 990 . SMA Negeri 5 Bengkulu Eddy Hermanto, ST 317 Olimpiade Sains Nasional 2009 Solusi Bidang : Mat emat ika 6. f (x) = x 2008 − 2 x 2007 + 3 x 2006 − 4 x 2005 + 5 x 2004 + ⋅⋅⋅ − 2006 x 3 + 2007 x 2 − 2008x + 2009 f (x) = x 2006(x − 1) 2 + 2x 2004(x − 1) 2 + 3x 2002(x − 1) 2 + ⋅⋅⋅ + 1004(x − 1) 2 + 1005 Karena bilangan kuadrat t idak mungkin negat if maka f (x) akan minimal saat x = 1 f (x) minimal = 1005 ∴ Jadi, nilai t erkecil dari f (x) adalah 1005 . 7. Karena FPB(a, b) = 1 maka past i ada pasangan bilangan bulat (x, y) yang memenuhi ax + by = −1. Misalkan m = ax dan n = by. Jelas bahwa m dan n keduanya bilangan bulat . n2 + n = by(by +1) = by( −ax) = −mn yang berart i habis dibagi m. m 2 + m = ax(ax +1) = ax( −by) = −mn yang berart i habis dibagi n. Jadi m ⏐n2 + n dan n⏐m 2 + m Selain it u, dari m = ax dan n = by akan didapat a⏐m dan b ⏐n. Jika a⏐n maka a⏐y. Dari ax + by = −1 akan didapat bahwa a⏐1 sehingga a = 1 (kont radiksi). Jadi, a t idak membagi n. Dengan cara yang sama akan didapat bahwa b t idak membagi m. ∴ Terbukti bahwa terdapat pasangan baik (m, n) dengan a⏐m dan b⏐n tet api a tidak membagi n dan b tidak membagi m. 8. a) ∠LIC = 180o − ∠AIC = 180o − (180o − 1 2 A− 1 2 Karena ∆BDF sama kaki maka ∠BDF = 90 − o ∠CDL = 180o − ∠BDF = 180o − (90o − 1 2 (180o − (A + B)) = 90o − 1 2 1 2 B ⋅⋅⋅⋅⋅⋅⋅⋅⋅⋅⋅⋅⋅⋅⋅ (1) B ⋅⋅⋅⋅⋅⋅⋅⋅⋅⋅⋅⋅⋅⋅⋅⋅⋅⋅⋅⋅ (2) B) = 90o + 1 2 B ⋅⋅⋅⋅⋅⋅⋅⋅⋅⋅⋅⋅⋅⋅⋅⋅⋅⋅⋅⋅⋅⋅⋅⋅⋅ (3) Karena ∠LIC + ∠CDL = 180 maka CDLI adalah segiempat t alibusur. Karena CI adalah t alibusur suat u lingkaran sedangkan t it ik D dan L t erlet ak pada lingkaran t ersebut maka ∠CLI = ∠CDI = 90o. ∴ Jadi, CL tegak lurus AK yang merupakan garis bagi sudut BAC (t erbukt i). ∠AEK = 180o − ∠CEK = 180o − (90o − 12 (180o − (A + B)) = 180o − 12 (A + B) o ∠DKI = 180o − ∠AEK − ∠CAK = 180o − (180o − 1 2 (A + B)) − 1 2 A= 1 2 B Karena ∠DKI = ∠DBI maka t it ik-t it ik D, K, B dan I adalah segiempat t alibusur. Karena BI adalah t alibusur suat u lingkaran sedangkan t it ik D dan K t erlet ak pada lingkaran t ersebut maka ∠BKI = ∠BDI = 90o. ∴ Jadi, BK t egak lurus AK yang merupakan garis bagi sudut BAC (t erbukt i). SMA Negeri 5 Bengkulu Eddy Hermanto, ST 318 Olimpiade Sains Nasional 2009 Solusi b) Bidang : Mat emat ika Misalkan garis BC dan KL berpot ongan di t it ik T. Karena ∠CLT = ∠BKT = 90o dan ∠BTK = ∠CTL maka ∆CLT dan ∆BKT sebangun BK CL = BT CT c sin 12 A = TK LT = a−CT CT a−CT CT = b sin 12 A c b = = TK AK − AL −TK ⋅⋅⋅⋅⋅⋅⋅⋅⋅⋅⋅⋅⋅⋅⋅⋅⋅⋅⋅⋅⋅⋅⋅⋅⋅⋅⋅⋅⋅⋅⋅⋅⋅⋅⋅⋅⋅⋅⋅⋅⋅⋅⋅⋅⋅⋅⋅⋅ (3) TK c cos 12 A − b cos 12 A −TK Dari persamaan (3) didapat c ⋅ CT = ab − b ⋅ CT ⋅⋅⋅⋅⋅⋅⋅⋅⋅⋅⋅⋅⋅⋅⋅⋅⋅⋅⋅⋅⋅⋅⋅⋅⋅⋅⋅⋅⋅⋅⋅⋅⋅⋅⋅⋅⋅⋅⋅⋅⋅⋅⋅⋅⋅⋅⋅ (4) CT = bab +c Juga didapat c(c − b) cos 12 A − c ⋅ TK = b ⋅ TK TK = LT = c (c − b ) cos 12 A b+c b c ⋅ TK ⋅⋅⋅⋅⋅⋅⋅⋅⋅⋅⋅⋅⋅⋅⋅⋅⋅⋅⋅⋅⋅⋅⋅⋅⋅⋅⋅⋅⋅⋅⋅⋅⋅⋅⋅⋅⋅⋅⋅⋅⋅ (5) Misalkan lingkaran luar ∆A1LK memot ong garis BC di t it ik N. Karena ∠A1LT = ∠KNT dan ∠A1TL = ∠KTN maka ∆KTN sebangun dengan ∆A1TL. Jadi, A1T ⋅ TN = LT ⋅ TK (CT − CA1) ⋅ TN = bc ⋅ TK2 (bab+c − b cos C ) ⋅ TN = b c ⋅ ( c 2 (c −b )2 2 (b + c )2 Dengan menggunakan cos C = a ( c −b ) TN = 2(b + c ) a 2 + b 2 −c 2 2 ab maka didapat a ( c −b ) + 2(b + c ) = a2 Maka, N adalah pert engahan BC. Jadi, N = M. Jadi, t it ik A1, L, K dan M t erlet ak pada sat u lingkaran. ∴ Terbukti bahwa A1 , L, M dan K siklik . CN = CT + TN = ab b+c (cos C + 1)) SMA Negeri 5 Bengkulu Eddy Hermanto, ST 319 RIWAYAT HIDUP PENULIS Eddy Hermanto lahir di Desa Bunut Tinggi, Kecamatan Talo, Kabupaten Bengkulu Selatan (sekarang Kabupaten Seluma) pada tanggal 9 September 1979. Pendidikan SD dan SLTP diselesaikannya di Lampung, yaitu SD di SD Negeri 2 Bandar Jaya, Lampung Tengah dan SLTP di SMP Negeri Bandar Jaya, Lampung Tengah. Sedangkan pendidikan SLTA dilaluinya di SMU Negeri 5 Bengkulu. Penulis yang juga merupakan putera asli Bengkulu ini kemudian melanjutkan pendidikan S1 ke Jurusan Teknik Sipil Fakultas Teknik Universitas Gadjah Mada Yogyakarta pada tahun 1997 yang diselesaikannya pada bulan Februari 2002 dengan predikat Cum Laude. Saat ini Penulis bekerja sebagai PNS di Pemerintah Kota Bengkulu pada Bagian Penyusunan Program Setda Kota Bengkulu yang telah digeluti sejak Desember 2002. Selain bekerja di Pemerintah Kota Bengkulu, Penulis juga aktif membina siswa-siswa di SMA N 5 Bengkulu baik dalam persiapan menghadapi Ujian Masuk Universitas Gadjah Mada (UM-UGM), Seleksi Penerimaan Mahasiswa baru (SPMB) maupun ketika SMA N 5 Bengkulu akan menghadapi perlombaan-perlombaan baik tingkat kota, provinsi maupun nasional. Penulis juga pernah beberapa kali membina siswa-siswa dari Provinsi Bengkulu yang akan mengikuti Olimpiade Sains Nasional Bidang Matematika.